Property case digests 2013

September 16, 2017 | Author: Karen Supapo | Category: Mortgage Law, Property, Foreclosure, Surety Bond, Lawsuit
Share Embed Donate


Short Description

Comprehensive Property case digests MBA-JD Syllabus 2013...

Description

IMMOVABLE AND MOVABLE PROPERTY (ART. 415-418) JRPA Lopez v. Orosa G.R. Nos. L-10817-18, 103 SCRA 98 DOCTRINE: For while it is true that generally, real estate connotes the land and the building constructed thereon, it is obvious that the inclusion of the building in the enumeration of what may constitute real properties could only mean one thing—that a building is by itself an immovable property FACTS: Lopez was engaged in business under the name Lopez-Castelo Sawmill. Orosa, who lived in the same province as Lopez, one day approached Lopez and invited the latter to make an investment in the theatre business. Orosa, his family and close friends apparently were forming a corporation named Plaza Theatre. Lopez expressed his unwillingness to invest. Nonetheless, there was an oral agreement between Lopez and Orosa that Lopez would be supplying the lumber for the construction of the theatre. The terms were the following: one, Orosa would be personally liable for any account that the said construction would incur; two, payment would be by demand and not by cash on delivery. Pursuant to the agreement, Lopez delivered the lumber for the construction. Lopez was only paid one-third of the total cost. The land on which the building has been erected was previously owned by Orosa, which was later on purchased by the corporation. Due to the incessant demands of Lopez, the corporation mortgaged its properties. On an earlier relevant date, the corporation obtained a loan with Luzon Surety Company as surety and in turn, the corporation executed a mortgage over the land and building. In the registration of the land under Act 496, such mortgage wasn‘t revealed. Also due to the demands of Lopez, Orosa issued a deed of assignment over his shares of stock in the corporation. As there was still an unpaid balance, Lopez filed a case against Orosa and Plaza theatre. He asked that Orosa and Plaza theatre be held liable solidarily for the unpaid balance; and in case defendants failed to pay, the land and building should be sold in public auction with the proceeds to be applied to the balance; or that the shares of stock be sold in public auction. Lopez also had lis pendens be annotated in the OCT. The trial court decided that there was joint liability between defendants and that the material man‘s lien was only confined to the building. ISSUE: W/N the material men‘s lien for the value of the materials used in the construction of the building attaches to said structure alone and doesn‘t extend to the land on which the building is adhered to? HELD: The contention that the lien executed in favor of the furnisher of materials used for the construction and repair of a building is also extended to land on which the building was constructed is without merit. For while it is true that generally, real estate connotes the land and the building constructed thereon, it is obvious that the inclusion of the building in the enumeration of what may constitute real properties could only mean one thing—that a building is by itself an immovable property. Moreover, in the absence of any specific provision to the contrary, a building is an immovable property irrespective of whether or not said structure and the land on which it is adhered to belong to the same owner.

Appellant invoked Article 1923 of the Spanish Civil Code, which provides—―With respect to determinate real property and real rights of the debtor, the following are preferred: xxx Credits for reflection, not entered or recorded, and only with respect to other credits different from those mentioned in four next preceding paragraphs.‖ Close examination of the abovementioned provision reveals that the law gives preference to unregistered refectionary credits only with respect to the real estate upon which the refectionary or work was made. This being so, the inevitable conclusion must be that the lien so created attaches merely to the immovable property for the construction or repair of which the obligation was incurred. Therefore, the lien in favor of appellant for the unpaid value of the lumber used in the construction of the building attaches only to said structure and to no other property of the obligors. ABB Associated Insurance and Surety Company v. Iya 103 SCRA 972 DOCTRINE: A building is an immovable property irrespective of where or not said structure and the land on which it is adhered to belong to the same owner. FACTS: Spouses Adriano Valino and Lucia A. Valino own a house of strong materials. They filed a bond of P 11,000.00 subscribed by the Associated Insurance and Surety Co., Inc. and as a counterguaranty, the spouses Valino executed an alleged chattel mortgage on the aforementioned house in favor of the surety company. The parcel of land on which the house is erected was still registered in the name of the Philippine Realty Corporation but was able to obtain the same from them after full payment of the purchase price. The Valinos acquired another loan from Isabel Iya for P12,000.00, executing a real estate mortgage over the house and lot. However, they were unable to pay off their other loan which caused the foreclosure of the chattel mortgage. The surety company was awarded the land as the highest bidder in the auction but later on discovered that the land was subject to a real estate mortgage. The surety company then requested that the house and lot be excluded from the real estate mortgage. Iya, in her answer, said that she had a real right over the property and that the chattel mortgage on which the foreclosure was based should be declared null and void for non-compliance with the form required by law. The CA ruled that the foreclosure of the real estate mortgage is limited to the land alone and they awarded the structure to the surety company saying that the house is a personal property and may be subject to chattel mortgage. ISSUE: Which of the mortgages should have preference? HELD: It was held in Lopez vs. Orosa that the building is an immovable itself, separate and distinct from the land. A building is an immovable property irrespective of whether or not said structure and the land on which it is adhered to belong to the same owner. Only personal properties can be the subject of a chattel mortgage and since the structure in this case is an immovable, it cannot subject to a chattel mortgage. Therefore the chattel mortgage and the sale on which it was based should be declared null and void. Also, while it is true that said document was registered in the Chattel Mortgage Register of Rizal, this act produced no effect whatsoever for where the interest conveyed is in the nature of a real property, the

registration of the document in the registry of chattels is merely a futile act which would produce no legal effect insofar as the building is concerned. FZC Bicerra v. Teneza G.R. No. L-16218, 6 SCRA 648 DOCTRINE: A house is classified as immovable property by reason of its adherence to the soil on which it is built (Article 415, paragraph 1, Civil Code). This classification holds true regardless of the fact that the house may be situated on land belonging to a different owner. But once the house is demolished it ceases to exist, hence its character as an immovable likewise ceases. FACTS: The Bicerras are supposedly the owners of the house (PhP 20,000) built on a lot owned by them in Lagangilang, Abra which the Tenezas forcibly demolished in January 1957, claiming to be the owners thereof. The materials of the house were placed in the custody of the barrio lieutenant. The Bicerras filed a complaint claiming actual damages of P200, moral and consequential damages amounting to P600, and the costs. The CFI Abra dismissed the complaint claiming that the action was within the exclusive (original) jurisdiction of the Justice of the Peace Court of Lagangilang, Abra. The Supreme Court affirmed the order appealed. Having been admitted in forma pauperis, no costs were adjudged. ISSUE: WON the house is immovable property even if it is on the land of another HELD: House is immovable property even if situated on land belonging to a different owner; Exception, when demolished. LNAC Evangelista v. Alto Surety & Insurance Co., Inc. G.R. No. L-11139 DOCTRINE: Sales on execution affect the public and third persons. The regulation governing sales on execution are for public officials to follow. The form of proceedings prescribed for each kind of property is suited to its character, not to the character, which the parties have given to it or desire to give it. When the rules speak of personal property, property which is ordinarily so considered is meant; and when real property is spoken of, it means property which is generally known as real property. The regulations were never intended to suit the consideration that parties may have privately given to the property levied upon. FACTS: Petitioner Santos Evangelista instituted a Civil Case for a sum of money. He obtained a writ of attachment, which levied upon a house, built by Rivera on a land situated in Manila and leased to him, by filing copy of said writ and the corresponding notice of attachment with the Office of the Register of Deeds of Manila. Judgment was rendered in favor of Evangelista, who, bought the house at public auction held in compliance with the writ of execution issued in said case. The definite deed of sale was issued to him upon expiration of the period of redemption. When

Evangelista sought to take possession of the house, Rivera refused to surrender it, upon the ground that he had leased the property to the Alto Surety & Insurance Co., Inc., respondent herein, and that the latter is now the true owner of said property. It appears that a definite deed of sale of the same house had been issued to respondent, as the highest bidder at an auction sale held in compliance with a writ of execution. Hence, Evangelista instituted the present action against respondent and Rivera, for the purpose of establishing his (Evangelista) title over said house, securing possession thereof, apart from recovering damages. In its answer, respondent alleged that it has a better right to the house, because the sale made, and the definite deed of sale executed, in its favor, on September 29, 1950 and May 10, 1952, respectively, precede the sale to Evangelista (October 8, 1951) and the definite deed of sale in his favor (October 22, 1952). Rivera, in effect, joined forces with respondent. CFI rendered judgment for Evangelista, sentencing Rivera and respondent to deliver the house in question to petitioner herein and to pay him, jointly and severally from October, 1952, until said delivery, plus costs. On appeal taken by respondent, the above decision was reversed by the CA which absolved Alto Surety from the complaint on account that although the writ of attachment in favor of Evangelista had been filed with the Register of Deeds of Manila prior to the sale in favor of Alto Surety, Evangelista did not acquire thereby a preferential lien, the attachment having been levied as if the house in question were immovable property. Evangelista now seeks a review by certiorari. ISSUE: Whether a house, constructed by the lessee of the land on which it is built, should be dealt with, for purpose of attachment, as immovable property, or as personal property. HELD: Said house is not personal property, much less a debt, credit or other personal property not capable of manual delivery, but immovable property. As explicitly held, in Laddera vs. Hodges, "a true building (not merely superimposed on the soil) is immovable or real property, whether it is erected by the owner of the land or by usufructuary or lessee. It is true that the parties to a deed of chattel mortgage may agree to consider a house as personal property for purposes of said contract. However, this view is good only insofar as the contracting parties are concerned. It is based, partly, upon the principle of estoppel. Neither this principle, nor said view, is applicable to strangers to said contract. Much less is it in point where there has been no contract whatsoever, with respect to the status of the house involved, as in the case at bar. The rules on execution do not allow, and, we should not interpret them in such a way as to allow, the special consideration that parties to a contract may have desired to impart to real estate, for example, as personal property, when they are, not ordinarily so. Sales on execution affect the public and third persons. The regulation governing sales on execution are for public officials to follow. The form of proceedings prescribed for each kind of property is suited to its character, not to the character, which the parties have given to it or desire to give it. When the rules speak of personal property, property which is ordinarily so considered is meant; and when real property is spoken of, it means property which is generally known as real property. The regulations were never intended to suit the consideration that parties may have privately given

to the property levied upon. Enforcement of regulations would be difficult were the convenience or agreement of private parties to determine or govern the nature of the proceedings. The mere fact that a house was the subject of the chattel mortgage and was considered as personal property by the parties does not make said house personal property for purposes of the notice to be given for its sale of public auction. This ruling is demanded by the need for a definite, orderly and well defined regulation for official and public guidance and would prevent confusion and misunderstanding. The foregoing considerations apply, with equal force, to the conditions for the levy of attachment, for it similarly affects the public and third persons. TKDC Leung Yee v. Strong Machinery Co. G.R. No. L-11658 DOCTRINE: The mere fact that the parties decided to deal with the building as personal property does not change its character as real property. Neither the original registry in the chattel mortgage registry nor the annotation in said registry of the sale of the mortgaged property had any effect on the building. FACTS: Compañia Agricola Filipina bought several rice-cleaning machinery from a machinery company, Frank L. Strong Machinery Company and executed a chattel mortgage to secure payment of the purchase price. The deed of mortgage includes the building where the machinery was installed without any reference to the land on which it stood. Since Compañia Agricola Filipina failed to pay when due, the mortgaged property was sold by the sheriff and was bought by the machinery company. Few weeks later, Compañia Agricola Filipina executed a deed of sale of the land where the building stood to the machinery company. In effect, the machinery company possessed the building when the sale took place and continued its possession ever since. When the chattel mortgage was executed, Compañia Agricola Filipina executed another mortgage in favor of Yee over the building to pay its debt to the machinery company. Since Compañia Agricola Filipina failed to pay when due, Yee secured a judgment to levy execution upon the building and bought the building at the sheriff‘s sale; Yee secured the sheriff‘s certificate of sale and registered it in the land registry. When the execution was levied upon the building, the machinery company filed with the sheriff a sworn statement setting up its claim of title and demanding the release of property from the levy. On the other hand, Yee filed an action to recover possession of the building from the machinery company. Trial court ruled in favor of the machinery company on the basis of Article 1473 of the Civil Code; it ruled that the machinery company registered the title to the building prior to the registration date of Yee‘s certificate. ISSUE: Whether or not the nature of property is changed by its registration in the Chattel Mortgage Registry. -- NO HELD:

The registry under Article 1473 of the Civil Code refers to registry of real property and the annotation or inscription of a deed of sale of real property in a chattel mortgage registry cannot be given the legal effect of an inscription in the registry of real property. The Chattel Mortgage Law contemplates mortgages of personal property. The sole purpose and object of the chattel mortgage registry is the registration of personal property mortgages executed in the manner and form prescribed in the statute. In this case, the building where the rice-cleaning machinery was installed was real property. The mere fact that the parties dealt with it as separate and apart from the land on which it stood does not change its character as real property. Neither the original registry of the building in the chattel mortgage nor the annotation of sale of the mortgaged property in the registry had any effect on the building‘s nature as immovable property. AMD Standard Oil Co. of New York v. Jaramillo 44 SCRA 630 DOCTRINE: The duties of a register of deeds in respect to the registration of chattel mortgage are of a purely ministerial character; and no provision of law can be cited which confers upon him any judicial or quasi-judicial power to determine the nature of any document of which registration is sought as a chattel mortgage. FACTS: Gervasia de la Rosa, Vda. de Vera, was the lessee of a parcel of land situated in Manila and owner of the house built thereon. She executed a chattel mortgage in favor of Standard Oil Co. to convey both the leasehold interest in said lot and the building. After the document had been duly acknowledge and delivered, petitioner presented it to the respondent, Joaquin Jaramillo, as ROD of Manila, to be recorded in the book of record of chattel mortgages. Upon examination of the instrument, Jaramillo was of the opinion that it was not a chattel mortgage as the interest mortgaged did not appear to be personal property, within the meaning of the Chattel Mortgage Law, and registration was refused on this ground only. So, petitioner sought for a peremptory mandamus to compel the respondent to record the said document in the register. Jaramillo interposed a demurrer before the SC. ISSUE: W/N the ROD can refuse the registration of a Chattel Mortgage? NO HELD: It is his duty to accept the proper fee and place the instrument on record. The duties of a register of deeds in respect to the registration of chattel mortgage are of a purely ministerial character; and no provision of law can be cited which confers upon him any judicial or quasijudicial power to determine the nature of any document of which registration is sought as a chattel mortgage. The original provisions touching this matter are contained in section 15 of the Chattel Mortgage Law (Act No. 1508), as amended by Act No. 2496; but these have been transferred to section 198 of the Administrative Code. There is nothing in any of these provisions conferring upon the register of deeds any authority whatever in respect to the "qualification‖, of chattel mortgage. His duties in respect to such instruments are ministerial only. The efficacy of the act of recording a chattel mortgage consists in the fact that it operates as constructive notice of the existence of

the contract, and the legal effects of the contract must be discovered in the instrument itself in relation with the fact of notice. Registration adds nothing to the instrument, considered as a source of title, and affects nobody's rights except as a specifies of notice. Articles 334 and 335 of the Civil Code supply no absolute criterion for discriminating between real property and personal property for purpose of the application of the Chattel Mortgage Law. Those articles state rules which, considered as a general doctrine, are law in this jurisdiction; but it must not be forgotten that under given conditions property may have character different from that imputed to it in said articles. It is undeniable that the parties to a contract may by agreement treat as personal property that which by nature would be real property; and it is a familiar phenomenon to see things classed as real property for purposes of taxation which on general principle might be considered personal property. CRF Sibal v. Valdez G.R. No. L-27532 DOCTRINE: For the purpose of attachment and execution, and for the purposes of the Chattel Mortgage Law, "ungathered products" have the nature of personal property. (batasnatin) FACTS: As a first cause of action the plaintiff alleged that the defendant Vitaliano Mamawal, deputy sheriff of the Province of Tarlac, by virtue of a writ of execution issued by the Court of First Instance of Pampanga, attached and sold to the defendant Emiliano J. Valdez the sugar cane planted by the plaintiff and his tenants on seven parcels of land. That within one year from the date of the attachment and sale the plaintiff offered to redeem said sugar cane and tendered to the defendant Valdez the amount sufficient to cover the price paid by the latter, the interest thereon and any assessments or taxes which he may have paid thereon after the purchase, and the interest corresponding thereto and that Valdez refused to accept the money and to return the sugar cane to the plaintiff. One of the defenses of the defendant Emiliano J. Valdez is that the sugar cane in question had the nature of personal property and was not, therefore, subject to redemption. The trial court hold that the sugar cane in question was personal property and, as such, was not subject to redemption. ISSUE: Whether the sugar cane in question is personal or real property under civil code? Under chattel mortgage law? HELD: The court ruled that It is contended that sugar cane comes under the classification of real property as "ungathered products" in paragraph 2 of article 334 of the Civil Code. Said paragraph 2 of article 334 enumerates as real property the following: Trees, plants, and ungathered products, while they are annexed to the land or form an integral part of any immovable property." We may, therefore, conclude that paragraph 2 of article 334 of the Civil Code has been modified by section 450 of the Code of Civil Procedure and by Act No. 1508, in the sense that, for the purpose of attachment and execution, and for the purposes of the Chattel Mortgage Law, "ungathered products" have the nature of personal property. The lower court, therefore,

committed no error in holding that the sugar cane in question was personal property and, as such, was not subject to redemption. MPF Tsai v. CA G.R. No. 120098 DOCTRINE: Even if the properties are immovable by nature, nothing detracts the parties from treating them as chattels to secure an obligation under the principle of estoppel. FACTS: ● EVERTEX secured a loan from PBC, guaranteed by real estate and chattel mortgage over a parcel of land where the factory stands, and the chattels located therein, as included in a schedule attached to the mortgage contract. another loan was obtained secured by a chattel mortgage over properties with similar descriptions listed in the first schedule. ● During the date of execution of the second mortgage. EVERTEX purchased machineries and equipment. ● Due to business reverses, EVERTEX filed for insolvency proceedings. It failed to pay its obligation and thus, PBC initiated extrajudicial foreclosure of the mortgages. ● PBC was the highest bidder in the public auctions, making it the owner of the properties. It then leased the factory premises to Tsai. ● Afterwards, EVERTEX sought the annulment of the sale and conveyance of the properties to PBC as it was allegedly a violation of the insolvency law. ● The RTC held that the lease and sale were irregular as it involved properties not included in the schedule of the mortgage contract. ISSUE: Whether or not the (immovable) properties in question can be entered into a chattel mortgage. -YES HELD: An immovable may be considered a personal property if there is a stipulation as when it is used as security in the payment of an obligation where a chattel mortgage is executed over it, as in the case at bar. While it is true that the controverted properties appear to be immobile, a perusal of the contract of real estate mortgage and chattel mortgage by the parties gives a contrary indication. Both the trial and appellate courts show that the intention was to treat the machineries as movables or personal property. Assuming that the properties were considered immovables, nothing detracts the parties from treating it as chattels to secure an obligation under the principle of estoppel. AMDG Yap v. Tanada G.R. No. L-32917 DOCTRINE: The Civil code considers as immovable property among others, anything ―attached to an immovable in a fixed manner, in such a way that it cannot be separated therefrom without breaking the material or deterioration of the object‖ FACTS: Goulds Pumps International (Phil.), Inc. (herein Goulds) filed a complaint against Yap and his

wife seeking to recover the balance of the price and installation of the water pump in the latter‘s residence. The city court declared Yap and his wife in default and rendered a judgment in favor of Goulds. Yap appealed to the CFI wherein Judge Tanada was residing. Yap was again declared in default and judgment was again rendered in favor of Goulds. Later on, Judge Tanada granted Gould‘s Motion for Issuance of Writ of Execution. Subsequently, the water pump was sold in a public auction in favor of Goulds being the highest bidder. Another writ of execution was issued as regards the removal of the water pump and delivery of such to Goulds. Yap is questioning validity of the auction sale and praying that it be annulled as well as the writ of execution. Yap is alleging that the water pump is considered as an immovable property because it is installed in his residence. He also argued that being an immovable property, a notice must be made before the auction sale pursuant to the Rules of Court. It is noted that Yap filed several motion of reconsideration to which all were denied. ISSUE: Whether or not the water pump is an immovable property -- NO HELD: The Civil code considers as immovable property among others, anything ―attached to an immovable in a fixed manner, in such a way that it cannot be separated therefrom without breaking the material or deterioration of the object‖ The water pump involved in this case does not satisfy the above description. It is highly possible to remove the water pump without it breaking or deteriorating by simply loosening the bolts or dismantling the fasteners that were used to attach or install it in his house. GCG Mindanao Bus Co. v. City Assessor and Treasurer G.R. No. L-17870 DOCTRINE: Movable equipment, to be immobilized in contemplation of Article 415 of the Civil Code, must be the essential and principal elements of an industry or works which are carried on in a building or on a piece of land. Thus, where the business is one of transportation, which is carried on without a repair or service shop, and its rolling equipment is repaired or serviced in a shop belonging to another, the tools and equipment in its repair shop which appear movable are merely incidentals and may not be considered immovables, and, hence, not subject to assessment as real estate for purposes of the real estate tax. FACTS: Petitioner is a public utility solely engaged in transporting passengers and cargoes by motor trucks, over its authorized lines in the Island of Mindanao, collecting rates approved by the Public Service Commission. The petitioner is the owner of the land where it maintains and operates a garage for its TPU motor trucks; a repair shop; blacksmith and carpentry shops, and with these machineries which are placed therein, its TPU trucks are made; body constructed; and same are repaired in a condition to be serviceable in the TPU land transportation business it operates. These machineries have never been or were never used as industrial equipments to produce finished products for sale, nor to repair machineries, parts and the like offered to the general public indiscriminately for business or commercial purposes for which petitioner has never

engaged in, The City Assessor of CDO then assessed a P4,400 realty tax on said machineries and repair equipment. This was then appealed to the Court of Tax Appeals (CTA) who sustained the respondent city assessor's ruling. ISSUE: Whether or not the machineries and the equipments are considered immobilized and thus subject to a realty tax. -- NO HELD: The Supreme Court held a decision for the petition for review to be set aside and the equipments in question declared not subject to assessment as real estate for the purposes of the real estate tax. The law that governs the determination of the question at issue is as follows: Art. 415. The following are immovable property: xxx

xxx

xxx

(5) Machinery, receptacles, instruments or implements intended by the owner of the tenement for an industry or works which may be carried on in a building or on a piece of land, and which tend directly to meet the needs of the said industry or works; (Civil Code of the Phil.) Aside from the element of essentiality the above-quoted provision also requires that the industry or works be carried on in a building or on a piece of land. Thus in the case of Berkenkotter vs. Cu Unjieng, supra, the "machinery, liquid containers, and instruments or implements" are found in a building constructed on the land. A sawmill would also be installed in a building on land more or less permanently, and the sawing is conducted in the land or building. But in the case at bar the equipments in question are destined only to repair or service the transportation business, which is not carried on in a building or permanently on a piece of land, as demanded by the law. Said equipments may not, therefore, be deemed real property. Resuming what we have set forth above, we hold that the equipments in question are not absolutely essential to the petitioner's transportation business, and petitioner's business is not carried on in a building, tenement or on a specified land, so said equipment may not be considered real estate within the meaning of Article 415 (c) of the Civil Code. Said equipments are not considered immobilized as they are merely incidental, not essential and principal to the business of the petitioner. The transportation business could be carried on without repair or service shops of its rolling equipment as they can be repaired or services in another shop belonging to another VCL IV Fels Energy, Inc. v. Province of Batangas, et al. G.R. No. 168557 DOCTRINE: Article 415 (9) of the New Civil Code provides that ―docks and structures which,

though floating, are intended by their nature and object to remain at a fixed place on a river, lake, or coast‖ are considered immovable property. Thus, power barges are categorized as immovable property by destination, being in the nature of machinery and other implements intended by the owner for an industry or work which may be carried on in a building or on a piece of land and which tend directly to meet the needs of said industry or work. FACTS: On January 18, 1993, National Power Corporation (NPC) entered into a lease contract with Polar Energy, Inc. over 3×30 MW diesel engine power barges moored at Balayan Bay in Calaca, Batangas. The contract, denominated as an Energy Conversion Agreement, was for a period of five years. Article 10 states that NPC shall be responsible for the payment of taxes. (other than (i) taxes imposed or calculated on the basis of the net income of POLAR and Personal Income Taxes of its employees and (ii) construction permit fees, environmental permit fees and other similar fees and charges. Polar Energy then assigned its rights under the Agreement to Fels despite NPC‘s initial opposition. FELS received an assessment of real property taxes on the power barges from Provincial Assessor Lauro C. Andaya of Batangas City. FELS referred the matter to NPC, reminding it of its obligation under the Agreement to pay all real estate taxes. It then gave NPC the full power and authority to represent it in any conference regarding the real property assessment of the Provincial Assessor. NPC filed a petition with the Local Board Assessment Appeals (LBAA). The LBAA ordered Fels to pay the real estate taxes. The LBAA ruled that the power plant facilities, while they may be classified as movable or personal property, are nevertheless considered real property for taxation purposes because they are installed at a specific location with a character of permanency. The LBAA also pointed out that the owner of the barges–FELS, a private corporation–is the one being taxed, not NPC. A mere agreement making NPC responsible for the payment of all real estate taxes and assessments will not justify the exemption of FELS; such a privilege can only be granted to NPC and cannot be extended to FELS. Finally, the LBAA also ruled that the petition was filed out of time. Fels appealed to the Central Board Assessment Appeals (CBAA). The CBAA reversed and ruled that the power barges belong to NPC; since they are actually, directly and exclusively used by it, the power barges are covered by the exemptions under Section 234(c) of R.A. No. 7160. As to the other jurisdictional issue, the CBAA ruled that prescription did not preclude the NPC from pursuing its claim for tax exemption in accordance with Section 206 of R.A. No. 7160. Upon MR, the CBAA reversed itself. ISSUE: Whether or not barges are considered as real property, thus can be subject to real property tax -- YES HELD: The CBAA and LBAA power barges are real property and are thus subject to real property tax. In Consolidated Edison Company of New York, Inc., et al. v. The City of New York, et al., a power company brought an action to review property tax assessment. On the city‘s motion to dismiss, the Supreme Court of New York held that the barges on which were mounted gas turbine power plants designated to generate electrical power, the fuel oil barges which supplied fuel oil to the power plant barges, and the accessory equipment mounted on the barges were subject to real property taxation. Moreover, Article 415 (9) of the New Civil Code provides that ―docks and structures which,

though floating, are intended by their nature and object to remain at a fixed place on a river, lake, or coast‖ are considered immovable property. Thus, power barges are categorized as immovable property by destination, being in the nature of machinery and other implements intended by the owner for an industry or work which may be carried on in a building or on a piece of land and which tend directly to meet the needs of said industry or work. FXRL Davao Sawmill Co. v. Castillo G.R. No. 40411, DOCTRINE: Generally, machinery becomes immobilized when placed by the owner of the plant or property. This rule does not apply should the machinery be placed by any other person such as a tenant or usufructuary. FACTS: ● The petitioner company operates a sawmill in barrio Tigatu, Davao. ● Said facility contained both movable and immovable property (machines and other such implements). ● However, the land on which it is situated belongs to another person. ● The parties executed a lease contract providing that upon the expiration or termination of such lease, the following shall happen: o The ownership of all structures and improvements introduced by the petitioner company shall be transferred to the respondents without any cost or obligation to pay. o The machines and their accessories shall not be included in said transfer. ● It was noted by the court that in a previous case between the two parties, judgment was rendered against the petitioner company upon which a writ of execution was brought against its machines (as personalty) in favor of Castilllo, et al. ● Additionally, the records of the current case reflected that the petitioner company had treated its machinery as personal property by executing chattel mortgages on them in favor of third persons. ● Petitioner company contends that its machines are immovable under the first and fifth paragraphs of Article 334 (now Article 415) of the Civil Code. ISSUE: W/N the machines of the petitioner company are movable or immovable property. HELD: The machines are movable. The court observed that the petitioner company failed to register its protest at the time its machines were sold. Generally, this inaction would be inconclusive but it is indicative of the intention impressed upon the property in question. This is so because while machines are generally movable property, they may nevertheless be ―immobilized‖ by destination or purpose subject to several conditions. This conclusion finds its ground under the fifth paragraph of Article 415. Here, machinery becomes immobilized when placed by the owner of the plant or property. This rule does not apply should the machinery be placed by any other person such as a tenant or usufructuary.

Applying the rule to the case on hand, the machinery was placed by the petitioner company who was merely a lessee. As such, the equipment was never immobilized in the first place. RSDM Makati Leasing and Financial Corporation v. Wearever Textile Mills, Inc. G.R. No. L-58469 DOCTRINE: If a house of strong materials, like what was involved in the above Tumalad case, may be considered as personal property for purposes of executing a chattel mortgage thereon as long as the parties to the contract so agree and no innocent third party will be prejudiced thereby, there is absolutely no reason why a machinery, which is movable in its nature and becomes immobilized only by destination or purpose, may not be likewise treated as such. This is really because one who has so agreed is estopped from denying the existence of the chattel mortgage. FACTS: The private respondent Wearever Textile Mills, Inc., discounted and assigned several receivables with the former under a Receivable Purchase Agreement in order to obtain financial accommodations from herein petitioner Makati Leasing and Finance Corporation. To secure the collection of the receivables assigned, private respondent executed a Chattel Mortgage over certain raw materials inventory as well as a machinery described as an Artos Aero Dryer Stentering Range. Upon default, petitioner filed a petition for extrajudicial foreclosure of the properties mortgage to it. The Deputy Sheriff assigned to implement the foreclosure failed to gain entry into private respondent's premises and was not able to effect the seizure of the aforedescribed machinery. Petitioner thereafter filed a complaint for judicial foreclosure with the Court of First Instance of Rizal. Acting on petitioner's application for replevin, the lower court issued a writ of seizure, the enforcement of which was however subsequently restrained upon private respondent's filing of a motion for reconsideration. After several incidents, the lower court finally issued an order lifting the restraining order for the enforcement of the writ of seizure and an order to break open the premises of private respondent to enforce said writ. The lower court reaffirmed its stand upon private respondent's filing of a further motion for reconsideration. The Court of Appeals, in certiorari and prohibition proceedings subsequently filed by herein private respondent, set aside the Orders of the lower court and ordered the return of the drive motor seized by the sheriff pursuant to said Orders, after ruling that the machinery in suit cannot be the subject of replevin, much less of a chattel mortgage, because it is a real property pursuant to Article 415 of the new Civil Code, the same being attached to the ground by means of bolts and the only way to remove it from respondent's plant would be to drill out or destroy the concrete floor, the reason why all that the sheriff could do to enfore the writ was to take the main drive motor of said machinery. The appellate court rejected petitioner's argument that private respondent is estopped from claiming that the machine is real property by constituting a chattel mortgage thereon. ISSUE: Whether or not the property in suit is real property – NO. It is a personal property HELD: Examining the records of the instant case, We find no logical justification to exclude the rule out,

as the appellate court did, the present case from the application of the abovequoted pronouncement. If a house of strong materials, like what was involved in the above Tumalad case, may be considered as personal property for purposes of executing a chattel mortgage thereon as long as the parties to the contract so agree and no innocent third party will be prejudiced thereby, there is absolutely no reason why a machinery, which is movable in its nature and becomes immobilized only by destination or purpose, may not be likewise treated as such. This is really because one who has so agreed is estopped from denying the existence of the chattel mortgage. In rejecting petitioner's assertion on the applicability of the Tumalad doctrine, the Court of Appeals lays stress on the fact that the house involved therein was built on a land that did not belong to the owner of such house. But the law makes no distinction with respect to the ownership of the land on which the house is built and We should not lay down distinctions not contemplated by law. It must be pointed out that the characterization of the subject machinery as chattel by the private respondent is indicative of intention and impresses upon the property the character determined by the parties. As stated in Standard Oil Co. of New York v. Jaramillo, 44 Phil. 630, it is undeniable that the parties to a contract may by agreement treat as personal property that which by nature would be real property, as long as no interest of third parties would be prejudiced thereby. MRAM Board of Assessment Appeals v. MERALCO 10 SCRA 68

DOCTRINE: The steel towers or poles of MERALCO are not real properties because 1) they are not adhered to the soil, 2) they are not attached to an immovable property and can be dismantled without breaking or deteriorating the material and 3) they are not machineries nor instruments or implements intended for the industry or works on the land FACTS: Generated by its hydroelectric plant, MERALCO‘s electric power is transmitted from Laguna to Manila through electric transmission wires. These electric transmission wires which carry high voltage current, are fastened to insulators attached on steel towers. MERALCO has constructed 40 of these steel towers within Quezon City, on land belonging to it. Three steel towers were the subject of this dispute. When inspected, the findings disclose that there was no concrete foundation but there was adobe stone underneath. Further, it could not be ascertained whether said adobe stone was purposely or not. From this, the City Assessor of Quezon City declared the steel towers subject to real property tax. MERALCO, however, protested the assessment saying that the steel towers are considered poles and according to their franchise, it is exempt from taxation ISSUE: Whether or not the steel towers or poles of the MERALCO are considered real properties, hence subject to real property tax? HELD:

The Supreme Court held in the negative. The Court said that the steel towers are personal properties. The Court based their ruling on the enumeration of immovable properties in Art. 415 of the Civil Code. First, the steel towers do not come within the objects mentioned in par. 1, because they do not constitute buildings or constructions adhered to the soil.Moreover, they are not construction analogous to buildings nor adhering to the soil because as per description, they are removable and merely attached to a square metal frame by means of bolts, which when unscrewed could easily be dismantled and moved from place to place. Second, they can not be included under paragraph 3 since they are not attached to an immovable in a fixed manner; they can be separated without breaking the material or causing deterioration upon the object to which they are attached. In fact, each of these steel towers or supports consists of steel bars joined together by means of bolts, which can be disassembled by unscrewing the bolts and reassembled by screwing the same. Lastly, they do not fall under paragraph 5, as they are not machineries, receptacles, instruments or implements. SC said that even if they were machineries, receptacles, instruments or implements, they are not intended for industry or works on the land. MERALCO is not engaged in an industry or works in the land in which the steel supports or towers are constructed. FMM Machinery & Engineering Supplies, Inc. v. CA G.R. No. L-7057 DOCTRINE: When the machinery and equipment in question appeared to be attached to the land, particularly to the concrete foundation of said premises, in a fixed manner, in such a way that the former could not be separated from the latter "without breaking the material or deterioration of the object or that in order to remove said outfit, it became necessary, not only to unbolt the same, but , also, to cut some of its wooden supports and when, said machinery and equipment were "intended by the owner of the tenement for an industry" carried on said immovable and tended, it becomes immovable property pursuant to paragraphs 3 and 5 of Article 415 of Civil Code of the Philippines. FACTS: ▪ On 13 March 1953, Machinery & Engineering Supplies, Inc. (the ―Petitioner‖) filed a complaint for replevin in the Court of First Instance (―CFI‖) of Manila for the recovery of the machinery and equipment sold and delivered to Ipo Limestone Co., Inc and Dr. Antonio Villarama (the ―Respondents‖) at their factory in Barrio Bigti, Norzagaray, Bulacan. ▪ Upon application ex-parte of the Petitioner and upon approval of its bond sum of P15,769.00, herein Respondent Judge issued an order directing the Provincial Sheriff of Bulacan to seize and take immediate possession of the properties specified in the said order. ▪ On 19 March 1953, two deputy sheriffs of Bulacan, Ramon S. Roco and a crew of technician and laborers proceeded to Bigti to carry out the CFI‘s order. ▪ Leonardo Contreras, herein Respondent Company‘s Manager met the sheriffs and handed the latter a letter addressed to Atty. Leopoldo C. Paled, ex-officio

Provincial Sheriff of Bulacan, signed by the Respondent Company‘s counsel, protesting against the seizure of the properties on the ground that the same are not personal properties. ▪ Roco and the deputy sheriffs contended that their duty is ministerial and went ahead to the factory. At the factory, Rocco‘s attention was called to the fact that the equipment could not possibly be dismantled without causing damages or injuries to the wooden frames attached to them but Roco insisted in dismantling the same on his own responsibility and alleged that the bond was posted for such eventuality. Thus, the deputy sheriffs directed that some of the machine‘s supports be cut. ▪ On 20 March 1953, the Respondent Company filed an urgent motion, with a counter-bond in the amount of P15,769 for the return of the properties seized by the sheriffs. On the same day, the trial court issued an order, directing the Provincial Sheriff of Bulacan to return the machinery and equipment to the place where they were installed at the time of seizure. ▪ On 2 March 1953, the deputy sheriffs returned the said properties by depositing them along the road near the quarry of the Respondent Company, without inventory and re-installation in its former position and replacing the destroyed posts, which rendered its use impracticable. ▪ On 23 March 1953, Respondents‘ counsel asked the provincial sheriff if the machinery and equipment dumped on the road would be re-installed to their former position and condition. The next day, the provincial sheriff filed an urgent motion in court manifesting the Roco had been asked to furnish the sheriff‘s office with the expenses, laborers, technical men and equipment to carry into effect the courts order, among other things but that Roco absolutely refused and asking the Court that Respondent Company be ordered to provide the required aid or relieve the sheriff of the duty of complying to the said order. ▪ On 30 March 1953, the trial court ordered the provincial sheriff and the Petitioner Company to reinstate the machinery and equipment removed by them in their original condition. An urgent motion of the provincial sheriff dated 15 April 1953 requesting for an extension was denied and on 4 May 1953, the trial court ordered the Petitioner Company to furnish the provincial sheriff with the necessary funds and technical crew and laborers to reinstate the machinery and equipment. ▪ The case was appealed before the Court of Appeals but the latter dismissed the same for lack of merit. ▪ Hence this petition filed before the Supreme Court (the ―SC‖). The Petitioner argued that the respondent judge had completely disregarded his manifestation that the machinery and equipment seized were and still are the Petitioner's property until fully paid for and such never became immovable. The question of ownership and the applicability of Art. 415 of the new Civil Code are immaterial in the determination of the only issue involved in this case. ISSUE: Whether the machineries and equipments can be considered as personal properties subject to replevin. -- NO

HELD: The SC held that the special civil action known as replevin, governed by Rule 62 of Court, is applicable only to "personal property". When the sheriff repaired to the premises of respondent company, the machinery and equipment in question appeared to be attached to the land, particularly to the concrete foundation of said premises, in a fixed manner, in such a way that the former could not be separated from the latter "without breaking the material or deterioration of the object." Hence, in order to remove said outfit, it became necessary, not only to unbolt the same, but, also, to cut some of its wooden supports. Moreover, said machinery and equipment were "intended by the owner of the tenement for an industry" carried on said immovable and tended." For these reasons, they were already immovable property pursuant to paragraphs 3 and 5 of Article 415 of Civil Code of the Philippines, which are substantially identical to paragraphs 3 and 5 of Article 334 of the Civil Code of Spain. As such immovable property, they were not subject to replevin. RGGM Punsalan, Jr. v. Vda. De Lacsamana 121 SCRA 331 DOCTRINE: Buildings are always immovable under the Civil Code. Separate treatment by the parties of building from the land in which it stood does not change the immovable character of the building. FACTS: Punsalan was the owner of a piece of land, which he mortgaged in favor of PNB. Due to his failure to pay, the mortgage was foreclosed and the land was sold in a public auction to which PNB was the highest bidder. On a relevant date, while Punsalan was still the possessor of the land, it secured a permit for the construction of a warehouse. A deed of sale was executed between PNB and Punsalan. This contract was amended to include the warehouse and the improvement thereon. By virtue of these instruments, respondent Lacsamana secured title over the property in her name. Petitioner then sought for the annulment of the deed of sale. Among his allegations was that the bank did not own the building and thus, it should not be included in the said deed. Petitioner‘s complaint was dismissed for improper venue. The trial court held that the action being filed in actuality by petitioner is a real action involving his right over a real property. ISSUE: Whether or not the warehouse is an immovable and must be tried in the province where the property lies. HELD: Warehouse claimed to be owned by petitioner is an immovable or real property. Buildings are always immovable under the Civil Code. A building treated separately from the land on which it is stood is immovable property and the mere fact that the parties to a contract seem to have dealt with it separate and apart from the land on which it stood did not change its character as immovable property.

MCSS Prudential Bank v. Panis 153 SCRA 390 FACTS: Plaintiff-spouses Magcale secured two loans over a 2-storey residential building. For failure of the plaintiffs to pay their obligation to defendant Bank after it became due, the deed of the Real Estate Mortgage were extrajudicially foreclosed. ISSUE: WON a valid real estate mortgage can be constituted on the building. -- YES HELD: Inclusion of building separate and distinct from land, in the provision of law can only mean that a building is by itself an immovable property. A building by itself may be mortgaged apart from the land on which it has been built. NKVS Tumalad v. Vicencio 41 SCRA 143 DOCTRINE: The view that parties to a deed of chattel mortgage may agree to consider a house as personal property for the purposes of said contract, "is good only insofar as the contracting parties are concerned. It is based, partly, upon the principle of estoppel.‖ FACTS: On 1 September 1955 defendants executed a chattel mortgage in favor of plaintiffs over their house located at Quiapo, Manila, which were being rented from Madrigal & Company, Inc. The mortgage was registered in the Registry of Deeds of Manila on 2 September 1955. The mortgage was executed to guarantee a loan of P4,800.00 received from plaintiffs. It was also agreed that default in the payment of any of the amortizations, would cause the remaining unpaid balance to become immediately due and Payable and the Chattel Mortgage will be enforceable in accordance with the provisions of Special Act No. 3135, and for this purpose, the Sheriff of the City of Manila or any of his deputies is hereby empowered and authorized to sell all the Mortgagor's property after the necessary publication in order to settle the financial debts of P4,800.00, plus 12% yearly interest, and attorney's fees. When defendants defaulted in paying, the mortgage was extrajudicially foreclosed, and the house was sold at public auction pursuant to the said contract. As highest bidder, plaintiffs were issued the corresponding certificate of sale. Thereafter, plaintiffs commenced Civil Case No. 43073 in the municipal court of Manila, praying, among other things, that the house be vacated and its possession surrendered to them, and for defendants to pay rent of P200.00 monthly from 27 March 1956 up to the time the possession is surrendered. MTC granted petition. Defendants, in their answers in both the municipal court and court a quo impugned the legality of the chattel mortgage, claiming that they are still the owners of the house. During the pendency of the appeal to the Court of First Instance, defendants failed to deposit the rent as ordered in the decision of the municipal court. As a result, the court granted plaintiffs motion for execution. However, the judgment regarding the surrender of possession to plaintiffs could not be executed because the subject house had been already demolished pursuant to the order of

the court in a separate civil case for ejectment against the present defendants for non-payment of rentals on the land on which the house was constructed. ISSUE: W/N the house may be a subject of a Chattel Mortgage. – YES, it may be the subject of a chattel mortgage. HELD: Defendants predicate their theory of nullity of the chattel mortgage on the ground that the subject matter of the mortgage is a house of strong materials, and, being an immovable, it can only be the subject of a real estate mortgage and not a chattel mortgage. The rule about the status of buildings as immovable property is that it is obvious that the inclusion of the building, separate and distinct from the land, in the enumeration of what may constitute real properties could only mean one thing — that a building is by itself an immovable property irrespective of whether or not said structure and the land on which it is adhered to belong to the same owner. It is undeniable that the parties to a contract may by agreement treat as personal property that which by nature would be real property. The view that parties to a deed of chattel mortgage may agree to consider a house as personal property for the purposes of said contract, "is good only insofar as the contracting parties are concerned. It is based, partly, upon the principle of estoppel.‖ In a case, a mortgaged house built on a rented land was held to be a personal property, not only because the deed of mortgage considered it as such, but also because it did not form part of the land for it is now settled that an object placed on land by one who had only a temporary right to the same, such as the lessee or usufructuary, does not become immobilized by attachment. Hence, if a house belonging to a person stands on a rented land belonging to another person, it may be mortgaged as a personal property as so stipulated in the document of mortgage. It should be noted, however that the principle is predicated on statements by the owner declaring his house to be a chattel, a conduct that may conceivably estop him from subsequently claiming otherwise. Although there is no specific statement referring to the subject house as personal property, yet by ceding, selling or transferring a property by way of chattel mortgage defendants could only have meant to convey the house as chattel, or at least, intended to treat the same as such, so that they should not now be allowed to make an inconsistent stand by claiming otherwise. Moreover, the subject house stood on a rented lot to which defendants merely had a temporary right as lessee, and although this can not in itself alone determine the status of the property, it does so when combined with other factors to sustain the interpretation that the parties, particularly the mortgagors, intended to treat the house as personalty. Finally, because it is the defendants themselves, as debtors-mortgagors, who are attacking the validity of the chattel mortgage in this case, the doctrine of estoppel therefore applies to the defendants, having treated the subject house as personalty. AMPS Serg’s Products and Goquiola v. PCI Leasing and Finance 338 SCRA 499 DOCTRINE: After agreeing to a contract stipulating that a real or immovable property be

considered as personal or movable, a party is estopped from subsequently claiming otherwise. Hence, such property is a proper subject of a writ of replevin obtained by the other contracting party. FACTS: PCI Leasing and Finance, Inc. filed a complaint with the RTC for a sum of money with an application for a writ of replevin. Upon an ex-parte application of PCI Leasing, respondent judge issued a writ of replevin directing its sheriff to seize and deliver the machineries and equipment to PCI Leasing after 5 days and upon the payment of the necessary expenses. Serg‘s filed a motion for special protective order. This motion was opposed by PCI Leasing on the ground that the properties [were] still personal and therefore still subject to seizure and a writ of replevin. In their Reply, petitioners asserted that the properties sought to be seized were immovable as defined in Article 415 of the Civil Code, the parties‘ agreement to the contrary notwithstanding. They argued that to give effect to the agreement would be prejudicial to innocent third parties. They further stated that PCI Leasing was estopped from treating these machineries as personal because the contracts in which the alleged agreement were embodied were totally sham and farcical. Citing the Agreement of the parties, the appellate court held that the subject machines were personal property, and that they had only been leased, not owned, by petitioners. It also ruled that the ―words of the contract are clear and leave no doubt upon the true intention of the contracting parties.‖ ISSUE: Whether or not the machineries purchased and imported by SERG‘S became real property by virtue of immobilization. HELD: The machineries herein are real properties but are considered personal by the parties‘ agreement. The Court will resolve whether the said machines are personal, not immovable, property which may be a proper subject of a writ of replevin. Rule 60 of the Rules of Court provides that writs of replevin are issued for the recovery of personal property only. Section 3 thereof reads: ―SEC. 3. Order. -- Upon the filing of such affidavit and approval of the bond, the court shall issue an order and the corresponding writ of replevin describing the personal property alleged to be wrongfully detained and requiring the sheriff forthwith to take such property into his custody.‖ On the other hand, Article 415 of the Civil Code enumerates immovable or real property as follows: ―ART. 415. The following are immovable property: x x x....................................x x x....................................x x x (5) Machinery, receptacles, instruments or implements intended by the owner of the tenement for an industry or works which may be carried on in a building or on

a piece of land, and which tend directly to meet the needs of the said industry or works; x x x....................................x x x....................................x x x‖ In the present case, the machines that were the subjects of the Writ of Seizure were placed by petitioners in the factory built on their own land. Indisputably, they were essential and principal elements of their chocolate-making industry. Hence, although each of them was movable or personal property on its own, all of them have become ―immobilized by destination because they are essential and principal elements in the industry.‖ In that sense, petitioners are correct in arguing that the said machines are real, not personal, property pursuant to Article 415 (5) of the Civil Code. Be that as it may, we disagree with the submission of the petitioners that the said machines are not proper subjects of the Writ of Seizure. The Court has held that contracting parties may validly stipulate that a real property be considered as personal. After agreeing to such stipulation, they are consequently estopped from claiming otherwise. Under the principle of estoppel, a party to a contract is ordinarily precluded from denying the truth of any material fact found therein. Hence, in Tumalad v. Vicencio, the Court upheld the intention of the parties to treat a house as a personal property because it had been made the subject of a chattel mortgage. It should be stressed, however, that our holding -- that the machines should be deemed personal property pursuant to the Lease Agreement – is good only insofar as the contracting parties are concerned. Hence, while the parties are bound by the Agreement, third persons acting in good faith are not affected by its stipulation characterizing the subject machinery as personal. In any event, there is no showing that any specific third party would be adversely affected. KGS Manarang and Manarang v. Ofilada and Esteban 99 SCRA 108 DOCTRINE: House is personal property for purposes of chattel mortgage only; Remains real property. The mere fact that a house was the subject of a chattel mortgage and was considered as personal property by the parties does not make said house personal property for purposes of the notice to be given for its sale at public auction. It is real property within the purview of Rule 39, section 16, of the Rules of Court as it has become a permanent fixture on the land, which is real property. FACTS: Manarang obtained a loan from Esteban, and executed a chattel mortgage over a house of mixed materials as a security. Upon default, Estaban brought an action to foreclose the property mortgaged. At Manarang‘s request, the house mortgaged was to be sold at public auction to satisfy the debt. However, before the property could be sold, Manarang offered to pay the sum. But the sheriff refused the tender unless the additional amount is also paid representing the publication of the notice in two newspapers. Manarang contended that the house in question should be considered as personal property and the publication of the notice of its sale at public auction in execution considered unnecessary. The Court of First Instance held that although real

property may sometimes be considered as personal property, the sheriff was in duty bound to cause the publication of the notice of its sale in order to make the sale valid or to prevent its being declared void or voidable. ISSUE: Can the house of Manarang be classified as personal property since it was considered as such in a chattel mortgage? -- NO HELD: The house of mixed materials levied upon on execution, although subject of a contract of chattel mortgage between the owner and a third person, is real property within the purview of Rule 39, section 16, of the Rules of Court as it has become a permanent fixture on the land, which is real property. There cannot be any question that a building of mixed materials may be the subject of a chattel mortgage, in which case it is considered as between the parties as personal property. The matter depends on the circumstances and the intention of the parties. The general principle of law is that a building permanently fixed to the freehold becomes a part of it, that prima facie a house is real estate, belonging to the owner of the land on which it stands, even though it was erected against the will of the landowner, or without his consent. The general rule is otherwise, however, where the improvement is made with the consent of the landowner, and pursuant to an understanding either expressed or implied that it shall remain personal property. Nor does the general rule apply to a building, which is wrongfully removed from the land and placed on the land of the person removing it. Among the principal criteria for determining whether property remains personally or becomes realty are annexation to the soil, either actual or construction, and the intention of the parties. Personal property may retain its character as such where it is so agreed by the parties interested even though annexed to the realty, or where it is affixed in the soil to be used for a particular purpose for a short period and then removed as soon as it has served its purpose. These considerations notwithstanding, we hold that the rules on execution do not allow, and we should not interpret them in such a way as to allow, the special consideration that parties to a contract may have desired to impart to real estate, for example, as personal property, when they are not ordinarily so. Sales on execution affect the public and third persons. The regulation governing sales on execution are for public officials to follow. The form of proceedings prescribed for each kind of property is suited to its character, not to the character which the parties have given to it or desire to give it. When the rules speak of personal property, property which is ordinarily so considered is meant; and when real property is spoken of, it means property which is generally known as real property. The regulations were never intended to suit the consideration that parties, may have privately given to the property levied upon. Enforcement of regulations would be difficult were the convenience or agreement of private parties to determine or govern the nature of the proceedings. We, therefore, hold that the mere fact that a house was the subject of a chattel mortgage and was considered as personal property by the parties does not make said house personal property for purposes of the notice to be given for its sale at public auction. This ruling is demanded by the need for a definite, orderly and well-defined regulation for official and public guidance and which would prevent confusion and misunderstanding

JPOT Navarro v. Pineda 9 SCRA 631 DOCTRINE: Estoppel, in that "the parties have so expressly agreed" in the mortgage to consider the house as chattel "for its smallness and mixed materials of sawali and wood". FACTS: December 14, 1959, Rufino G. Pineda and his mother Juana Gonzales (married to Gregorio Pineda), borrowed from plaintiff Conrado P. Navarro, the sum of P2,500.00, payable 6 months after said date or on June 14, 1959. To secure the indebtedness, Rufino executed a document captioned "DEED OF REAL ESTATE and CHATTEL MORTGAGES", whereby Juana Gonzales, by way of Real Estate Mortgage hypothecated a parcel of land, belonging to her, registered with the Register of Deeds of Tarlac, under Transfer Certificate of Title No. 25776, and Rufino G. Pineda, by way of Chattel Mortgage, mortgaged his two-story residential house, having a floor area of 912 square meters, erected on a lot belonging to Atty. Vicente Castro, located at Bo. San Roque, Tarlac, Tarlac; and one motor truck, registered in his name, under Motor Vehicle Registration Certificate No. A-171806. Both mortgages were contained in one instrument, which was registered in both the Office of the Register of Deeds and the Motor Vehicles Office of Tarlac. After failing to settle amount due, respondent was then granted an extension on June 30,1960 and consequently July 30th of the same year for still being unable to comply. Rufino Pineda then issued a document entitled "Promise," stating that defendant would no longer ask for further extension and there would be no need for any formal demand, and plaintiff could proceed to take whatever action he might desire to enforce his rights, under the said mortgage contract. On August 10, 1960, plaintiff filed a complaint for foreclosure of the mortgage and for damages, which consisted of liquidated damages in the sum of P500.00 and 12% per annum interest on the principal, effective on the date of maturity, until fully paid. Defendants admit that the loan is overdue but deny that portion of paragraph 4 of the First Cause of Action which states that the defendants unreasonably failed and refuse to pay their obligation to the plaintiff the truth being the defendants are hard up these days and pleaded to the plaintiff to grant them more time within which to pay their obligation and the plaintiff refused; WHEREFORE, in view of the foregoing it is most respectfully prayed that this Honorable Court render judgment granting the defendants until January 31, 1961, within which to pay their obligation to the plaintiff. November 11, 1960, however, the parties submitted a Stipulation of Facts, wherein the defendants admitted the indebtedness, the authenticity and due execution of the Real Estate and Chattel Mortgages; that the indebtedness has been due and unpaid since June 14, 1960; that a liability of 12% per annum as interest was agreed, upon failure to pay the principal when due and P500.00 as liquidated damages; that the instrument had been registered in the Registry of Property and Motor Vehicles Office, both of the province of Tarlac. ISSUE: W/N the residential house, subject of the mortgage therein, can be considered a Chattel and the propriety of the attorney's fees.

HELD: The court ruled "a property may have a character different from that imputed to it in said articles. It is undeniable that the parties to a contract may by agreement, treat as personal property that which by nature would be real property" (Standard Oil Co. of N.Y. v. Jaranillo, 44 Phil. 632633)."There can not be any question that a building of mixed materials may be the subject of a chattel mortgage, in which case, it is considered as between the parties as personal property. ... The matter depends on the circumstances and the intention of the parties". "Personal property may retain its character as such where it is so agreed by the parties interested even though annexed to the realty ...". (42 Am. Jur. 209-210, cited in Manarang, et al. v. Ofilada, et al., G.R. No. L-8133, May 18, 1956; 52 O.G. No. 8, p. 3954.) Moreover, the court continues and makes plain that it "is good only insofar as the contracting parties are concerned. It is based partly, upon the principles of estoppel ..." (Evangelista v. Alto Surety, No. L-11139, Apr. 23, 1958). WHEREBY, previous judgment is hereby AFFIRMED costs against appelant. MLAV Manila Electric Co., v. Central Board of Assessment Appeals 114 SCRA 273 DOCTRINE: Oil storage tanks were held to be taxable realty. For purposes of taxation, the term "real property" may include things which should generally be regarded as personal property. FACTS: The case is about the imposition of the realty tax on two oil storage tanks installed in 1969 by Manila Electric Company in Batangas which it leased in 1968 from Caltex (Phil.), Inc. The tanks are within the Caltex refinery compound, and are used for storing fuel oil for Meralco's power plants. According to Meralco, the storage tanks are made of steel plates welded and assembled on the spot. Their bottoms rest on a foundation consisting of compacted earth as the outermost layer, a sand pad as the intermediate layer and a two-inch thick bituminous asphalt stratum as the top layer. The bottom of each tank is in contact with the asphalt layer. Hence, it is not attached to its foundation. On the other hand, according to the hearing commissioners of the Central Board of Assessment Appeals (CBAA) states that while the tanks rest or sit on their foundation, the foundation itself and the walls, dikes and steps, which are integral parts of the tanks, are affixed to the land while the pipelines are attached to the tanks. The CBAA ruled that the tanks together with the foundation, walls, dikes, steps, pipelines and other appurtenances constitute taxable improvements. Meralco filed a motion for reconsideration which the Board denied. They elevated the case to the SC. ISSUE: WON the storage tanks are considered ―improvements‖ on real property such that it is subject to real property tax. -- YES HELD: Meralco contends that the said oil storage tanks do not fall within any of the kinds of real property enumerated in article 415 of the Civil Code and, therefore, they cannot be categorized

as realty by nature, by incorporation, by destination nor by analogy. Stress is laid on the fact that the tanks are not attached to the land and that they were placed on leased land, not on the land owned by Meralco. The issue raised by Meralco has to be resolved in the light of the provisions of the Assessment Law, Commonwealth Act No. 470, and the Real Property Tax Code, Presidential Decree No. 464 which took effect on June 1, 1974. Section 2 of the Assessment Law provides that the realty tax is due "on real property, including land, buildings, machinery, and other improvements" not specifically exempted in section 3 thereof. This provision is reproduced with some modification in the Real Property Tax Code which provides: Sec. 38. Incidence of Real Property Tax. — They shall be levied, assessed and collected in all provinces, cities and municipalities an annual ad valorem tax on real property, such as land, buildings, machinery and other improvements affixed or attached to real property not hereinafter specifically exempted. The Code contains the following definition in its section 3: k) Improvements — is a valuable addition made to property or an amelioration in its condition, amounting to more than mere repairs or replacement of waste, costing labor or capital and intended to enhance its value, beauty or utility or to adapt it for new or further purposes. The SC holds that while the two storage tanks are not embedded in the land, they may, nevertheless, be considered as improvements on the land, enhancing its utility and rendering it useful to the oil industry. It is undeniable that the two tanks have been installed with some degree of permanence as receptacles for the considerable quantities of oil needed by Meralco for its operations. Oil storage tanks were held to be taxable realty in Standard Oil Co. of New Jersey vs. Atlantic City, 15 Atl. 2nd 271. For purposes of taxation, the term "real property" may include things which should generally be regarded as personal property. It is a familiar phenomenon to see things classed as real property for purposes of taxation which on general principle might be considered personal property (Standard Oil Co. of New York vs. Jaramillo, 44 Phil. 630, 633). DJTV Caltex Philippines v. Central Board of Assessment Appeals 114 SCRA 296 DOCTRINE: Gasoline station equipment and machineries are permanent fixtures for purposes of realty taxation. FACTS: Machines and equipment are loaned by Caltex to gas station operators under an appropriate lease agreement or receipt. It is stipulated in the lease contract that the operators, upon demand, shall return to Caltex the machines and equipment in good condition as when received, ordinary wear and tear excepted. The city assessor of Pasay City characterized the said items of gas station equipment and

machinery as taxable realty. The city board of tax appeals ruled that they are personalty. The assessor appealed to the Central Board of Assessment Appeals. The Board said machines and equipment are real property within the meaning of sections 3(k) & (m) and 38 of the Real Property Tax Code, Presidential Decree No. 464, which took effect on June 1, 1974, and that the definitions of real property and personal property in articles 415 and 416 of the Civil Code are not applicable to this case. Caltex filed this certiorari petition wherein it prayed for the setting aside of the Board's decision and for a declaration that t he said machines and equipment are personal property not subject to realty tax. ISSUE: Whether or not Gasoline station equipment and machineries are permanent fixtures for purposes of realty taxation. -- YES HELD: The Supreme Court held that gasoline station equipment and machineries are permanent fixtures for purposes of realty taxation. Thus, they are subject to the real property tax. The said equipment and machinery, as appurtenances to the gas station building or shed owned by Caltex (as to which it is subject to realty tax) and which fixtures are necessary to the operation of the gas station, for without them the gas station would be useless, and which have been attached or affixed permanently to the gas station site or embedded therein, are taxable improvements and machinery within the meaning of the Assessment Law and the Real Property Tax Code. The Central Board of Assessment Appeals did not commit a grave abuse of discretion in upholding the city assessor's is imposition of the realty tax on Caltex's gas station and equipment. WHEREFORE, the questioned decision and resolution of the Central Board of Assessment Appeals are affirmed. The petition for certiorari is dismissed for lack of merit. JGY Phil. Refining Co., Inc. v. Jarque G.R. No. 41506 DOCTRINE: Vessels are considered personal property under the civil law. Similarly under the common law, vessels are personal property although occasionally referred to as a peculiar kind of personal property. Since the term "personal property" includes vessels, they are subject to mortgage agreeably to the provisions of the Chattel Mortgage Law. FACTS: Philippine Refining Co., Inc., and Jarque executed three chattel mortgages on the motor vessels Pandan and Zaragoza. Neither of the first two mortgages had appended an affidavit of good faith. The third mortgage contained such an affidavit, but this mortgage was not registered in the customs house within the period of thirty days prior to the commencement of insolvency proceedings against Jarque. A fourth mortgage was executed and was entered in the chattel mortgage registry of the register of deeds within the thirty-day period before the institution of insolvency proceedings. CFI declared Jarque to be an insolvent debtor. As a result, all his properties were assigned to Corominas. CFI Judge declined to order the foreclosure of the

mortgages, but on the contrary sustained the special defenses of fatal defectiveness of the mortgages. ISSUES: 1. WON the vessels are considered as personal property and can be the subject of Chattel Mortgage. Yes 2. WON the Chattel Mortgages are valid. No. HELD: Vessels are considered personal property under the civil law. Similarly under the common law, vessels are personal property although occasionally referred to as a peculiar kind of personal property. Since the term "personal property" includes vessels, they are subject to mortgage agreeably to the provisions of the Chattel Mortgage Law. Indeed, it has heretofore been accepted without discussion that a mortgage on a vessel is in nature a chattel mortgage. The only difference between a chattel mortgage of a vessel and a chattel mortgage of other personalty is that it is not now necessary for a chattel mortgage of a vessel to be noted n the registry of the register of deeds, but it is essential that a record of documents affecting the title to a vessel be entered in the record of the Collector of Customs at the port of entry. Otherwise a mortgage on a vessel is generally like other chattel mortgages as to its requisites and validity. The Chattel Mortgage Law in its section 5, in describing what shall be deemed sufficient to constitute a good chattel mortgage, includes the requirement of an affidavit of good faith appended to the mortgage and recorded therewith. The absence of the affidavit vitiates a mortgage as against creditors and subsequent encumbrancers. As a consequence a chattel mortgage of a vessel wherein the affidavit of good faith required by the Chattel Mortgage Law is lacking, is unenforceable against third persons. JRPA Rubiso v. Rivera 37 Phil 72 DOCTRINE: The requisite of registration in the registry, of the purchase of the vessel, is necessary and indispensable in order that the purchaser‘s rights may be maintained against a third person. FACTS: Rubiso filed a complaint against Rivera for the recovery of a pilot boat. He alleged that he is the rightful owner of a pilot boat, which was stranded and recovered by Rivera. The latter refused to return the said boat as he alleged too that he was the owner thereof. It was known that the original owners of the boat had secretly sold the pilot boat to Rivera on an earlier date than the sale in a public auction to Rubiso. Nonetheless, material is the fact that the entry into the customs registry of the sale of the boat was later than the recording of the sale to Rubiso. ISSUE: WON it is Rubiso or Rivera who has a better right to the boat? -- Rubiso has a better right HELD: The requisite of registration in the registry, of the purchase of the vessel, is necessary and indispensable in order that the purchaser‘s rights may be maintained against a third

person. Such registration is required both by the Code of Commerce and Act 1900. It is undeniable, ergo, that Rivera doesn‘t have a better right than Rubiso over the pilot boat. Ships and vessels, whether moved by steam or by sail, partake, to a certain extent of the nature and conditions of real property, on account of their value and importance in world commerce; and for this, the provisions of the Code of Commerce are nearly identical with Article 1473 of the Civil Code ABB U.S. v. Carlos 21 Phil. 553 DOCTRINE: The true test of what constitutes the proper subject of [theft] is not whether the subject is corporeal or incorporeal, but whether it is capable of appropriation by another other than the owner. FACTS: Ignacio Carlos has been a consumer of electricity furnished by MERALCO for a building containing the residence of the accused and 3 other residences. Representatives of the company believing that more light is consumed than what is shown in the meter installed an additional meter on the pole outside Carlos‘ house to compare the actual consumption and found out that a jumper was used to manipulate the readings of the first meter. Further, a jumper was found in a drawer of a small cabinet in the room of the defendant‘s house where the meter was installed. In the absence of any explanation for Carlos‘ possession of said device, the presumption raised was that Carlos was the owner of the device whose only use was to deflect the current from the meter. Thus, he was charged with the crime of theft amounting to 2,273KW of electric power worth 909.20 pesos. Carlos claimed that what he did failed to constitute an offense because the crime of theft applies only to tangibles, chattels and objects that can be taken into possession. Deliberation quickly followed at the court which subsequently sentenced him to over a year in jail. Carlos contested saying that electrical energy can‘t be stolen because of its nature of being incorporeal. He filed an appeal on such grounds which the CFI affirmed. ISSUE: Whether or not theft can be committed against an intangible such as electricity. -- YES HELD: Theft of incorporeal objects is possible. The right of ownership of electrical current was secured by Art 517 and 518 of the Penal Code which applies to gas. Analogically, electricity can be considered as ‗gas‘ which can be stolen. However, the true test of what constitutes the proper subject of larceny is not whether the subject is corporeal or incorporeal, but whether it is capable of appropriation by another other than the owner. It is a valuable article of merchandise, a force of nature brought under the control of science (under Art. 416 of the New Civil Code). Carlos secretly and with intent to deprive the company of its rightful property, used jumper cables to appropriate the same for his own use. Such acts constitute theft. FZC

Piansay v. David 12 SCRA 227 DOCTRINE: regardless of the validity of a contract constituting a chattel mortgage on a house, the same cannot and does not bind third persons, who are not parties to the contract of their privies. FACTS: David obtained a loan from Uy Kim secured with a chattel mortgage on a house in Tondo. David failed to pay, thus was Kim foreclosed. Kim then bought the house at the resulting public auction. Thereafter, Kim sold the same to Marcos Mangubat, who later filed a complaint against David for the collection of a portrion of the loan. The complaint was later amended to implead Uy Kim and Piansay praying that the auction sale and deed of absolute sale executed by Uy Kim in favor of Piansay be annulled. CFI Manila ordered David to pay and annulled the chattel mortgage. CA affirmed. David was ordered to pay and the house was levied upon. To prevent the sale at the public auction, Piansay and Uy Kim filed a petition for certiorari and mandamus with preliminary injunction before the CA; it was denied. Subsequently, Piansay and Uy Kim instituted an action against David and Mangubat praying that judgment be rendered declaring Piansay as the true owner and restrain the levy and sale to public auction. David demanded from Piansay the payment of the rentals for the use and occupation of the house; the latter claims it is his property. Mangubat, on one hand, moved to dismiss the complaint on the ground of res adjudicata and lackof personality to sue; it was granted. CA affirmed explaining that Uy Kim had no right to foreclose the chattel mortgage because it was in reality a mere contract of an unsecured loan. Piansay assailed Mangubat's right to levy execution upon the house alleging that the same belongs to him, he having bought it from Uy Kim who acquired it at the auction sale. ISSUE: WON the chattel mortgage and sale are valid - NO HELD: Upon the theory that the chattel mortgage and sale in favor of Uy Kim had been annulled in the original decision, as affirmed by the CA, the fact is that said order became final and executory upon the denial of the petition for certiorari and mandamus. Hence, Uy Kim and Piansay are now barred from asserting that the chattel mortgage and sale are valid. At any rate, regardless of the validity of a contract constituting a chattel mortgage on a house, as between the parties to said contract, the same cannot and does not bind third persons, who are not parties to the contract of their privies. As a consequence, the sale of the house in question in the proceedings for the extrajudicialforeclosure of said chattel mortgage, is null and void insofar as defendant Mangubat is concerned, anddid not confer upon Uy Kim, as buyer in said sale, any dominical right in and to said house, so that shecould not have transmitted to her assignee Piansay any such right as against Mangubat. In short, they donot have a cause of action against Mangubat and David LNAC U.S. v. Tambunting 41 Phil 364 DOCTRINE: There is nothing in the nature of gas used for illuminating purposes which renders it incapable of being feloniously taken and carried away. It is a valuable article of merchandise,

bought and sold like other personal property, susceptible of being severed from a mass or larger quantity and of being transported from place to place. Likewise water which is confined in pipes and electricity which is conveyed by wires are subjects of larceny. FACTS: This appeal was instituted for the purpose of reversing a judgment of the CFI, finding the accused, Manuel Tambunting, guilty of stealing a quantity of gas belonging to the Manila Gas Corporation, and sentencing him to undergo imprisonment with the accessories prescribed by law; to indemnify the said corporation, with subsidiary imprisonment in case of insolvency; and to pay the costs. The evidence submitted in behalf of the prosecution shows that in January of the year 1918, the accused and his wife became occupants of the upper floor of the house situated at No. 443, Calle Evangelista, Manila. In this house the Manila Gas Corporation had previously installed apparatus for the delivery of gas on both the upper and lower floors, consisting of the necessary piping and a gas meter, which last mentioned apparatus was installed below. When the occupants at whose request this installation had been made vacated the premises, the gas company disconnected the gas pipe and removed the meter, thus cutting off the supply of gas from said premises. Upon June 2, 1919, one of the inspectors of the gas company visited the house in question and found that gas was being used, without the knowledge and consent of the gas company, for cooking in the quarters occupied by the defendant and his wife: to effect which a short piece of iron pipe had been inserted in the gap where the gas meter had formerly been placed, and piece of rubber tubing had been used to connect the gas pipe of rubber tubing had been used to connect the gas pipe in kitchen with the gas stove, or plate, used for cooking. At the time this discovery was made, accused Tambunting was not at home. However, he presently arrived and admitted to the agent to the gas company that he had made the connection with the rubber tubing between the gas pipe and the stove, though he denied making the connection below. He also admitted that he knew he was using gas without the knowledge of the company and that he had been so using it for probably two or three months. The clandestine use of gas by the accused in the manner stated is thus established in our opinion beyond a doubt; and inasmuch as the animo lucrandi is obvious. ISSUE: Whether gas can be the subject to larceny. -- YES HELD:. The right of the ownership of electric current is secured by article 517 and 518 of the Penal Code; the application of these articles in cases of subtraction of gas, a fluid used for lighting, and in some respects resembling electricity, is confirmed by the rule laid down in the decisions of the SC of Spain, construing and enforcing the provisions of articles 530 and 531 of the Penal Code of that country, articles identical with articles 517 and 518 of the code in force in these Islands. These expressions were used in a case which involved the subtraction and appropriation of electrical energy and the court held, in accordance with the analogy of the case involving the theft of gas, that electrical energy could also be the subject of theft. In this connection it will suffice to quote the following from the topic "Larceny," at page 34, Vol. 17, of Ruling Case Law: There is nothing in the nature of gas used for illuminating purposes

which renders it incapable of being feloniously taken and carried away. It is a valuable article of merchandise, bought and sold like other personal property, susceptible of being severed from a mass or larger quantity and of being transported from place to place. Likewise water which is confined in pipes and electricity which is conveyed by wires are subjects of larceny. TKDC Involuntary Insolvency of Strochecker v. Ramirez 44 Phil. 933 DOCTRINE: All personal properties may be mortgaged. Interest in business is personal property capable of appropriation and not included in the enumeration of real properties under Article 335 of the Civil Code. Thus, interest in business may be subject of mortgage. FACTS: Three mortgages seek preference in the lower court: one in favor of Fidelity and Surety Co., another in favor of Ramirez, and the last one in favor of Ayala. Ayala‘s claim was rejected by trial court from which she didn‘t appeal. As to the time of the mortgages, the one in favor of Fidelity and Surety Co. is preferred because it was executed and registered in the registry of property prior to that of Ramirez‘s. However, Ramirez claimed that the mortgage in favor of Fidelity and Surety Co. is invalid because the property, the half interest in the drug business, is incapable of being mortgaged. Trial court ruled that the mortgage in favor of Fidelity and Surety Co. is entitled to preference. ISSUE: Whether or not one-half interest in the business is capable of being mortgaged. -- YES HELD: All personal properties may be mortgaged. Interest in business is personal property capable of appropriation and not included in the enumeration of real properties under Article 335 of the Civil Code. Thus, interest in business may be subject of mortgage. In this case, the mortgaged property of one-half interest in the drug business in favor of Fidelity and Surety Co. is a valid subject of mortgage. AMD Laurel v. Abrogar G.R. No. 155076 DOCTRINE: Telecommunication services and the business of providing said services are not personal properties and cannot be subject to Article 308 of the Revised Penal Code Services in business, although properties, are not proper subjects of theft under the Revised Penal Code because the same cannot be "taken" or "occupied". FACTS: PLDT claims that Luis Marcos P. Laurel, board member and corporate secretary of Baynet Co., Ltd., stole and used the international long distance calls belonging to PLDT by conducting International Simple Resale (ISR) – a method of routing and completing international long distance calls using lines, cables, antennae, and/or air wave frequency which connect directly to the local or domestic exchange facilities of the country where the call is destined. PLDT alleged

that such business was effectively stolen while using their facilities leading to great damage and prejudice amounting to P20,370,651.92. Laurel however alleged that the allegations do not constitue the felony of theft under Article 308 of the RPC or any special law. He claimed that, telephone calls with the use of PLDT telephone lines, whether domestic or international, belong to the persons making the call, not to PLDT. He argued that the caller merely uses the facilities of PLDT, and what the latter owns are the telecommunication infrastructures or facilities through which the call is made. He also asserted that PLDT is compensated for the caller‘s use of its facilities by way of rental; for an outgoing overseas call, PLDT charges the caller per minute, based on the duration of the call. Thus, no personal property was stolen from PLDT. The prosecution asserted that the use of PLDT‘s intangible telephone services/facilities allows electronic voice signals to pass through the same, and ultimately to the called party‘s number. It averred that such service/facility is akin to electricity which, although an intangible property, may, nevertheless, be appropriated and be the subject of theft. The prosecution further alleged that "international business calls and revenues constitute personal property envisaged in Article 308 of the Revised Penal Code." Moreover, the intangible telephone services/facilities belong to PLDT and not to the movant and the other accused, because they have no telephone services and facilities of their own duly authorized by the NTC; thus, the taking by the movant and his coaccused of PLDT services was with intent to gain and without the latter‘s consent. ISSUE: W/N telephone calls placed by Bay Super Orient Card holders through the telecommunication services provided by PLDT are considered as personal property, and thus, proper subjects of theft under Article 308 of the Revised Penal Code. -- NO HELD: The court finds that the international telephone calls placed by Bay Super Orient Card holders, the telecommunication services provided by PLDT and its business of providing said services are not personal properties under Article 308 of the Revised Penal Code. The rule is that, penal laws are to be construed strictly. Penal statutes may not be enlarged by implication or intent beyond the fair meaning of the language used; and may not be held to include offenses other than those which are clearly described. One is apt to conclude that "personal property" standing alone, covers both tangible and intangible properties and are subject of theft under the Revised Penal Code. But the words "Personal property" under the Revised Penal Code must be considered in tandem with the word "take" in the law. The statutory definition of "taking" and movable property indicates that, clearly, not all personal properties may be the proper subjects of theft. The general rule is that, only movable properties which have physical or material existence and susceptible of occupation by another are proper objects of theft. According to Cuello Callon, in the context of the Penal Code, only those movable properties which can be taken and carried from the place they are found are proper subjects of theft. Intangible properties such as rights and ideas are not subject of theft because the same cannot be "taken" from the place it is found and is occupied or appropriated. Gas and electrical energy should not be equated with business or services provided by business entrepreneurs to the public. Business does not have an exact definition. Business is referred as that which occupies the time, attention and labor of men for the purpose of livelihood

or profit. It embraces everything that which a person can be employed. Business may also mean employment, occupation or profession. Business is also defined as a commercial activity for gain benefit or advantage. Business, like services in business, although are properties, are not proper subjects of theft under the Revised Penal Code because the same cannot be "taken" or "occupied." PLDT does not acquire possession, much less, ownership of the voices of the telephone callers or of the electronic voice signals or current emanating from said calls. The human voice and the electronic voice signals or current caused thereby are intangible and not susceptible of possession, occupation or appropriation by PLDT or even the petitioner, for that matter. PLDT merely transmits the electronic voice signals through its facilities and equipment. Baynet Card Ltd., through its operator, merely intercepts, reroutes the calls and passes them to its toll center.

PROPERTY IN RELATIONSHIP TO THE PERSON WHOM IT BELONGS (ART. 419-426) CRF Villanueva v. Castañeda 154 SCRA 142 DOCTRINE: Article 344 of the Civil Code: "Property for public use in provinces and in towns comprises the provincial and town roads, the squares, streets, fountains, and public waters, the promenades, and public works of general service supported by said towns or provinces.‖ Such is outside the commerce of man and cannot be the object of a valid contract. (Article 1271) FACTS: The subject of the herein petition is a ―talipapa‖ that the petitioners claim they have a right to remain in and conduct business in this area by virtue of a previous authorization granted to them by the municipal government. The respondents deny this and justify the demolition of their stalls as illegal constructions on public property. This dispute goes back to when the municipal council of San Fernando adopted Resolution No. 218 authorizing some 24 members of the Fernandino United Merchants and Traders Association to construct permanent stags and sell in the above-mentioned place. 2 The action was protested where the Court of First Instance of Pampanga, Branch 2, issued a writ of preliminary injunction that prevented the defendants from constructing the said stalls until final resolution of the controversy. 3 On January 18, 1964, while this case was pending, the municipal council of San Fernando adopted Resolution G.R. No. 29, which declared the subject area as "the parking place and as the public plaza of the municipality. Four years later, the judge decided that the land occupied by the petitioners, being public in nature, was beyond the commerce of man and therefore could not be the subject of private occupancy. 5 The writ of preliminary injunction was made permanent. 6 The decision was apparently not enforced, for the petitioners were not evicted from the place; in fact, according to then they and the 128 other persons were in 1971 assigned specific areas or space allotments therein for which they paid daily fees to the municipal government. Then, on January 12, 1982, the Association of Concerned Citizens and Consumers of San Fernando filed a petition for the immediate implementation of Resolution No. 29, to restore the subject property "to its original and customary use as a public plaza thereby the respondent was tasked to demolish the stalls in the subject place which was favored by the trial court. The basic contention of the petitioners is that the disputed area is under lease to them by virtue of contracts they had entered into with the municipal government. ISSUE: WON the lease agreement whereby the municipality of Cavite leased to the petitioner valid given the fact the said area is dedicated for public use? -- NO HELD: Applying this well-settled doctrine, we rule that the petitioners had no right in the first place to occupy the disputed premises and cannot insist in remaining there now on the strength of their alleged lease contracts. The lease agreement is null and void. According to article 344 of the Civil Code: "Property for public use in provinces and in towns

comprises the provincial and town roads, the squares, streets, fountains, and public waters, the promenades, and public works of general service supported by said towns or provinces. The said Plaza Soledad being a promenade for public use, the municipal council of Cavite could not in 1907 withdraw or exclude from public use a portion thereof in order to lease it for the sole benefit of the defendant Hilaria Rojas. Furthermore, the Civil Code, article 1271, prescribes that everything which is not outside the commerce of man may be the object of a contract, and plazas and streets are outside of this commerce. MPF Maneclang v. IAC 144 SCRA 553 DOCTRINE: Finding that subject body of water is a creek belonging to the public domain,not susceptible to private appropriation, a factual determination binding on the Supreme Court. FACTS: ● Maneclang filed a complaint for quieting of title over a certain fishpond located within the four parcels of land belonging to them. ● The trial court dismissed the complaint upon finding that the body of water traversing the titled properties of petitioners is a creek constituting a tributary of a river; therefore public in nature and not subject to private appropriation. ISSUE: Whether or not a creek can be registered under the Torrens System -- NO HELD: A creek is a recess/arm extending from a river and participating in the ebb and flow of the sea. It is a property belonging to the public domain, It is a property belonging to the public domain. it is not susceptible to appropriation and acquisitive prescription. As a public water, it cannot be registered under the Torrens System in the name of any individual. Its nature as property of the public domain cannot be modified by the construction of irrigation dikes by the National Irrigation Authority, or by its conversion into a fishpond. Hence, a compromise agreement adjudicating the ownership of such property in favor of an individual is null and void. The compromise agreement has no legal effect since it is contrary to law and public policy. AMDG Chavez v. Public Estates Authority 384 SCRA 152 DOCTRINE: Until now, the only way the government can sell to private parties government reclaimed and marshy disposable lands of the public domain is for the legislature to pass a law authorizing such sale. However, there exists a constitutional ban wherein private corporations are prohibited from acquiring alienable lands of the public domain. These corporations may only lease the lands from a period granted by the law. FACTS: The government, through the Commissioner of Public Highways, signed a contract with CDCP

to reclaim certain foreshore and offshore areas of Manila Bay under the MCCRRP. Later on President Marcos signed PD No. 1084 and 1085 creating PEA and transferring to PEA the reclaimed lands in the foreshore and offshore of the Manila Bay. In addition, a Memorandum of Agreement was executed between PEA and CDCP wherein the latter acceded and transferred its rights and interest in favor of the former as regards CDCP‘s reclaimed lands under MCCRRP. During Aquino‘s administration, special patents as well as 3 TCTs (the lands were known as Freedom Islands) were issued in favor of PEA. PEA and AMARI, a private corporation, through negotiation but without conducting any public bidding entered into a Joint Venture Agreement (JVA for brevity) for the development of the Freedom Islands. A year later, Senate President Maceda described such JVA during his privileged speech as the ―grandmother of all scams‖. Consequently, a joint investigation was conducted and the report concluded that the JVA is illegal because what PEA seeks to do is to transfer ownership of the reclaimed lands which are public lands hence inalienable to AMARI. However, the Legal Task formed by Pres. Ramos upheld the legality of the JVA. Phillipine Daily Inquirer and Today published reports that Pres. Ramos ordered that renegotiations regarding the JVA be again made. Such JVA (now called Amended JVA) was later on approved by Pres. Estrada. Petitioner Chavez prays that the Amended JVA be declared null and void for it violating the Constitutional and statutory provisions. ISSUE: Whether or not AMARI, a private corporation may acquire the reclaimed lands? NO HELD: In this case, the SC traced back the laws governing reclaimed lands as regards its alienability. The previous Constitutions including the 1987 Constitution has adopted the Regalian Doctrine wherein it states that all public lands and waters are owned by the State. The court discussed and emphasized also CA No. 141 which states that the only way the government can sell to private parties‘ government reclaimed and marshy disposable lands of the public domain is for the legislature to pass a law authorizing such sale. In addition, the Constitution has established that private corporations (such as AMARI) cannot acquire the reclaimed lands however; these corporations are allowed to lease them. This rule is absolute. Applying these provisions to the case, the reclaimed lands are classified as public property and in order for PEA to sell these lands; there must be a legislative act granting such right to sell. In addition, even if there exist an express provision in favor of PEA, such would still subject of the constitutional ban as regards private corporation acquiring reclaimed alienable lands. As mentioned and established already, these reclaimed lands are considered inalienable public property. PD No. 1085 granting PEA the power to oversee the Freedom Islands did not in any way convert the lands into alienable or disposable lands. The issuance of special patents by Pres. Aquino as well as the TCTs also did not convert it into private lands. It must be noted that the registration of public lands under Torrens system cannot convert it into private property. GCG Republic v. Court of Appeals 281 SCRA 639 DOCTRINE: When the sea moved towards the estate and the tide invaded it, the invaded property became foreshore land and passed the realm of the public domain and accordingly

cannot be a subject of a free patent. FACTS: Respondent Josefina Morato filed a Free Patent Application on a parcel of land. The patent was approved at the Register of Deeds of Quezon. Both the free patent and the title specifically mandate that the land shall not be alienated nor encumbered within five years from the date of the issuance of the patent (Sections 118 and 124 of CA No. 141, as amended). Subsequently, the District Land Officer in Lucena City, acting upon reports that respondent Morato had encumbered the land in violation of the condition of the patent, conducted an investigation. Thereafter, it was established that the subject land is a portion of the Calauag Bay, five (5) to six (6) feet deep under water during high tide and two (2) feet deep at low tide, and not suitable to vegetation. Moreover, a portion of the land was mortgaged by respondent Morato to respondents Nenita Co and Antonio Quilatan. The spouses Quilatan constructed a house on the land. Another portion of the land was leased to Perfecto Advincula, where a warehouse was constructed. Petitioner filed an amended complaint against respondents Morato, spouses Nenita Co and Antonio Quilatan, and the Register of Deeds of Quezon for the cancellation of title and reversion of a parcel of land to the public domain, subject of a free patent in favor of respondent Morato, on the grounds that the land is a foreshore land and was mortgaged and leased within the fiveyear prohibitory period. The lower court ruled that there was no violation of the 5-year period ban against alienating or encumbering the land, because the land was merely leased and not alienated. It also found that the mortgage to Nenita Co and Antonio Quilatan covered only the improvement and not the land itself. On appeal, the Court of Appeals affirmed the decision of the trial court. Thereafter, the Republic of the Philippines filed the present petition. ISSUE: Whether the questioned land is a foreshore land and thus must be reverted to the public domain. -- YES HELD: The Supreme Court found that the subject land was foreshore land, it nevertheless sustained the award thereof to Respondent Morato. It defined a foreshore land as ―that parcel of land which is between high and low water and left dry by the flux and reflux of the tides‖; it is that ―strip of land that lies between the high and low water marks and that is alternatively wet and dry according to the flow of the tide‖. From the actual findings of the lower court, it was found out that years before the issuance of the free patent to private respondent, the questioned land was subjected to several natural calamities like earthquakes and typhoons that caused severe erosion of the land. Then private respondent introduced improvements and developments to the land. At the time then of the issuance of free patent of land to Morato, it was not covered by water but due to the gradual sinking of the land caused by natural calamities, the sea advances had permanently invaded a portion of subject land. During high tide, at least half of the land is 6 feet deep under water and three feet deep during low tide. The Calauag Bay has extended up to a portion of the land.

Thus, uncontestedly, the land has become a foreshore land and is now a part of the public domain pursuant to Article 420 of the New Civil Code – being part of the ―shores‖ defined therein. Accordingly, it cannot be disposed of by the government and appropriated by a private individual, i.e. be a subject of a free patent. VCL IV MIAA v. Court of Appeals G.R. No. 155650 DOCTRINE: The term ―ports‖ includes seaports and airports. The MIAA Airport Lands and Buildings constitute a ―port‖ constructed by the State. Under Article 420 of the Civil Code, the MIAA Airport Lands and Buildings are properties of public dominion and thus owned by the State or the Republic of the Philippines. FACTS: Manila International Airport Authority (MIAA) operates the Ninoy Aquino International Airport Complex in Parañaque City. As operator of the international airport, MIAA administers the land, improvements and equipment within the NAIA Complex. The MIAA Charter transferred to MIAA approximately 600 hectares of land including the runways and buildings (―Airport Lands and Buildings‖) then under the Bureau of Air Transportation. The MIAA Charter further provides that no portion of the land transferred to MIAA shall be disposed of through sale or any other mode unless specifically approved by the President of the Philippines. OGCC (Office of the Government Corporate Counsel) issued Opinion No. 061, in which it said that the Local Government Code of 1991 withdrew the exemption for real estate tax granted to MIAA under Section 21 of the MIAA charter. Therefore, MIAA was held to be delinquent in paying its taxes. The City of Parañaque Levied upon the properties of MIAA, and posted invitations for public biddings of MIAA‘s properties. MIAA filed with CA an action for prohibition / injunction. The City of Parañaque averred that Section 193 of the Local Government code expressly withdrew tax exemptions from government owned and controlled corporations (GOCCs). CA dismissed the petition for filing beyond the 60 day reglementary period ISSUE: Whether properties of the MIAA are subject to real estate taxes. -- NO HELD: In the first place, MIAA is not a GOCC, it is an instrumentality of the government. MIAA is a government instrumentality vested with corporate powers to perform efficiently its governmental functions. MIAA is like any other government instrumentality, the only difference is that MIAA is vested with corporate powers. As operator of the international airport, MIAA administers the land, improvements and equipment within the NAIA Complex. The MIAA Charter transferred to MIAA approximately 600 hectares of land, including the runways and buildings (―Airport Lands and Buildings‖) then under the Bureau of Air Transportation. The MIAA Charter further provides that no portion of the land transferred to MIAA shall be disposed of through sale or any other mode unless specifically approved by the President of the Philippines. Furthermore, Airport Lands and Buildings of MIAA are property of public dominion and therefore

owned by the State or the Republic of the Philippines. Article 419 of the Civil Code provides, The Airport Lands and Buildings of MIAA are property of public dominion and therefore owned by the State or the Republic of the Philippines. The Civil Code provides: ARTICLE 419. Property is either of public dominion or of private ownership. ARTICLE 420. The following things are property of public dominion: (1) Those intended for public use, such as roads, canals, rivers, torrents, ports and bridges constructed by the State, banks, shores, roadsteads, and others of similar character; (2) Those which belong to the State, without being for public use, and are intended for some public service or for the development of the national wealth. (Emphasis supplied) ARTICLE 421. All other property of the State, which is not of the character stated in the preceding article, is patrimonial property. ARTICLE 422. Property of public dominion, when no longer intended for public use or for public service, shall form part of the patrimonial property of the State. No one can dispute that properties of public dominion mentioned in Article 420 of the Civil Code, like ―roads, canals, rivers, torrents, ports and bridges constructed by the State,‖ are owned by the State. The term ―ports‖ includes seaports and airports. The MIAA Airport Lands and Buildings constitute a ―port‖ constructed by the State. Under Article 420 of the Civil Code, the MIAA Airport Lands and Buildings are properties of public dominion and thus owned by the State or the Republic of the Philippines. The Airport Lands and Buildings are devoted to public use because they are used by the public for international and domestic travel and transportation. The fact that the MIAA collects terminal fees and other charges from the public does not remove the character of the Airport Lands and Buildings as properties for public use. The operation by the government of a tollway does not change the character of the road as one for public use. Someone must pay for the maintenance of the road, either the public indirectly through the taxes they pay the government, or only those among the public who actually use the road through the toll fees they pay upon using the road. The tollway system is even a more efficient and equitable manner of taxing the public for the maintenance of public roads. The charging of fees to the public does not determine the character of the property whether it is of public dominion or not. Article 420 of the Civil Code defines property of public dominion as one ―intended for public use.‖ Even if the government collects toll fees, the road is still ―intended for public use‖ if anyone can use the road under the same terms and conditions as the rest of the public. The charging of fees, the limitation on the kind of vehicles that can use the road, the speed restrictions and other conditions for the use of the road do not affect the public character of the road. The terminal fees MIAA charges to passengers, as well as the landing fees MIAA charges to airlines, constitute the bulk of the income that maintains the operations of MIAA. The collection of such fees does not change the character of MIAA as an airport for public use. Such fees are often termed user‘s tax. This means taxing those among the public who actually use a public facility instead of taxing all the public including those who never use the particular public facility.

A user‘s tax is more equitable — a principle of taxation mandated in the 1987 Constitution. The Airport Lands and Buildings of MIAA, which its Charter calls the ―principal airport of the Philippines for both international and domestic air traffic,‖ are properties of public dominion because they are intended for public use. As properties of public dominion, they indisputably belong to the State or the Republic of the Philippines. Being a property of public dominion, the properties of MIAA are beyond the commerce of man. FXRL Lanzar v. Director of Lands 78 SCRA 130 DOCTRINE: Properties formed by accretion through the action of the sea belong to the public domain unless the government decrees otherwise. FACTS: ● Petitioner Lanzar filed an application for the registration of a property in Molo, Iloilo City. ● The Director of Lands filed an opposition to such application on the following grounds: o the said property is foreshore land o the City of Iloilo needs the same property as a road right of way for Molo Blvd. o the petitioner had not possessed the property in such manner as to grant him ownership ● The CFI ruled in favor of the plaintiff on the ground of valid prescription. ● The CA reversed, ruling in favor of the respondent on the ground that the property was formed by accretion from the sea making it public domain. ISSUE: W/N the title to property formed by action of the sea as an accretion may be registered to an applicant on the basis of adverse possession for over 30 years -HELD: The land is of public domain. Article 4 of the Law of Waters provides that: ―Lands added to the shores by accretions and alluvium deposits caused by the action of the sea, form part of the public domain. When they are no longer washed by the waters of the sea, and are not necessary for the purposes of public utility, or for the establishment of special industries, or for the coastguard service, the Government shall declare them to be the property of the owners of the estates adjacent thereto and as an increment thereof.‖ In sum, properties formed by accretion through the action of the sea belong to the public domain unless the government decree otherwise. As applied to the case on hand, it is undisputed that the property was formed by accretion through the action of the sea and that there was no such declaration by the government that the said property is no longer part of public domain. As such, the petitioner could not have acquired the property by mere adverse possession for the requisite number of years. RSDM Ignacio v. Director of Lands 108 Phil. 335

DOCTRINE: Citing Article 457 of the New Civil Code (Article 366, Old Civil Code), which provides that: To the owners of lands adjoining the banks of rivers belong the accretion which they gradually receive from the effects of the current of the waters. The article cited is clearly inapplicable because it refers to accretion or deposits on the banks of rivers, while the accretion in the present case was caused by action of the Manila Bay. FACTS: Ignacio filed an application alleging that among others that he owned the parcel applied for by right of accretion. The Director of Lands, Laureano Valeriano and Domingo Gutierrez filed oppositions. Gutierrez later withdrew his opposition. The Director of Lands claimed the parcel applied for as a portion of the public domain, for the reason that neither the applicant nor his predecessor-in-interest possessed sufficient title thereto, not having acquired it either by composition title from the Spanish government or by possessory information title under the Royal Decree and that he had not possessed the same openly, continuously and adversely under a bona fide claim of ownership since July 26, 1894. In his turn, Valeriano alleged he was holding the land by virtue of a permit granted him by the Bureau of Fisheries and approved by the President. It is not disputed that the land applied for adjoins a parcel owned by the applicant which he had acquired from the Government by virtue of a free patent title in 1936. It has also been established that the parcel in question was formed by accretion and alluvial deposits caused by the action of the Manila Bay which boarders it on the southwest. On the other hand, the Director of Lands sought to prove that the parcel is foreshore land, covered by the ebb and flow of the tide and, therefore, formed part of the public domain. After hearing, the trial court dismissed the application, holding that the parcel formed part of the public domain. ISSUE: Whether or not the disputed land is part of public domain -- Yes. It is part of public domain HELD: Appellant contends that the parcel belongs to him by the law of accretion, having been formed by gradual deposit by action of the Manila Bay, and he cites Article 457 of the New Civil Code (Article 366, Old Civil Code), which provides that: To the owners of lands adjoining the banks of rivers belong the accretion which they gradually receive from the effects of the current of the waters. The article cited is clearly inapplicable because it refers to accretion or deposits on the banks of rivers, while the accretion in the present case was caused by action of the Manila Bay. Interpreting Article 4 of the Law of Waters of 1866, in the case of Natividad vs. Director of Lands, (CA) 37 Off. Gaz., 2905, it was there held that: Article 4 of the Law of Waters of 1866 provides that when a portion of the shore is no longer washed by the waters of the sea and is not necessary for purposes of public utility, or for the establishment of special industries, or for coastguard service, the government shall declare it to be the property of the owners of the estates adjacent thereto and as an increment thereof. We believe that only the executive and possibly the legislative departments have the authority and the power to make the declaration that any land so gained by the sea, is not necessary for

purposes of public utility, or for the establishment of special industries, on for coast-guard service. If no such declaration has been made by said departments, the lot in question forms part of the public domain. MRAM Villarico v. Court of Appeals 309 SCRA 193 DOCTRINE: Private persons cannot own forest lands. Possession thereof, no matter how long, does not ripen into a registrable title. FACTS: Spouses Villarico filed an application for confirmation of title over a parcel of land in Meycauayan, Bulacan. The applicants alleged that 1)they are the absolute owners of the property having bought the same from Segundo Villarico and Mercedes Cardenas, 2) they and their predecessors-in-interest have been in actual, open, adverse and continuous possession thereof for more than 30 years, 3) they are not aware of any mortgage or encumbrance thereon nor of any person having an estate or interest therein, and 4) the land involved is not within the forest zone or government reservation. Marcos Camargo opposed the application for the registration of the land claiming that he is the real owner thereof. The Government interposed its opposition, through the Director of Forestry, averring that the land in question is part of the public domain, within the unclassified area in Meycauayan, Bulacan hence, it is not available for private appropriation. The trial court dismissed the case saying that “a certificate of title is void when it covers property of the public domain classified as forest or timber and mineral lands. Any title thus issued on non-disposable lots, even in the hands of an innocent purchaser for value, should be cancelled.” ISSUE: W/N the property in question can be registered by the applicant or by the oppositor or by their respective predecessors-in-interest? HELD: The Supreme Court Held in the negative. Since there is no showing that a declassification has been made by the Director of Forestry declaring the land in question as disposable or alienable, the land remains to be part of the public domain as a forest land. Considering this, possession of the land in question by the applicants and/or their predecessors-in-interest even for more than 30 years does not convert the land into private property capable of private appropriation. The SC stated that forest lands cannot be owned by private persons. Its possession, no matter how long it is, does not ripen into a registrable title. Furthermore, it must be noted that the adverse possession which may be the basis of a grant of title or confirmation of an imperfect title refers only to alienable or disposable portions of the public domain. FMM Villanueva v. Castaneda 154 SCRA 142 DOCTRINE: A public plaza is beyond the commerce of man and so cannot be the subject of lease or any other contractual undertaking.

FACTS: On 7 November 1961, the municipal council of San Fernando adopted Resolution No. 28 authorizing some members of the Fernandino United Merchants and Traders Association (herein ―Petitioners‖) to construct permanent stags and sell on a strip of land measuring 12 by 77 meters along Mercado Street, San Fernando, Pampanga. However, the action was protested on 10 November 1961 through a civil case. The issuance by the Court of First Instance of Pampanga of a writ of preliminary injunction prevented the Petitioners from constructing the stalls until final resolution of the controversy. On 18 January 1964, while the civil case was pending, the municipal council of San Fernando adopted Resolution G.R. No. 29 declaring the subject area as ―the parking place and as the public plaza of the municipality‖, thereby impliedly revoking Resolution No. 218, series of 1961. Four years later, on 2 November 1968, Judge Andres C. Aguilar decided the case and held that the land occupied by the Petitioners, being public in nature, was beyond the commerce of man and thus could not be subject of private occupancy. The writ of preliminary injunction was made permanent. However, the decision was not enforced for the Petitioners were not evicted from the subject area. The latter claimed that they and 128 other persons were in 1971 assigned specific areas for which they paid daily fees to the municipal government. This problem appears to have worsened for some more years under a presumable uneasy truce among the protagonists, none of whom made any move, for some reason that does not appear on record. On 12 January 1982, the Association of Concerned Citizens and Consumers of San Fernando filed a petition for the immediate implementation of Resolution No. 29, to restore the subject property ―to its original and customary use as a public plaza‖. Acting thereon after an investigation conducted by the municipal attorney, Vicente a. Macalino (herein ―Respondent‖), as officer in charge of the office of the mayor of San Fernando, issued on 14 June 1982, a resolution requiring the municipal treasurer and the municipal engineer to demolish the stalls in the subject area beginning 1 July 1982. The Petitioners reacted by filing a petition for prohibition with the Court of First Instance of Pampanga on 26 June 1982. However, herein Respondent judge denied the petition as well as the subsequent motion for reconsideration. Thus this petition for certiorari to challenge the decision. The basic contention of the Petitioners is that the disputed area is under lease to them by virtue of contracts they had entered into with the municipal government, first in 1961 insofar as the original occupants were concerned, and later with them and the other petitioners by virtue of the space allocations made in their favor in 1971 for which they are paying daily fees. Upon request by the Petitioners, the Supreme Court (SC) issued a temporary restraining order to preserve the status quo between the parties pending the decision. ISSUE: Whether the lease of the subject area is valid. -- NO HELD: The SC held that a public plaza is beyond the commerce of man and cannot be subject of lease or any other contractual undertaking.

The SC cited the following cases: 1. In Municipality of Cavite vs. Rojas, the SC declared as null and void the lease of a public plaza of the said municipality in favor of a private person; 2. In Muyot vs. de la Fuente, the SC held that the City of Manila could not lease a portion of a public sidewalk on Plaza Sta. Cruz, being likewise beyond the commerce of men; and 3. In Espiritu vs. Municipal Council of Pozorrubio, the SC declared: ―There is absolutely no question that the town plaza cannot be used for the construction of market stalls, specially of residences, and that such structures constitute a nuisance subject to abatement according to law. Town plazas are properties of public dominion, to be devoted to public use and to be made available to the public in general They are outside the common of man and cannot be disposed of or even leased by the municipality to private parties.‖ And ruled that applying this well-settled doctrine, the petitioners had no right in the first place to occupy the disputed premises and cannot insist in remaining there now on the strength of their alleged lease contracts. The SC dismissed the petition and affirmed the decision and order of the lower court. It also lifted the temporary restraining order requested by the petitioners. RGGM Dacanay Jr. v. Asistio Jr. 208 SCRA 404 DOCTRINE: A public street is property for public use hence outside the commerce of men. The right of the public to use the city streets may not be bargained away through a contract. FACTS: This is a petition for mandamus to the non-action of the city government of Caloocan in accordance with the decision of the RTC to evict the occupants of a flea market located in the streets of Caloocan. January 5, 1979 – Metropolitan Manila Commission enacted an ordinance allowing the use of streets for the purpose of flea markets subject to several conditions. 1987 – Mayor Martinez caused the demolition of the flea markets and the stall owners filed a case against such action. RTC dismissed the case on the ground that the streets in questions are of public dominion, hence outside the commerce of man. After the decision came out, there was a change in the city administration and current mayor (Asistio) did not pursue the action of the previous mayor and left the flea markets in the streets as is. Dacanay filed a petition for mandamus to remove the stalls in their street. ISSUE: May public streets be leased or licensed to market stallholders by virtue of a city ordinance or resolution of Metropolitan Manila Commission? -- NO

HELD: A public street is property for public use hence outside the commerce of man. Being outside the commerce of man, it may not be the subject of lease or other contract. Any executive order or city resolution cannot change the nature of the public street because it is going to be contrary to the general law. MCSS Cebu Oxygen and Acetylene Co. v. Bercilles 66 SCRA 431 FACTS: A portion of land, sought to be registered, was declared, through law, an abandoned road. The lot was awarded to the petitioner for being the highest bidder. The Assistant Provincial Fiscal of Cebu filed a motion to dismiss the application on the ground that the property sought to be registered being a public road intended for public use is considered part of the public domain and therefore outside the commerce of man. ISSUE: WON the portion of land is susceptible to registration by a private individual. -- YES HELD: Revised Charter of Cebu, under section 31, provides that, the City Council shall have the power to close any city road, street or alley, etc, withdrawn from public servitude, may be used or conveyed for any purpose. It is undoubtedly clear that the City of Cebu is empowered to close a city road or street. Such power is discretionary and will not ordinarily be controlled or interfered with by the courts, absent a plain case of abuse or fraud or collusion. It follows that such withdrawn portion becomes patrimonial property which can be the object of an ordinary contract. NKVS Laurel v. Garcia 187 SCRA 797 DOCTRINE: An abandonment of the intention to use the property for public service and to make it patrimonial property under Article 422 of the Civil Code must be definite Abandonment and it cannot be inferred from the non-use alone specially if the non-use was attributable not to the government's own deliberate and indubitable will but to a lack of financial support to repair and improve the property Abandonment must be a certain and positive act based on correct legal premises. FACTS: These are two petitions for prohibition seeking to enjoin respondents, their representatives and agents from proceeding with the bidding for the sale of the 3,179 square meters of land at Tokyo, Japan scheduled on February 21, 1990. The subject property in this case is 1 of the 4 properties in Japan acquired by the Philippine government under the Reparations Agreement entered into with Japan on May 9, 1956. The properties and the capital goods and services procured from the Japanese government for national development projects are part of the indemnification to the Filipino people for their

losses in life and property and their suffering during World War II. A proposal was presented to President Corazon C. Aquino by former Philippine Ambassador to Japan, Carlos J. Valdez, to make the property the subject of a lease agreement with a Japanese firm. No change of ownership or title shall occur. The Philippine government retains the title all throughout the lease period and thereafter. However, the government has not acted favorably. On July 25, 1987, the President issued Executive Order No. 296 entitling non-Filipino citizens or entities to avail of separations' capital goods and services in the event of sale, lease or disposition. The four properties in Japan including the Roppongi were specifically mentioned in the first "Whereas" clause. Amidst opposition by various sectors, the Executive branch of the government has been pushing its decision to sell the reparations properties starting with the Roppongi lot. The property has twice been set for bidding at a minimum floor price of $225 million. ISSUE: W/N the Roppongi property and others of its kind be alienated by the Philippine Government. -NO HELD: NO, the subject property cannot be alienated by the government, even if the property has not been in use for a long time. Vice President Laurel asserts that the lands were acquired as part of the reparations for diplomatic and consular use by the Philippine government. Laurel states that the Roppongi property is classified as one of public dominion, and not of private ownership under Article 420 of the Civil Code. The petitioner submits that the Roppongi property comes under "property intended for public service" in paragraph 2 of the above provision. He states that being one of public dominion, no ownership by anyone can attach to it, not even by the State. The Roppongi and related properties were acquired for "sites for chancery, diplomatic, and consular quarters, buildings and other improvements. The petitioner states that they continue to be intended for a necessary service. They are held by the State in anticipation of an opportune use. (Citing 3 Manresa 6566). Hence, it cannot be appropriated, is outside the commerce of man, or to put it in more simple terms, it cannot be alienated nor be the subject matter of contracts (Citing Municipality of Cavite v. Rojas, 30 Phil. 20 [1915]). Noting the non-use of the Roppongi property at the moment, the petitioner avers that the same remains property of public dominion so long as the government has not used it for other purposes nor adopted any measure constituting a removal of its original purpose or use. As property of public dominion, the Roppongi lot is outside the commerce of man. It cannot be alienated. Its ownership is a special collective ownership for general use and enjoyment, an application to the satisfaction of collective needs, and resides in the social group. The purpose is not to serve the State as a juridical person, but the citizens; it is intended for the common and public welfare and cannot be the object of appropration The applicable provisions of the Civil Code are: ART. 419. Property is either of public dominion or of private ownership.

ART. 420. The following things are property of public dominion (1) Those intended for public use, such as roads, canals, rivers, torrents, ports and bridges constructed by the State, banks shores roadsteads, and others of similar character; (2) Those which belong to the State, without being for public use, and are intended for some public service or for the development of the national wealth. ART. 421. All other property of the State, which is not of the character stated in the preceding article, is patrimonial property. The Roppongi property is correctly classified under paragraph 2 of Article 420 of the Civil Code as property belonging to the State and intended for some public service. The fact that the Roppongi site has not been used for a long time for actual Embassy service does not automatically convert it to patrimonial property. Any such conversion happens only if the property is withdrawn from public use. A property continues to be part of the public domain, not available for private appropriation or ownership until there is a formal declaration on the part of the government to withdraw it from being such. An abandonment of the intention to use the Roppongi property for public service and to make it patrimonial property under Article 422 of the Civil Code must be definite Abandonment cannot be inferred from the non-use alone specially if the non-use was attributable not to the government's own deliberate and indubitable will but to a lack of financial support to repair and improve the property Abandonment must be a certain and positive act based on correct legal premises. A mere transfer of the Philippine Embassy to Nampeidai in 1976 is not relinquishment of the Roppongi property's original purpose. Moreover, President Aquino‘s approval of the recommendation by the investigating committee to sell the Roppongi property was premature or, at the very least, conditioned on a valid change in the public character of the Roppongi property. It does not have the force and effect of law since the President already lost her legislative powers. The Congress had already convened for more than a year. Assuming that the Roppongi property is no longer of public dominion, there is another obstacle to its sale by the respondents. There is no law authorizing its conveyance, and thus, the Court sees no compelling reason to tackle the constitutional issue raised by petitioner Ojeda. AMPS Palanca v. Commonwealth G.R. No. 46373 DOCTRINE: A private person may not acquire ownership of a property of public dominion (such as navigable waters) through prescription or even by virtue of a Torrens title. FACTS: On the 17th of July 1919 Carlos Palanca obtained a decision from the Court of First Instance of Bulacan for the registration of 4 parcels of land, each adjacent to the others and separated only among them by some waterways called ―esteros‖. Before the decision is read, the government through the fiscal general presented a petition for reopening of the case to check on the existence of ―esteros‖ and rivers that are navigable in the terrain, with the intention of

setting/including the said circumstance in the map and so that it be excluded from the registration. The Court denied this petition because they consider it unnecessary. Despite its presence in the terrain, the registration of these will not affect the rights of propriety of the Insular Government or the public use of the said waterways, which will always remain safe with article 39 of the Law Register of Propriety. Much later, the Government of the Islands of the Philippines presented the current action against Carlos Palanca claiming that he is illegally occupying portions of the Viray River and Sapang Sedaria ―estero‘, which are navigable and was asked that he be obliged to open it, and leave it in its primitive state. The court discontinues this action; but elevated it to the Court of Appeals. The Court of Appeals declared that the said River Viray and Sapang Sedaria ―estero‘ are of the possession and use of the public and that the right acquired by Carlos Palanca over the land in which the waterways are located does not affect the right of the State over them, as goods destined for public use. Carlos Palanca in his turn elevated an appeal through ―certiorari‖ to this decision of the Court of Appeals to the Supreme Court. ISSUE: What is the status of the river and estero? HELD: The river and the “estero” being navigable, useful for commerce, the navigation and fishing, have the character of public possession and its legal condition in this sense has not been affected by the possession of Carlos Palance, whenever has been the time of this possession, because it does not admit any prescription against the State over goods of public use. The Court of Appeals declared in view of the proofs presented in the case that the River Viray and the Sapang Sedaria ―Estero‖ are found within the land of Carlos Palanca that was the object of the decision of the Court of First Instance, and that said river and ―estero‖ are navigable and of common use, serves as a communication between two courses of water which flows into the Manila Bay, and that the water current in them are of the possession and use of the public, useful for commerce, navigation and fishing and that they have these conditions when the land that surrounds them still pertains to the State. These being the fact, the request or the recourse opened before this tribunal should be denied, thus confirming the decision pronounced by the Court of Appeals. We cannot find basis for the request of the appellant that the decision pronounced by the Court of First Instance and the Torrens title issued, in its consequence, in favor of Carlos Palanca establish the non-existence of the river and ―estero‖ in question as navigable. In that decision, it is well said that the terrain is crossed by some waterways called ―esteros‖. There is no declaration whatsoever that these ―esteros‖ are not navigable. On the other hand, when the government asked for opportunity to prove that some of these ―esteros‖ are navigable, the court denied them this opportunity due to the reason that, if they really are navigable, the right of the Government over them, by reason of such condition of the River Viray and the Sapang Sedaria ―estero‖ as navigable, have not been the object of the judicial decision.

As for the rest, the river and the ―estero‖ being navigable, useful for commerce, the navigation and fishing, have the character of public possession and its legal condition in this sense has not been affected by the possession of Carlos Palance, whenever has been the time of this possession, because it does not admit any prescription against the State over goods of public use. For these considerations, the appeal is denied and the decision of the court of appeals is confirmed. KGS Rabuco v. Villegas G.R. No. L-24661 DOCTRINES: 1. RA 3120 converting communal lands into alienable lands is constitutional. 2. Legislative body has wide discretion to reclassify State property. FACTS: The legislatures enacted RA3120 converting certain parcels of land in the City of Manila which are reserved as communal property into disposable or alienable lands of the state and providing for their subdivision and sale. It includes a lot located in San Andres, Malate, which was occupied by petitioners (illegal settlers). The City Mayor of Manila wanted to demolish and eject said occupants. Subsequently, a large fire gutted the Malate area, which includes said property. City officials then took over the lot and kept petitioners reconstructing or repairing their burned dwellings. The petitioners insisted that RA 3120 should be implemented to them as the tenants and bona fide occupants thereof. ISSUE: Can a communal property be converted into disposable or alienable land through legislation? -YES HELD: The Court herein upholds the constitutionality of Republic Act 3120 on the strength of the established doctrine that the subdivision of communal land of the State (although titled in the name of the municipal corporation) and conveyance of the resulting subdivision lots by sale on installment basis to bona fide occupants by Congressional authorization and disposition does not constitute infringements of the due process clause or the eminent domain provisions of the Constitution but operates simply as a manifestation of the legislature's right of control and power to deal with State property. Respondents city officials' contention that the Act must be stricken down as unconstitutional for depriving the city of Manila of the lots in question and providing for their sale in subdivided small lots to bona fide occupants or tenants without payment of just compensation is untenable and without basis, since the lots in question are manifestly owned by the city in its public and governmental capacity and are therefore public property over which Congress had absolute control as distinguished from patrimonial property owned by it in its private or proprietary capacity of which it could not be deprived without due process and without just compensation. Here, Republic Act 3120 expressly declared that the properties were "reserved as communal property" and ordered their conversion into "disposable and alienable lands of the State" for sale

in small lots to the bona fide occupants thereof. It is established doctrine that the act of classifying State property calls for the exercise of wide discretionary legislative power which will not be interfered with by the courts. The case of Salas vs. Jarencio wherein the Court upheld the constitutionality of Republic Act 4118 whereby Congress in identical terms as in Republic Act 3120 likewise converted another city lot (Lot 1-B-2-B of Block 557 of the cadastral survey of Manila also in Malate) which was reserved as communal property into disposable land of the State for resale in small lots by the Land Tenure, Administration to the bona fide occupants is controlling in the case at bar. The Court therein reaffirmed the established general rule that "regardless of the source or classification of land in the possession of a municipality, excepting those acquired with its own funds in its private or corporate capacity, such property is held in trust for the State for the benefit of its inhabitants, whether it be for governmental or proprietary purposes. It holds such lands subject to the paramount power of the legislature to dispose of the same, for after all it owes its creation to it as an agent for the performance of a part of its public work, the municipality being but a subdivision or instrumentality thereof for purposes of local administration. Accordingly, the legal situation is the same as if the State itself holds the property and puts it to a different use" 9 and stressed that "the property, as has been previously shown, was not acquired by the City of Manila with its own funds in its private or proprietary capacity. That it has in its name a registered title is not questioned, but this title should be deemed to be held in trust for the State as the land covered thereby was part of the territory of the City of Manila granted by the sovereign upon its creation." There as here, the Court holds that the Acts in question (Republic Acts 4118 in Salas and Republic Act 3120 in the case at bar) were intended to implement the social justice policy of the Constitution and the government program of land for the landless and that they were not "intended to expropriate the property involved but merely to confirm its character as communal land of the State and to make it available for disposition by the National Government: ... The subdivision of the land and conveyance of the resulting subdivision lots to the occupants by Congressional authorization does not operate as an exercise of the power of eminent domain without just compensation in violation of Section 1, subsection (2), Article III of the Constitution, but simply as a manifestation of its right and power to deal with state property." JPOT Macasiano v. Diokno G.R. No. 97764 DOCTRINE: "Verily, the powers of a local government unit are not absolute. They are subject to limitations laid down by the Constitution and the laws such as our Civil Code. Moreover, the exercise of such powers should be subservient to paramount considerations of health and wellbeing of the members of the community." FACTS: On June 13, 1990, the respondent municipality passed Ordinance No. 86, Series of 1990 which authorized the closure of J. Gabriel, G.G. Cruz, Bayanihan, Lt. Garcia Extension and Opena Streets located at Baclaran, Paranaque, Metro Manila and the establishment of a flea market thereon. July 20, 1990, the Metropolitan Manila Authority approved Ordinance No. 86, s. 1990 of the municipal council of respondent municipality subject to the following conditions: 1. That the aforenamed streets are not used for vehicular traffic, and that the majority of the residents do not oppose the establishment of the flea market/vending areas thereon;

2. That the 2-meter middle road to be used as flea market/vending area shall be marked distinctly, and that the 2 meters on both sides of the road shall be used by pedestrians; 3. That the time during which the vending area is to be used shall be clearly designated; 4. That the use of the vending areas shall be temporary and shall be closed once the reclaimed areas are developed and donated by the Public Estate Authority. June 20, 1990, Mayor Walfrido N. Ferrer to enter into contract with any service cooperative for the establishment, operation, maintenance and management of flea markets and/or vending areas. On August 8, 1990, respondent municipality and respondent Palanyag, a service cooperative, entered into an agreement whereby the latter shall operate, maintain and manage the flea market in the aforementioned streets with the obligation to remit dues to the treasury of the municipal government of Paranaque. Consequently, market stalls were put up by respondent Palanyag on the said streets. On September 13, 1990, petitioner Brig. Gen. Macasiano, PNP Superintendent of the Metropolitan Traffic Command, ordered the destruction and confiscation of stalls along G.G. Cruz and J. Gabriel St. in Baclaran. These stalls were later returned to respondent Palanyag. October 16, 1990, petitioner Brig. General Macasiano wrote a letter to respondent Palanyag giving the latter ten (10) days to discontinue the flea market; otherwise, the market stalls shall be dismantled. Hence, on October 23, 1990, respondents municipality and Palanyag filed with the trial court a joint petition for prohibition and mandamus with damages and prayer for preliminary injunction, to which the petitioner filed his memorandum/opposition to the issuance of the writ of preliminary injunction. October 24, 1990, the trial court issued a temporary restraining order to enjoin petitioner from enforcing his letterorder of October 16, 1990 pending the hearing on the motion for writ of preliminary injunction. On December 17, 1990, the trial court issued an order upholding the validity of Ordinance No. 86 s. 1990 of the Municipality of Paranaque and enjoining petitioner Brig. Gen. Macasiano from enforcing his letter-order against respondent Palanyag. ISSUE: W/N an ordinance or resolution issued by the municipal council of Paranaque authorizing the lease and use of public streets or thoroughfares as sites for flea markets is valid. -- NO HELD: The Executive Order issued by acting Mayor Robles authorizing the use of Heroes del '96 Street as a vending area for stallholders who were granted licenses by the city government contravenes the general law that reserves city streets and roads for public use. Mayor Robles' Executive Order may not infringe upon the vested right of the public to use city streets for the purpose they were intended to serve: i.e., as arteries of travel for vehicles and pedestrians. The Solicitor General furthers the matter with his observation, "Verily, the powers of a local government unit are not absolute. They are subject to limitations laid down by the Constitution and the laws such as our Civil Code. Moreover, the exercise of such powers should be subservient to paramount considerations of health and well-being of the members of the community. Every local government unit has the sworn obligation to enact measures that will enhance the public health, safety and convenience, maintain peace and order, and promote the general prosperity of the inhabitants of the local units. Based on this objective, the local government should refrain from acting towards that which might prejudice or adversely affect the general welfare." Moreover, the municipality did not even comply with the guidelines set forth by the Metropolitan Manila Authority. Even if we were to argue for purposes of debate, the city of Paranaque's claim would still be bereft and lacking in reason. ACCORDINGLY, the petition is GRANTED and the decision of the respondent Regional Trial Court dated December 17, 1990 which granted the writ of preliminary injunction enjoining

petitioner as PNP Superintendent, Metropolitan Traffic Command from enforcing the demolition of market stalls along J. Gabriel, G.G. Cruz, Bayanihan, Lt. Garcia Extension and Opena streets is hereby RESERVED and SET ASIDE. SO ORDERED. MLAV Province of Zamboanga del Norte v. City of Zamboanga G.R. No. L-24440 DOCTRINE: Under the law of Municipal Corporations, properties which are devoted to public service are deemed public and the rest remain patrimonial. Under this norm, to be considered public, it is enough that the property be held and, devoted for governmental purposes like local administration, public education, public health, etc. FACTS: On October 12, 1936, Commonwealth Act 39 was approved converting the Municipality of Zamboanga into Zamboanga City. It further provided that buildings and properties which the province shall abandon upon the transfer of the capital to another place will be acquired and paid for by the City of Zamboanga at a price to be fixed by the Auditor General. The properties and buildings referred to consisted of 50 lots and some buildings constructed thereon, located in the City of Zamboanga and covered individually by Torrens certificates of title in the name of Zamboanga Province. Pursuant to CA 39, the Auditor General fixed the value of the properties and buildings in question left by Zamboanga Province in Zamboanga City at P1,294,244.00. When RA 711 was approved dividing the province of Zamboanga into Zamboanga del Norte and Zamboanga del Sur, assets and obligations of the previous Zamboanga province were divided as follows: 54.39% for Zamboanga del Norte and 45.61% for Zamboanga del Sur. Zamboanga del Norte therefore became entitled to 54.39% of P1,294,244.00, the total value of the lots and buildings in question, or P704,220.05 payable by Zamboanga City. The Executive Secretary issued a ruling holding that Zamboanga del Norte had a vested right as owner of the properties mentioned in Sec. 50 of CA 39, and is entitled to the price thereof, payable by Zamboanga City. This ruling revoked the previous Cabinet Resolution conveying all the said 50 lots and buildings thereon to Zamboanga City when the provincial capital of the then Zamboanga Province was transferred to Dipolog. The Secretary of Finance then authorized the Commissioner of Internal Revenue to deduct an amount equal to 25% of the regular internal revenue allotment for the City of Zamboanga. The deductions, all aggregating P57,373.46, was credited to the province of Zamboanga del Norte, in partial payment of the P704,220.05 due it. On June 17, 1961, RA 3039 was approved amending Sec. 50 of CA 39 by providing that buildings, properties and assets belonging to the former province of Zamboanga and located within the City of Zamboanga are transferred, free of charge, in favor of the said City of Zamboanga. Consequently, the Secretary of Finance ordered the CIR to stop from effecting further payments to Zamboanga del Norte and to return to Zamboanga City the sum taken from it out of the internal revenue allotment of Zamboanga del Norte.

Zamboanga del Norte filed a complaint for relief with Preliminary Mandatory Injunction against Zamboanga City, the Secretary of Finance and the Commissioner of Internal Revenue. They prayed that 1) RA 3039 be declared unconstitutional, 2) plaintiff's rights and obligations under said law be declared, 3) they be reimbursed the sum paid to defendant City, and 4) the latter be ordered to continue paying the balance of its internal revenue allotments. The lower court ruled in favor of the plaintiff. ISSUE: WON RA 3039 is valid – PARTLY VALID HELD: Applying the norm obtaining under the principles constituting the law of Municipal Corporations, all those of the 50 properties in question which are devoted to public service are deemed public; the rest remain patrimonial. Under this norm, to be considered public, it is enough that the property be held and, devoted for governmental purposes like local administration, public education, public health, etc. Following this classification, RA 3039 is valid insofar as it affects the lots used as capitol site, school sites and its grounds, hospital and leprosarium sites and the high school playground sites — a total of 24 lots — since these were held by the former Zamboanga province in its governmental capacity and therefore are subject to the absolute control of Congress. Regarding the several buildings existing on the lots above-mentioned, the records do not disclose whether they were constructed at the expense of the former Province of Zamboanga. Considering however the fact that said buildings must have been erected even before 1936 when CA 39 was enacted and the further fact that provinces then had no power to authorize construction of buildings at their own expense, it can be assumed that said buildings were erected by the National Government, using national funds. Even assuming that provincial funds were used, still the buildings constitute mere accessories to the lands, which are public in nature, and so, they follow the nature of said lands, i.e., public. But RA 3039 cannot be applied to deprive Zamboanga del Norte of its share in the value of the rest of the 26 remaining lots which are patrimonial properties since they are not being utilized for distinctly, governmental purposes. Moreover, the fact that these 26 lots are registered strengthens the proposition that they are truly private in nature. However, the fact that the 24 lots used for governmental purposes are also registered is of no significance since registration cannot convert public property to private. It results then that Zamboanga del Norte is still entitled to collect from the City of Zamboanga the former's 54.39% share in the 26 properties which are patrimonial in nature. DJTV Director of Lands v. MERALCO 153 SCRA 686 DOCTRINE: Open, exclusive and undisputed possession of alienable public land for the period prescribed by law creates the legal fiction whereby the land ceases to be public land and becomes private property.

FACTS: Manila Electric Company (MERALCO) filed an amended application for registration of a parcel of land located in Taguig, Metro Manila. Applicant acquired the land applied for registration by purchase from Ricardo Natividad who in turn acquired the same from his father Gregorio Natividad as evidenced by a Deed of Original Absolute Sale. Applicant's predecessors-ininterest have possessed the property under the concept of an owner for more than 30 years. The property was declared for taxation purposes under the name of the applicant and the taxes due thereon have been paid. The respondent Judge rendered a decision ordering the registration of the property in the name of the private respondent. The petitioner Director of Lands interposed this petition raising the issue of whether or not a corporation may apply for registration of title to land. Petitioner contends that a corporation is not among those that may apply for confirmation of title under Section 48 of Commonwealth Act No. 141, the Public Land Act. ISSUES: 1. Whether or not a corporation may apply for registration of titles to public land. 2. Whether or not open, exclusive and undisputed possession of alienable public land for the period prescribed by law creates the legal fiction whereby the land ceases to be public land and becomes private property. HELD: 1. Yes. The legal issue raised by the petitioner Director of Lands has been squarely dealt with in two recent cases (The Director of Lands v. Intermediate Appellate Court and Acme Plywood & Veneer Co., Inc., etc., No. L-73002 (December 29, 1986), 146 SCRA 509. The Director of Lands v. Hon. Bengzon and Dynamarine Corporation, etc., No. 54045 (July 28, 1987)], and resolved in the affirmative. There can be no different answer in the case at bar. Coming to the case at bar, if the land was already private at the time Meralco bought it from Natividad, then the prohibition in the 1973 Constitution against corporations holding alienable lands of the public domain except by lease (1973 Const., Art. XIV, See. 11) does not apply. The fact that the confirmation proceedings were instituted by a corporation is simply another accidental circumstance, "productive of a defect hardly more than procedural and in nowise affecting the substance and merits of the right of ownership sought to be confirmed in said proceedings." Considering that it is not disputed that the Natividads could have had their title confirmed, only a rigid subservience to the letter of the law would deny private respondent the right to register its property which was validly acquired. 2. Yes. In the Acme decision, Supreme Court upheld the doctrine that open, exclusive and undisputed possession of alienable public land for the period prescribed by law creates the legal fiction whereby the land, upon completion of the requisite period ipso jure and without the need of judicial or other sanction, ceases to be public land and becomes private property. WHEREFORE, the petition is DENIED. The questioned decision of the respondent Judge is AFFIRMED.

OWNERSHIP (ART. 427-439) JGY Chiao Liong Tan v. Court of Appeals 228 SCRA 75 DOCTRINE: Although a "replevin" action is primarily one for the possession of personality, yet it is sufficiently flexible to authorize a settlement of all equities between the parties, arising from or growing out of the main controversy. FACTS: Petitioner Chiao Liong Tan claims to be the owner of an Isuzu Elf van. He claims that he has been in possession, enjoyment and utilization of the said motor vehicle until it was taken from him by his older brother, Tan Ban Yong, the private respondent herein. Petitioner relies principally on the fact that the Isuzu Elf van is registered in his name. He claims in his testimony before the trial court that the said vehicle was purchased from Balintawak Isuzu Motor Center for a price of over P100K; that he sent his brother to pay for the van and the receipt for payment was placed in his name because it was his money that was used to pay for the vehicle; that he allowed his brother to use the van because the latter was working for his company. Private respondent testified that CLT Industries is a family business. When the family business needed a vehicle, he asked petitioner to look for a vehicle and gave him the amount of P5K to be deposited as down payment and he himself paid the whole price out of a loan of P140K. Since he was still on good terms with the petitioner then, he allowed the registration of the vehicle in petitioner's name. TC: Ruled in favor of private respondent. CA: Affirmed. ISSUE: WON questions of ownership may be resolved in a replevin proceeding. -- YES HELD: It is true that the judgment in a replevin suit must only resolve in whom is the right of possession. Primarily, the action of replevin is possessory in character and determined nothing more than the right of possession. However, when the title to the property is distinctly put in issue by the defendant's plea and by reason of the policy to settle in one action all the conflicting claims of the parties to the possession of the property in controversy, the question of ownership may be resolved in the same proceeding. Although a "replevin" action is primarily one for the possession of personality, yet it is sufficiently flexible to authorize a settlement of all equities between the parties, arising from or growing out of the main controversy. Thus, in an action for replevin where the defendant is adjudged entitled to possession, he need not go to another forum to procure relief for the return of the replevied property or secure a judgment for the value of the property in case the adjudged return thereof could not be had. Appropriately, the trial court rendered an alternative judgment. CRF Calub v. Court of Appeals 331 SCRA 55

DOCTRINE: Replevin cannot be issued to recover a property lawfully taken by virtue of legal process and considered in the custody of the law. A replevin case against the State, without its consent, cannot prosper. (batasnatin) FACTS: On January 28, 1992, the Forest Protection and Law Enforcement Team of the Community Environment and Natural Resources Office (CENRO) of the DENR apprehended two (2) motor vehicles by whose drivers failed to present proper documents and/or licenses. Thus, the apprehending team seized and impounded the vehicles and its load of lumber at the DENRPENR. Seizure receipts were issued but the drivers refused to accept the receipts. Abuganda, one of the drivers was charged with for violation of Revised Forestry Code. On January 31, 1992, the impounded vehicles were forcibly taken by Gabon and Abuganda from the custody of the DENR. On February 11, 1992, one of the two vehicles was again apprehended by a composite team of DENR-CENR loaded with forest products. A corresponding criminal case was again filed against Abuganda because of this incident. In both criminal cases however, Abegonia and Abuganda were acquitted on the ground of reasonable doubt. Subsequently, herein private respondents Manuela Babalcon, the vehicle owner, and Constancio Abuganda, the driver, filed a complaint for the recovery of possession of the two (2) impounded vehicles with an application for replevin against herein petitioners before the RTC of Catbalogan which was granted by the RTC. The Court issued a TRO, enjoining respondent RTC judge from conducting further proceedings in the civil case for replevin; and enjoining private respondents from taking or attempting to take the motor vehicles and forest products seized from the custody of the petitioners. The Court further instructed the petitioners to see to it that the motor vehicles and other forest products seized are kept in a secured place and protected from deterioration, said property being in custodia legis and subject to the direct order of the Supreme Court. On May 27, 1994, the Court of Appeals denied said petition for lack of merit. It ruled that the mere seizure of a motor vehicle pursuant to the authority granted by Section 68 [78] of P.D. No. 705 as amended by E.O. No. 277 does not automatically place said conveyance in custodia legis. ISSUE: Whether or not the DENR-seized motor vehicle in custodia legis can be a subject to an action for replevin? - NO HELD: Since there was a violation of the Revised Forestry Code and the seizure was in accordance with law, in our view the subject vehicles were validly deemed in custodia legis. It could not be subject to an action for replevin. For it is property lawfully taken by virtue of legal process and considered in the custody of the law, and not otherwise. JRPA Sarmiento v. Court of Appeals 250 SCRA 108

DOCTRINE: A boundary dispute is neither a case of forcible entry nor of unlawful detainer, but an issue of ownership. FACTS: Cruz was the owner of a parcel of land. Adjacent to this lot is one wherein Sarmiento had a house built on. On trying to cause the relocation of her lot, Cruz found out that Sarmiento was encroaching on her property. When Cruz talked to Sarmiento about constructing a new fence, which will cover her true property, the latter vehemently refused to do so and threatened Cruz with legal action. For fear of being sued in court, she sought judicial relief. The trial court decided in favor of Cruz. Sarmiento tried to assail this decision by saying that the issue was on ownership of the portion of land and thus, the action should have been an accion reivindicatoria and not forcible entry. ISSUE: WON Cruz filed the proper judicial relief. -- NO HELD: A careful reading of the facts averred in said complaint filed by Cruz reveals that the action is neither of forcible entry nor of unlawful detainer but essentially involves a boundary dispute, which must be resolved in an accion reivindicatoria on the issue of ownership over the portion of a lot. Forcible entry and unlawful detainer cases are distinct actions. Cruz cannot belatedly claim that petitioner‘s possession of the controverted portion was by mere tolerance. The complaint didn‘t characterize Sarmiento‘s alleged entry on the land—whether legal or illegal. The complaint admitted also of the fact that the fence had already preexisted on the lot when she acquired the same. This was definitely not a situation obtained in and gave rise to an ejectment suit for two reasons. First, forcible entry into the land is an open challenge to the right of the lawful possessor, the violation of which right authorizes the speedy redress in the inferior court provided for in the Rules. Second, if a forcible entry action in the court is allowed after the lapse of a number of years, then the result may well be no action of forcible entry can really prescribe. No matter how long such defendant is in physical possession, the plaintiff may just throw in a demand, file a suit in court and summarily throw him out of the land. ABB Bongato v. Malvar 387 SCRA 327 DOCTRINE: The one-year period within which to bring an action for forcible entry is generally counted from the date of actual entry to the land. However, when entry is made through stealth, then the one-year period is counted from the time the plaintiff learned about it. After the lapse of the one-year period, the party deprived of the parcel of land may file either accion publiciana; or an accion reivindicatoria, which is an action to recover ownership as well as possession. FACTS: Spouses Severo and Trinidad Malvar filed a complaint in the MTCC for forcible entry against Teresita Bongato, alleging that Bongato unlawfully entered a parcel of land belonging to the

spouses and erected thereon a house of light materials. MTCC decided in favor of Malvar and ordered Bongato to vacate the land. RTC affirmed the decision. CA also held that MTCC had jurisdiction. On appeal, Bongato raised the issue of MTCC jurisdiction because the complaint was filed beyond the one-year prescriptive period. ISSUE: Whether or not the MTCC had jurisdiction since the Complaint was filed beyond the one-year period from date of alleged entry? -- NO HELD: MTCC had no jurisdiction. Forcible entry is a quieting process, and that the restrictive time bar is prescribed to complement the summary nature of such process. Indeed, the one-year period within which to bring an action for forcible entry is generally counted from the date of actual entry to the land. However, when entry is made through stealth, then the one-year period is counted from the time the plaintiff learned about it. After the lapse of the one-year period, the party deprived of the parcel of land may file either accion publiciana; or an accion reivindicatoria, which is an action to recover ownership as well as possession. One the basis of the facts, it is clear that the cause of action for forcible entry filed by respondents had already prescribed when they filed the complaint on July 10, 1992 (the house was built as early as 1987), thus the MTCC had no more jurisdiction to hear and decide the case for forcible entry. FZC De La Cruz v. CA 286 SCRA 230 DOCTRINE: A positive act of the gov‘t is needed to reclassify land and until such reclassification, property remains part of the forest reserve incapable of alienation and cannot be acquired by prescription. FACTS: In 1973, a 407 sq. m. residential lot was the subject of an application under the Land Registration Act by the Ramos bros. Eugenio de la Cruz [petitioner] opposed. After trial, the application was dismissed on the ground that the land was not yet reclassified and remains part of the forest reserve. The Ramos bros. pursued the reclassification of the land and were subsequently awarded ownership of it. Cristina Villanueva, the private respondent, subsequently purchased the same lot from the brothers. Upon learning of the said sale, petitioner filed a complaint for reconveyance claiming ownership of the said land having possessed and occupied it openly, publicly, notoriously and adversely against the whole world and in the concept of an owner for more than 30 years. His complaint was dismissed. The CA affirmed in toto the decision of the trial court thus the case at bar. ISSUES: Whether or not petitioner is vested with a better right over the residential lot to which he possessed and devoted time, effort and resources -- NO HELD: Petitioner possessed and occupied the land after it was declared by the Gov‘t as part of the forest zone. Forest lands or forest reserves are not capable of private appropriation, and

possession thereof, however long, cannot convert them into private property. A positive act by the government is needed to declassify land and to convert it to alienable or disposable land. And until such declassification, there is no disposable land to speak of. LNAC Philippine Economic Zone Authority v. Fernandez 358 SCRA 489 DOCTRINE: Reconveyance is a remedy of those whose property has been wrongfully or erroneously registered in the name of another. Such recourse, however, cannot be availed of once the property has passed to an innocent purchaser for value. For an action for reconveyance to prosper, the property should not have passed into the hands of an innocent purchaser for value. FACTS: The subject of the present controversy is Lot No. 4673 of the Opon Cadastre situated in LapuLapu City, covered by Original Certificate of Title (OCT) No. RO-2537 (May 19, 1982) and registered in the names of Florentina Rapaya et al. On May 15, 1982, Jorgea Igot-Soroño, Frisca Booc and Felix Cuizon executed an Extrajudicial Partition, in which they declared themselves as the only surviving heirs of the registered owners of the aforesaid lot. Consequently, they were issued TCT No. 12467 on July 8, 1982. Considering that the said lot was among the objects of expropriation proceedings, the RTC rendered a partial Decision on August 11, 1982. In that Decision, the RTC approved the Compromise Agreement entered into between the Export Processing Zone Authority (EPZA) and the new registered owners of Lot No. 4673; namely, Jorgea Igot-Soroño, Frisca Booc and Felix Cuizon. In accordance with the approved Compromise Agreement, EPZA would pay P68,070 as just compensation for the expropriation of the subject property. Petitioner acquired title over Lot No. 4673 and the corresponding TCT No. 12788 issued by the Register of Deeds of Lapu-Lapu City on October 13, 1982. On July 29, 1996, private respondents filed a Complaint for Nullity of Documents, Redemption and Damages against petitioner and Jorgea-Igot Soroño et al. The Complaint alleged that herein private respondents had been excluded from the extrajudicial settlement of the estate. It likewise sought the nullification of several documents, including TCT No. 12788 dated October 13, 1992, issued in the name of herein petitioner. On February 17, 1997, petitioner filed a Motion to Dismiss the Complaint on the ground of prescription. This Motion was denied by respondent judge on January 12, 1998. A Motion for Reconsideration thereof was likewise denied on March 31, 1998. On April 30, 1998, petitioner elevated the matter to the CA through a Petition for Certiorari. CA dismissed the Petition. Hence, this recourse. ISSUES: 1. Whether private respondents‘ claim against expropriated property had prescribed. --

YES 2. Whether reconveyance lies against the expropriated property in this case. -- NO HELD: 1. The pertinent provisions of Section 4, Rule 74 of the Rules of Court, are reproduced for easy reference, as follows: Section 4. Liability of distributees and estate. - If it shall appear at any time within two (2) years after the settlement and distribution of an estate in accordance with the provisions of either of the first two sections of this rule, that an heir or other person has been unduly deprived of his lawful participation in the estate, such heir or such other person may compel the settlement of the estate in the courts in the manner hereinafter provided for the purpose of satisfying such lawful participation. And if within the same time of two (2) years, it shall appear that there are debts outstanding against the estate which have not been paid, or that an heir or other person has been unduly deprived of his lawful participation payable in money, the court having jurisdiction of the estate may, by order for that purpose, after hearing, settle the amount of such debts or lawful participation and order how much and in what manner each distributee shall contribute in the payment thereof, and may issue execution, if circumstances require, against the bond provided in the preceding section or against the real estate belonging to the deceased, or both. Such bond and such real estate shall remain charged with a liability to creditors, heirs, or other persons for the full period of two (2) years after such distribution, notwithstanding any transfers of real estate that may have been made. A perusal of the foregoing provision will show that persons unduly deprived of their lawful participation in a settlement may assert their claim only within the two-year period after the settlement and distribution of the estate. This prescription period does not apply, however, to those who had no part in or had no notice of the settlement. Section 4, Rule 74 of the Rules of Court, is not meant to be a statute of limitations. Moreover, by no reason or logic can one contend that an extrajudicial partition, being merely an ex parte proceeding, would affect third persons who had no knowledge thereof. Be that as it may, it cannot be denied, either, that by its registration in the manner provided by law, a transaction may be known actually or constructively. In the present case, private respondents are deemed to have been constructively notified of the extrajudicial settlement by reason of its registration and annotation in the certificate of title over the subject lot. From the time of registration, private respondents had two (2) years or until July 8, 1984, within which to file their objections or to demand the appropriate settlement of the estate. The only exception to the above-mentioned prescription is when the title remains in the hands of the heirs who have fraudulently caused the partition of the subject property or in those of their transferees who cannot be considered innocent purchasers for value. In this regard, title to the property in the present case was no longer in the name of the allegedly fraudulent heirs, but already in that of an innocent purchaser for value—the government. Even assuming that there was in fact fraud on the part of the other heirs, private respondents may proceed only against the defrauding heirs, not against petitioner which had no participation in or knowledge of the alleged fraud. 2. An action for reconveyance resulting from fraud prescribes four years from the discovery of the fraud; such discovery is deemed to have taken place upon the issuance of the certificate of title over the property. Registration of real property is considered a constructive notice to all persons and, thus, the four-year period shall be counted therefrom.

Clearly then, private respondents‘ action for reconveyance based on fraud has already prescribed. Even an action for reconveyance based on an implied or a constructive trust would have already prescribed just the same, because such action prescribes ten (10) years from the alleged fraudulent registration or date of issuance of the certificate of title over the property. The imprescriptibility of an action for reconveyance based on implied or constructive trust applies only when the plaintiff or the person enforcing the trust is in possession of the property. Undisputedly, private respondents are not in possession of the disputed property. Finally, it must be remembered that reconveyance is a remedy of those whose property has been wrongfully or erroneously registered in the name of another. Such recourse, however, cannot be availed of once the property has passed to an innocent purchaser for value. For an action for reconveyance to prosper, the property should not have passed into the hands of an innocent purchaser for value. We find that the property has already been conveyed to the government in appropriate expropriation proceedings, the regularity or validity of which has not been questioned. Petitioner should, therefore, enjoy the security afforded to innocent third persons under our registration laws. Equally important, its title to the property must be rightfully preserved. TKDC Idolor v. Court of Appeals 351 SCRA 399 DOCTRINE: Injunction is a preservative remedy aimed at protecting substantive rights and interests. Before an injunction can be issued, the following requisites must be present: (1) a right in esse or the existence of a right to be protected; and (2) the act, against which the injunction is to be directed, is a violation of such right. Hence, the existence of a violated right is a prerequisite to the granting of an injunction. FACTS: Idolor executed a deed of real estate mortgage with right of extra-judicial foreclosure upon failure to redeem over a property in favor of De Guzman to secure a loan. Upon Idolor‘s failure to pay, De Guzman filed a complaint against Idolor before the Office of the Barangay Captain that resulted in an agreement wherein Idolor is given another ninety days to settle her loan; otherwise, she will execute a deed of sale with the agreement to repurchase without interest within a year. Idolor again failed to pay back her loan so De Guzman filed an extra judicial foreclosure of the real estate mortgage resulting in a public auction where the property was sold to Gumersindo. Idolor filed a complaint with the RTC resulting to the court‘s issuance of a writ of preliminary injunction enjoining the issuance of a final deed of sale and consolidation of ownership of the property in favor of the De Guzmans. Idolor claims that her proprietary right over the property was not yet lost since her right to redeem the land for a year had neither lapsed nor run as the sheriff‘s certificate of sale was null and void. ISSUE: Whether or not the questioned writ of preliminary injunction was issued with grave abuse of

discretion. -- YES HELD: Injunction is a preservative remedy aimed at protecting substantive rights and interests. Before an injunction can be issued, the following requisites must be present: (1) a right in esse or the existence of a right to be protected; and (2) the act, against which the injunction is to be directed, is a violation of such right. Hence, the existence of a violated right is a prerequisite to the granting of an injunction. In this case, Idolor has no more proprietary right to speak of over the foreclosed property to entitle her to the issuance of a writ of injunction. She had one year from the registration of the sheriff‘s sale to redeem the property but she failed to exercise her right; she failed to show sufficient interest or title in the property as her right of redemption had already expired two days before the filing of the complaint. Thus, the de Guzmans are now entitled to a conveyance and possession of the foreclosed property. AMD Cagayan de Oro City Landless Residents v. Court of Appeals 254 SCRA 220 DOCTRINE: 1. Injunction is calculated to preserve or maintain the status quo of things and is generally availed of to prevent actual or threatened acts, until the merits of the case can be heard. Before an injunction can be issued, it is essential that the following requisites be present: a. There must be a right in esse or the existence of a right to be protected; and b. The act against which the injunction is to be directed is a violation of such right. 2. An Original Certificate of Title serves as a concrete and conclusive evidence of an indefeasible title to the property. The certificate of title vests not only ownership over the lot but also the right of possession as a necessary consequence of the right of ownership. FACTS: The subject of the dispute is a 12.82-hectare land located at Cagayan de Oro City. Said parcel of land was formerly classified as a timberland until1956, when the Bureau of Forestry released the said land as alienable and disposable public land. On January 1964, Petitioner Cagayan de Oro City Landless Residents Association, Inc (COCLAI) was authorized by the Bureau of Lands (BOL) to survey the land for purposes of subdivision into residential lots. On March 1964, after conducting an ocular survey, the BOL required COCLAI to file a Miscellaneous Sales Application over the land. The said sales application was held in abeyance by the BOL pending the final outcome of the civil case filed by the Republic of the Philippines and the City of CDO against Benedicta Salcedo for the annulment of an Original Certicate of Title covering the subject land. In this case, COCLAI was a party-intervenor. Meanwhile, the NHA initiated an expropriation proceeding to acquire an area which included the subject land. Petitioner intervened and claimed that instead of being paid through money, it preferred acquisition of residential lots in any housing area of NHA. Upon learning of the pending suit (v. Salcedo case) over the same land, the NHA sought the suspension of the expropriation proceedings. In 1982, the SC finally resolved the pending suit by annulling the

title and declaring the subject lot to be public land. In response to the BOL‘s order to take an inventory of the land, the Regional Land Director of Region 10 informed the Director of Lands that the members of COCLAI were occupying portions of the said lot by virtue of the Survey Authority. In 1983, Presidential Proclamation No. 2292 was issued, reserving the entire subject land for the Slum Improvement and Resettlement (SIR) Project of the NHA. This led to the rejection of the subdivision survey submitted by COCLAI. Eventually, the NHA caused the demolition of the settlement structures constructed by COCLAI members. This prompted petitioner to file a case for forcible entry, which the MTCC decided in petitioner‘s favor. On appeal, the RTC affirmed the MTCC decision. However, during the pendency of the case filed by COCLAI, Special Patent No. 3551 was issued by the President over the entire subject land, and by virtue of which the ROD of CDO issued an Original certificate of Title in the name of NHA. Thus, a day after COCLAI moved for the execution of judgment, the NHA filed a complaint for "Quieting of Title with Application for a Writ of Preliminary Injunction" against COCLAI. The RTC ruled in favour of COCLAI, denying NHA‘s Motion to Dismiss and prayer for the issuance of a preliminary injunction. NHA‘s MR was also denied. On appeal, the CA reversed the RTC decision. The CA ordered the issuance of a writ of preliminary injunction to respect the possession of the NHA over the land in dispute. Hence, this petition. ISSUES: 1. W/N the NHA is entitled to the injunction prayed for. -- YES 2. W/N the NHA has a better right to the possession of the lot involved. -- YES HELD: 1. As an extraordinary remedy, injunction is calculated to preserve or maintain the status quo of things and is generally availed of to prevent actual or threatened acts, until the merits of the case can be heard. As such, injunction is accepted as a strong arm of equity or a transcendent remedy to be used cautiously, as it affects the respective rights of the parties, and only upon full conviction on the part of the court of its extreme necessity. NHA was entitled to the writ of injunction because while the case for forcible entry was pending on appeal, NHA was granted an OCT over the subject land by virtue of the Special Patent issued by the President. In view of this intervening development, NHA filed a complaint for quieting of title before the RTC. Thus, it was only proper for the CA to direct the RTC, where the quieting case was pending, to grant the writ of preliminary injunction to restrain the enforcement of the decision of the MTCC in the forcible entry case as there was a material change in the status of the parties with regard to the said land. 2. The Original Certificate of Title issued to respondent NHA serves as a concrete and conclusive evidence of an indefeasible title to the property. Accordingly, once a decree of registration is issued under the Torrens systems and the one year period from the issuance of the decree of registration has lapsed, without said decree being controverted by any adverse party, the title becomes perfect and cannot later on be questioned. Moreover, the said certificate of title was not controverted by petitioner in a proper

proceeding nor did it show that the issuance of the Original Certificate of Title by the register of deeds to NHA was tainted with bad faith or fraud. Hence, said certificate of title enjoys the presumption of having been issued by the register of deeds in the regular performance of its official duty. The certificate of title vested not only ownership over the lot but also the right of possession as a necessary consequence of the right of ownership. MPF Lucero v. Loot 25 SCRA 687 DOCTRINE: When a final decree has been issued in a land registration case, the issuance of a writ of possession is only a matter of course if nothing in the past has been issued in favor of the registered owner. There is no period of prescription as to the issuance of writ of possession. FACTS: ● Julio Lucero filed and was granted a writ of possession of property (based on a final decree in a land registration proceeding). ○ Although the other party (all surnamed Loot) filed a motion to quash the writ, this was granted by CFI Iloilo‘s Judge Fernan on September 21, 1959. ● The Loots opposed the decision on the ground that there were defects in the reconstitution of the records and that the motion was not under oath. ○ The court dismissed these as trivial arguments. ○ Two motions for reconsideration were also denied. ○ The writ of possession prayed for was issued in favour of Lucero. ● The Loots were very adamant to their opposition. ○ They then went straight to the Supreme Court for an appeal for certiorari. ○ The Loots desperately tried to assert ISSUE: Whether or not the order granting the writ of possession was in accordance with law. -- YES HELD: The order granting the writ of possession was based on a decision promulgated on a land registration case in 1938, which became a final decree on October 29, 1941.
 After the final decree, the issuance of the writ of possession was only a ministerial duty of the court if no writ has been issued to the registered owner yet. The final decree, in effect, immediately empowered the court to enforce the order/judgment/decree. This automatic process is to avoid further delay and inconvenience to a successful land registration litigant if he were compelled to commence another action to secure possession. Furthermore, there is no period of prescription as to the issuance of a writ of possession. The writ may be issued not only against the person who has been defeated in a registration case, but also against anyone adversely occupying the land or any portion of the land. Even fraud shall not be a bar to the issuance of the writ of possession, which necessarily implied the delivery of possession of the land. As to the questions of fact raised by the Loots, the SC can do nothing. These must be raised at the CA of appeals; otherwise, the parties contesting the facts are deemed to have waived the opportunity to question the correctness of the findings.

AMDG Vencilao v. Vano 182 SCRA 492 DOCTRINE: Writ of Possession may be issued not only against the party defeated in the land registration case but also against anyone unlawfully and adversely occupying the land, or any portion thereof during the land registration proceedings up to issuance of final decree FACTS: (NOTE: Three consolidated cases) First case: Juan Reyes applied for the registrations over parcels of land, however, it was argued that Juan Reyes was not the lawful owner of a portion of the parcels that was registered. Second case: Bernardina Vda. de Lupso was the administratix over the lot subject in controversy. When she died, several TCTs were issued to Pedro Lupso and several persons. As a result, several writs of possession were also issued in their favor. When the writs were being served by the sheriff, the petitioners refused to vacate and even executed guerilla-like tactics as their sign of refusal. A petition for contempt was filed by one of the registered owners against the petitioners to which the court granted. The petitioners were contending that the writs of possession should not be issued in the first place since there were no complaints of forcible entry, unlawful detainer or recovery of ownership and possession that were filed against them. The petitioners also claim that they were not defeated oppositors in the first case. Third case: As a result of the writ of possessions being issued against the petitioners, the judge also issued writs of demolition. The petitioners opposed the validity of these writs. ISSUES: 1. 2.

Whether or not the writs of possession were valid? -- YES Whether or not the writ of demolition was valid? -- YES

HELD: The issuance of the TCTs carried with it the delivery of possession which is an inherent element of the right of ownership. The issuance of the writ of possession is, therefore, sanctioned by existing laws in this jurisdiction and by the generally accepted principle upon which the administration of justice rests. A writ of possession may be issued not only against the person who has been defeated in a registration case but also against anyone unlawfully and adversely occupying the land or any portion thereof during the land registration proceedings up to the issuance of the final decree. Applying this to the case, from the records, though the petitioners were not the defeated oppositors over the land registration case, they were unlawfully occupying the lots although writs of possession were already being served to them. Having said this, the writs of possession were properly issued against them. The writs of demotion were also issued properly. These writs were described by previous cases as a complement to the writs of possession. Without it, the writ of possessions would be ineffective. The writs are issued to prevent the lawful owner from being deprived of his lots. GCG German Management and Services v. CA 177 SCRA 495

DOCTRINE: Actual possessors can commence a forcible entry case because ownership is not in issue. Forcible entry is merely a quieting process and never determines the actual title to an estate. Title is not involved, only actual possession FACTS: Spouses Cynthia Cuyegkeng Jose and Manuel Rene Jose, residents of Pennsylvania, Philadelphia, USA are the owners of a parcel of land situated in Sitio Inarawan, San Isidro, Antipolo, Rizal. The land was originally registered in the Office of the Register of Deeds of Rizal, pursuant to a Homestead Patent granted by the President of the Philippines. Spouses Jose executed a special power of attorney authorizing petitioner German Management Services to develop their property into a residential subdivision. Consequently, petitioner obtained development permit from the Human Settlements Regulatory Commission for said development. Finding that part of the property was occupied by private respondents and twenty other persons, petitioner advised the occupants to vacate the premises but the latter refused. Nevertheless, petitioner proceeded with the development of the subject property which included the portions occupied and cultivated by private respondents. Private respondents filed an action for forcible entry against petitioner before the Municipal Trial Court of Antipolo, Rizal. They alleged that they are mountainside farmers of Sitio Inarawan, San Isidro, Antipolo, Rizal and members of the Concerned Citizens of Farmer's Association; that they have occupied and tilled their farmholdings some twelve to fifteen years prior to the promulgation of P.D. No. 27; that during the first week of August 1983, petitioner, under a permit from the Office of the Provincial Governor of Rizal, was allowed to improve the Barangay Road at Sitio Inarawan, San Isidro, Antipolo, Rizal at its expense, subject to the condition that it shall secure the needed right of way from the owners of the lot to be affected; that, petitioner deprived private respondents of their property without due process of law by: (1) forcibly removing and destroying the barbed wire fence enclosing their farmholdings without notice; (2) bulldozing the rice, corn fruit bearing trees and other crops of private respondents by means of force, violence and intimidation, in violation of P.D. 1038 and (3) trespassing, coercing and threatening to harass, remove and eject private respondents from their respective farmholdings in violation of P.D. Nos. 316, 583, 815, and 1028. Municipal Trial Court dismissed private respondents' complaint for forcible entry. On appeal, the Regional Trial Court of Antipolo, Rizal, Branch LXXI sustained the dismissal by the Municipal Trial Court. Private respondents then filed a petition for review with the Court of Appeals. The court gave due course to their petition and reversed the decisions of the Municipal Trial Court and the Regional Trial Court. The Appellate Court held that since private respondents were in actual possession of the property at the time they were forcibly ejected by petitioner, private respondents have a right to commence an action for forcible entry regardless of the legality or illegality of possession. Petitioner moved to reconsider but the same was denied by the Appellate Court. Hence, this recourse. ISSUE: Whether or not private respondents are entitled to file a forcible entry case against petitioner? -YES, they are entitled to file a forcible entry case.

HELD: Since private respondents were in actual possession of the property at the time they were forcibly ejected by petitioner, private respondents have a right to commence an action for forcible entry regardless of the legality or illegality of possession. Private respondents, as actual possessors, can commence a forcible entry case against petitioner because ownership is not in issue. Forcible entry is merely a quieting process and never determines the actual title to an estate. Title is not involved, only actual possession. It is undisputed that private respondents were in possession of the property and not the petitioners nor the spouses Jose. Although the petitioners have a valid claim over ownership this does not in any way justify their act of ―forcible entry. It must be stated that regardless of the actual condition of the title to the property the party in peaceable quiet possession shall not be turned out by a strong hand, violence or terror. Thus, a party who can prove prior possession can recover such possession even against the owner himself. Whatever may be the character of his possession, if he has in his favor priority in time, he has the security that entitles him to remain on the property until he is lawfully ejected by a person having a better right by accion publiciana or accion reivindicatoria. The doctrine of self help, which the petitioners were using to justify their actions, are not applicable in the case because it can only be exercised at the time of actual or threatened dispossession which is absent in the case at bar (in fact they are the ones who are threatening to remove the respondents with the use of force). Article 536 basically tells us that the owner or a person who has a better right over the land must resort to judicial means to recover the property from another person who possesses the land. When possession has already been lost, the owner must resort to judicial process for the recovery of property. As clearly stated in Article 536- ―In no case may possession be acquired through force or intimidation as long as there is a possessor who objects thereto. He who believes that he has an action or right to deprive another of the holding of a thing must invoke the aid of the competent court, if holder should refuse to deliver the thing. VCL IV Caisip v. People 36 SCRA 17 Doctrine: Art. 429 cannot be used as a defense of the petitioner to justify their action. The order to vacate was until June 26 (or 20 days from the execution of the decision.) On June 17, the spouses REMAINED in possession of the said lot. At the very least the owner of the hacienda is just a co-possessor of the land, thus the spouses still had rights over it. FACTS: Gloria Cabalag is the wife of Marcelino Guevarra who cultivated a parcel of land known as Lot 105-A of Hacienda Palico situated in Sitio Bote-bote, Barrio Tampisao, Nasugbu, Batangas. The said parcel of land used to be tenanted by the deceased father of the Cabalag. Hacienda Palico is owned by Roxas y Cia. and administered by Antonio Chuidian. The overseer of the said hacienda is Felix Caisip, one of the accused herein. Before the present case happened, Marcelino Guevarra filed an action with the Court of Agrarian Relations seeking recognition as a lawful tenant of Roxas y Cia. over lot No. 105-A of Hacienda Palico. In a decision of the Court of Agrarian Relations, it declared that Guevarra is not a tenant on the said parcel of land. Then after, Roxas y Cia filed a forcible entry case against Guevarra. The court decided in favour of Roxas y Cia and issued a writ of execution.

The return of the writ showed that possession of Lot 105-A was turned over to the owner thru Caisip and that spouses Guevarra and Cabalag were given 20 days from June 6, 1959 to vacate the premises. It also appears in the record that due to the tenacious attitude of Cabalag, Caisip sought the help of policemen Federico Villadelrey and Ignacio Rojales. On June 17, 1959, Gloria Cabalag was seen weeding the portion of Lot 105-A which was a ricefield. Caisip approached her and bade her to leave but Cabalag refused to do so claiming that she and her husband has a right over the property. She having stuck to this attitude, even when he threatened to call the police, Caisip went to his co-defendants, Sgt. Rojales and Cpl. Villadelrey, both of the local police, who were some distance away, and brought them with him. Gloria Cabalag Version: Rojales told her, who was then in a squatting position, to stop weeding. As she insisted on her right to stay in said lot, Rojales grabbed her right hand and, twisting the same, wrested therefrom the trowel she was holding. Thereupon, Villadelrey held her left hand and, together with Rojales, forcibly dragged her, which resulted for her dress to torn. Caisip et. al Version: Upon being asked by the policemen to stop weeding and leave the premises, Cabalag, not only refused to do so, but, also, insulted them, as well as Caisip. According to the defense, she was arrested because of the crime of slander then committed by her. Rojales and Villadelrey, moreover, testified that, as they were heading towards the barrio of Camachilihan, Gloria proceeded to tear her clothes. Due to the aforementioned incidents, A case filed against Caisip et al. for Grave Coercion, (Petitioners Caisip et al. also filed grave coercion and unjust vexation against Gloria Cabalag after 8 days) One of their defences was ART. 429 (including the doctrine of self help.) TC Ruled in favor of Gloria Cabalag and that Caisip et al. are guilty of Grave Coercion. CA Upheld the TC Ruling. *Caisip et al. argued in the SC that the Court of Appeals erred in not finding that their acts are justified under Article 429 of the Civil Code; ―The owner or lawful possessor of a thing has the right to exclude any person from the enjoyment and disposal thereof. For this purpose, he may use such force as may be reasonably necessary to repel or prevent an actual or threatened unlawful physical invasion or usurpation of his property.‖ ISSUE: Whether Article 429 of the Civil Code applies in the present case. -- NO HELD: No, Article 429 is inapplicable, Cabalag was given 20 days from June 6, 1959 within which to vacate the premises. Cabalag did not, on June 17, 1959 — or within said period — invade or usurp said lot. She had merely remained in possession thereof, even though the hacienda owner may have become its co-possessor. Appellants did not ―repel or prevent in actual or threatened . . . physical invasion or usurpation.‖ They expelled Gloria from a property of which she and her husband were in possession. It is urged, that, by weeding and refusing to leave Lot 105-A, Gloria had committed a crime in the presence of the policemen, despite the aforementioned 20-day period, which, appellants claim, the sheriff had no authority to grant. This contention is manifestly untenable, because: (1) said period was granted in the presence of the hacienda owner‘s representative, appellant Caisip, who, by not objecting thereto, had impliedly consented to or ratified the act performed by

the sheriff; 2) Gloria and her husband were thereby allowed to remain, and had, in fact, remained, in possession of the premises, perhaps together with the owner of the hacienda or his representative, Caisip; (3) the act of removing weeds from the ricefield was beneficial to its owner and to whomsoever the crops belonged, and, even if they had not authorized it, does not constitute a criminal offense; and (4) although Gloria and her husband had been sentenced to vacate the land, the judgment against them did not necessarily imply that they, as the parties who had tilled it and planted thereon, had no rights, of any kind whatsoever, in or to the standing crops, inasmuch as ―necessary expenses shall be refunded to every possessor,‖ and the cost of cultivation, production and upkeep has been held to partake of the nature of necessary expenses. It is, accordingly, clear that appellants herein had, by means of violence, and without legal authority therefor, prevented the complainant from ―doing something not prohibited by law,‖ (weeding and being in Lot 105-A), and compelled her ―to do something against‖ her will (stopping the weeding and leaving said lot), ―whether it be right or wrong,‖ thereby taking the law into their hands, in violation of Art. 286 of the Revised Penal Code, Grave Coercion. FXRL Heirs of Vencilao, Sr. v. CA 288 SCRA 574 DOCTRINE: Tax declarations and receipts do not by themselves conclusively prove title to the land. They only constitute prima facie evidence of ownership or possession. Where the certificate of title is in the name of the vendor when the land is sold, the vendee for value has the right to rely on what appears on the face of the title. He is under no obligation to look beyond the certificate and investigate the title of the vendor appearing on the face of the certificate. However, the vendee is required to make the necessary inquiries if there is anything in the certificate of title which indicates any cloud or vice in the ownership of the property. Otherwise, his mere refusal to believe that such defect exists, he will not be deemed a purchaser in good faith should such title indeed be defective. FACTS: Both petitioner and respondent claim ownership over a parcel of land located at Bohol. Petitioner claims ownership by virtue of inheritance from their father who exercised uninterrupted possession over the land, declared the property for taxation purpose and religiously paid the real estate tax. The respondent claims ownership as registered owner of said lot as a portion of the lot owned by Pedro Luspo who mortgaged his land to PNB and subsequently was foreclosed. The respondents won as the highest bidder and they became the owner of that portion of land. The RTC ruled in favor to the petitioners holding that they had validly acquired the land by prescription. Although a Torrens Title is indefeasible and not subject for prescription it is not so when the respondents purchased the land from PNB with prior knowledge that the land was in possession of the petitioner‘s father. The CA reversed the court ruling and declared the petitioners as the true owners of the property. ISSUE:

W/N land registered under the Torrens system can be acquired by prescription -- NO HELD: No. The previously registered land may not be acquired by prescription. The Property Registration Decree states that no land under the Torrens System may be acquired through prescription. Such title is indefeasible. The tax declarations and tax receipts presented by petitioners as evidence of ownership prevail over respondents‘ certificate of title which is an incontrovertible proof of ownership. Tax declarations and receipts do not by themselves conclusively prove title to the land. They only constitute prima facie evidence of ownership or possession. Petitioners are wrong to contend that the prior knowledge of the respondents on the possession of the petitioner‘s father defies the Torrens title‘s imprescriptibility because there is no flaw on the title when they purchased it from PNB that was the registered owner of the land. The general rule is that where the certificate of title is in the name of the vendor when the land is sold, the vendee for value has the right to rely on what appears on the face of the title. He is under no obligation to look beyond the certificate and investigate the title of the vendor appearing on the face of the certificate. However, the vendee is required to make the necessary inquiries if there is anything in the certificate of title which indicates any cloud or vice in the ownership of the property. Otherwise, his mere refusal to believe that such defect exists, or his willful closing of his eyes to the possibility of the existence of a defect in his vendor‘s title, will not make him an innocent purchaser for value if it afterwards develops that the title was in fact defective, and it appears that he had such notice of the defect as would have led to its discovery had he acted with that measure of precaution which may reasonably be required of a prudent man in a like situation. In the instant case, there is nothing from the records showing that the title of PNB, the vendor, was flawed. Petitioners not only failed to substantiate their claim of acquisitive prescription as basis of ownership but they also failed to allege, and much less adduce, any evidence that there was a defect in the title of PNB. In the absence of such evidence, the presumption leans towards the validity of the vendor‘s title. RSDM Oclarit v. CA 233 SCRA 239 DOCTRINE: Although it is true that what defines a piece of land is not the area mentioned in its description but the boundaries therein laid down, in controversial cases as in this case where there appears to be an overlapping of boundaries, the actual sizeof the property gains importance. FACTS: The late Juan Oclarit, petitioners‘ predecessor-in-interest, allegedly purchased from Martin Macalos a parcel of unregistered land located in Antipolo, Garcia-Hernandez, Bohol, with no permanent landmarks . This particular land did not have specified boundaries, as it was only

indicated that the borders were a brook, lands of Gales, and another of Baja.Then he bought five more parcels of land located in Antipolo and Ulbujan, also in Garcia-Hernandez, Bohol, from Dalmacio Gales. The heirs of Oclarit filed an action for the quieting of title and damages against respondent Balasabas. The complaint alleged that private respondent entered the properties subject of the action. Failing to work on the area planted to palay, private respondent climbed the coconut trees, replaced the "J.O." markings on the trees with "F.G.", representing Felipa Gales, his mother, and caused to be recorded in the cadastral survey of the land the name of Felipa Gales as claimant against Juan Oclarit. The heirs of Oclarit considered the acts of private respondent as having cast a cloud of doubt over their title to the property and therefore deprived them of the enjoyment of the fruits of the coconut trees. Petitioners further alleged that the late Juan Oclarit, from the time of the acquisition of said properties, had exercised dominion and ownership thereon openly, peacefully, adversely and uninterruptedly. It was also claimed even after the death of Oclarit they still enjoyed the property. Respondent Balasabas claims to have actually and lawfully possessed the disputed parcels of land "since time immemorial". The first parcel of land was owned by his mother, Felipa Gales, by virtue of inheritance, and declared in her name under Tax Declaration; while the second parcel of land was acquired by him from his own mother as evidenced by a deed of absolute sale and which he declared in his name under Tax Declaration. In addition, respondent likewise alleged possession of the parcels of land openly, peacefully, adversely and continuously without disturbance from any party until he was molested by the heirs of Oclarit. Trial court appointed an Assessor (commissioner) for the purpose of determining whether the lands described in the complaint overlapped with any of those lands claimed by respondents‘. Trial court dismissed the complaint. CA affirmed. ISSUE: Whether or not the heirs of Oclarit are lawfully entitled to the subject properties. -- NO HELD: Had the petitioners been in possession of solid evidence that the parcels of land they are claiming are "alien" or "foreign" to those declared by private respondent as his, they should have questioned the commissioner‘s report which was based on the relocation survey and ocular inspection which were conducted in their presence. Moreover, petitioners‘ claim that their property is different from those of private respondent‘s is indeed antithetical to their filing of the complaint for quieting of title — there would not have been any basis for claiming that private respondent cast a cloud of doubt to their title over their two parcels of land. The deed of sale wherein Martin Macalos conveyed to Oclarit a parcel of land did not even indicate with particularity the area of the land covered thereby. This explains why they indiscriminately pointed at boundaries which are even beyond what could have been bought by Oclarit. Although it is true that what defines a piece of land is not the area mentioned in its description but the boundaries therein laid down, in controversial cases as in this case where there appears to be an overlapping of boundaries, the actual size of the property gains importance. Thus, the lower court correctly stressed that it would have done petitioners some good had they correctly specified even in their tax declarations the areas of the land they were claiming. It is well settled that anyone who claims that he has a better right to the property, must prove both ownership and identity of the said property. An area delimited by boundaries properly identifies a parcel of land.

With regard to tax declarations as bases for claim of ownership, petitioners capitalize on what was obviously an obiter that no one in his right mind would be continuously paying taxes for property that is not in his actual possession. On the contrary, any person who claims ownership by virtue of tax declarations must also prove he is in actual possession of the property. Thus, proof that the property involved had been declared for taxation purposes did not constitute proof of possession, nor is it proof of ownership in the absence of the claimant‘s actual possession of said property. MRAM Cutanda v. Heirs of Roberto Cutanda 335 SCRA 418 DOCTRINE: Under the Code of Civil Procedure, therefore, ten years of actual adverse possession was required, regardless of how such occupancy may have commenced or continued, before possession ripened into full and complete title over the land. FACTS: Private respondents brought an action for recovery of possession, accounting and damages against petitioners in the RTC of Tagbilaran City. They alleged that in the 1900‘s, their grandfather, Roberto Cutanda, owned two parcels of land in Bohol. Tax declarations were used by them as evidence. Upon Roberto‘s death, these lands were inherited by his children, namely: Doque, Diego, Pedro, Andres, and Anastacia. Except for Doque who stayed in Bohol and administered the lands, all of Roberto‘s children established residence in Leyte. In 1987, they returned to Bohol to personally work the inherited lands. Their plan, however, was frustrated as petitioners, who were occupying the lands, refused to leave. Private respondent thus prayed that each be declared owner of 1/5 of the subject real properties and that petitioners be ordered to return to them said properties. The petitioners, on the other hand averred that the lands were owned by their uncle Anastacio who died without children. As such, petitioners, who were nieces and nephews of Anastacio occupied and cultivated the land. The petitioners were claiming rightful ownership of the land for having openly, contiguously, adversely, and continuously possessing the land for about 55 years. ISSUE: Whether or not private respondents‘ right over the land has prescribed? HELD: The Court held in the affirmative. The Civil Code provides that title to land by prescription is acquired if the land has been in open, continuous, adverse possession and occupancy of the land over 10 years. Under the Code of Civil Procedure, therefore, ten years of actual adverse possession was required, regardless of how such occupancy may have commenced or continued, before possession ripened into full and complete title over the land. Applying this to the present case, by 1943, ten years after his possession of the subject parcel of land had begun, Anastacio Cutanda became owner of the land in question through acquisitive prescription. Hence, the rightful owner of the land is the petitioners. FMM

Sps. Luis Cruz v. Sps. Alejando Fernando, Sr. G.R. No. 145470 DOCTRINE: Occupation of a property merely through the tolerance of the owners could neither ripen into ownership nor operate to bar any action by the subsequent owners to recover absolute possession thereof. FACTS: Spouses Luis V. Cruz and Aida Cruz (the ―Spouses Cruz‖) are occupants of the front portion of a 710-square meter property located in Sto. Cristo, Baliuag, Bulacan. On 21 October 1994, Spouses Alejandro Fernando, Sr. and Rita Fernando (the ―Spouses Fernando‖) filed before the Regional Trial Court (the ―RTC‖) a complaint for accion publiciana against the Spouses Cruz, demanding the latter to vacate the premises and to pay the amount of P500.00 a month as reasonable rental for the use thereof. The Spouses Fernando alleged that they are owners of the property having bought the same from Spouses Clodualdo and Teresita Glorioso (the ―Gloriosos‖) pursuant to a Deed of Sale dated 9 March 1987. They also alleged that the Gloriosos offered to sell to the Spouses Cruz the rear portion of the property but the transaction did not materialize due to the latter‘s failure to exercise their option and for this reason, the Spouses Fernando were able to buy the whole property. For their part, the Spouses Cruz argued that the Kasunduan dated 6 August 1983 they made with the Gloriosos is a perfected contract of sale and that the agreement has already been ―partially consummated‖ as they already relocated their house from the rear portion of the lot to the front portion that was sold to them. After due proceedings, the RTC rendered a decision on 3 April 1998 in favor of the Spouses Fernando and ordered the Spouses Cruz to vacate the property and pay the rent beginning from 2 October 1994 when the case was filed before the RTC. The Spouses Cruz appealed the RTC decision but the Court of Appeals affirmed it on 3 October 2000. Thus, the Spouses Cruz herein petition before the Supreme Court. The Spouses Cruz argued that the Kasunduan is a perfected contract of sale and thus they have ownership over the property. ISSUE: Whether the Spouses Cruz have a right of ownership over the property. -- NO HELD: The SC held that the terms and conditions of the Kasunduan show that it is a contract to sell and that a contract of sale is yet to be consummated and thus ownership of the property remained in the Gloriosos. The SC also held that the Spouses Cruz have no superior right of ownership or possession to speak of. Their occupation of the property was merely through the tolerance of the owners. Evidence on record shows that the Spouses Cruz and their predecessors were able to live and build their house on the property through the permission and kindness of the previous owner, Pedro Hipolito, who was their relative, and subsequently, Teresita Glorioso, who is also their relative. They have no title or, at the very least, a contract of lease over the property. Based as

it was on mere tolerance, the Spouses Cruz possession could neither ripen into ownership nor operate to bar any action by the Spouses Fernando to recover absolute possession thereof.

RIGHTS OF ACCESSION (ART. 440-475) Accession Industrial RGGM Depra v. Dumlao 136 SCRA 475 DOCTRINE: The owner of land on which improvement was built by another in good faith is entitled to removal of improvement only after land owner has opted to sell the land and the builder refused to pay for the same. where the land‘s value is considerably more than the improvement, the landowner cannot compel the builder to buy the land. In such event, a ―forced lease‖ is created and the court shall fix the terms thereof in case the parties disagree thereon. FACTS: Francisco Depra, is the owner of a parcel of land registered, situated in the municipality of Dumangas, Iloilo. Agustin Dumlao, defendant-appellant, owns an adjoining lot. When Dumlao constructed his house on his lot, the kitchen thereof had encroached on an area of thirty four (34) square meters of Depra‘s property, After the encroachment was discovered in a relocation survey of Depra‘s lot made on November 2, 1972, his mother, Beatriz Depra after writing a demand letter asking Dumlao to move back from his encroachment, filed an action for Unlawful Detainer. Said complaint was later amended to include Depra as a party plaintiff. After trial, the Municipal Court found that Dumlao was a builder in good faith, and applying Article 448 of the Civil Code. Depra did not accept payment of rentals so that Dumlao deposited such rentals with the Municipal Court. In this case, the Municipal Court, acted without jurisdiction, its Decision was null and void and cannot operate as res judicata to the subject complaint for Quieting of Title. The court conceded in the MCs decision that Dumlao is a builder in good faith. ISSUE: Whether or not the factual situations of Dumlao and Depra conform to the juridical positions respectively defined by law, for a "builder in good faith" under Article 448, a "possessor in good faith" under Article 526 and a "landowner in good faith' under Article 448? HELD: Owner of the land on which improvement was built by another in good faith is entitled to removal of improvement only after landowner has opted to sell the land and the builder refused to pay for the same. Res judicata doesn‘t apply wherein the first case was for ejectment and the other was for quieting of title. ART. 448. The owner of the land on which anything has been built sown or planted in good faith, shall have the right to appropriate as his own the works, sowing or planting, after payment of the indemnity provided for in articles 546 and 548, or to oblige the one who built or planted to pay the price of the land, and the one who sowed, the proper rent. However, the builder or planter cannot be obliged to buy the land if its value is considerably more than that of the building or trees. In such case, he shall pay reasonable rent, if the owner of the land does not choose to appropriate the building or trees after proper indemnity. The parties shall agree upon the terms of the lease and in case of disagreement, the court shall fix the terms thereof.

MCSS Sarmiento v. Agana 129 SCRA 122 FACTS: While one Ernesto was still courting his wife, the latter's mother had told him the couple could build a residential house on a certain lot. They constructed a residential house. Subsequently, the land was sold to petitioner Sarmiento, who asked Ernesto and his wife to vacate. Sarmiento filed an Ejectment suit against them. In the evidentiary hearing, Sarmiento submitted the deed of sale of the land which showed the price to be PHP15,000. On the otherhand, Ernesto testified that the residential house then cost PHP30,000-40,000, which was not questioned. The MTC found that Ernesto was a builder in good faith and the house had a value of PHP20,000. When the case was elevated, the CFI of Pasay ordered Sarmiento to exercise his option, to reimburse Ernesto for the sum of the house or allow them to purchase the land, within sixty days. Upon expiration of the period, Ernesto was allowed to deposit the sum of PHP25,000 with the Court as the purchase price for the land. ISSUE: WON private respondents are builders in good faith. -- YES HELD: Ernesto and his wife were builders in good faith in view of the peculiar circumstance under which they had constructed the residential house. As far as they knew, the land was owned by Ernesto's mother-in-law, and could reasonably be expected to later on give them the land. The owner of the building erected in good faith on a land owned by another, is entitled to retain the possession of the land until he is paid the value of his building. The owner of the land, has the option either to pay for the building or to sell his land to the owner of the building. But he cannot, as Sarmiento did, refuse both to pay for the building and to sell the land and compel the owner of the building to remove it from the land where it is erected. NKVS Nuguid v. CA 452 SCRA 243 FACTS: Pedro P. Pecson owned a commercial lot located at Quezon City, on which he built a four-door two-storey apartment building. For failure to pay realty taxes, the lot was sold at public auction by the City Treasurer of Quezon City to Nepomuceno, who in turn sold it for P103,000 to the spouses Juan and Erlinda Nuguid. Pecson challenged the validity of the auction sale before the RTC of Quezon City. In its Decision, the RTC upheld the spouses‘ title but declared that the four-door two-storey apartment building was not included in the auction sale. This was affirmed in toto by the Court of Appeals and thereafter by this Court, in its Decision dated May 25, 1993, in G.R. No. 105360 entitled Pecson v. Court of Appeals.

Nuguid spouses moved for delivery of possession of the lot and the apartment building. In its Order of November 15, 1993, the trial court, relying upon Article 546 of the Civil Code, ruled that the Spouses Nuguid were to reimburse Pecson for his construction cost of P53,000, following which, the spouses Nuguid were entitled to immediate issuance of a writ of possession over the lot and improvements. In the same order the RTC also directed Pecson to pay the same amount of monthly rentals to the Nuguids as paid by the tenants occupying the apartment units or P21,000 per month from June 23, 1993, and allowed the offset of the amount of P53,000 due from the Nuguids against the amount of rents collected by Pecson from June 23, 1993 to September 23, 1993 from the tenants of the apartment. After conducting a hearing, the lower court issued an Order dated, directing the spouses to pay the sum of P1,344,000 as reimbursement of the unrealized income of Pecson for the period beginning November 22, 1993 up to December 1997. The sum was based on the computation of P28,000/month rentals of the four-door apartment Pecson filed a petition for review before this Court. The Court handed down the decision remanding to the trial court for it to determine the current market value of the apartment building on the lot. The value so determined shall be forthwith paid by Spouses Juan and Erlinda Nuguid to Pedro Pecson otherwise the petitioner shall be restored to the possession of the apartment building until payment of the required indemnity. On the basis of this Court‘s decision, Pecson filed a Motion to Restore Possession and a Motion to Render Accounting, praying respectively for restoration of his possession over the subject 256-square meter commercial lot and for the spouses Nuguid to be directed to render an accounting under oath, of the income derived from the subject four-door apartment from November 22, 1993 until possession of the same was restored to him. ISSUE: Whether or not the petitioners should reimburse the respondent for the improvements of the building -- YES HELD: It is not disputed that the construction of the four-door two-storey apartment, subject of this dispute, was undertaken at the time when Pecson was still the owner of the lot. When the Nuguids became the uncontested owner of the lot on June 23, 1993, by virtue of entry of judgment of the Court‘s decision, dated May 25, 1993, in G.R. No. 105360, the apartment building was already in existence and occupied by tenants. In its decision dated May 26, 1995 in G.R. No. 115814, the Court declared the rights and obligations of the litigants in accordance with Articles 448 and 546 of the Civil Code. These provisions of the Code are directly applicable to the instant case. Under Article 448, the landowner is given the option, either to appropriate the improvement as his own upon payment of the proper amount of indemnity or to sell the land to the possessor in good faith. Relatedly, Article 546 provides that a builder in good faith is entitled to full reimbursement for all the necessary and useful expenses incurred; it also gives him right of retention until full reimbursement is made. While the law aims to concentrate in one person the ownership of the land and the improvements thereon in view of the impracticability of creating a state of forced co-ownership,it

guards against unjust enrichment insofar as the good-faith builder‘s improvements are concerned. The right of retention is considered as one of the measures devised by the law for the protection of builders in good faith. Its object is to guarantee full and prompt reimbursement as it permits the actual possessor to remain in possession while he has not been reimbursed (by the person who defeated him in the case for possession of the property) for those necessary expenses and useful improvements made by him on the thing possessed. Accordingly, a builder in good faith cannot be compelled to pay rentals during the period of retention nor be disturbed in his possession by ordering him to vacate. In addition, as in this case, the owner of the land is prohibited from offsetting or compensating the necessary and useful expenses with the fruits received by the builder-possessor in good faith. Otherwise, the security provided by law would be impaired. This is so because the right to the expenses and the right to the fruits both pertain to the possessor, making compensation juridically impossible; and one cannot be used to reduce the other. The text of the decision in G.R. No. 115814 expressly exempted Pecson from liability to pay rentals, for we found that the Court of Appeals erred not only in upholding the trial court‘s determination of the indemnity, but also in ordering him to account for the rentals of the apartment building from June 23, 1993 to September 23, 1993, the period from entry of judgment until Pecson‘s dispossession. As pointed out by Pecson, the dispositive portion of our decision in G.R. No. 115814 need not specifically include the income derived from the improvement in order to entitle him, as a builder in good faith, to such income. The right of retention, which entitles the builder in good faith to the possession as well as the income derived therefrom, is already provided for under Article 546 of the Civil Code. AMPS Sulo ng Nayon v. Nayong Filipino G.R. No. 170923 DOCTRINE: Article 448 does not apply to a lessee-BPS. The basis of the application of Article 448 is the BPS‘s (good faith) belief of a claim of title. FACTS: Respondent Nayong Pilipino Foundation, a government-owned and controlled corporation, is the owner of a parcel of land in Pasay City, known as the Nayong Pilipino Complex. Petitioner Philippine Village Hotel, Inc. (PVHI), formerly called Sulo sa Nayon, Inc., is a domestic corporation duly organized and existing under Philippine laws. Contract of lease: respondent leased a portion of the Nayong Pilipino Complex, consisting of 36,289 square meters, to petitioner Sulo sa Nayon, Inc. for the construction and operation of a hotel building, to be known as the Philippine Village Hotel. The lease was for an initial period of 21 years, or until May 1996. The contract was renewed for another 25 years or until 2021. Beginning January 2001, petitioners defaulted in the payment of their monthly rental. Respondent repeatedly demanded petitioners to pay the arrears and vacate the premises. Hence, respondent filed a complaint for unlawful detainer before the MeTC of Pasay City. ISSUE: Should the rules on accession, as found in Articles 448 and 546 of the Civil Code, apply to the improvements made by the lessee? -- NO. Article 1678 applies. HELD:

The late Senator Arturo M. Tolentino, a leading expert in Civil Law, explains: This article [Article 448] is manifestly intended to apply only to a case where one builds, plants, or sows on land in which he believes himself to have a claim of title, and not to lands where the only interest of the builder, planter or sower is that of a holder, such as a tenant. In the case at bar, petitioners have no adverse claim or title to the land. In fact, as lessees, they recognize that the respondent is the owner of the land. What petitioners insist is that because of the improvements, which are of substantial value, that they have introduced on the leased premises with the permission of respondent, they should be considered builders in good faith who have the right to retain possession of the property until reimbursement by respondent. We affirm the ruling of the CA that introduction of valuable improvements on the leased premises does not give the petitioners the right of retention and reimbursement which rightfully belongs to a builder in good faith. Otherwise, such a situation would allow the lessee to easily "improve" the lessor out of its property. We reiterate the doctrine that a lessee is neither a builder in good faith nor in bad faith that would call for the application of Articles 448 and 546 of the Civil Code. His rights are governed by Article 1678 of the Civil Code, which reads: Art. 1678. If the lessee makes, in good faith, useful improvements which are suitable to the use for which the lease is intended, without altering the form or substance of the property leased, the lessor upon the termination of the lease shall pay the lessee one-half of the value of the improvements at that time. Should the lessor refuse to reimburse said amount, the lessee may remove the improvements, even though the principal thing may suffer damage thereby. He shall not, however, cause any more impairment upon the property leased than is necessary. With regard to ornamental expenses, the lessee shall not be entitled to any reimbursement, but he may remove the ornamental objects, provided no damage is caused to the principal thing, and the lessor does not choose to retain them by paying their value at the time the lease is extinguished. Under Article 1678, the lessor has the option of paying one-half of the value of the improvements which the lessee made in good faith, which are suitable for the use for which the lease is intended, and which have not altered the form and substance of the land. On the other hand, the lessee may remove the improvements should the lessor refuse to reimburse. Petitioners argue that to apply Article 1678 to their case would result to sheer injustice, as it would amount to giving away the hotel and its other structures at virtually bargain prices. They allege that the value of the hotel and its appurtenant facilities amounts to more than two billion pesos, while the monetary claim of respondent against them only amounts to a little more than twenty six-million pesos. Thus, they contend that it is the lease contract that governs the relationship of the parties, and consequently, the parties may be considered to have impliedly waived the application of Article 1678. We cannot sustain this line of argument by petitioners. Basic is the doctrine that laws are deemed incorporated in each and every contract. Existing laws always form part of any contract. Further, the lease contract in the case at bar shows no special kind of agreement between the parties as to how to proceed in cases of default or breach of the contract. KGS

Pecson v. CA 244 SCRA 407 DOCTRINES: 1. Article 448 of the Civil Code does not apply to a case where the owner of the land is the builder, sower, or planter who then later loses ownership of the land by sale or donation. 2. The provision of Art. 448 on indemnity may be applied by analogy to a case where one loses the ownership of the land on which he earlier built an apartment. FACTS: Pecson was the owner of a commercial lot on which he built a 4-door, 2-storey apartment. For his failure to pay realty tax on said property, the lot was sold at public auction by the City Treasurer of Quezon City to Nepomuceno, who in turn the property to Sps. Nuguid. Pecson questioned the validity of the auction sale. RTC then, dismissed the complaint and ruled that the apartment building was included in the sale. It reached the SC, but the same was denied. Thereafter, Sps. Nuguid filed a Motion for delivery of possession, which was granted by the trial court. Pecson contested. The Court of Appeals affirmed in part the order of the trial court citing Article 448 of the Civil Code. CA ordered that Pecson should be indemnified with the construction cost of the apartment. ISSUE: Is Article 448 applicable in a case wherein the owner of the land is also the builder who then later loses ownership of the land by sale? -- NO HELD: By its clear language, Article 448 refers to a land whose ownership is claimed by two or more parties, one of whom has built some works, or sown or planted something. The building, sowing or planting may have been made in good faith or in bad faith. The rule on good faith laid down in Article 526 of the Civil Code shall be applied in determining whether a builder, sower or planter had acted in good faith. Article 448 does not apply to a case where the owner of the land is the builder, sower, or planter who then later loses ownership of the land by sale or donation. Elsewise stated, where the true owner himself is the builder of works on his own land, the issue of good faith or bad faith is entirely irrelevant. Thus in strict point of law, Article 448 is not apposite to the case at bar. Nevertheless, we believe that the provision therein on indemnity may be applied by analogy considering that the primary intent of Article 448 is to avoid a state of forced co-ownership and that the parties, including the two courts below, in the main agree that Articles 448 and 546 of the Civil Code are applicable and indemnity for the improvements may be paid although they differ as to the basis of the indemnity. JPOT Spouses Benitez v. CA 266 SCRA 242 DOCTRINE: "His right is older, and because, by the principle of accession, he is entitled to the ownership of the accessory thing." FACTS: Spouses Macapagal bought a 303 sq. m. lot and found that Spouses Benitez had encroached

on their land. Both parties were amenable to a compromise and had sold the land in excess for a P1000 per sq.m. The former spouses again acquired a lot adjacent to the Benitezes and after a survey had discovered that their lot was encroached on yet again by the latter. They went to court and filed legal action in the MeTC (San Juan), RTC (Pasig) as well as with the CA. It was held by each respectively below: On January 18, 1990, private respondents filed with the Metropolitan Trial Court of San Juan, Branch 58, Civil Case No. 61004 for ejectment against petitioners. The MeTC of San Juan decided in favor of the former, with the following disposition: "WHEREFORE, in view of all the foregoing, judgment is hereby rendered for the plaintiffs and against the defendants ordering them and all persons claiming rights under them to vacate and surrender possession of the subject premises to the plaintiffs as well as to pay the following: 1. The amount of P930.00 a month starting July 17, 1989 until they finally vacate the subject premises; 2. The amount of P5,000.00 for and as attorney's fees; and 3. Cost of suit." On appeal, the Regional Trial Court of Pasig, Branch 167, affirmed said decision. The RTC said: "The controversy in this case is not an encroachment or overlapping of two (2) adjacent properties owned by the parties. It is a case where a part of the house of the defendants is constructed on a portion of the property of the plaintiffs. So that as new owner of the real property, who has a right to the full enjoyment and possession of the entire parcel covered by Transfer Certificate of Title No. 41961, plaintiffs have the right to demand that defendants remove the portion of the house standing on plaintiff's realty. . . ." The dispositive portion thereof reads: "WHEREFORE, finding no reversible error in the decision appealed from, it being more consistent with the facts and the law applicable, the same is hereby AFFIRMED in toto. Costs against the defendant-appellants. SO ORDERED." On further appeal, the respondent Court found no merit in petitioners' plea. In a Resolution dated March 24, 1992, the Sixth Division of said Court found the petition to be a mere rehash of the issues and arguments presented before the lower courts. It ruled in part that: "3) Petitioners were fully aware that part of their house encroached on their neighbor's property, while respondents became aware of it only after purchasing said property. Petitioners cannot claim good faith as against the respondents. "4) Since petitioners are not builders in good faith, they cannot demand that respondents sell the disputed portion; what the law provides is that the builders in bad faith can be ordered to dismantle said structure at their own expense. In the interim period that petitioners' structure remains, they should pay reasonable rent until they remove the structure." The dispositive portion thereof reads: "For reasons indicated, We find the appeal without merit and deny it due course, with costs against the petitioners. SO ORDERED." Spouses Benitez contest and have firm belief to the contrary, for which they assailed tweaked claims to be reviewed by the SC.

ISSUE: W/N possession of a lot encroached upon by a part of another's house be recovered in an action for ejectment? -- YES HELD: The jurisdictional requirements for ejectment, as borne out by the facts, are: after conducting a relocation survey, private respondents discovered that a portion of their land was encroached by petitioners' house; notices to vacate were sent to petitioners, the last one being dated October 26, 1989; and private respondents filed the ejectment suit against petitioners on January 18, 1990 or within one (1) year from the last demand. Prior possession is not always a condition sinequa non in ejectment. This is one of the distinctions between forcible entry and unlawful detainer. Actual or physical occupation is not always necessary. Hence, ejectment is the proper remedy for the case at bar. In addition, the court also ruled that the MeTC had jurisdiction over the matter and were appaled by the audacity of petitioners persistently arguing otherwise after receiving a decision which was sound from the beginning. Furthermore, the "rental" issue is technically damages for which the land owner is entitled to for the non-enjoyment and deprivation of his property. Moreover, the option to sell rests solely with the land owner and only his. Article 448 is accorded the landowner because "his right is older, and because, by the principle of accession, he is entitled to the ownership of the accessory thing." In sum, the SC affirms the decision of all lower courts and said decision is unanimous. MLAV Technogas Phil. V. CA 268 SCRA 5 DOCTRINE: Possession acquired in good faith continues to be enjoyed in the same character in which it was acquired, until the contrary is proved. Good faith consists in the belief of the builder that the land he is building on is his, and his ignorance of any defect or flaw in his title. The good faith ceases from the moment defects in the title are made known to the possessor, by extraneous evidence or by suit for recovery of the property by the true owner. FACTS: Technogas Philippines Manufacturing Corp is the registered owner of Lot 4531-A of Lot 4531 with all buildings, walls, and improvements therein, which they bought from Pariz Industries Inc. The lot that it adjoined, Lot 4531-B, is owned and registered under the name of Eduardo Uy. It was later found out that a portion of the walled lot encroached on the land owned by Uy. Technogas offered to by the encroachment but Uy refused. They entered into a private agreement to demolish the wall on the encroached land. Consequently, Uy filed a complaint for encroachment before the office of the Municipal Engineer and the Provincial Fiscal. However, the complaint of did not prosper. This prompted Uy to dig a canal along the wall, causing it to collapse. Due to the damage, petitioner filed a complaint against Uy for malicious mischief. The RTC ruled in favor of Technogas and ordered Uy to sell the encroached land and pay for damages to the wall.

The CA reversed the decision of the RTC and ordered Technogas to pay rent for the encroachment. ISSUE: WON Technogas is a builder in good faith -- YES HELD: Article 527 of the Civil Code presumes good faith, and since no proof exists to show that the encroachment over a narrow, needle-shaped portion of private respondent‘s land was done in bad faith by the builder of the encroaching structures, the latter should be presumed to have built them in good faith. It is presumed that possession continues to be enjoyed in the same character in which it was acquired, until the contrary is proved. Good faith consists in the belief of the builder that the land he is building on is his, and his ignorance of any defect or flaw in his title. Hence, such good faith, by law, passed on to Pariz‘s successor, petitioner in this case. The good faith ceases from the moment defects in the title are made known to the possessor, by extraneous evidence or by suit for recovery of the property by the true owner. Consequently, the builder, if sued by the aggrieved landowner for recovery of possession, could have invoked the provisions of Art. 448 of the Civil Code. The benefit to the builder under this article is that, instead of being outrightly ejected from the land, he can compel the landowner to make a choice between the two options: (1) to appropriate the building by paying the indemnity required by law, or (2) sell the land to the builder. The landowner cannot refuse to exercise either option and compel instead the owner of the building to remove it from the land. In view of the good faith of both petitioner and private respondent, their rights and obligations are to be governed by Art. 448. Hence, his options are limited to: (1) appropriating the encroaching portion of petitioner‘s building after payment of proper indemnity, or (2) obliging the latter to buy the lot occupied by the structure. He cannot exercise a remedy of his own liking. Petitioner, however, must also pay the rent for the property occupied by its building only up to the date private respondent serves notice of its option upon petitioner and the trial court; that is, if such option is for private respondent to appropriate the encroaching structure. In such event, petitioner would have a right of retention which negates the obligation to pay rent. The rent should however continue if the option chosen is compulsory sale, but only up to the actual transfer of ownership. DJTV Manotok Reality v. Tecson 164 SCRA 587 DOCTRINES: ● Issuance of writ of execution is proper even if private respondent was adjudged a builder in good faith or peculiar circumstances supervened; Option to retain the premises and pay for improvements or to sell the premises to the builder in good faith belongs to the owner of the property. ● Where the improvements have been gutted by fire, the basis for private respondent‘s right to retain the premises has already been extinguished without petitioner‘s fault.

FACTS: In a complaint filed by the petitioner for recovery of possession and damages against the private respondent, the then Court of First Instance of Manila rendered judgment declaring the defendant Nilo Madlangawa as a builder or possessor in good faith; ordering the plaintiff to recognize the right of said defendant to remain in Lot No. 345, Block 1, of the Clara Tambunting Subdivision until after he shall have been reimbursed by the plaintiff the sum of P7,500.00, without pronouncement as to costs. Petitioner filed with the trial court, presided over by respondent Judge Jose H. Tecson, a motion for the approval of petitioner's exercise of option and for satisfaction of judgment, praying that the court issue an order: a) approving the exercise of petitioner's option to appropriate the improvements introduced by the private respondent on the property; b) thereafter, private respondent be ordered to deliver possession of the property in question to the petitioner. The respondent judge denied the motion by issuing the disputed order that under the peculiar circumstances which supervened after the institution of this case, like, for instance, the introduction of certain major repairs of and other substantial improvements on the controverted property, the instant motion of the plaintiff is not well-taken and therefore not legally proper and tenable. After a denial of its motion for reconsideration, the petitioner filed the present petition for mandamus alleging that the respondent judge committed grave abuse of discretion in denying his motion to exercise option and for execution of judgment on the grounds that under Articles 448 and 546 of the Civil Code, the exercise of option belongs to the owner of the property, who is the petitioner herein, and that upon finality of judgment, the prevailing party is entitled, as a matter of right, to its execution which is only a ministerial act on the part of the respondent judge. The private respondent filed his comment on the petition alleging that the same has already become moot and academic because fire gutted not only the house of the private respondent but the majority of the houses in Tambunting Estate. Petitioner argues that since the judgment of the trial court has already become final, it is entitled to the execution of the same and that moreover, since the house of the private respondent was gutted by fire, the execution of the decision would now involve the delivery of possession of the disputed area by the private respondent to the petitioner. ISSUES: 1.

Whether or not the private respondent is a builder in good faith. --

NO 2. Whether or not the issuance of writ of execution is proper even if private respondent was adjudged a builder in good faith or peculiar circumstances supervened. -- YES 3. Whether or not the basis for private respondent‘s right to retain the premises has already been extinguished without petitioner‘s fault when the improvements have been gutted by fire. -- YES HELD: 1. The private respondent's good faith ceased after the filing of the complaint by the petitioner. A possessor in good faith is entitled to the fruits only so long as his possession is not legally interrupted, and such interruption takes place upon service of judicial summons

(Arts. 544 and 1123, Civil Code). Thus, the repairs and improvements introduced by the said respondents after the complaint was filed cannot be considered to have been built in good faith, much less, justify the denial of the petitioner's option. 2. When the decision of the trial court became final and executory, it became incumbent upon the respondent judge to issue the necessary writ for the execution of the same. There is, therefore, no basis for the respondent judge to deny the petitioner's motion to avail of its option to approriate the improvements made on its property. Neither can the respondent judge deny the issuance of a writ of execution because the private respondent was adjudged a builder in good faith or on the ground of "peculiar circumstances which supervened after the institution of this case, like, for instance, the introduction of certain major repairs of and other substantial improvements..." because the option given by law either to retain the premises and pay for the improvements thereon or to sell the said premises to the builder in good faith belongs to the owner of the property. 3. Since the improvements have been gutted by fire, and therefore, the basis for private respondent's right to retain the premises has already been extinguished without the fault of the petitioner, there is no other recourse for the private respondent but to vacate the premises and deliver the same to herein petitioner.

WHEREFORE, IN VIEW OF THE FOREGOING, the petition is GRANTED and the respondent judge is hereby ordered to immediately issue a writ of execution ordering the private respondent to vacate the disputed premises and deliver possession of the same to the petitioner. JGY Ballatan v. CA 304 SCRA 34 DOCTRINE: In the event that the owner elects to sell to the builder, planter or sower the land on which the improvement stands, the price must be fixed at the prevailing market value at the time of payment. FACTS: The parties herein are owners of adjacent lots. Lot 24 is registered in the name of petitioners Ballatan. Lots 25 & 26 are registered in the name of respondent Go Sr. His son, Winston Go constructed a house on Lot 25. Adjacent to Lot 26 is Lot 27, 4 registered in the name of respondent Li Ching Yao. Petitioner Ballatan constructed her house on Lot 24. During the construction, she noticed that the concrete fence and side pathway of respondent‘s house encroached her property. Ballatan informed Go of this discrepancy and his encroachment on her property. Surveys were made and it was found out that the lot area of petitioner Ballatan was less by a few meters and that of respondent Li Ching Yao, which was three lots away, increased by two 2 meters. Petitioner Ballatan made a written demand on respondents Go to remove and dismantle their improvements on Lot 24. Respondents Go refused. Hence, Ballatan filed for recovery of possession of real property. TC decided in favor of petitioners. It ordered the Go's to vacate the subject portion, demolish their improvements. CA modified: ordered Go to pay Ballatan, and respondent Li Ching Yao to pay Go and the value to be fixed at the time of the taking.

ISSUE: WON the value should be fixed at the time of the taking. -- NO HELD: All the parties are presumed to have acted in good faith. Their rights must, therefore, be determined in accordance with the appropriate provisions of the Civil Code on property specifically Art. 448. Petitioners, as owners of Lot 24, may choose to purchase the improvement made by respondents Go on their land, or sell to respondents Go the subject portion. If buying the improvement is impractical as it may render the Go's house useless, then petitioners may sell to respondents Go that portion of Lot 24 on which their improvement stands. If the Go's are unwilling or unable to buy the lot, then they must vacate the land and, until they vacate, they must pay rent to petitioners. Petitioners, however, cannot compel respondents Go to buy the land if its value is considerably more than the portion of their house constructed thereon. If the value of the land is much more than the Go's improvement, then respondents Go must pay reasonable rent. If they do not agree on the terms of the lease, then they may go to court to fix the same. In the event that petitioners elect to sell to respondents Go the subject portion of their lot, the price must be fixed at the prevailing market value at the time of payment. JRPA Geminiano v. CA 259 SCRA 344 DOCTRINE: Article 448 of the Civil Code, in relation to Article 546 of the same Code, which allows full reimbursement of useful improvements and retention of the premises until reimbursement is made, applies only to a possessor in good faith, i.e., one who builds on land with the belief that he is the owner thereof. It does not apply where one's only interest is that of a lessee under a rental contract; otherwise, it would always be in the power of the tenant to "improve" his landlord out of his property. FACTS: Lot No. 3765-B-1 containing an area of 314sq was originally owned by the petitioners' mother, Paulina Amado vda. de Geminiano. On a 12-square-meter portion of that lot stood the petitioners' unfinished bungalow, which the petitioners sold to the private respondents with an alleged promise to sell to the latter that portion of the lot occupied by the house. Subsequently, the petitioners' mother executed a 7-year contract of lease over a 126 square-meter portion of the lot, including that portion on which the house stood, in favor of the private respondents. The private respondents then introduced additional improvements and registered the house in their names. After the expiration of the lease contract however, the petitioners' mother refused to accept the monthly rentals. It turned out that the lot in question was the subject of a suit, which resulted in its acquisition by one Maria Lee. Lee sold the lot to Lily Salcedo, who in turn sold it to the spouses Agustin and Ester Dionisio. Dionisio spouses executed a Deed of Quitclaim over the said property in favor of the petitioners. As such, the lot was registered in the latter's name. The petitioners sent, via registered mail, a letters addressed to private respondent Mary Nicolas demanding that she vacate the premises and pay the rentals in arrears. Upon failure of the private respondents to heed the demand, the petitioners filed with the MTCC of Dagupan City a complaint for unlawful

detainer and damages. ISSUES: 1. Whether or not Art 448 applies to this case? -- NO 2. Whether or not the private respondents are builder in good faith or mere lessees? -- LESSEES HELD: The private respondents claim they are builders in good faith, hence, Article 448 of the Civil Code should apply. They rely on the lack of title of the petitioners' mother at the time of the execution of the contract of lease, as well as the alleged assurance made by the petitioners that the lot on which the house stood would be sold to them. It has been said that while the right to let property is an incident of title and possession, a person may be lessor and occupy the position of a landlord to the tenant although he is not the owner of the premises let. After all, ownership of the property is not being transferred, only the temporary use and enjoyment thereof. In this case, both parties admit that the land in question was originally owned by the petitioners' mother. The land was allegedly acquired later by one Maria Lee by virtue of an extrajudicial foreclosure of mortage. Lee, however, never sought a writ of possession in order that she gain possession of the property in question. The petitioners' mother therefore remained in possession of the lot. It is undisputed that the private respondents came into possession of 126 square-meter portion of the said lot by virtue of contract of lease executed by the petitioners' mother as lessor, and the private respondents as lessees, is therefore well-established, and carries with it a recognition of the lessor's title. The private respondents, as lessees who had undisturbed possession for the entire term under the lease, are then estopped to deny their landlord's title, or to assert a better title not only in themselves, but also in some third person while they remain in possession of the leased premises and until they surrender possession to the landlord. This estoppel applies even though the lessor had no title at the time the relation of lessor and lessee was created, and may be asserted not only by the original lessor, but also by those who succeed to his title. Being mere lessees, the private respondents knew that their occupation of the premises would continue only for the life of the lease. Plainly, they cannot be considered as possessors nor builders in good faith. Article 448 of the Civil Code, in relation to Article 546 of the same Code, which allows full reimbursement of useful improvements and retention of the premises until reimbursement is made, applies only to a possessor in good faith, i.e., one who builds on land with the belief that he is the owner thereof. It does not apply where one's only interest is that of a lessee under a rental contract; otherwise, it would always be in the power of the tenant to "improve" his landlord out of his property. Anent the alleged promise of the petitioners to sell the lot occupied by the private respondents' house, the same was not substantiated by convincing evidence. Neither the deed of sale over the house nor the contract of lease contained an option in favor of the respondent spouses to purchase the said lot. And even if the petitioners indeed promised to sell, it would not make the private respondents possessors or builders in good faith so as to covered by the provision of

Article 448 of the Civil Code. The latter cannot raise the mere expectancy or ownership of the aforementioned lot because the alleged promise to sell was not fulfilled nor its existence even proven. ABB Spouses del Campo v. Obesia 160 SCRA DOCTRINE: When the co-ownership is terminated by the partition and it appears that the house of the defendants occupies a portion of 5 square meters of the land pertaining to plaintiffs which the defendant obviously built in good faith, the provisions of Article 448 of the new Civil Code should apply. FACTS: An action for partition of a parcel of land was filed by the spouses Del Campo in the CFI of Cebu. Plaintiffs and defendants are co-owners pro indiviso of this lot in the proportion of and 1/3 share each, respectively. The trial court appointed a commissioner in accordance with the agreement of the parties. The commissioner conducted a survey, prepared a sketch plan and submitted a report to the trial court on May 29, 1976, recommending that the property be divided into two lots: Lot 1161-A with an area of 30 square meters for the spouses Del Campo and Lot No. 1161-B with an area of 15 square meters for Obesia. Upon surveying, it was shown that the house of Obesia occupied the portion with an area of 5 square meters of Lot 1161-A of the spouses Del Campo. The parties manifested their conformity to the report and asked the trial court to finally settle and adjudicate who among the parties should take possession of the 5 square meters of the land in question. ISSUE: Whether or Not Article 448 of the Civil Code is applicable to a builder in good faith when the property involved is owned in common. HELD: When the co-ownership is terminated by the partition and it appears that the house of Obesia occupies a portion of 5 square meters of the land pertaining to spouses Del Campo which Obesia obviously built in good faith, the provisions of Article 448 of the new Civil Code should apply. In applying Article 448 of the New Civil Code, the plaintiffs have the right to appropriate said portion of the house of defendants upon payment of indemnity to defendants as provided for in Article 546 of the Civil Code. Otherwise, the plaintiffs may oblige the defendants to pay the price of the land occupied by their house. But if the price asked for is considerably much more than the value of the portion of the house of defendants built thereon, then the latter cannot be obliged to buy the land. The defendants shall then pay the reasonable rent to the plaintiff upon such terms and conditions that they may agree. In case of disagreement, the trial court shall fix the terms thereof. The defendants may opt to demolish or remove the said portion of their house, at their own expense, if they so decide. FZC Rosales, et al. v. Castellfort, et al. G.R. No. 157044 DOCTRINE: Where the builder, planter or sower has acted in good faith, a conflict of rights

arises between the owners, and it becomes necessary to protect the owner of the improvements without causing injustice to the owner of the land. In view of the impracticability of creating a state of forced co-ownership, the law has provided a just solution by giving the owner of the land the option to acquire the improvements after payment of the proper indemnity, or to oblige the builder or planter to pay for the land and the sower the proper rent. He cannot refuse to exercise either option. It is the owner of the land who is authorized to exercise the option, because his right is older, and because, by the principle of accession, he is entitled to the ownership of the accessory thing. FACTS: Spouses-petitioners Rodolfo V. Rosales and Lily Rosqueta-Rosales (petitioners) are the registered owners of a parcel of land designated as Lot 17, Block 1 of Subdivision Plan situated in Los Baños, Laguna. Petitioners discovered that a house was being constructed on their lot, without their knowledge and consent, by respondent Miguel Castelltort (Castelltort). It turned out that respondents Castelltort and his wife Judith had purchased a lot, Lot 16 of the same Subdivision Plan, from respondent Lina Lopez-Villegas (Lina) through her son-attorney-in-fact Rene Villegas (Villegas) but that after a survey thereof by geodetic engineer Augusto Rivera, he pointed to Lot 17 as the Lot 16 the Castelltorts purchased. Negotiations for the settlement of the case thus began, with Villegas offering a larger lot near petitioners‘ lot in the same subdivision as a replacement thereof. In the alternative, Villegas proposed to pay the purchase price of petitioners‘ lot with legal interest. Both proposals were, however, rejected by petitioners whose counsel directed Castelltort to stop the construction of and demolish his house and any other structure he may have built thereon, and desist from entering the lot. Petitioners subsequently filed a complaint for recovery of possession and damages with prayer for the issuance of a restraining order and preliminary injunction against spouses-respondents Miguel and Judith Castelltort before the RTC of Calamba, Laguna ISSUE: Under Art 448, who has the right of option? HELD: Under Article 448, the landowner can choose between appropriating the building by paying the proper indemnity or obliging the builder to pay the price of the land, unless its value is considerably more than that of the structures, in which case the builder in good faith shall pay reasonable rent. If the parties cannot come to terms over the conditions of the lease, the court must fix the terms thereof.The choice belongs to the owner of the land, a rule that accords with the principle of accession, i.e.,that the accessory follows the principal and not the other way around. Even as the option lies with the landowner, the grant to him, nevertheless, is preclusive. The landowner cannot refuse to exercise either option and compel instead the owner of the building to remove it from the land.The raison d‘etre for this provision has been enunciated thus: Where the builder, planter or sower has acted in good faith, a conflict of rights arises between the owners, and it becomes necessary to protect the owner of the improvements without causing injustice to the owner of the land. In view of the impracticability of creating a state of forced co-ownership, the law has provided a just solution by giving the owner of the land the option to acquire the improvements after payment of the proper indemnity, or to oblige the builder or planter to pay for the land and the sower the proper rent. He cannot refuse to exercise either option. It is the owner of the land who is authorized to exercise the option, because his right is older, and because, by the principle of accession, he is entitled to the ownership of the

accessory thing. LNAC Ignao v. IAC, G.R. No. 72876 193 SCRA 17 DOCTRINE: When the co-ownership is terminated by a partition and it appears that the house of an erstwhile co-owner has encroached upon a portion pertaining to another co-owner which was however made in good faith, then the provisions of Article 448 should apply to determine the respective rights of the parties. FACTS: Petitioner Florencio Ignao and his uncles private respondents Juan Ignao and Isidro Ignao were co-owners of a parcel of land with an area of 534 square meters situated in Barrio Tabon, Municipality of Kawit, Cavite. Pursuant to an action for partition filed by petitioner, the then CFI directed the partition of the aforesaid land, allotting 133.5 square meters or 2/8 thereof to private respondents Juan and Isidro, and giving the remaining portion with a total area of 266.5 square meters to petitioner Florencio. However, no actual partition was ever effected. Petitioner instituted a complaint for recovery of possession of real property against private respondents Juan and Isidro before the CFI. In his complaint petitioner alleged that the area occupied by the two (2) houses built by private respondents exceeded the 133.5 square meters previously allotted to them by the trial court. It was found that the houses of Juan and Isidro actually encroached upon a portion of the land belonging to Florencio. Upon agreement of the parties, the trial court ordered a licensed geodetic engineer to conduct a survey to determine the exact area occupied by the houses of private respondents. The survey subsequently disclosed that the house of Juan occupied 42 square meters while that of Isidro occupied 59 square meters of Florencio's land or a total of 101 square meters. Trial court ruled that although private respondents occupied a portion of Florencio's property, they should be considered builders in good faith. Petitioner appealed to IAC which subsequently affirmed the decision of the trial court. Thus, herein petition. ISSUE: Whether the provisions of Article 448 should apply to a builder in good faith on a property held in common. -- YES HELD: It should be noted that prior to partition, all the co-owners hold the property in common dominion but at the same time each is an owner of a share which is abstract and undetermined until partition is effected. As co-owners, the parties may have unequal shares in the common property, quantitatively speaking. But in a qualitative sense, each co-owner has the same right as any one of the other co-owners. Every co-owner is therefore the owner of the whole, and over the whole he exercises the right of dominion, but he is at the same time the owner of a portion which is truly abstract, because until division is effected such portion is not concretely determined.

Article 448 provides: The owner of the land on which anything has been built, sown or planted in good faith, shall have the right to appropriate as his own the works, sowing or planting, after payment of the indemnity provided for in articles 546 and 548, or to oblige the one who built or planted to pay the price of the land, and the one who sowed, the proper rent. However, the builder or planter cannot be obliged to buy the land if its value is considerably more than that of the building or trees. In such case, he shall pay reasonable rent, if the owner of the land does not choose to appropriate the building or trees after proper indemnity. The parties shall agree upon the terms of the lease and in case of disagreement, the court shall fix the terms thereof. Whether or not the provisions of Article 448 should apply to a builder in good faith on a property held in common has been resolved in the case of Spouses del Campo vs. Abesia, wherein the Court ruled that: The court a quo correctly held that Article 448 of the Civil Code cannot apply where a co-owner builds, plants or sows on the land owned in common for then he did not build, plant or sow upon land that exclusively belongs to another but of which he is a co-owner. The co-owner is not a third person under the circumstances, and the situation is governed by the rules of coownership. However, when, as in this case, the ownership is terminated by the partition and it appears that the home of defendants overlaps or occupies a portion of 5 square meters of the land pertaining to plaintiffs which the defendants obviously built in good faith, then the provisions of Article 448 of the new Civil Code should apply. Both the trial court and the Appellate Court erred when they peremptorily adopted the "workable solution" in the case of Grana vs. CA, and ordered the owner of the land, petitioner Florencio, to sell to private respondents, Juan and Isidro, the part of the land they intruded upon, thereby depriving petitioner of his right to choose. Such ruling contravened the explicit provisions of Article 448 to the effect that "(t)he owner of the land xxx shall have the right to appropriate xxx or to oblige the one who built xxx to pay the price of the land xxx." The law is clear and unambiguous when it confers the right of choice upon the landowner and not upon the builder and the courts TKDC MWSS v. C.A. 143 SCRA 623 DOCTRINE: The right of a possessor in bad faith to remove improvements applies only to improvements for pure luxury or mere pleasure, provided the thing does not suffer any injury and the lawful possessor does not prefer to retain them by paying their value at the time of his possession. FACTS: Dagupan City filed a complaint against MWSS for recovery of ownership and possession of the Dagupan Waterworks System. MWSS interposed R.A. 1383 as its defense; it vested to MWSS the ownership, possession, and control of all waterworks system throughout the Philippines. MWSS also filed a counterclaim for reimbursement of expenses it incurred for necessary and useful improvements. Trial court ruled that MWSS is a possessor in bad faith so it is not entitled to claim

reimbursement. MWSS appealed to the Court of Appeals arguing that Dagupan City should be liable for payment of the balance of the loan secured by MWSS for the improvement of the Dagupan Waterworks System; however the Court of Appeals affirmed trial court‘s judgment. MWSS appealed to the Supreme Court for the removal of useful improvements. Dagupan City argues that MWSS is a possessor in bad faith so it has absolutely no right to the useful improvements. ISSUE: Whether or not a possessor in bad faith has the right to remove useful improvements. -- NO HELD: Under Article 499 of the Civil Code, ―he who builds, plants, or sows in bad faith on the land of another, loses what is built, planted, or sown without right to indemnity.‖ Additionally, under Article 546 of the Civil Code, only a possessor in good faith shall be refunded for useful expenses with the right of retention until reimbursed. Finally, under Article 547 of the Civil Code, only a possessor in good faith may remove useful improvements if this can be done without damage to the principal thing and if the person who recovers the possession does not exercise the option of reimbursing the useful expenses. The right of a possessor in bad faith to remove improvements applies only to improvements for pure luxury or mere pleasure, provided the thing does not suffer any injury and the lawful possessor does not prefer to retain them by paying their value at the time of his possession. In this case, MWSS is a builder in bad faith so it loses whatever useful improvements it made without right to indemnity. AMD Alviola v. CA 289 SCRA 537 DOCTRINE: For Article 448 to apply, the construction must be of permanent character, attached to the soil with an idea of perpetuity; but if it is of a transitory character or is transferable, there is no accession, and the builder must remove the construction. The proper remedy of the landowner is an action to eject the builder from the land. FACTS: In 1950, Victoria Tinagan purchased from Mauro Tinagan 2 parcels of land situated in Valencia, Negros Occidental. Victoria and her son, Agustin Tinagan took possession of said parcels of land. In 1960, the Alviola spouses occupied portions thereof whereat they built a copra dryer and put up a store engaged in the buying and selling of copra. Both Victoria and Augustin died in 1975. The latter was survived by private respondents, his wife Florencia and their children. Editha Aviola, assisted by her husband, filed a complaint for partition and damages before the CFI, claiming to be an acknowledged natural child of Augustin TInagan and demanding the delivery of her share in the estate. The case was dismissed on the ground that recognition of natural children should be brought only during the lifetime of the presumed parent. In 1988, private respondents filed a complaint for recovery of possession against the Alviolas. The RTC ruled in favor of private respondents and ordered the Alviolas to peacefully vacate and to surrender the possession of the premises. The latter may remove their store and dryer

without injury and prejudice to the plaintiffs. Petitioners appealed to the CA, said court upheld the RTC decision. Hence, this petition. ISSUE: W/N the copra dryer and the store are susceptible of accession (as accession industrial). -- NO HELD: The SC ruled that private respondents adduced overwhelming evidence to prove their ownership and possession of the 2 parcels of land on portions of which petitioners built the copra dryer and a store. In addition, the Alviolas‘ stay thereon, since 1961, was merely by tolerance on the part of the private respondents and their predecessor-in-interest. The evidence shows that the petitioners were permitted by Victoria Tinagan to build a copra dryer on the land when they got married. As correctly ruled by the respondent court, there was bad faith on the part of the petitioners when they constructed the copra dryer and store on the disputed portions since they were fully aware that the parcels of land belonged to Victoria Tinagan. And, there was likewise bad faith on the part of the private respondents, having knowledge of the arrangement between petitioners and Victoria Tinagan relative to the construction of the copra dryer and store. Thus, for purposes of indemnity, Article 448 of the New Civil Code should be applied. However, the copra dryer and the store, as determined by the trial court and respondent court, are transferable in nature. Thus, it would not fall within the coverage of Article 448. As the noted civil law authority, Senator Arturo Tolentino, aptly explains: "To fall within the provision of this Article, the construction must be of permanent character, attached to the soil with an idea of perpetuity; but if it is of a transitory character or is transferable, there is no accession, and the builder must remove the construction. The proper remedy of the landowner is an action to eject the builder from the land." The private respondents‘ action for recovery of possession was the suitable solution to eject petitioners from the premises. MPF Arangote v. Maglunob GR 178906 DOCTRINE: The rights under Article 448 and 546 of the Civil Code are applicable only to builders in good faith and not to possessors in good faith. Possession in Good Faith; Every possessor in good faith becomes a possessor in bad faith from the moment he becomes aware that what he believed to be true is not so. FACTS: ● All the respondents in the case are siblings of Esperanza. ● 1985. Esperanza executed a Last Will and Testament bequeathing the subject property she inherited from her father based on the Partition Agreement to Arangote. ● 1986. Esperanza executed an affidavit waiving and quitclaiming all her rights, interest, and participation in the subject property in favor of Arangote. ○ Tax Declaration of the subject property was cancelled from Esperanza and was issued in the name of Arangote ● 1989. Arangote constructed a house on the subject property. ● 1993. OCT was issued in the name of Arangote on the subject property. ○ However, respondents (Maglunobs) entered the subject property and built a concrete wall behind and front of Arangote‘s house effectively blocking the entrance.

● Arangote file a complaint against respondent for quieting title, declaration of ownership and possession at MTC. ISSUES: Whether or not Arangote were builders in good faith. -- NO Whether or not Arangote were entitled rights under Article 448 and 526. -- NO HELD: Possessors in good faith are those who are not aware that there exists in his title or mode of acquisition and flaw which invalidates it. Good faith of the possessor consists in the reasonable belief that the person from whom he received the thing was the owner thereof, and could transmit his ownership. Every possessor in good faith becomes possessor in bad faith from the moment he becomes aware that what believed to be true is not so. In the case, the subject property waived and quitclaimed by Esperanza to the petitioners in affidavit was only covered by a tax declaration in the name of Esperanza. Arangote did not look into the origin if the subject property and to probe if Esperanza has the right to relinquish the subject property. Thus, when Arangote built their house, they cannot be considered to have acted in good faith because they only relied on tax declaration. Settles is the rule that a tax declaration does not prove ownership and the Supreme Court also added that payment of taxes is not a proof of ownership. Hence, Arangote is not a builder and possessor in good faith. Accession Natural AMDG Reynante v. C.A. 207 SCRA 794 DOCTRINE: Granting that the lots were created by alluvial formation and while it is true that accretions which the bank of rivers may gradually receive from the effect of the current become the property of the owner of the banks, such accretion to registered land does not preclude acquisition of the additional are by another person through prescription FACTS: Petitioner, Jose Reynante was a tenant over the two lots of Don Cosme Carlos for more than 50 years. A fishpond is situated between the two lots. During his tenancy, Reynante constructed a nipa hut, and planted and harvested nipa plams, to which he appropriated it as his own. When Don Cosme Carlos died, his heirs, private respondents, executed a written agreement with Reynante wherein the latter would turn over the fishpond to the former. After executing the agreement, Reynante surrendered the fishponds as well as the lots. The fishpond was leased to one Carlos de la Cruz. However, Reynante did not vacate the lots and continued to live there and harvest the nipa palms he planted. The private respondents formally demanded that Reynante vacate the lots to which the latter refused to do. A forcible entry with preliminary injunction was filed against Reynante however the trial court dismissed the complaint basing from the fact that Reynante was the prior possessor of the lots. The trial court‘s decision was affirmed by the Court of Appeals. ISSUES: 1.

Who between the petitioner and private respondents has prior physical

possession the two lots? -- PETITIONER REYNANTE 2. Whether or not the disputed lots belong to private respondents as a result of accretion? -- NO HELD: It has been held that party who can prove prior possession can recover such possession even against the owner himself. As long as the party is able to prove prior possession he is entitled to remain on the property until he is lawfully ejected. It is clear from the records that Reynante has been in possession of the lots for more than 50 years. Consequently, the court cannot legally grant the possession over the two lots to the private respondents. The Court of Appeals ruled that the two lots were created by alluvial formation hence under Article 457 of the New Civil code the ownership of such is granted to the private respondents. However, although the lands to which the additional areas (which are the two lots subject in this case) are attached to are registered; the failure to register the additional lands subjected it to acquisition through prescription. Applying this to the case, the private respondents never registered the two lots and since Reynante has been in possession of such for more than 50 years already, its possession over it must be respected unless the private respondents are able to show that they have the better title over it. GCG Vda. de Nazareno v. C.A. 257 SCRA 589 DOCTRINE: Accretion, as a mode of acquiring property under Article 457 of the Civil Code, requires the concurrence of these requisites: (1) that the deposition of soil or sediment be gradual and imperceptible; (2) that it be the result of the action of the waters of the river (or sea); and (3) that the land where accretion takes place is adjacent to the banks or rivers (or the sea coast). These are called the rules on alluvion which if present in a case, give to the owners of lands adjoining the banks of rivers or streams any accretion gradually received from the effects of the current of waters. FACTS: A parcel of land situated in Telegrapo, Puntod, Cagayan de Oro City is said to have been formed as a result of sawdust dumped into the dried-up Balacanas Creek and along the banks of the Cagayan river. Jose Salasalan and Leo Rabaya leased the subject lots on which their houses stood from one Antonio Nazareno, petitioners‘ predecessor-in-interest. Salasalan and Rabaya allegedly stopped paying rentals. As a result, Antonio Nazareno and petitioners filed a case for ejectment with the MTC. A decision was rendered against Salasalan and Rabaya, which decision was affirmed by the RTC. The case was remanded to the Municipal trial court for execution of judgment after the same became final and executory. Private respondents filed a case for annulment of judgment before the RTC Misamis Oriental which was dismissed. The decision of the lower court was finally enforced with the private respondents being ejected from portions of the subject lots they occupied. Before he died, Antonio Nazareno caused the approval by the Bureau of Lands of the survey plan designated with a view to perfecting his title over the accretion area being claimed by him. Before the approved survey plan could be released to the applicant, however, it was protested by private respondents before the Bureau of Lands. The report of the Land Investigator, made in

compliance with the order of the District Land Officer, recommended the Survey Plan MSI-1006-000571-D (Lot 36302, Cad. 237) in the name of Antonio Nazareno be cancelled and that private respondents be directed to file appropriate public application. Based on the report, the Regional Director of the Bureau of Lands rendered a decision ordering an amendment to the survey plain of Nazareno by segregating therefrom the areas occupied by the private respondents. Antonio Nazareno filed a motion for reconsideration with the Undersecretary of Natural Resources and OIC of the Bureau of Lands; which was denied. The petitioners Desamparada vda. De Nazareno and Leticia Tapia Nazero filed a case before the RTC for the annulment of the decision and order of the Bureau of Lands regarding the parcel of land. The RTC dismissed the complaint for failure to exhaust administrative remedies, resulting to the finality of the administrative decision of the Bureau of Lands. On appeal, the Court of Appeals affirmed the decision of the RTC dismissing the complaint. Hence, the petition. ISSUE: Whether or not accretion belongs to the riparian owners -- NO HELD: No. Supreme Court dismissed the petition for lack of merit. Article 457 of the Civil Code provides that ―to the owners of land adjoining the banks of rivers belong the accretion which they gradually receive from the effects of the current of the waters.‖ In the case of Meneses v. CA, it was held that accretion, as a mode of acquiring property under Article 457 of the Civil Code, requires the concurrence of these requisites: (1) that the deposition of soil or sediment be gradual and imperceptible; (2) that it be the result of the action of the waters of the river (or sea); and (3) that the land where accretion takes place is adjacent to the banks or rivers (or the sea coast). These are called the rules on alluvion which if present in a case, give to the owners of lands adjoining the banks of rivers or streams any accretion gradually received from the effects of the current of waters. The application of the rules on alluvion cannot be made in the present case as the first and second requirements of the rules were not met. Thus, the Nazarenos cannot claim the rights of a riparian owner. By their own admission, the accretion was formed by the dumping of boulders, soil and other filling materials on portions of the Balacanas Creek and the Cagayan River bounding their land. It cannot be claimed, therefore, that the accumulation of such boulders, soil and other filling materials was gradual and imperceptible, resulting from the action of the waters or the current of the Balacanas Creek and the Cagayan River. In Hilario v. City of Manila, it was held that the word ―current‖ indicate the participation of the body of water in the ebb and flow of waters due to high and low tide. Petitioners are estopped from denying the public character of the subject land, as well as the jurisdiction of the Bureau of Lands when the late Antonio Nazareno filed his Miscellaneous Sales Application MSA (G-6) 571. The mere filing of said Application constituted an admission that the land being applied for was public land, having been the subject of Survey Plan MSI-1006-000571-D which was conducted as a consequence of Antonio Nazareno‘s Miscellaneous Sales Application wherein said land was described as an orchard. Said description by Antonio Nazareno was controverted by the findings of the ocular inspection that said land actually

covers a dry portion of Balacanas Creek and a swampy portion of Cagayan River. The Bureau of Lands classified the subject land as an accretion area which was formed by deposits of sawdust in the Balacanas Creek and the Cagayan river, in accordance with the ocular inspection conducted by the Bureau of Lands. It has often enough held that findings of administrative agencies which have acquired expertise because their jurisdiction is confined to specific matters are generally accorded not only respect but even finality. Again, when said factual findings are affirmed by the Court of Appeals, the same are conclusive on the parties and not reviewable by the Supreme Court. In Republic v. CA, it was ruled that the requirement that the deposit should due to the effect of the current of the river is indispensable. This excludes from Article 457 of the Civil Code all deposits caused by human intervention. Putting it differently, alluvion must be the exclusive work of nature. Thus, in Tiongco v. Director of Lands, et al., where the land was not formed solely by the natural effect of the water current of the river bordering said land but is also the consequence of the direct and deliberate intervention of man, it was deemed a man-made accretion and, as such, part of the public domain. In the present case, the subject land was the direct result of the dumping of sawdust by the Sun Valley Lumber Co. consequent to its sawmill operations. As the accretion site was the result of the late Antonio Nazareno‘s labor consisting in the dumping of boulders, soil and other filling materials into the Balacanas Creek and Cagayan River bounding his land, the same would still be part of the public domain. VCL IV Bagaipo v. C.A. 347 SCRA 443 DOCTRINE: In the absence of evidence that the change in the course of the river was sudden or that it occurred through avulsion, the presumption is that the change was gradual and was caused by alluvium and erosion. FACTS: Petitioner Dionisia P. Bagaipo is the registered owner of Lot which located southeast of Davao river. While respondent Leonor Lozano is the owner of a registered parcel of land located across and opposite the southeast portion of petitioner‘s lot facing the Davao River. On May 26, 1989, Bagaipo filed a complaint for Recovery of Possession with Mandatory Writ of Preliminary Injunction and Damages against Lozano for: (1) the surrender of possession by Lozano of a certain portion of land measuring 29,162 square meters which is supposedly included in the area belonging to Bagaipo under TCT No. T-15757; and (2) the recovery of a land area measuring 37,901 square meters which Bagaipo allegedly lost when the Davao River traversed her property. Bagaipo contended that as a result of a change in course of the said river, her property became divided into three lots, namely: Lots 415-A, 415-B and 415-C.In January 1988, Bagaipo commissioned a resurvey of Lot 415 and presented before the trial court a survey plan prepared by Geodetic Engineer Gersacio A. Magno which concluded that the land presently located across the river and parallel to Bagaipo‘s property still belonged to the latter and not to Lozano who planted some 350fruit-bearing trees on Lot 415-C and the old abandoned river bed. For his part, Lozano insisted that the land claimed by Bagaipo is actually an accretion to their titled property. He asserted that the Davao River did not change its course and that the reduction in Bagaipo‘s domain was caused by gradual erosion due to the current of the Davao

River. Lozano added that it is also because of the river‘s natural action that silt slowly deposited and added to his land over a long period of time. He further averred that this accretion continues up to the present and that registration proceedings instituted by him over the alluvial formation could not be concluded precisely because it continued to increase in size. The Trial Court, upon inspection, found that the decrease in land area was brought about by erosion and not a change in the river‘s course. CA affirmed the TC ruling. ISSUE: Whether or not there was a change in the river‘s course which resulted to avulsion? -- NO HELD: The trial court and the appellate court both found that the decrease in land area was brought about by erosion and not a change in the river‘s course. This conclusion was reached after the trial judge observed during ocular inspection that the banks located on petitioner‘s land are sharp, craggy and very much higher than the land on the other side of the river. Additionally, the riverbank on respondent‘s side is lower and gently sloping. The lower land therefore naturally received the alluvial soil carried by the river current. These findings are factual, thus conclusive on this Court, unless there are strong and exceptional reasons, or they are unsupported by the evidence on record, or the judgment itself is based on a misapprehension of facts. The decrease in petitioner‘s land area and the corresponding expansion of respondent‘s property were the combined effect of erosion and accretion respectively. Art. 461 of the Civil Code is inapplicable. Petitioner cannot claim ownership over the old abandoned riverbed because the same is inexistent. The riverbed‘s former location cannot even be pinpointed with particularity since the movement of the Davao River took place gradually over an unspecified period of time, up to the present. The rule is well-settled that accretion benefits a riparian owner when the following requisites are present: 1) That the deposit be gradual and imperceptible; 2) That it resulted from the effects of the current of the water; and 3) That the land where accretion takes place is adjacent to the bank of the river. These requisites were sufficiently proven in favor of respondents. In the absence of evidence that the change in the course of the river was sudden or that it occurred through avulsion, the presumption is that the change was gradual and was caused by alluvium and erosion. FXRL Agustin v. IAC 187 SCRA 218 DOCTRINE: Accretion benefits a riparian owner when the following requisites are present: (1) that the deposit be gradual and imperceptible; (2) that it resulted from the effects of the current of the water; and (3) that the land where accretion takes place is adjacent to the bank of a river . FACTS: ● Petitioner Agustin is the owner of a parcel of land on the eastern side of the Cagayan river. ● Private respondents Binuyag, et al. own parcels of land on the western side of

the Cagayan river. ● From 1919 to 1968, the river transferred land (by alluvium) from the property of Agustin to that of the respondents. ● In 1968, a flood made the river changed its course and now cut through the land of the respondents. ● This change in course forced the respondents to cross the river whenever they have to cultivate their land ● Seeing this, the petitioner sought the assistance of the local government in displacing the respondents while they were planting crops on the land now separated by the river ● RTC ruled in favor of the respondents and ordered the petitioners to vacate the land. ● IAC affirmed the ruling. ISSUE: W/N the respondents‘ ownership of the land was affected by the sudden change in the course of the river. -- NO HELD: No. The land separated by the course of the river remains to be that of the respondents. Art. 457 of the civil code provides that: ―To the owners of lands adjoining the banks of rivers belong the accretion which they gradually receive from the effects of the current of the waters. (366)‖ Accretion benefits a riparian owner when the following requisites are present: (1) that the deposit be gradual and imperceptible; (2) that it resulted from the effects of the current of the water; and (3) that the land where accretion takes place is adjacent to the bank of a river . All these requisites of accretion are present in this case as the river had indeed, over a period of time (49 years), deposited the land gradually and imperceptibly from the current if its water and that it is adjacent to the riverbank. Further, the private respondents' ownership of the accretion to their lands was not lost upon the sudden and abrupt change of the course of the Cagayan River in 1968 or 1969 when it reverted to its old 1919 bed, and separated or transferred said accretions to the eastern bank of the river. Articles 459 and 463 of the Civil Code apply to this situation. Art. 459. Whenever the current of a river, creek or torrent segregates from an estate on its bank a known portion of land and transfers it to another estate, the owner of the land to which the segregated portion belonged retains the ownership of it, provided that he removes the same within two years. Art. 463. Whenever the current of a river divides itself into branches, leaving a piece of land or part thereof isolated, the owner of the land retains his ownership. He also retains it if a portion of land is separated from the estate by the current. In the case at bar, the sudden change of course of the Cagayan River as a result of a strong typhoon in 1968 caused a portion of the lands of the private respondents to be "separated from the estate by the current." The private respondents have retained the ownership of the portion that was transferred by avulsion to the other side of the river.

RSDM Viajar v. CA 168 SCRA 405 DOCTRINE: Registration does not protect the riparian owner against the diminution of the area of his land through gradual changes in the course of the adjoining stream. Accretions which the banks of rivers may gradually receive from the effect of the current become the property of the owners of the banks (Art. 366 of the Old Civil Code; Art. 457 of the New). Such accretions are natural incidents to land bordering on running streams and the provisions of the Civil Code in that respect are not affected by the Registration Act. FACTS: The spouses Ricardo Y. Ladrido and Leonor P. Ladrido were the owners of Lot No. 7511 of the Cadastral Survey of Pototan situated in barangay Cawayan, Pototan, Iloilo. This lot contained an area of 154,267 square meters and was registered in the names of the spouses under Transfer Certificate of Title No. T-21940 of the Register of Deeds of Iloilo. Spouses Rosendo H. Te and Ana Te were also the registered owners of a parcel of land described in their title as Lot No. 7340 of the Cadastral Survey of Pototan. Rosendo H. Te, with the conformity of Ana Te, sold this lot to Angelica F. Viajar and Celso F. Viajar. A Torrens title was later issued in the names of Angelica F. Viajar and Celso F. Viajar. Later, Angelica F. Viajar had Lot No. 7340 relocated and found out that the property was in the possession of Ricardo Y. Ladrido. Consequently, she demanded its return but Ladrido refused. Angelica F. Viajar and Celso F. Viajar instituted a civil action for recovery of possession and damages against Ricardo Y. Ladrido. Summoned to plead, defendant Ladrido filed his answer with a counterclaim. Plaintiffs filed their reply to the answer. CFI ruled in favor of the defendants which the CA confirmed. There was a mention in the case that the issue from which the decision of the CFI was not the issue appealed in the CA so the affirmation made by the CA should be void. ISSUES: 1. Whether or not the change in the course of the Suague River was sudden 2. Whether or not the plaintiffs are protected by the Torrens Title. HELD: No it was gradual. The trial court found that the change in the course of the Suague River was gradual and this finding was affirmed by the respondent Court of Appeals. We do not find any valid reason to disturb this finding of fact. Article 457 of the New Civil Code must be construed to limit the accretion mentioned therein as accretion of unregistered land to the riparian owner, and should not extend to registered land. Thus, the lot in question having remained the registered land of the petitioners, then the private respondents cannot acquire title there in derogation to that of the petitioners, by accretion, for that will defeat the indefeasibility of a Torrens Title. The rule that registration under the Torrens System does not protect the riparian owner against

the diminution of the area of his registered land through gradual changes in the course of an adjoining stream is well settled. MRAM Binalay v. Manalo 195 SCRA 374 DOCTRINE: For accretion to take place as a mode of acquiring ownership over the land, the land formed should be directly adjacent to the land owned. FACTS: Guillermo Manalo bought parcels of land in Isabela; 8.65 hectares from was acquired from Faustino Taccad and 1.80 hectares was bought from Gregorio Taguba. The parcels of land were described as having the Cagayan River on their west. During a cadastral survey conducted, the two parcels of land was consolidated into one lot and was designated as Lot No. 307. But since the survey was conducted on a rainy day, a portion of the land bought from Taccad was covered with water and was not included in Lot No. 37. The Sketch Plan shows that the Cagayan River running from south to north, forks at a certain point to form 2 branches (eastern and western branches) and then unites at the other end, further north, to form a narrow strip of land. It appears that eastern branch of the river cuts through the land of Manalo and is inundated with water during rainy season. The bed of the eastern branch is the unsurveyed portion of the land belonging to Manalo, and is, for most part of the year (about 8 months), dry and susceptible to cultivation. Lot 821 is located directly opposite Lot 307 and is separated from it during the rainy season. Being a portion of the land bought from Taccad, Manalo claims that Lot 821 also belongs to him by way of accretion to the submerged portion of the property to which it is adjacent. Petitioners (Binalay et al) however claims ownership over the land as they have possessed it, occupied it and have cultivated it. ISSUES: Whether or not Manalo owns Lot 821? HELD: The Court held in the negative saying that 1.) it is part of public dominion and 2.) it fails to meet all the requisites needed for accretion to take place. First, it must be noted that Art. 70 of the Law of Waters defines the natural bed or channel of a creek or river as the ground covered by its waters during the highest floods. The periodic swelling of the water was taken into consideration and it was concluded that the submerged portion of the land during rainy days forms part of the natural bed of the river. In connection with this conclusion, Art. 420 of the Civil Code provides that rivers form part of public dominion. Second, accretion as a mode of acquiring property under Art. 457 of the Civil Code requires the concurrence of 3 requisites: 1) that the deposition of soil or sediment be gradual and imperceptible; 2) that it be the result of the action of the waters of the river (or sea); and 3) that the land where accretion takes place is adjacent to the banks of rivers (or the sea coast). In this case, the claimed accretion lies on the bank of the river not adjacent to Lot 307 but directly opposite Lot 307 across the river.

Lastly, SC held that it is difficult to suppose that a land with an area of 22.72 hectares resulted from slow accretion to another lot of almost equal size. If Manalo‘s contention is accepted, then his land would have doubled in a span of 10 years. All these considered, the Court held that the land in question is part of public dominion and neither Manalo nor the petitioners were held owners of the land. FMM Republic v. CA 132 SCRA 514 DOCTRINE: Article 457 of the New Civil Code states that, ―To the owners of lands adjoining the banks of rivers belong the accretion which they gradually receive from the effects of the current of the waters.‖ Article 457 requires the concurrence of three requisites before an accretion is said to have taken place: (1) That the deposit be gradual and imperceptible; (2) that it be made through the effects of the current of the water; and (3) that the land where accretion takes place is adjacent to the banks of the rivers. FACTS: Tancinco (the ―Tancincos‖) are registered owners of a parcel of land situated at Barrio Ubihan, Meycauayan, Bulacan bordering on the Meycauayan and Bocaue Rivers. On 24 June 1973, the Tancincos filed the application for the registration of three lots adjacent to their fishpond property (Lots 1, 2 and 3 of plan Psu-131892). However, on 5 April 1974, Assistant Provincial Fiscal Amando C. Vicente, representing the Bureau of Lands (the ―Republic‖) filed a written opposition to the said application for registration. On 6 March 1975, in line with the recommendation of the Commissioner appointed by the Court, the Tancincos filed a partial withdrawal of the application for registration of Lot 3. On 7 March 1975, Lot 3 was ordered withdrawn from the application and trial proceeded only with respect to Lots 1 and 2. On 26 June 1976, the Court of First Instance of Bulacan rendered a decision granting the application on the finding that the lots (1 and 2) are accretions to the Tancincos‘ fishpond property. On 30 July 1976, the Republic appealed to the Court of Appeals (the ―CA‖) However, the CA, on 19 August 1982, rendered a decision affirming in toto the decision of the lower court. Hence, the Republic filed this petition with the Supreme Court (the ―SC‖). The Republic contended that there is no accretion to speak of under Article 457 of the New Civil Code because the Tancincos simply transferred their dikes further down the riverbed of the Meycauayan River, and thus, if there is any accretion to speak of, it is man-made and artificial and not the result of the gradual and imperceptible sedimentation by the waters of the river.

ISSUE: Whether there was accretion to the fishpond property of the Tancincos. -- NO HELD: The SC agreed with the Republic‘s contention that there is no accretion to speak of under Article 457 of the New Civil Code. The SC held that Article 457 of the New Civil Code which provides that ―To the owners of lands adjoining the banks of rivers belong the accretion which they gradually receive from the effects of the current of the waters‖, requires the concurrence of three requisites before an accretion is said to have taken place, namely: (1) that the deposit be gradual and imperceptible; (2) that it be made through the effects of the current of the water; and (3) that the land where accretion takes place is adjacent to the banks of rivers. The SC emphasized that the requirement that the deposit should be due to the effect of the current of the river is indispensable. It held that this excludes from Article 457 of the New Civil Code all deposits caused by human intervention. It further held that alluvion must be the exclusive work of nature. On the contrary, in the instant case, there is no evidence to prove that the addition to the fishpond property was made gradually through the effects of the current of the Meycauayan and Bocaue rivers. The alleged alluvial deposits came into being not because of the sole effect of the current of the rivers but as a result of the transfer of the dike towards the river and encroaching upon it by reclamation. Thus, the SC granted the Republic‘s petition, reversed and set aside the decisions of the lower courts and ordered the Tancincos to move back the dikes of their fishpond to their original location and return the disputed property to the river to which it belongs. RGGM Republic v. CA 131 SCRA 532 DOCTRINE: Laguna de bay is a lake and that part around it which becomes covered with water four to five months a year, not due to tidal actions, but due to rains cannot be considered as part of the bed or basin of Laguna de Bar nor as a foreshore land; Property being so, the land is registerable under the LRA. FACTS: The subject land in this case is situated 20 meters away from the shores of Laguna de Bay. Said land was owned by Benedicto del Rio. After the death of Benedicto, the land was acquired by his son Santos Del Rio. The private oppositors in this case sought and obtained permission from Santos Del Rio to construct duck houses on said land. The private oppositors, however, violated their agreement and instead constructed residential houses thereon. Santos then filed an ejectment suit against the private oppositors and later on sought to register the land. Meanwhile, private oppositors simultaneously filed their respective sales applications with Bureau of Lands, and they opposed Santos del Rio‘s application for registration. The CFI of Laguna dismissed the application for registration. Applicant appealed and obtained a favourable judgment from the Court of Appeals. The Director of Lands and the private oppositors filed their respective petitions for review on said decision to the Supreme Court.

The Director of Lands contends that since a portion of the land is covered with water four to five months a year, the same is part of the lake bed of Laguna de Bay and therefore it cannot be the subject of registration. ISSUE: 1. Whether or not the parcel of land in question is public land; and 2. Whether or not applicant private respondent has registerable title to the land. HELD: The inundation of a portion of the land is not due to "flux and reflux of tides." It cannot be considered a foreshore land, hence it is not a public land and therefore capable of registration as private property provided that the applicant proves that he has a registerable title. The purpose of land registration under the Torrens System is not the acquisition of lands but only the registration of title which applicant already possesses over the land. While it is true that by themselves tax receipts and declarations of ownership for taxation purposes are not incontrovertible evidence of ownership, they become strong evidence of ownership acquired by prescription when accompanied by proof of actual possession of the property. Applicant by himself and through his father before him, has been in open, continuous, public, peaceful, exclusive and adverse possession of the disputed land for more than thirty (30) years and has presented tax declarations and tax receipts. Applicant has more than satisfied the legal requirements. Thus, he is clearly entitled to the registration in his favor of said land. MCSS Heirs of Navarro v. IAC 268 SCRA 74 FACTS: On October 3, 1946, Sinforoso Pascual filed an application for foreshore lease covering a tract of foreshore land in Sibocon, Balanga, Bataan, having an area of approximately seventeen (17) hectares. This application was denied on January 15, 1953. So was his motion for reconsideration. Subsequently, petitioners' predecessor-in-interest, also now deceased, Emiliano Navarro, filed a fishpond application with the Bureau of Fisheries covering twenty five (25) hectares of foreshore land also in Sibocon, Balanga, Bataan. Initially the application was denied, eventually however the grant was given. Pascual claimed that this land is an accretion to his property, The Talisay River as well as the Bulacan River flow downstream and meet at the Manila Bay thereby depositing sand and silt on Pascual's property resulting in an accretion thereon. Sinforoso Pascual claimed the accretion as the riparian owner. On March 25, 1960, the Director of Lands, represented by the Assistant Solicitor General, filed an opposition thereto stating that neither Pascual nor his predecessors-in-interest possessed sufficient title to the subject property, the same being a portion of the public domain and, therefore, it belongs to the Republic of the Philippines. On November 10, 1975, the courta quorendered judgment finding the subject property to be foreshore land and, being a part of the public domain, it cannot be the subject of land registration proceedings.

On appeal, the respondent court reversed the findings of the court a quo and granted the petition for registration of the subject property but excluding certain areas. A motion for reconsideration was filed by in the CA but the same was denied. Anchoring their claim of ownership on Article 457 of the Civil Code, petitioners vigorously argue that the disputed 14hectare land is an accretion caused by the joint action of the Talisay and Bulacan Rivers which run their course on the eastern and western boundaries, respectively, of petitioners' own tract of land. ISSUE: WON the petitioners can rightfully claim the land under the principle of accretion. HELD: The petitioner‘s claim is misplaced. The principle of accretion is only applicable to owners whose estates are adjacent to rivers as stated in Article 457 of the Civil Code. The disputed land is an accretion not on a river bank but on a sea bank, or on what used to be the foreshore of Manila Bay which adjoined petitioners' own tract of land on the northern side. As such, the applicable law is not Article 457 of to Civil Code but Article 4 of the Spanish Law of Waters of 1866. The disputed property is an accretion on a sea bank, Manila Bay being an inlet or an arm of the sea; as such, the disputed property is, under Article 4 of the Spanish Law of Waters of 1866, part of the public domain. As part of the public domain, the herein disputed land is intended for public uses, and "so long as the land in litigation belongs to the national domain and is reserved for public uses, it is not capable of being appropriated by any private person, except through express authorization granted in due form by a competent authority."Only the executive and possibly the legislative departments have the right and the power to make the declaration that the lands so gained by action of the sea is no longer necessary for purposes of public utility or for the cause of establishment of special industries or for coast guard services. Petitioners utterly fail to show that either the executive or legislative department has already declared the disputed land as qualified, under Article 4 of the Spanish Law of Waters of 1866, to be the property of petitioners as owners of the estates adjacent thereto. NKVS Grande v. CA 5 SCRA 524 FACTS: Petitioners are the owners of a parcel of land, with an area of 3.5032 hectares, located at the province of Isabela, by inheritance from their deceased mother Patricia. Said property is identified as Lot No. 1. When it was surveyed for purposes of registration sometime in 1930, its northeastern boundary was the Cagayan River. Since then, and for many years thereafter, a gradual accretion on the northeastern side took place, by action of the current of the Cagayan River, so much so, that by 1958, the bank thereof had receded to a distance of about 105 meters from its original site, and an alluvial deposit of 19,964 square meters, more or less, had been added to the registered area. On January 25, 1958, petitioners instituted the present action in the CFI of Isabela against respondents, to quiet title to said portion formed by accretion, alleging in their complaint that they and their predecessors-in-interest, were formerly in peaceful and continuous possession thereof, until September, 1948, when respondents entered upon the land under claim of

ownership. Petitioners also asked for damages corresponding to the value of the fruits of the land as well as attorney's fees and costs. In their answer, respondents claim ownership in themselves, asserting that they have been in continuous, open, and undisturbed possession of said portion, since prior to the year 1933 to the present. ISSUE: 1. W/N the land formed through accretion becomes automatically registered land of the riparian owner. -- NO 2. W/N defendants acquired the property through prescription. -- YES HELD: 1st Issue: NO, land formed through accretion does not become automatically registered. There can be no dispute that both under Article 457 of the New Civil Code and Article 366 of the old, petitioners are the lawful owners of said alluvial property, as they are the registered owners of the land which it adjoins. The question is whether the accretion becomes automatically registered land just because the lot which receives it is covered by a Torrens title thereby making the alluvial property imprescriptible. We agree with the Court of Appeals that it does not, just as an unregistered land purchased by the registered owner of the adjoining land does not, by extension, become ipso facto registered land. Ownership of a piece of land is one thing, and registration under the Torrens system of that ownership is quite another. Ownership over the accretion received by the land adjoining a river is governed by the Civil Code. Imprescriptibility of registered land is provided in the registration law. Registration under the Land Registration and Cadastral Acts does not vest or give title to the land, but merely confirms and thereafter protects the title already possessed by the owner, making it imprescriptible by occupation of third parties. But to obtain this protection, the land must be placed under the operation of the registration laws wherein certain judicial procedures have been provided. The fact remain, however, that petitioners never sought registration of said alluvial property (which was formed sometime after petitioners' property covered by Original Certificate of Title No. 2982 was registered on June 9, 1934) up to the time they instituted the present action in the Court of First Instance of Isabela in 1958. The increment, therefore, never became registered property, and hence is not entitled or subject to the protection of imprescriptibility enjoyed by registered property under the Torrens system. Consequently, it was subject to acquisition through prescription by third persons. 2nd Issue: YES, the defendants acquired the property through prescription. This is a question which requires determination of facts: physical possession and dates or duration of such possession. The Court of Appeals, after analyzing the evidence, found that respondents-appellees were in possession of the alluvial lot since 1933 or 1934, openly, continuously and adversely, under a claim of ownership up to the filing of the action in 1958. This finding of the existence of these facts, arrived at by the Court of Appeals after an examination of the evidence presented by the parties, is conclusive as to them and can not be reviewed by us. The law on prescription applicable to the case is that provided in Act 190 and not the provisions of the Civil Code, since the possession started in 1933 or 1934 when the pertinent articles of the old Civil Code were not in force and before the effectivity of the new Civil Code in 1950. Hence, the conclusion of the Court of Appeals that the respondents acquired alluvial lot in question by acquisitive prescription is in accordance with law. AMPS

Ronquillo v. CA 195 SCRA 433 DOCTRINE: The rules of accretion do not apply where the drying-up of river is not due to a ―natural‖ change in the course of the waters. Herein, the change was man-made (pollution). The dried-up portion belongs to the State as land of public domain. FACTS: Plaintiff Rosendo del Rosario was a registered owner of a certain Lot No. 34. Adjoining said lot is a dried-up portion of the old Estero Calubcub occupied by the defendant since 1945. Plaintiffs claim that long before the year 1930, when title was issued over the lot to his name, Rosendo del Rosario had been in possession of this including the adjoining dried-up portion of the old Estero Calubcub having bought the same from Arsenio Arzaga. In the early part of 1945 defendant (Ronquillo) occupied the eastern portion of said titled lot as well as the dried-up portion of the old Estero Calubcub which abuts plaintiffs' titled lot. After a relocation survey of the land in question sometime in 1960, plaintiffs learned that defendant was occupying a portion of their land and thus demanded defendant to vacate said land when the latter refused to pay the reasonable rent for its occupancy. However, despite said demand defendant refused to vacate. Rosendo, Amparo and Florencia, all surnamed del Rosario (Del Rosarios), lodged a complaint with the Court of First Instance of Manila praying, among others, that they be declared the rightful owners of the dried-up portion of Estero Calubcub. The trial court upheld the complaint. On appeal, respondent court, in affirming the decision of the trial court, declared that since Estero Calubcub had already dried-up way back in 1930 due to the natural change in the course of the waters, under Article 370 of the old Civil Code, the abandoned river bed belongs to the Del Rosarios as riparian owners. Hence, the petition before the SC. Before the SC, the Director of Lands in his Comment stated: We do not see our way clear to subscribe to the ruling of the Honorable Court of Appeals on this point for Article 370 of the Old Civil Code, insofar as ownership of abandoned river beds by the owners of riparian lands are concerned, speaks only of a situation where such river beds were abandoned because of a natural change in the course of the waters. Conversely, we submit that if the abandonment was for some cause other than the natural change in the course of the waters, Article 370 is not applicable and the abandoned bed does not lose its character as a property of public dominion not susceptible to private ownership in accordance with Article 502 (No. 1) of the New Civil Code. In the present case, the drying up of the bed, as contended by the petitioner, is clearly caused by human activity and undeniably not because of the natural change of the course of the waters. ISSUE: Whether the dried-up portion of Estero Calubcub being claimed by herein petitioner was caused

by a natural change in the course of the waters; and, corollary thereto, is the issue of the applicability of Article 370 of the old Civil Code. HELD: Caused by man. Article 370, thus, does not apply. A careful perusal of the evidence presented by both parties in the case at bar will reveal that the change in the course of Estero Calubcub was caused, not by natural forces, but due to the dumping of garbage therein by the people of the surrounding neighborhood. Private respondent Florencia del Rosario, in her testimony, made a categorical statement which in effect admitted that Estero Calubcub changed its course because of the garbage dumped therein, by the inhabitants of the locality. In addition, the relocation plan which also formed the basis of respondent court's ruling, merely reflects the change in the course of Estero Calubcub but it is not clear therefrom as to what actually brought about such change. There is nothing in the testimony of lone witness Florencia del Rosario nor in said relocation plan which would indicate that the change in the course of the estero was due to the ebb and flow of the waters. On the contrary, the aforequoted testimony of the witness belies such fact, while the relocation plan is absolutely silent on the matter. The inescapable conclusion is that the dried-up portion of Estero Calubcub was occasioned, not by a natural change in the course of the waters, but through the active intervention of man. The foregoing facts and circumstances remove the instant case from the applicability of Article 370 of the old Civil Code which provides: Art. 370. The beds of rivers, which are abandoned because of a natural change in the course of the waters, belong to the owners of the riparian lands throughout the respective length of each. If the abandoned bed divided tenements belonging to different owners the new dividing line shall be equidistant from one and the other. Hence, the Del Rosarios cannot be entitled thereto supposedly as riparian owners. The dried-up portion of Estero Calubcub should thus be considered as forming part of the land of the public domain which cannot be subject to acquisition by private ownership. KGS Baes v. CA 224 SCRA 562 DOCTRINE: If the riparian owner is entitled to compensation for the damage to or loss of his property due to natural causes, there is all the more reason to compensate him when the change in the course of the river is effected through artificial means. FACTS: The government dug a canal on a private parcel of land to streamline the Tripa de Gallina creek. This lot was later acquired by Baes and registered it under his name. Said lot was subdivided into 3 parts: A, B, and C. However, Lot B was totally occupied by the canal. To resolve this, the government gave Baes a lot exactly the same area in exchange for Lot B. It was near but not contiguous to Lot C. The soil displaced by the canal was used to fill up the old bed of the creek.

Thereafter, the lots were resurveyed and subdivided wherein Lot A was designated as Lot 1-A and B. After years, the Republic of the Philippines discovered that Lot 1-B on which Baes erected an apartment building, was a filled-up portion of the Tripa de Gallina creek. Baes claimed that they own said lot as they became the owners of the old bed by virtue of Article 461. The government rejects this claim and avers that sps. Baes had already been fully compensated for it when they agreed to exchange their Lot B to another lot belonging to the government. ISSUE: Is Baes entitled for compensation for his lot which was totally occupied by the man-made canal by virtue of Article 461? -- YES HELD: If the riparian owner is entitled to compensation for the damage to or loss of his property due to natural causes, there is all the more reason to compensate him when the change in the course of the river is effected through artificial means. The loss to the petitioners of the land covered by the canal was the result of a deliberate act on the part of the government when it sought to improve the flow of the Tripa de Gallina creek. It was therefore obligated to compensate the Baeses for their loss. We find, however, that the petitioners have already been so compensated. Felix Baes was given Lot 3271-A in exchange for the affected Lot 2958-B through the Deed of Exchange of Real Property dated June 20, 1970. This was a fair exchange because the two lots were of the same area and value and the agreement was freely entered into by the parties. The petitioners cannot now claim additional compensation because, as correctly observed by the Solicitor General,. . . to allow petitioners to acquire ownership of the dried-up portion of the creek would be a clear case of double compensation and unjust enrichment at the expense of the state. The exchange of lots between the petitioners and the Republic was the result of voluntary negotiations. If these had failed, the government could still have taken Lot 2958-B under the power of eminent domain, upon payment of just compensation, as the land was needed for a public purpose. JPOT Jagualing v. CA 194 SCRA 607 DOCTRINE: Article 465 of the Civil Code -an island belongs to the owner of the land along the nearer margin as sole owner thereof; or more accurately, because the island is longer than the property of private respondents, they are deemed ipso jure to be the owners of that portion which corresponds to the length of their property along the margin of the river. FACTS: Janita Eduave inherited a parcel of land in Mindoro which was eroded and became surmountably underwater because of typhoon Ineng. She has actual possesion of land and tends to its needs. She pays taxes, even though, declared land fails in comparison to its present size. Eduave also made a loan with Luzon Surety in consideration of said land for P6000. Eventually, because of the sudden increase in size of said lot from 4,937 sq.m. to16,452 sq.m. and the formation of an island, Janita permitted petitioners to occupy land in support of her endeavors warding away informal settlers as well as planting corn and bananas in duration of their stay. Afterwhich, petitioners assail that they would be the rightful owners of the said land by

admission stating that they acquired it through prescription. ISSUE: Between the one who has actual possession of an island that forms in a non-navigable and nonfloatable river and the owner of the land along the margin nearest the island, who has the better right thereto? HELD: Although there is much controversy regarding the matter, the meat of it is stated on the issue aforementioned. In this regard the Court of Appeals did not err in applying Article 465 of the Civil Code. 12 Under this provision, the island belongs to the owner of the land along the nearer margin as sole owner thereof; or more accurately, because the island is longer than the property of private respondents, they are deemed ipso jure to be the owners of that portion which corresponds to the length of their property along the margin of the river. What then, about the adverse possession established by petitioners? Are their rights as such not going to be recognized? It is well-settled that lands formed by accretion belong to the riparian owner. 13This preferential right is, under Article 465, also granted the owners of the land located in the margin nearest the formed island for the reason that they are in the best position to cultivate and attend to the exploitation of the same. 14 In fact, no specific act of possession over the accretion is required. 15 If, however, the riparian owner fails to assert his claim thereof, the same may yield to the adverse possession of third parties, as indeed even accretion to land titled under the torrens system must itself still be registered. WHEREFORE, We find no error committed by respondent court and DENY the petition for lack of sufficient merit. The decision of respondent Court of Appeals is hereby AFFIRMED, without pronouncement as to costs. SO ORDERED. MLAV Payatas-Estate Improvement Co. v. Tuason G.R. No. L-30067 DOCTRINE: Accretions, as contemplated in Article 366 of the Civil Code, are natural incidents to land bordering on running streams and are not affected by the registration laws. It follows that registration does not protect the riparian owner against diminution of the area of his land through gradual changes in the course of the adjoining stream. FACTS: Maria de la Concepcion Martinez Canas was originally the owner of Payatas estate, principal part was bounded by the east side Mariquina river. An estate belonging to the Tuasons adjoined the river on the other side. After the initiation of the Torrens system of land registration, Maria de la Concepcion Canas had the property surveyed and obtained the certificate of title. Later on, the land, consisting of 3 parcels, A, B and C, was sold to the Payatas Estate Improvement Company. In 1920, another survey was made for subdivision purposes, and on October 15, 1924, the subdivision plans were submitted to the Court of First Instance of Rizal for approval. On October 25, 1924, the Payatas Estate Improvement Company filed another motion in which it asked that transfer certificate of title no. 8691 be cancelled as to parcels A and C but not in

regard to parcel B, the latter not being included in the subdivision. The CFI approved, and certificate of titles were issued accordingly. On March 18, 1925, the Payatas Estate Improvement Co., filed another motion alleging that the area of the subdivided land parcels A and C together with parcel B did not include all the land to which the company was entitled. The motion was accompanied by a plan of two strips of land situated along the eastern side of the Mariquina River, and the company asked that a certificate of title be issued in its favor, to which the Tuasons opposed. The CFI Denied the motion of Payatas Estate. ISSUE: WON Payatas Estate has right of ownership over the disputed area -- NO HELD: Article 366 of the Civil Code states: "any accretions which the banks of rivers may gradually receive from the effect of the current belong to the owners of the estates bordering thereon." Accretions of that character are natural incidents to land bordering on running streams and are not affected by the registration laws. It follows that registration does not protect the riparian owner against diminution of the area of his land through gradual changes in the course of the adjoining stream. Mariquina river separates the Payatas estate from the Mariquina estate and constitutes the boundary between the two estates. The river has changed its course to the prejudice of the Payatas estate on the western side of the stream and to the benefit of Mariquina estate by increasing the latter's area, and the 22 hectares now in controversy which formerly were on the Payatas side of the river are now on the Mariquina estate side. Assuming this to be true, was the change in the course of the direction of the river was caused by erosion and accretion or has it occurred through avulsion? The presumption is that the change is gradual and caused by the erosion of the Payatas bank of the river and consequent accretion to the Mariquina estate. (Martinez Canas vs. Tuason (5 Phil., 688)) It follows that the land in question is now a part of the estate and no longer pertains to the Payatas estate. Accession with Respect to Movable Property DJTV Santos v. Bernabe 54 Phil 19 DOCTRINE: Article 381 of the Civil Code states: ―If, by the will of their owners, two things of identical or dissimilar nature are mixed, or if the mixture occurs accidentally, if in the latter case the things cannot be separated without injury, each owner shall acquire a right in the mixture proportionate to the part belonging to him, according to the value of the things mixed or commingled.‖ FACTS: Plaintiff Urbano Santos deposited 778 cavans and 38 kilos of palay and appellant Pablo Tiongson deposited 1,026 cavans and 9 kilos of the same grain in defendant Jose C. Bernabe's warehouse. It does not appear that the sacks of palay deposited in Jose C. Bernabe's warehouse bore any marks or signs nor were they separated one from the other.

Pablo Tiongson filed with the Court of First Instance of Bulacan a complaint against Jose C. Bernabe, to recover the cavans and kilos palay he deposited in the defendant's warehouse. At the same time, the application of Pablo Tiongson for a writ of attachment was granted, and the attachable property of Jose C. Bernabe, including 924 cavans and 31 1/2 kilos of palay found by the sheriff in his warehouse, were attached, sold at public auction, and the proceeds thereof delivered to said defendant Pablo Tiongson, who obtained judgment in said case. Plaintiff, Urbano Santos, intervened in the attachment of the palay, but upon Pablo Tiongson's filing the proper bond, the sheriff proceeded with the attachment, giving rise to the present complaint. ISSUE: Whether or not plaintiff acquired right over the mixture of cavans and kilos of palay. -- YES HELD: The sheriff having found only 924 cavans and 31 1/2 kilos of palay in said warehouse at the time of the attachment thereof and there being no means of separating form said 924 cavans and 31 1/2 of palay belonging to Urbano Santos and those belonging to Pablo Tiongson, the following rule prescribed in article 381 of the Civil Code for cases of this nature, is applicable: Art. 381. If, by the will of their owners, two things of identical or dissimilar nature are mixed, or if the mixture occurs accidentally, if in the latter case the things cannot be separated without injury, each owner shall acquire a right in the mixture proportionate to the part belonging to him, according to the value of the things mixed or commingled. The number of kilos in a cavan not having been determined, we will take the proportion only of the 924 cavans of palay which were attached and sold, thereby giving Urbano Santos, who deposited 778 cavans, 398.49 thereof, and Pablo Tiongson, who deposited 1,026 cavans, 525.51, or the value thereof at the rate of P3 per cavan. Wherefore, the judgment appealed from is hereby modified, and Pablo Tiongson is hereby ordered to pay the plaintiff Urbano Santos the value of 398.49 cavans of palay at at the rate of P3 a cavan, without special pronouncement as to costs. So ordered. JGY Siari Valley Estate v. Lucasan 97 Phil. 987 DOCTRINE: One who has stolen a part of the stolen money must have taken the larger sum lost by the offended party. If the commingling of two things is made in bad faith, the one responsible for it will lose his share. FACTS: Siara Valley Estate filed an action to recover 200 head of cattle that were driven to the adjoining ranch of Lucasan, which the latter denied having appropriated or retained any cattle belonging to the former. Lucasan alleging that there‘s no actual evidence on the number of missing bulls and that plaintiff‘s cattle comingle with his. Trial Court: Ruled in favor of Siara Valley. ISSUE: WON Lucasan can recover his share of the cattle. -- NO

HELD: Defendant‘s cowboys and even his sons Rafael and Vicente- rounded up and drove plaintiff's cattle into his pasture; he knew he had plaintiff's cattle, but refused toreturn them despite demands by plaintiff; he even threatened plaintiff's men when the latter tried to retrieve its animals; he harassed them with false prosecutions for their attempts to get back the company's animals; he wouldn't allow plaintiff' s cowboys to get into his pasture to identify its flock; he rebranded several Siari Valley cattle with his own brand; he sold cattle without registering the sales; after some cattle impounded were entrusted to his custody as trustee, he disposed of not less than 5 head of cattle among those he received as such trustee; lastly, he disposed of much more cattle than he had a right to. One who has stolen a part of the stolen money must have taken the larger sum lost by the offended party. If the commingling of two things is made in bad faith, the one responsible for it will lose his share. JRPA Aguirre v. Pheng 18 SCRA 18 DOCTRINE: Although ordinarily, the owner would be entitled to any accession thereto, the rule is different where the works or improvements or the accession was made on the property by one who acted in good faith. FACTS: On June 28, 1954, Vicente Aldaba and Teresa V. Aldaba sold to Jesus Aguirre a circular bolted steel tank with a capacity of 5,000 gallons, for the sum of P900.00, for which the latter delivered to the sellers duly endorsed, Security Bank & Trust Company check No. 281912, in the amount of P900.00. Aguirre, however, failed to, take physical possession of the tank, having been prevented from doing so by the municipal authorities of Los Baños, Laguna (where the tank was located), in view of the claim of ownership being made by the Bureau of Public Highways. It appears, however, that Vicente and Teresa Aldaba again sold the same tank on December 2, 1954 to Zosimo Gabriel, for P900.000. Gabriel, in turn, sold it to the Leonora & Company on December 5, 1954, for P2,500.00. After some alterations and improvements made on the tank, Leonora & Company was able to sell the tank to National Shipyards & Steel Corporation (Nassco), for P14,500.00. ISSUE: 1. WON Aguirre can take ownership of the property -- YES 2. WON Aguirre should reimburse Leonora for the improvements -- YES HELD: It is clear that we have here a case of accession by specification: Leonora and Company, as purchaser acting in good faith, spending P11,299.00 for the reconditioning of the tank which is later adjudged to belong to petitioner Aguirre. There is no showing that without the works made by Leonora & Company, the tank in its original condition when Aguirre paid P900.00 therefor, would command the price of P14,500 which Nassco was willing to pay. Although ordinarily, therefore, Aguirre, as owner of the tank, would be entitled to any accession thereto, the rule is different where the works or improvements or the accession was made on the property by one who acted in good faith.2 And, it is not contended that the making of the improvements and incurring of expenses amounting to P11,299.00 by Leonora & Company was done in bad faith.

Furthermore, to uphold petitioner's contention that he is entitled to the sum of P14,500.00 the price of the tank in its present condition, would be to allow him to enrich himself at the expense of another. The lower courts, therefore, acted correctly in ordering the reimbursement to Leonora & Company of the expenses it made on the tank.

QUIETING OF TITLE AND CO-OWNERSHIP (ART. 476-501) Quieting of Title ABB Realty Sales Enterprises, Inc. v. IAC G.R. No. L-67451 DOCTRINE: One is considered an innocent purchaser for value only if, relying on the title, he bought the property from the registered owner without notice that some other person has a right to or interest in such property and pays a full and fair price for the same, at the time such purchase or before he receives notice of the claim or interest of some other persons in the property. FACTS: Two adjacent lands situated in Las Pinas were covered by three (3) distinct sets of Torrents titles. Morris Carpo, filed a complaint with the CFI (Vera Court) for declaration of nullity of Decree no. N-66934 and TCT 20408 against Realty Sales Enterprise, Inc. Macondray Farms and the Commission of Land Registration. Prior to all these, Estanislao Mayuga, father of Dominador Mayuga (Realty‘s predecessor-ininterest) originally filed on June 24, 1927 a registration proceeding to confirm his title over parcels of land (Lots 1, 2, 3; lots 2 and 3 currently the subject of the litigation). The LRC case was jointly tried with the cases filed by Eduardo Guico and Florentino Baltazar involving the same pieces of land. The Baltazars herein impleaded in this case are the heirs of Florentino Baltazar of the original application filed by Estanislao Mayuga. Carpo bought the disputed property from them. The CFI confirmed title of Estanislao Mayuga to Lots 1, 2, and 3 and held that the succesors of Florentino Baltazar cannot feign ignorance to the land registration proceedings initiated by the elder Baltazar, Guico and Mayuga. The title in the name of Dominador Mayuga from whom Realty Sales Enterprise, Inc. derived its title was issued in 1958, 12 years before the issuance of the title in the name of Baltazar in 1970. ISSUE: Whether or not Carpo was an innocent purchaser of value and in good faith. -- NO HELD: One is considered an innocent purchaser for value only if, relying on the title, he bought the property from the registered owner without notice that some other person has a right to or interest in such property and pays a full and fair price for the same, at the time such purchase or before he receives notice of the claim or interest of some other persons in the property. In the case at bar, at the time of the sale, there was as yet no Torrens title which Carpo could have relied upon so that he may qualify as an innocent purchaser for value. Therefore, Carpo‘s claim should be discredited by the court because Realty Sales Enterprise, Inc. is the one with the valid title to these properties.

Also, the general rule is that in case of two (2) certificates of title purporting to include the same land, the one earlier in date prevails. Since the title in the name of Dominado Mayuga (Predecessor-in-interest of Realty Sales Enterprise, Inc) was issued in 1958, 12 years earlier from the issuance of title in the name of Baltazar in 1970, it is held that Mayuga hold the better right to the property in question. FZC Lucasan v. PDIC G.R. No. 176926 DOCTRINE: To avail of the remedy of quieting of title, two (2) indispensable requisites must concur, namely: (1) the plaintiff or complainant has a legal or an equitable title to or interest in the real property subject of the action; and (2) the deed, claim, encumbrance or proceeding claimed to be casting a cloud on his title must be shown to be in fact invalid or inoperative despite its prima facie appearance of validity or legal efficacy. For levied properties sold on public auctions, the right to redeem becomes functus officio on the date of its expiry, and its exercise after the period is not really one of redemption but a repurchase. The judgment debtor therefore, no longer has legal or equitable title to or interest in the real property subject of the action. Thus, he may not avail of the remedy of quieting of title. FACTS: Petitioner Inocencio Y. Lucasan (Lucasan) and his wife Julianita Sorbito (now deceased) were the owners of Lot situated in Bacolod City. Pacific Banking Corporation (PBC) extended a loan to Lucasan, with Carlos Benares as his co-maker. Lucasan and Benares failed to pay the loan when it became due and demandable. Consequently, PBC filed a collection case with the RTC of Bacolod City. The RTC rendered a decision ordering Lucasan and Benares to jointly and severally pay PBC until the full payment of the obligation. Lucasan failed to pay the monetary award; thus, to satisfy the judgment, the RTC issued a writ of execution directing the sheriff to effect a levy on the properties owned by Lucasan and sell the same at public auction. In compliance with the writ, the City Sheriff of Bacolod issued a Notice of Embargo. Annotated therefore on Lucasan‘s TCTs as prior encumbrances on the same titles were the mortgages in favor of Philippine National Bank (PNB) and Republic Planter‘s Bank (RPB) executed to secure Lucasan‘s loans with the banks. The lots were later sold at a public auction where PBC was awarded the highest bidder. A certificate of sale was executed in its favor and was registered. Neither PNB nor RPB, the mortgagees, assailed the auction sale. Lucasan, as well as the mortgagee banks, PNB and RPB, did not redeem the properties within the redemption period. Nevertheless, PBC did not file a petition for consolidation of ownership. Lucasan, through counsel, wrote a letter to the Philippine Deposit Insurance Corporation (PDIC), PBC‘s receiver and liquidator seeking the cancellation of the certificate of sale and offering to pay PBC‘s claim against Lucasan. Not long thereafter, Lucasan paid his loans with the PNB and RPB. Consequently, the mortgagee banks executed their respective releases of mortgage, resulting in the cancellation of the prior encumbrances in favor of PNB and RPB. Later, PDIC denied Lucasan‘s request for the cancellation of the certificate of sale stating that the subject lots have already become part of the acquired assets of Pacific Banking Corporation by virtue of a Certificate of Sale executed by the City Sheriff of Bacolod. That reacquisition of the subject properties have to be through PDIC‘s public bidding disposal policy.

Lucasan then filed a petition denominated as declaratory relief with the RTC of Bacolod City. He sought confirmation of his rights provided in the second paragraph of Section 1, Rule 63 of the Rules of Court in relation to Section 75 of Presidential Decree (P.D.) No. 1529 and pleaded for the lifting and/or cancellation of the notice of embargo and the certificate of sale, offering to pay as consideration for the cancellation. PDIC moved to dismiss the complaint for lack of cause of action. It averred that an action to quiet title under Section 1 of Rule 63 may only be brought when there is a cloud on, or to prevent a cloud from being cast upon, the title to real property. It asseverated that a cloud on the title is an outstanding instrument record, claim, encumbrance or proceeding which is actually invalid or inoperative, but which may nevertheless impair or affect injuriously the title to property. PDIC claimed that the notice of embargo was issued pursuant to a writ of execution while the certificate of sale was executed as a result of a public bidding. Thus, their annotations on the titles were valid, operative or effective. PDIC asserted that Lucasan‘s petition is nothing but a disguised attempt to compel PDIC to resell the properties at a reduced price. Accordingly, it prayed for the dismissal of the petition. Lucasan opposed the motion. He countered that the subject properties were still in his possession, and neither PBC nor PDIC instituted an action for consolidation of ownership. Since the certificate of title was still in his name, he contended that he could pursue all legal and equitable remedies, including those provided for in Section 1, Rule 63 of the Rules of Court to reacquire the properties. He also claimed that PDIC‘s policy of disposing the subject properties through public bidding was unjust, capricious and arbitrary, considering that the judgment debt was lower in amount. The RTC granted PDIC‘s motion to dismiss, finding the claim of any cloud over the titles of [Lucasan] to be bereft of basis in fact and in law. Lucasan filed a motion for reconsideration, but the RTC denied. On appeal, the CA affirmed in toto the RTC ruling. It declared that Lucasan already lost his right to redeem the properties when he failed to exercise it within the prescribed period. The effect of such failure was to vest in PBC absolute ownership over the subject properties. Lucasan sought a reconsideration of the CA Decision, but the same was denied. ISSUE: WON Lucasan has sufficient cause of action for quieting of title. -- NO HELD: The requisites of quieting of title are wanting in this case. Quieting of title is a common law remedy for the removal of any cloud of doubt or uncertainty with respect to real property. To avail of the remedy of quieting of title, two (2) indispensable requisites must concur, namely: (1) the plaintiff or complainant has a legal or an equitable title to or interest in the real property subject of the action; and (2) the deed, claim, encumbrance or proceeding claimed to be casting a cloud on his title must be shown to be in fact invalid or inoperative despite its prima facie appearance of validity or legal efficacy. Stated differently, the plaintiff must show that he has a legal or at least an equitable title over the real property in dispute, and that some deed or proceeding beclouds its validity or efficacy. Admittedly, the subject parcels of land were levied upon by virtue of a writ of execution. They were later sold on a public auction, awarded to PBC as the highest bidder. A certificate of sale

in favor of PBC was issued. Under the Rules of Court, the judgment debtor or redemptioner had the right to redeem the property from PBC within twelve (12) months from the registration of the certificate of sale. With the expiration of the twelve-month period of redemption and no redemption having been made, the judgment debtor or the redemptioner lost whatever right he had over the land in question. Lucasan admitted that he failed to redeem the properties within the redemption period, on account of his then limited financial situation.It was only fifteen (15) years later that he manifested his desire to reacquire the properties. Clearly thus, he had lost whatever right he had over the subject lots. The payment of loans made by Lucasan to PNB and RPB cannot, in any way, operate to restore whatever rights he had over the subject properties. Such payment only extinguished his loan obligations to the mortgagee banks and the liens which Lucasan claimed were subsisting at the time of the registration of the notice of embargo and certificate of sale. Neither can Lucasan capitalize on PBC‘s failure to file a petition for consolidation of ownership after the expiration of the redemption period. Certainly, Lucasan no longer possess any legal or equitable title to or interest over the subject parcels of land; hence, he cannot validly maintain an action for quieting of title. LNAC Coronel v. IAC G.R. No. 70191 DOCTRINE: Laches has been defined as the failure or neglect, for an unreasonable and unexplained length of time, to do that which by exercising due diligence could or should have been done earlier; it is negligence or omission to assert a right within a reasonable time, warranting a presumption that the party entitled to assert it either has abandoned it or declined to assert it. Private respondents have always been in peaceful possession of the 1/3 portion of the subject lot, exercising ownership thereto for more than 25 years disrupted only in 1975 when the petitioner tried to remove them by virtue of his torrens title covering the entire Lot 1950-A of the Naic Estate. It was only at this point that private respondents knew about the supposed sale of their 1/3 portion of Lot 1950-A of the Naic Estate. FACTS: The complaint was filed against the private respondents Elias Merlan, Brigido Merlan, Jose Merlan, Teodorico Nostrates, Severo Jeciel Santiago Fernan and Fortunato Ocampo before the then CFI. Coronel alleged in his complaint that at the time he purchased the subject parcel of land, the defendants (private respondents herein) were already occupying a portion thereof as "tenants at will" and that despite demands to vacate the premises, the defendants failed and refused to move out from the land. In their Answer with Counterclaim and With Third-Party Complaint, the defendants denied that Coronel was the owner of the whole parcel of land and alleged that the lots occupied by them form part of a 1/3 undivided share of brothers Brigido Merlan and Jose Merlan which they inherited from their deceased father Gabriel Merlan, one of the three heirs of Bernabela Lontoc, the original owner of Lot No. 1950-A of the Naic Estate; that the Merlan brothers together with

their two brothers and a sister never sold their undivided 1/3 share of the lot to anybody; that it was actually their other co-heirs who sold their undivided portions and that the plaintiff's claim of ownership of the whole parcel of land, if ever it has basis, is fraudulent, void, and without effect; that the Merlans have always been in open and peaceful possession of their undivided share of the lot throughout the years from the first sale by their co-heirs of Lot No. 1950-A in 1950; and that the other defendants were legitimate tenants. They prayed that the plaintiff respect their rights over 1/3 (4,063 square meters) of Lot No. 1950-A of the Naic Estate. In their Third-Party Complaint, the defendants charged that the third-party defendants, owners of the remaining portion of Lot No. 1950-A, defrauded them when they sold the entire parcel. Third-Party Defendants Marcelo Novelo, Paz Anuat Daniel Anuat and Rosario Cailao, the defendants' co-owners of Lot No. 1950-A, denied that they had something to do with the fraudulent acts or illegal machinations which deprived the defendants of their share in the subject parcel of land, and that what they sold was only their 2/3 undivided shares in said parcel. They also filed a cross-claim against their co-defendant Mariano Manalo whom they charged might have connived with others, including the plaintiff to deprive the defendants and their coheirs of their share in the subject parcel of land. Lower court ruled in favor of the defendants and on appeal, the lower court's decision was affirmed with modification by the then IAC. Thus, herein petition. ISSUE: Whether the claim of private respondents to the land in question is barred by the statute of limitation or by estoppel by laches. -- NO HELD: No. In dispute is the 2/8 share of Bernabela Lontoc which is equivalent to 12,189 square meters. When Lontoc died in 1945, she was survived by three sets of heirs: 1) Bernardino Merlan, a grandson by her son Enrique Merlan who died in 1918; 2) Jose Merlan and Brigido Merlan, defendants in the case below and private respondents herein, Graciano Merlan, Agapito Merlan and Corazon Merlan, children of her son Gabriel who died in 1937; and 3) Daniel Anuat and Paz Anuat children of her daughter Francisca Merlan. In 1950, Bernardino Merlan, Daniel Anuat and Paz Anuat sold their 2/3 undivided portion of the lot to spouses Ignacio Manalo and Marcela Nobelo. In 1960, TCT No. (T-3116) RT-5010 was cancelled by TCT No. T-1444 but carried the same afore-specified registered co-owners with an annotation carried from the former TCT. In 1968, Lot No. 1950 of the Naic Estate was subdivided according to a Sketch Plan. The sketch plan was approved by the Commission on Land Registration. Bernabela Lontoc's 2/8 portion of Lot No. 1950 became Lot No. 1950-A with an area of 12,189 square meters. Sometime in 1970, Ignacio Manalo sold his interest in Lot 1950-A to Mariano Manalo. Thereafter, TCT No. T-1444 was cancelled and TCT No. T-41175 was issued for Lot No. 1950A of the Naic Estate in the name of Mariano Manalo married to Jorga Lagos. The certificate of

title issued in the name of spouses Mariano Manalo and Jorga Lagos covered the whole Lot No. 1950-A without any mention of the 1/3 share of the private respondents in the parcel of land which was not sold to them. Relying on the TCT of the spouses Mariano Manalo and Jorga Lagos and the Sketch Plan, petitioner Coronel then bought Lot No. 1950-A of the Naic Estate from the former for the consideration of P27,000.00. The deed of sale was registered on December 19, 1974 causing the cancellation of TCT No. T-41175 and the issuance of TCT No. T-75543 in the name of petitioner Coronel. It is evident that the private respondents never sold their 1/3 share over Lot No. 1950-A of the Naic Estate; that what their co-owners sold to Ignacio Manalo was their 2/3 share of the same lot; and that Ignacio Manalo sold only the 2/3 share to third-party defendant Mariano Manalo, the predecessor-in-interest of petitioner Coronel. There was a mistake when TCT No. 41175 was issued to Mariano Manalo covering the whole area of Lot No. 1950-A. Unfortunately, Mariano Manalo who was included as third-party defendant as well as the subject of a crossclaim filed by the other third-party defendants, and who could have shed light on this controversy was at the time residing abroad and was not served with the third-party complaint. There is no bar based on laches to assert their right over 1/3 of the disputed property. Laches has been defined as the failure or neglect, for an unreasonable and unexplained length of time, to do that which by exercising due diligence could or should have been done earlier; it is negligence or omission to assert a right within a reasonable time, warranting a presumption that the party entitled to assert it either has abandoned it or declined to assert it. Private respondents have always been in peaceful possession of the 1/3 portion of the subject lot, exercising ownership thereto for more than 25 years disrupted only in 1975 when the petitioner tried to remove them by virtue of his torrens title covering the entire Lot 1950-A of the Naic Estate. It was only at this point that private respondents knew about the supposed sale of their 1/3 portion of Lot 1950-A of the Naic Estate and they immediately resisted. Neither the private respondents nor their co-owners of the subject parcel of land sold the former's share of the lot. Furthermore, even Ignacio Manalo to whom the third-party defendants sold their share resold only the 2/3 shares to Mariano Manalo, the successor-in-interest of the petitioner. Whether or not there was fraud or just a mistake or oversight of an employee of the Register of Deeds of Cavite is not clear from the records. The point is that the 1/3 undivided portion of the private respondents over Lot No. 1950-A was mistakenly included in the TCT of Mariano Manalo. TKDC Sapto v. Fabiana 103 Phil 683 DOCTRINE: The statute of limitations is not a defense to an action to remove a cloud from title when complainant is in possession of the property. However, the statute of limitations is a defense when the property is in another‘s possession; the complainant must invoke his remedy within the statutory period. FACTS: Upon his death, Moro was left a parcel of land registered under his name to his children, Samuel, Constancio, and Ramon.

After Ramon‘s death, Samuel and Constancio executed a deed of sale of a portion of the property in favor of Fabiana; the sale was approved by the Provincial Governor but was unregistered. Since 1931 when possession of conveyed land was transferred, Fabiana has possessed the property. After Constancio‘s death, Samuel married Dora and had two children. After Samuel‘s death, his widow and children filed an action with the Court of First Instance to recover the parcel of land sold to Fabiana. Trial court held that, although the sale was unregistered, it was valid and binding upon the parties and also the vendors‘ heirs; trial court ordered the Samuel‘s heirs to execute the necessary deed of conveyance in Fabiana‘s favor. Samuel‘s heirs appealed to the Supreme Court claiming that Fabiana‘s action had long prescribed; twenty years had lapsed since the original sale. ISSUE: Whether or not Fabiana‘s action had already prescribed since twenty years had passed since the original sale. -- NO HELD: The statute of limitations is not a defense to an action to remove a cloud from title when complainant is in possession of the property. However, the statute of limitations is a defense when the property is in another‘s possession; the complainant must invoke his remedy within the statutory period. In this case, prescription does not run again Fabiana because she already possesses the property. Thus, her action may prosper. AMD Titong v. CA 287 SCRA 102 DOCTRINE: The ground or reason for filing a complaint for quieting of title must therefore be "an instrument, record, claim, encumbrance or proceeding" which constitutes or casts a cloud, doubt, question or shadow upon the owner's title to or interest in real property. FACTS: Mario Titong filed an action for quieting of title against Victorico and Angeles Laurio. Petitioner alleged that he was the owner of an unregistered parcel of land. He claimed that private respondents (Laurios), with heir hired laborers, forcibly entered a portion of the land containing an area of approximately 2 hectares; and began plowing the same under pretext of ownership. The Laurios denied this allegation, and averred that the disputed property formed part of the 5.5-hectare agricultural land which they had purchased from their predecessor-in-interest, Pablo Espinosa, an adjoining owner of Titong‘s land. The RTC ruled in favor of private respondents, declaring Laurio as the true and absolute owner of the property. Petitioner appealed to the CA, the RTC decision was affirmed and the MR was denied. Hence, this petition. ISSUE: W/N the action for quieting of title should have prospered. -- NO HELD: The SC held that the instant petition must be denied for the reason that the lower court should have outrightly dismissed the complaint for quieting of title. The remedy of quieting of title may be availed of under the circumstances enumerated in the Civil Code:

“Art. 476. Whenever there is a cloud on title to real property or any interest therein, by reason of any instrument, record, claim, encumbrance or proceeding which is apparently valid or effective but is in truth and in fact invalid, ineffective, voidable, or unenforceable, and may be prejudicial to said title, an action may be brought to remove such cloud or to quiet the title. An action may also be brought to prevent a cloud from being cast upon title to real property or any interest therein.” Under this provision, a claimant must show that there is an instrument, record, claim, encumbrance or proceeding which constitutes or casts a cloud, doubt, question or shadow upon the owner's title to or interest in real property. The ground or reason for filing a complaint for quieting of title must therefore be "an instrument, record, claim, encumbrance or proceeding." Under the maxim expressio unius est exclusio alterius, these grounds are exclusive so that other reasons outside of the purview of these reasons may not be considered valid for the same action. Had the lower court thoroughly considered the complaint filed, it would have had no other course of action under the law but to dismiss it. The complaint failed to allege that an "instrument, record, claim, encumbrance or proceeding" beclouded the plaintiff's title over the property involved. Petitioner merely alleged that the respondents, together with their hired laborers and without legal justification, forcibly entered the southern portion of the land of the plaintiff and plowed the same. He then proceeded to claim damages and attorney's fees. Hence, through his allegations, what petitioner imagined as clouds cast on his title to the property were private respondents' alleged acts of physical intrusion into his purported property. Clearly, the acts alleged may be considered grounds for an action for forcible entry but definitely not one for quieting of title. In addition, when the issues were joined by the filing of the answer to the complaint, it would have become apparent to the court that the case was a boundary dispute. MPF Pingol v. CA 226 SCRA 118 DOCTRINE: A vendee (buyer) in an oral contract to convey land who had made part payment thereof, entered upon the land and had made valuable improvements thereon is entitled to bring suit to clear his title against the vendor who had refused to transfer the title to him. It is not necessary that the vendee should have an absolute title, an equitable title being sufficient to clothe him with personality to bring an action to quiet title. FACTS: ● 1969 - Pingol, the owner of a lot in Caloocan City, executed a DEED OF ABSOLUTE SALE OF ONE-HALF OF AN UNDIVIDED PORTION OF [his] PARCEL OF LAND (274.5 sqm) in favor of Donasco (private respondent), payable in 6 years. And Donasco agreed to pay in monthly basis (for 72 months). ○ Both agreed that in case of default in the payment due the same should earn a legal interest. ○ Donasco took possession immediately and constructed a house thereon. ● 1984 - Donasco died and was only able to pay P8,369 plus P2,000 downpayment, leaving a balance of P10,161. ○ The heirs of Donasco remained in possession of such lot and offered to settle the



balance with Pingol. However, Pingol refused to accept the offer and demanded a larger amount. ■ Thus, the heirs of Donasco filed an action for specific performance (with Prayer for Writ of Prelim. Injunction, because Pingol were encroaching upon Donasco‘s lot). ■ Pingol averred that the sale and transfer of title was conditional upon the full payment of Donasco (contract to sell, not contract of sale). ■ With Donasco‘s breach of the contract in 1976 and death in 1984, the sale was deemed cancelled, and the continuous occupancy of the heirs was only being tolerated by Pingol.

RTC: ordering heirs of Donasco to pay monthly rental to Pingol CA: ordering Pingol to accept the sum of P10,161 plus legal interest from heirs of Donasco as payment to the land in question ISSUES: 1. Whether or not Pingol can refuse to transfer title to Donasco. -- NO 2. Whether or not Donasco has the right to quiet title. HELD: 1. The contract between Pingol and Donasco is a contract of sale and not a contract to sell. The acts of the parties, contemporaneous and subsequent to the contract, clearly show that the parties intended an absolute deed of sale; the ownership of the lot was transferred to the Donasco upon its actual (upon Donasco‘s possession and construction of the house) and constructive delivery (upon execution of the contract). The delivery of the lot divested Pingol of his ownership and he cannot recover the title unless the contract is resolved or rescinded under Art. 1592 of NCC. It states that the vendee may pay even after the expiration of the period stipulated as long as no demand for rescission has been made upon him either judicially or by notarial act. Pingol neither did so. Hence, Donasco has equitable title over the property. 2. Although the complaint filed by the Donascos was an action for specific performance, it was actually an action to quiet title. A cloud has been cast on the title, since despite the fact that the title had been transferred to them by the execution of the deed of sale and the delivery of the object of the contract, Pingol adamantly refused to accept the payment by Donascos and insisted that they no longer had the obligation to transfer the title. Donasco, who had made partial payments and improvements upon the property, is entitled to bring suit to clear his title against Pingol who refused to transfer title to him. It is not necessary that Donasco should have an absolute title, an equitable title being sufficient to clothe him with personality to bring an action to quiet title. Prescription cannot also be invoked against the Donascos because an action to quiet title to property in ONE‘s POSSESSION is imprescriptible. SC: appealed decision affirmed AMDG Gallar v. Hussain 20 SCRA 186

DOCTRINE: An action to quiet title, brought by a person who is in possession of the property, is imprescriptible. However, if the plaintiff is not in possession, the action would prescribe within the proper period. FACTS: Teodoro Husain is the owner of the land to which he sold to Serapio Chichirita for P30 with the right to repurchase of such land within six years. Teodoro did not avail his right to repurchase however, his sister, Graciana Husain, bought the said land shortly after the sale between Teodoro and Chichirita. It is alleged that that the sale was described to be a resale of the land. Afterwhich, Graciana sold it to appellee, Elias Gallar in exchange for one cow. Owner‘s duplicate of the TCT was given to Gallar and since then has been the possessor of the subject land. Gallar was unsuccessful when he went to the Cadastral Court to ask that the TCT be issued under his name. It is noted however that the Cadastral Court granted his request as regards amending the certificate and changing the registered owner of the land to Husain. Gallar then filed the suit in the CFI (now RTC) to compel the heirs of Teodoro Husain (Heremenegilda and Bonifacio) to execute the deed of conveyance in his favor to which the latter refused to execute. The heirs were denied and alleged that when Graciana bought the land from Serapio, she was exercising the right to repurchase granted to Teodoro. Also, they invoked that prescription has set in which bars the appellee‘s action. ISSUES: 1. WON the land was acquired by Graciana Husain on behalf of his brother Teodoro? -- NO 2. WON this action this is action is for specific performance? -- NO 3. WON appellee‘s action is imprescriptible? -- YES HELD: The right to repurchase may only be availed by the person to whom such right was given to or to the person to whom such right was transferred to. There was no evidence whatsoever to prove that Graciana acquired the land on behalf of his brother. Graciana bought the land subject only to his brother‘s right of remdeption. Therefore, being the owner, she has every right to sell the land which she did to Elias. When Teodoro failed to exercise his right of redemption, its ownership became consolidated in the appellee. Though the sale was in a private document, such was still considered valid. The delivery of the land and title to Gallar indicates that a sale was consummated. This action is not for a specific performance but a mere quieting of title. The object of the action was to remove the cloud cast on Gallar‘s ownership because of the heir‘s refusal to recognize the sale. Lastly being the Gallar is in possession when this action was constituted, the action is imprescriptible. The heir‘s argument as regards prescription would have been granted by the court in their favor if the land was not possessed by Gallar. GCG Vda de Aviles v. CA 264 SCRA 473 DOCTRINE: Quieting of Title Not Proper Remedy For Settling Boundary Dispute. Quieting of title is a common law remedy for the removal of any cloud upon or doubt or uncertainty with

respect to title to real property. FACTS: Petitioners aver that they are the actual possessors of a parcel of land situated in Malawa, Lingayen, Pangasinan, more particularly described as fishpond, cogonal, unirrigated rice and residential land, bounded on the N by Camilo Aviles; on the E by Malawa River, on the S by Anastacio Aviles and on the W by Juana and Apolonio Joaquin, with an area of 18,900 square meters and declared under Tax Declaration No. 31446. This property is the share of their father, Eduardo Aviles and brother of the defendant, in the estate of their deceased parents. Eduardo Aviles was in actual possession of the afore-described property since 1957. In fact, the latter mortgaged the same with the Rural Bank and Philippine National Bank branch in Lingayen. When the property was inspected by a bank representative, Eduardo Aviles, in the presence of the boundary owners, namely, defendant Camilo Aviles, Anastacio Aviles and Juana and Apolonio Joaquin pointed to the inspector the existing earthen dikes as the boundary limits of the property and nobody objected. When the real estate mortgage was foreclosed, the property was sold at public auction but this was redeemed by plaintiffs‘ mother and the land was subsequently transferred and declared in her name. Defendant Camilo Aviles asserted a color of title over the northern portion of the property with an area of approximately 1,200 square meters by constructing a bamboo fence (thereon) and moving the earthen dikes, thereby molesting and disturbing the peaceful possession of the plaintiffs over said portion. Defendant Camilo Aviles admitted the agreement of partition executed by him and his brothers, Anastacio and Eduardo. The respective area(s) alloted to them was agreed and measured before the execution of the agreement but he was not present when the measurement was made. Defendant agreed to have a smaller area because his brother Eduardo asked him that he wanted a bigger share because he has several children to support. The portion in litigation however is part of the share given to him in the agreement of partition. At present, he is only occupying a smaller than his actual share. Tax Declarations Nos. 23575, 481 and 379 covering his property from 1958 show that the area of his property is 14,470 square meters. The riceland portion of his land is 13,290 square meters, the fishpond portion is 500 square meters and the residential portion is 680 square meters, or a total of 14,470 square meters. That the topography of his land is not the same, hence, the height of his pilapils are likewise not the same. The trial court disposed of the case thus ordering the parties to employ the services of a Land Surveyor of the Bureau of Lands, Region I, San Fernando, La Union, to relocate and determine the extent and the boundary limit of the land of the defendant on its southern side in order that the fourteen thousand four hundred seventy (14,470) square meters which is the actual area given to the defendant be determined. It ordered the complaint dismissed for lack of basis and merits Dissatisfied with the trial court‘s decision, petitioners appealed to the respondent appellate Court. In its now-assailed Decision, the Court of Appeals affirmed in part the decision of the trial court, reasoning that a special civil action for quieting of title is not the proper remedy for settling a boundary dispute, and that petitioners should have instituted an ejectment suit instead. It affirmed the decision of the trial court in dismissing the complaint.

ISSUE: Whether or not the Hon. Court of Appeals is correct when it opined that the complaint for quieting of title instituted by the petitioners against private respondent before the court a quo is not the proper remedy but rather, it should be a case for ejectment. HELD: The Supreme Court ruled that Quieting of Title Not Proper Remedy For Settling Boundary Dispute. Quieting of title is a common law remedy for the removal of any cloud upon or doubt or uncertainty with respect to title to real property. The Civil Code authorizes the said remedy in the following language: ―Art. 476. Whenever there is a cloud on title to real property or any interest therein, by reason of any instrument, record, claim, encumbrance or proceeding which is apparently valid or effective but is, in truth and in fact, invalid, ineffective, voidable, or unenforceable, and may be prejudicial to said title, an action may be brought to remove such cloud or to quiet the title. An action may also be brought to prevent a cloud from being cast upon a title to real property of any interest therein." In fine, to avail the remedy of quieting of title, a plaintiff must show that there is an instrument, record, claim, encumbrance or proceeding which constitutes or casts a cloud, doubt, question or shadow upon the owner‘s title to or interest in real property. Thus, petitioners have wholly misapprehended the import of the foregoing rule by claiming that respondent Court erred in holding that there was ―no xxx evidence of any muniment of title, proceeding, written contract, xxx‖, and that there were, as a matter of fact, two such contracts, (i) the Agreement of Partition executed by private respondent and his brothers (including the petitioners‘ father and predecessor-in-interest), in which their respective shares in the inherited property were agreed upon, and (ii) the Deed of Sale evidencing the redemption by petitioner Anastacia Vda. de Aviles of the subject property in a foreclosure sale. However, these documents in no way constitute a cloud or cast a doubt upon the title of petitioners. Rather, the uncertainty arises from the parties‘ failure to situate and fix the boundary between their respective properties. As correctly held by the respondent Court, both plaintiffs and defendant admitted the existence of the agreement of partition and in accordance therewith, a fixed area was alloted (sic) to them and that the only controversy is whether these lands were properly measured. There is no adverse claim by the defendant ―which is apparently valid, but is, in truth and in fact, invalid, ineffective, voidable, or unenforceable‖ and which constitutes a cloud thereon. Corollarily, and equally as clear, the construction of the bamboo fence enclosing the disputed property and the moving of earthen dikes are not the ―clouds‖ or ―doubts‖ which can be removed in an action for quieting of title. An action to quiet title or to remove cloud may not be brought for the purpose of settling a boundary dispute. The precedent on this matter cited by the respondent Court in its Decision is herewith reproduced in full. VCL IV Oblea v. CA 244 SCRA 101 DOCTRINE: The pendency of an action for quieting of title before the RTC does not divest the

MTC of its jurisdiction to proceed with the ejectment case over the same property. The subsequent acquisition of ownership by petitioners is not a supervening event that will bar the execution of the judgment in said unlawful detainer case. FACTS: The lot in issue was originally registered in the names of Manuel Melencio, Pura Melencio, Wilfredo Wico and Mariabelle Wico. But was subsequently re-registered in the name of Ramon Melencio (son of deceased Manuel Melencio), Pura Melencio and the Wicos via a deed of sale. On 6 June 1958 subject lot was bought by private respondent Juan S. Esteban from Mauricio Ramos who claimed to have acquired the property from Ursula Melencio, the alleged administratrix of the estate of Manuel and Pura Melencio. Meanwhile, petitioner Romeo V. Oblea leased a building located on the subject lot from a certain Marius Esteban, an alleged son of private respondent Juan S. Esteban. Oblea eventually bought from Marius the lot on which the building stood. As a consequence, on 4 July 1991 Juan Esteban filed an ejectment suit against petitioner Oblea. MTC decided for Juan Esteban and ordered Oblea to vacate and pay arrears. RTC affirmed MTC‘s decision. On 3 June 1993, the registered owners (Ramon Melencio, Pura Melencio and Wilfredo Wico and Mariabelle Wico) sold the disputed lot to petitioner Oblea. Afterwards, Oblea together with the registered owners filed before the RTC an action for quieting of title against Juan Esteban. They contended that the deeds of sale executed by Mauricio Ramos in favor of Juan Esteban and by Ursula Melencio in favor of Mauricio Ramos were a nullity. Meanwhile, the ejectment case was appealed thrice in the CA, but all were denied. In the appeal to the SC, Oblea asserts that the subsequent sale to him by the registered owners is a supervening event that gave him a better right of possession and ownership. Hence the judgment of eviction can no longer be enforced. ISSUE: Whether or not a subsequent action to quiet title in the RTC divests the MTC of its jurisdiction over an ejectment case -- NO HELD: The sole issue in an action for unlawful detainer is physical or material possession, i.e., possession de facto and not possession de jure. The pendency of an action for quieting of title before the RTC does not divest the MTC of its jurisdiction to proceed with the ejectment case over the same property. The subsequent acquisition of ownership by petitioners is not a supervening event that will bar the execution of the judgment in said unlawful detainer case, the fact remaining that when judgment was rendered by the MTC in the ejectment case, petitioner Oblea was a mere possessor of the subject lot. Similarly, the fact that petitioners instituted a separate action for quieting of title is not a valid reason for defeating the execution of the summary remedy of ejectment. On the contrary., it bolsters the conclusion that the eviction case did not deal with the issue of ownership which was precisely the subject matter of the action for quieting of title before the RTC. With the finality of the decision in the ejectment case, execution in favor of the prevailing party has become a

matter of right; its implementation mandatory. It cannot be avoided. FXRL Gapacan v. Omipet 387 SCRA 383 DOCTRINE: An action to Quiet Title is a valid remedy to ascertain property rights. FACTS: ● Gapacan is a native Igorot possessor of a parcel of land in Bauko, Mr. Province. The property is divided into three and declared by him for taxation purposes. ● He has two children. Maria and Antonio. ● Antonio left to work in mining while Maria remained and eventually took over management and cultivation of the property. ● Antonio returned to their home and thereafter executed an Affidavit of Transfer of Real Property which now makes him the legal owner of the property in question as it transfers ownership from his sister to him. ● Since then, the family of Antonio (Gapacan) had been occupying and cultivating the property. ● The family of Maria (Omipet) ISSUE: W/N property rights may be determined by an action to Quiet Title. -- YES HELD: Art. 476 of the Civil Code provides that an action to quiet title may be brought when there exists a cloud on the title to a real property or any interest therein. The property owner whose property rights were being disturbed may ask a competent court for a proper determination of the respective rights of the party-claimants, places things in their proper place by: 1. Requiring the party with no right over the property to refrain from acts injurious to the peaceful enjoyment of the property by rightful owner and 2. Mutually benefitting both parties with the view of dissipating any cloud of doubt over the property. The appellate court in resolving the present controversy is well within its authority to adjudicate on the respective rights of the parties, that is, to pass upon the ownership of the property; hence to declare the same as common property. The court also noted that Omipet did not present sufficient evidence to overcome Gapacan‘s better right to possession. The Supreme Court ruled that CA was correct in its determination that the land in dispute is common property and must be partitioned. RSDM Robles v. CA 328 SCRA 97 DOCTRINE: An action may also be brought to prevent a cloud from being cast upon title to real

property or any interest therein. FACTS: Leon Robles primitively owned the land situated in Kay Taga, Lagundi, Morong, Riza. He also declared the same in his name for taxation purposes as early as 1916 and paid the corresponding taxes thereon. When Leon Robles died, his son Silvino Robles inherited the land, who took possession of the land, declared it in his name for taxation purposes and paid the taxes thereon. Upon the death of Silvino Robles, his widow Maria de la Cruz and his children inherited the property. They took adverse possession of said property and paid taxes thereon. The task of cultivating the land was assigned to plaintiff Lucio Robles who planted trees and other crops. The plaintiffs entrusted the payment of the land taxes to their co-heir and half-brother, Hilario Robles. In 1962, for unknown reasons, the tax declaration of the parcel of land in the name of Silvino Robles was canceled and transferred to one Exequiel Ballena, father of Andrea Robles who is the wife of defendant Hilario Robles. Thereafter, Exequiel Ballena secured a loan from the Antipolo Rural Bank, using the tax declaration as security. Somehow, the tax declaration was transferred to the name of Antipolo Rural Bank and later on, was transferred to the name of defendant Hilario Robles and his wife. In 1996, Andrea Robles secured a loan from the Cardona Rural Bank, Inc., using the tax declaration as security. Andrea Robles testified without contradiction that somebody else, not her husband Hilario Robles, signed the loan papers because Hilario Robles was working in Marinduque at that time as a carpenter. For failure to pay the mortgage debt, foreclosure proceedings were had and defendant Rural Bank emerged as the highest bidder during the auction sale in October 1968. "The spouses Hilario Robles failed to redeem the property and so the tax declaration was transferred in the name of defendant Rural Bank. Rural Bank sold the same to the Spouses Vergel Santos and Ruth Santos. Plaintiff discovered the mortgage and attempted to redeem the property, but was unsuccessful. Defendant spouses Santos took possession of the property in question and was able to secure Free Patent in their names. ISSUE: Whether or not the petitioners have the appropriate title that will entitle them to avail themselves of the remedy of quieting of title. -- YES HELD: Undisputed is the fact that the land had previously been occupied by Leon and later by Silvino Robles, petitioners' predecessor-in-interest, as evidenced by the different tax declarations issued in their names. Also undisputed is the fact that the petitioners continued occupying and possessing the land from the death of Silvino in 1942 until they were allegedly ousted therefrom in 1988. The failure to show the indubitable title of Exequiel to the property in question is vital to the resolution of the present Petition. It was from him that Hilario had allegedly derived his title thereto as owner, an allegation which thereby enabled him to mortgage it to the Rural Bank of Cardona. The occupation and the possession thereof by the petitioners and their predecessors-

in-interest until 1962 was not disputed, and Exequiel's acquisition of the said property by prescription was not alleged. Thus, the deed of conveyance purportedly evidencing the transfer of ownership and possession from the heirs of Silvino to Exequiel should have been presented as the best proof of that transfer. No such document was presented, however. Therefore, there is merit to the contention of the petitioners that Hilario mortgaged the disputed property to the Rural Bank of Cardona in his capacity as a mere co-owner thereof. Clearly, the said transaction did not divest them of title to the property at the time of the institution of the Complaint for quieting of title. Contrary to the disquisition of the Court of Appeals, Hilario effected no clear and evident repudiation of the co-ownership. It is a fundamental principle that a co-owner cannot acquire by prescription the share of the other co-owners, absent any clear repudiation of the co-ownership. In order that the title may prescribe in favor of a co-owner, the following requisites must concur: (1) the co-owner has performed unequivocal acts of repudiation amounting to an ouster of the other co-owners; (2) such positive acts of repudiation have been made known to the other coowner; and (3) the evidence thereof is clear and convincing. In the present case, Hilario did not have possession of the subject property; neither did he exclude the petitioners from the use and the enjoyment thereof, as they had indisputably shared in its fruits. Likewise, his act of entering into a mortgage contract with the bank cannot be construed to be a repudiation of the co-ownership. As absolute owner of his undivided interest in the land, he had the right to alienate his share, as he in fact did. Neither should his payment of land taxes in his name, as agreed upon by the co-owners, be construed as a repudiation of the co-ownership. The assertion that the declaration of ownership was tantamount to repudiation was belied by the continued occupation and possession of the disputed property by the petitioners as owners. MRAM Metropolitan Bank v. Alejo 364 SCRA 812 DOCTRINE: In a suit to nullify an existing TCT in which a real estate mortgage is annotated, the mortgagee is an indispensable party. In such suit, a decision canceling the TCT and the mortgage annotation is subject to a petition for annulment of judgment, because the non-joinder of the mortgagee deprived the court of jurisdiction to pass upon the controversy. FACTS: Spouses Acampado obtained a loan from petitioner and as security for their payment of the credit accommodations, they executed a real estate mortgage in Valenzuela City. The land was also located in Valenzuela and the TCT covering it is registered in the Registry of deeds in the same city. On June 1996, a Complaint for Declaration of Nullity of TCT was filed by Respondent Sy Tan Se against Spouses Acampado. Despite being the registered mortgagee of the real property covered by the title sought to be annulled, petitioner was not made a party to the case nor was it notified of its existence. The spouses defaulted in the payment of their loan. Thereafter,extrajudicial foreclosure proceedings over the mortgaged property were initiated and then the sheriff of Valenzuela conducted an auction sale of the property, during which petitioner submitted the highest and

winning bid. A Certificate of Sale was issued in its favor and the sale was entered in the Registry of Deeds of Valenzuela. When the redemption period lapsed exactly a year after, petitioner executed an Affidavit of Consolidation of Ownership to enable the Registry of Deeds of Valenzuela to issue a new TCT in its name however petitioner was informed that an RTC decision declared the TCT as null and void for having proceeded from an illegitimate source. ISSUE: Whether or not the judgment of the RTC that declared the TCT as null and void should be annulled? HELD: The Court held in the affirmative. The Rules of Court provide that no final determination can be had of an action wherein the indispensable party is not joined in the complaint either as the defendant or the complainant. It is undisputed that the property covered by the TCT was mortgaged to petitioner, and that the mortgage was annotated on it before the institution of Civil Cas. It is also undisputed that all subsequent proceedings pertaining to the foreclosure of the mortgage were entered in the Registry of Deeds. The nullification and cancellation of TCT carried with it the nullification and cancellation of the mortgage annotation. Although a mortgage affects the land itself and not merely the TCT covering it, the cancellation of the TCT and the mortgage annotation exposed petitioner to real prejudice. Evidently, the nullification of the TCT adversely affected its property rights, considering that a real mortgage is a real right and a real property by itself. Because petitioner falls under the definition of an indispensable party, it should have been impleaded as defendant in the civil case questioning the title. FMM Sps. Benito v. Saquitan Ruiz 394 SCRA 250 DOCTRINE: A petition for the quieting of title, although essentially an action for reconveyance, should not be dismissed on the ground of prescription, if it is alleged that the plaintiff is in possession of the property. FACTS: On 1 April 1999, Agapita Saquitan-Ruiz (―Saquitan-Ruiz‖) filed against Spouses Horacio and Felisa Benito (―Spouses Benito‖) a civil suit for ―specific performance with declaration of nullity of titles and damages‖. The Complaint alleged that the couple had sold in her favor a property located in Pasig City and despite repeated demands, they failed to deliver or cause the issuance of a new certificate of title in her name. However, it turned out that instead of issuing or delivering to Saquitan-Ruiz her certificate of title over the said property, Spouses Benito re-subdivided the property into five lots. On 28 June 1999, the Regional Trial Court (the ―RTC‖) dismissed Saquitan-Ruiz‘s Complaint on

the grounds of prescription and/or laches. It held that from the moment the contract was perfected, the parties could reciprocally demand performance of their obligations. There was a breach of obligation when, despite repeated demands, Spouses Benito failed to deliver to respondent the corresponding certificate of title to the lot. She, however, failed to file any action to compel performance until 16 April 1999, or 20 years from the time of the execution of the Deed of Absolute Sale on 17 April 1979. Moreover, the assailed Certificates of Title had been issued 25 March 1996, or more than one year before the Complaint was filed. An action to invalidate title certificates on the ground of fraud prescribes upon the expiration of one year from the entry of the decree of registration. Upon appeal with the Court of Appeals (the ―CA‖), the latter reversed the RTC‘s decision. It held that the Saquitan-Ruiz‘s second cause of action was for reconveyance, not for the invalidation of certificates of title. As long as the property was still in the name of the person who had caused the wrongful registration, and as long as it had not yet passed to an innocent purchaser for value, an action for reconveyance was still available. Such cause of action prescribes in ten (10) years, counted from the date of the issuance of the assailed certificate of title. Since the Complaint alleged that the questioned titles had been issued on 25 March 1996, the cause of action for reconveyance has not prescribed. Thus this petition filed by Spouses Benito. They contended that the action for reconveyance has been rendered moot and academic, because the disputed lot was already sold to Basilia dela Cruz at a public auction. They maintain that although the action for reconveyance may not have expired, the exercise of the right is no longer feasible, because the property was already transferred in good faith and for value to a third party. ISSUE: Whether the cause of action for reconveyance has already prescribed -- NO HELD: The SC noted that the Spouses Benito is in possession of the disputed property. Thus the SC held that if a person claiming to be the owner of a wrongfully registered parcel of land is in actual possession, the right to seek reconveyance does not prescribe. A petition for the quieting of title, although essentially an action for reconveyance, should not be dismissed on the ground of prescription, if it is alleged that the plaintiff is in possession of the property. Ruinous Buildings CRF Nakpil and Sons v. CA 144 SCRA 596 DOCTRINE: To be exempt from liability due to an act of God, the engineer/architect/contractor must not have been negligent in the construction of the building. (batasnatin) FACTS: The plaintiff, Philippine Bar Association decided to construct an office building on its 840 square meters lot located at the comer of Aduana and Arzobispo Streets, Intramuros, Manila. The construction was undertaken by the United Construction, Inc. on an "administration" basis, on the suggestion of Juan J. Carlos, the president and general manager of said corporation. The

plans and specifications for the building were prepared by the other third-party defendants Juan F. Nakpil & Sons. The building was completed in June, 1966. In the early morning of August 2, 1968 an unusually strong earthquake hit Manila and its environs and the building in question sustained major damage. The tenants vacated the building in view of its precarious condition. As a temporary remedial measure, the building was shored up by United Construction, Inc. at the cost of P13,661.28. On November 29, 1968, the plaintiff commenced this action for the recovery of damages arising from the partial collapse of the building against United Construction, Inc. and its President and General Manager Juan J. Carlos as defendants. Plaintiff alleges that the collapse of the building was accused by defects in the construction, the failure of the contractors to follow plans and specifications and violations by the defendants of the terms of the contract. Defendants in turn filed a third-party complaint against the architects who prepared the plans and specifications, alleging in essence that the collapse of the building was due to the defects in the said plans and specifications. Finally, on April 30, 1979 the building was authorized to be demolished at the expense of the plaintiff, but not another earthquake of high intensity on April 7, 1970 followed by other strong earthquakes on April 9, and 12, 1970, caused further damage to the property. The actual demolition was undertaken by the buyer of the damaged building. ISSUE: Whether or not an act of God-an unusually strong earthquake-which caused the failure of the building, exempts from liability, parties who are otherwise liable because of their negligence (NAKPILS and UNITED) -- NO HELD: There should be no question that the NAKPILS and UNITED are liable for the damage resulting from the partial and eventual collapse of the PBA building as a result of the earthquakes. The applicable law governing the rights and liabilities of the parties herein is Article 1723 of the New Civil Code, which provides: Art. 1723. The engineer or architect who drew up the plans and specifications for a building is liable for damages if within fifteen years from the completion of the structure the same should collapse by reason of a defect in those plans and specifications, or due to the defects in the ground. The contractor is likewise responsible for the damage if the edifice fags within the same period on account of defects in the construction or the use of materials of inferior quality furnished by him, or due to any violation of the terms of the contract. If the engineer or architect supervises the construction, he shall be solidarily liable with the contractor. Thus it has been held that when the negligence of a person concurs with an act of God in producing a loss, such person is not exempt from liability by showing that the immediate cause of the damage was the act of God. To be exempt from liability for loss because of an act of God, he must be free from any previous negligence or misconduct by which that loss or damage may have been occasioned. Co-ownership RGGM Robles v. C.A. 328 SCRA 97

DOCTRINE: It is a fundamental principle that a co-owner cannot acquire by prescription the share of the other co-owners, absent any clear repudiation of the co-ownership. in order that the title may prescribe in favor of a co-owner, the following requisites must concur: 1.) the co-owner has performed unequivocal acts of repudiation amounting to an ouster of the other co-owners; 2.) such positive acts of repudiation have been made known to the other co-owners; and 3.) the evidence thereof is clear and convincing. FACTS: Leon Robles owned a land in Morong Rizal. When Leon died, his son Silvino Robles inherited the land. Both of them declared the property under their name for taxation purposes. Upon the death of Silvino, his widow Maria dela Cruz and his children inherited the property. They took adverse possession of it and paid the taxes thereon. The task of cultivating the land was assigned to one of Silvino‘s son, Lucio Roles while the payment of the taxes was entrusted to their half-brother, Hilario Robles. In 1962, for unknown reasons, the tax declaration of the parcel of land in the name of Silvino Robles was canceled and transferred to one Exequiel Ballena, father of Andrea Robles who is the wife of defendant Hilario Robles. Thereafter, Exequiel Ballena secured a loan from the Antipolo Rural Bank, using the tax declaration as security. Somehow, the tax declaration was transferred to the name of Antipolo Rural Bank and later on, was transferred to the name of defendant Hilario Robles and his wife. In 1996, Andrea Robles secured a loan from the Cadona Rural Bank, Inc., using the tax declaration as security. For failure to pay the mortgage debt, foreclosure proceedings were had and defendant Rural Bank emerged as the highest bidder during the auction sale in October 1968. The spouses Hilario Robles failed to redeem the property and so the tax declaration was transferred in the name of defendant Rural Bank. On September 25, 1987, defendant Rural Bank sold the same to the Spouses Vergel Santos and Ruth Santos who took possession of the property and was able to secure Free Patent No. IV-1-010021 in their names. ISSUES: 1. Whether petitioners have the appropriate title essential to an action for quieting of title (relevant issue) and whether title claimed by respondents is valid 2. Whether real estate mortgage between Hilario and RBC is valid 3. Whether issuance of free patent is valid

HELD: 1. Petitioners have valid title by virtue of their continued and open occupation and possession as owners of the subject property. In this case, the cloud on petitioners‘ title emanate from the apparent validity of the free patent issued and the tax declarations and other evidence in favour of respondents ultimately leading to the transfer of the property to spouses Santos. WRT title of the spouses Santos, such is deemed invalid/inoperative insofar as it is rooted in the title and appropriation of Hilario. Hilario could not have prejudiced the rights of his co-heirs as coowners of the real estate. He must have first repudiated the ownership clearly and evidently. CA failed to consider the irregularities in the transactions involving the property. No instrument/deed of conveyance was presented to show any transaction between petitioners

and Ballane or even Hilario. 2. Mortgage was only valid insofar as Hilario‘s undivided interest is concerned there being co-ownership between the heirs. Court also delved into gross negligence which amounted to bad faith on part of bank by not exercising due diligence in verifying the ownership of the land considering such was unregistered. Free patent was also not valid, the land in question having been converted ipso jure to private land by virtue of the adverse possession in the concept of owners since. 3. 1916 by the petitioners. Issuance of patents covering private lands is out of the jurisdiction of the Director of Lands or Bureau of Lands. MCSS Acevedo v. Abesamis 217 SCRA 186 FACTS: Herodotus and 7 others were left an estate consisting of real properties in Quezon City and Caloocan City. Herodotus became the administrator pending partition. For the meantime, the property is owned in common by the heirs. The case pended for 16 years with the court. Miguel et al (respondents) then filed a Motion for Approval of Sale for them to sell their shares of the estate. The court approved the motion. Respondents were able to find a buyer in the person of Yu Hwa Ping who agreed to buy the properties for P12 Million. He paid P6 million as earnest money. Herodotus assailed the approval of the sale claiming that the price is quite low. The court ordered Miguel et al to find a higher bidder within a specified time frame which was later extended to 7 months but still no other buyer could provide better terms. Finally, it was agreed by the parties that respondents sell their share to the price already agreed upon with Ping and that the Herodotus can negotiate his price with Ping. But Herodotus still filed a Supplemental Opposition against the approval of the conditional sale. The court affirmed the approval of the sale and ordered Herodotus to sell his share at the same rate that the other heirs sold their share to Ping. ISSUE: WON the other heirs can sell their shares of the estate prior to adjudication. -- YES HELD: An heir can sell his share without final adjudication. An heir is a co-owner of the property (estate) before adjudication. Although the Rules of Court do not specifically state that the sale of an immovable property belonging to an estate of a decedent, in a special proceeding, should be made with the approval of the court, this authority is necessarily included in its capacity as a probate court. Therefore, it is clear that the probate court in the case at bar, acted within its jurisdiction in issuing the Order approving the Deed of Conditional Sale.

The right of an heir to dispose of the decedent‘s property, even if the same is under administration, is based on the Civil Code provision stating that the possession of hereditary property is deemed transmitted to the heir without interruption and from the moment of the death of the decedent, in case the inheritance is accepted. Where there are however, two or more heirs, the whole estate of the decedent is, before its partition, owned in common by such heirs. NKVS Paulmitan v. C.A. 215 SCRA 866 DOCTRINE: Even if a co-owner sells the whole property as his, the sale will affect only his own share but not those of the other co-owners who did not consent to the sale. This is because under the aforementioned codal provision, the sale or other disposition affects only his undivided share and the transferee gets only what would correspond to his grantor in the partition of the thing owned in common. FACTS: Agatona Sagario Paulmitan, who died sometime in 1953, left the two parcels of land located in the Province of Negros Occidental. From her marriage with Ciriaco Paulmitan, who is also now deceased, Agatona begot two legitimate children, namely: Pascual Paulmitan, who also died in 1953, apparently shortly after his mother passed away, and Donato Paulmitan, who is one of the petitioners. Petitioner Juliana P. Fanesa is Donato's daughter while the third petitioner, Rodolfo Fanes, is Juliana's husband. Pascual Paulmitan, the other son of Agatona Sagario, is survived by the respondents, who are his children, name: Alicio, Elena, Abelino, Adelina, Anita, Baking and Anito, all surnamed Paulmitan. Donato Paulmitan executed an Affidavit of Declaration of Heirship, extrajudicially adjudicating unto himself Lot No. 757 based on the claim that he is the only surviving heir of Agatona Sagario. As regards Lot No. 1091, Donato executed on May 28, 1974 a Deed of Sale over the same in favor of petitioner Juliana P. Fanesa, his daughter. For non-payment of taxes, Lot No. 1091 was forfeited and sold at a public auction, with the Provincial Government of Negros Occidental being the buyer. Juliana P. Fanesa redeemed the property. On learning of these transactions, respondents children of the late Pascual Paulmitan filed on with the Court of First Instance of Negros Occidental a Complaint against petitioners to partition the properties plus damages. Petitioners set up the defense of prescription with respect to Lot No. 757 contending that the Complaint was filed more than eleven years after the issuance of a transfer certificate of title to Donato Paulmitan over the land as consequence of the registration with the Register of Deeds, of Donato's affidavit extrajudicially adjudicating unto himself Lot No. 757. As regards Lot No. 1091, petitioner Juliana P. Fanesa claimed in her Answer to the Complaint that she acquired exclusive ownership thereof not only by means of a deed of sale executed in her favor by her father, petitioner Donato Paulmitan, but also by way of redemption from the Provincial Government of Negros Occidental. ISSUE: 1. W/N Donato and Pascual (through his heirs) are co-owners of the land. 2. W/N Juliana has acquired ownership of the property due to the same being forfeited and subsequently redeemed.

HELD: 1st Issue: YES, they are co-owners. When Agatona Sagario Paulmitan died intestate in 1952, her two (2) sons Donato and Pascual were still alive. From the time of the death of Agatona Sagario Paulmitan to the subsequent passing away of her son Pascual in 1953, the estate remained unpartitioned. Article 1078 of the Civil Code provides: "Where there are two or more heirs, the whole estate of the decedent is, before its partition, owned in common by such heirs, subject to the payment of debts of the deceased." Donato and Pascual Paulmitan were, therefore, co-owners of the estate left by their mother as no partition was ever made. When Pascual Paulmitan died intestate in 1953, his children, the respondents, succeeded him in the co-ownership of the disputed property. Pascual Paulmitan's right of ownership over an undivided portion of the property passed on to his children, who, from the time of Pascual's death, became co-owners with their uncle Donato over the disputed decedent estate. When Donato Paulmitan sold on May 28, 1974 Lot No. 1091 to his daughter Juliana P. Fanesa, he was only a co-owner with respondents and as such, he could only sell that portion which may be allotted to him upon termination of the co-ownership. The sale did not prejudice the rights of respondents to one half (1/2) undivided share of the land which they inherited from their father. It did not vest ownership in the entire land with the buyer but transferred only the seller's proindiviso share in the property 14 and consequently made the buyer a co-owner of the land until it is partitioned. Art. 493. Each co-owner shall have the full ownership of his part and of the fruits and benefits pertaining thereto, and he may therefore alienate, assign or mortgage it and even substitute another person its enjoyment, except when personal rights are involved. But the effect of the alienation or mortgage, with respect to the co-owners, shall be limited to the portion which may be allotted to him in the division upon the termination of the co-ownership. Even if a co-owner sells the whole property as his, the sale will affect only his own share but not those of the other co-owners who did not consent to the sale. This is because under the aforementioned codal provision, the sale or other disposition affects only his undivided share and the transferee gets only what would correspond to his grantor in the partition of the thing owned in common. From the foregoing, it may be deduced that since a co-owner is entitled to sell his undivided share, a sale of the entire property by one co-owner without the consent of the other co-owners is not null and void. However, only the rights of the co-owner-seller are transferred, thereby making the buyer a co-owner of the property. 2nd Issue: NO, she is only entitle to the spiritual share of Donato. The redemption of the land made by Fanesa did not terminate the co-ownership nor give her title to the entire land subject of the co-ownership. The right of repurchase may be exercised by co-owner with respect to his share alone. While the records show that petitioner redeemed the property in its entirety, shouldering the expenses therefor, that did not make him the owner of all of it. In other words, it did not put to end the existing state of co-ownership. There is no doubt that redemption of property entails a necessary expense. Under the Civil Code:

Art. 488. Each co-owner shall have a right to compel the other co-owners to contribute to the expenses of preservation of the thing or right owned in common and to the taxes. Any one of the latter may exempt himself from this obligation by renouncing so much of his undivided interest as may be equivalent to his share of the expenses and taxes. No such waiver shall be made if it is prejudicial to the co-ownership. Although petitioner Fanesa did not acquire ownership over the entire lot by virtue of the redemption she made, nevertheless, she did acquire the right to reimbursed for half of the redemption price she paid to the Provincial Government of Negros Occidental on behalf of her co-owners. Until reimbursed, Fanesa hold a lien upon the subject property for the amount due her. AMPS Del Campo v. C.A. 351 SCRA 1 DOCTRINE: 1. The co-owner‘s undivided interest could properly be the object of the contract of sale. 2. Also, undisturbed possession by a co-owner has the effect of a partial partition of the coowner property which entitles the possessor to the definite portion which he occupies. FACTS: Salome, Consorcia, Alfredo, Maria, Rosalia, Jose, Quirico and Julita, all surnamed Bornales, were the original co-owners of Lot 162 (27,179 sq.m.). It was divided in aliquot shares among the eight (8) co-owners as follows: Salome Bornales

4/16

Consorcia Bornales 4/16 Alfredo Bornales

2/16

Maria Bornales

2/16

Jose Bornales

1/16

Quirico Bornales

1/16

Rosalia Bornales

1/16

Julita Bornales

1/16

Sale by co-owner of her portion: Salome sold part of her 4/16 share to Soledad Daynolo. The land was specified in the deed of sale. Thereafter, Soledad Daynolo immediately took possession of the land and built a house thereon. Mortgage: A few years later, Soledad and her husband, Simplicio Distajo, mortgaged this portion to Jose Regalado. Soledad eventually died. The husband was able to redeem the mortgage (portion of) land. The heirs subsequently sold this to herein petitioners, the spouses Manuel Del Campo and Salvacion Quiachon. Another sale by the co-owners of the entire lot: Later on, three of the eight co-owners (Salome,

Consorcia and Alfredo) sold 24,993 sq. m. of said lot to Jose Regalado. Meanwhile, Jose Regalado was able to obtain a title in his name of the whole lot previously co-owned. The whole was subdivided and covered in further titles in his name. Hence, complaint by previous buyers: Thus, petitioners Manuel and Salvacion del Campo broug ht this complaint for "repartition, resurvey and reconveyance" against the heirs of the now deceased Jose Regalado. They claim that they owned an area of 1,544 square meters located within Lot 162-C-6 which was erroneously included in TCT No. 14566 in the name of Regalado. Petitioners alleged that they occupied the disputed area as residential dwelling ever since they purchased the property from the Distajos way back in 1951. They also declared the land for taxation purposes and paid the corresponding taxes. ISSUES: 1. Could Salome have validly sold her pro-indiviso share in Lot 162 by metes and bounds to Soledad, from whom petitioners derived their title? 2. May the ownership of the previous buyers be upheld against that of the second buyer who has now a registered title? HELD: 1. YES. Sale valid up to portion of entitlement. Even if a co-owner sells the whole property as his, the sale will affect only his own share but not those of the other co-owners who did not consent to the sale. Soledad became a co-owner of Lot 162 as of the year 1940 when the sale was made in her favor. It follows that Salome, Consorcia and Alfredo could not have sold the entire Lot 162 to Jose Regalado. Regalado merely became a new co-owner of Lot 162 to the extent of the shares which Salome, Consorcia and Alfredo could validly convey. Soledad retained her rights as co-owner and could validly transfer her share to petitioners in 1951. 2. Previous buyers who only had uninterrupted possession prevails over the second buyer despite the latter‘s registered title. His purchase constitutes fraud which defeats a purportedly indefeasible title. The area subject matter of this petition had already been effectively segregated from the ‗mother lot‘ even before title was issued in favor of Regalado. It must be noted that 26 years had lapsed from the time petitioners bought and took possession of the property in 1951 until Regalado procured the issuance of TCT No. 14566. X x x. In the case of Vda. De Cabrera vs. Court of Appeals, we had occasion to hold that where the transferees of an undivided portion of the land allowed a co-owner of the property to occupy a definite portion thereof and had not disturbed the same for a period too long to be ignored, the possessor is in a better condition or right than said transferees. (Potior est condition possidentis). Such undisturbed possession had the effect of a partial partition of the co-owner property which entitles the possessor to the definite portion which he occupies. Conformably, petitioners are entitled to the disputed land, having enjoyed uninterrupted possession thereof for a total of 49 years up to the present. Although Regalado‘s certificate of title became indefeasible after the lapse of one year from the date of the decree of registration, the attendance of fraud in its issuance created an implied trust in favor of petitioners and gave them the right to seek reconveyance of the

parcel wrongfully obtained by the former. Regalado was aware of petitioners‘ possession of the subject portion as well as the sale between Salome and Soledad.

KGS Villanueva v. Florendo 139 SCRA 329 DOCTRINE: Where a surviving spouse sold his undivided portion of the conjugal property to the wife of one of his sons, the vendor‘s other children are entitled to exercise the right of redemption as co-owners of the right portion sold. FACTS: Macario and Basilia are husband and wife. They have five children. After Basilia died intestate, the subject parcel of land (165sqm) was left undivided. Thereafter, Macario sold his share to said property (one-half) to his daughter-in-law, Erlinda. Having been informed of the sale, the petitioners (co-heirs) signified their intention to redeem the lot in question but Erlinda refused to allow such redemption contending that she is the wife of one of the legal heirs and therefore redemption will not lie against her because she is not the "third party" or "stranger" contemplated in the law. ISSUE: Is Erlinda considered as a co-heir and hence, the legitimate heirs are disallowed redemption contemplated in Article 1620 of the Civil Code? – NO HELD: NO. Erlinda is a third person. The co-owners have the right to redeem. Art. 1620 of the New Civil Code provides: A co-owner of a thing may exercise the right of redemption in case the shares of all the other co-owners or of any of them, are sold to a third person. If the price of the alienation is grossly excessive, the redemptioner shall pay only a reasonable one. Should two or more co-owners desire to exercise the right of redemption, they may only do so in proportion to the share they may respectively have in the thing owned in common. It is not disputed that co-ownership exists but the lower court disallowed redemption because it considered the vendee, Erlinda Vallangca, a co-heir, being married to Concepcion Villanueva, and the conveyance was held valid since it was in favor of the conjugal partnership of the spouses in the absence of any statement that it is paraphernal in character. Within the meaning of Art. 1620, the term "third person" or "stranger" refers to all persons who are not heirs in succession, and by heirs are meant only those who are called either by will or the law to succeed the deceased and who actually succeeds. In short, a third person is any one who is not a co-owner. The vendee is related by affinity to the deceased by reason of her marriage to one of the heirs and being married to Concepcion does not entitle the vendee to inherit or succeed in her own right. She is not an heir of Basilia Garcia nor included in the "family relations" of spouses Macario and Basilia as envisioned in Art. 217 of the Civil Code. Art. 217. Family relations shall include those: (1) Between Husband and wife; (2) Between parent and child; (3) Among other ascendants and their descendants; (4) Among brothers and sisters.

The co-owners should therefore be allowed to exercise their right to redeem the property sold to Erlinda Vallangca. To deny petitioners the right of redemption recognized in Art. 1620 of the Civil Code is to defeat the purpose of minimizing co-ownership and to contravene the public policy in this regard. Moreover, it would result in disallowing the petitioners a way out of what, in the words of Manresa, "might be a disagreeable or inconvenient association into which they have been thrust." Respondent seller Macario, as co-owner and before partition, has the right to freely sell and dispose of his undivided interest or his Ideal share but not a divided part and one with boundaries as what was done in the case at bar. It is an inherent and peculiar feature of coownership that although the co-owners may have unequal shares in the common property quantitatively speaking, each co-owner has the same right in a qualitative sense as any one of the other co-owners. In other words, every co-owner is the owner of the whole and over the whole, he exercises the right of dominion, but he is at the same time the owner of a portion which is truly abstract because until division is effected, such portion is not correctly determined. Separate Opinions AQUINO, J., dissenting: An undivided portion of a lot sold to a daughter-in-law is a sale to the conjugal partnership of gains; hence, her sisters and brothers-in-law cannot exercise the co-owner’s right of redemption. I am of the opinion that no right of redemption exists in favor of Erlinda's sisters-in-law and brothers-in-law because the sale was made to the (conjugal partnership of Erlinda and Concepcion), a co-owner. The sale made Concepcion and Erlinda the co-owners of 6/10 or 3/5 of the 165-square- meter lot. Erlinda is not a third person with respect to the co-ownership. JPOT Adille v. C.A. 157 SCRA 455 DOCTRINE: Prescription, as a mode of terminating a relation of co-ownership, must have been preceded by repudiation (of the co-ownership). The act of repudiation, in turn is subject to certain conditions: (1) a co-owner repudiates the co-ownership; (2) such an act of repudiation is clearly made known to the other co-owners; (3) the evidence thereon is clear and conclusive, and (4) he has been in possession through open, continuous, exclusive, and notorious possession of the property for the period required by law. FACTS: Felisa Alzul owns a parcel of land in Legazpi City, Albay. In her lifetime, she married twice. First,with Bernabe Adille which was Rustico Adille's father and second, with Prospero Asejo from whom she had other children. In 1939, she entered in a pacto de retro sale with a period of three years for repurchase. Unfortunately, she died in 1942 and was unable to settle her affairs accordingly. Rustico, acted on his own and acquired the land along with a OCT, which he was able to procure stating that he was the only child of Felisa and Bernabe. He has then been in charge of the land in question and has kept his actions from his siblings. Although there is one, namely, Emeteria who happens to live in the same land he resides in. He was then deemed absolute owner of the land and through his actuations seek to keep the land for himself not taking into account his other brothers nor his sister. It is because of this that plaintiffs have come here and contend that trial court erred in: I. ... declaring the defendant absolute owner of the property; II. ... not ordering the partition of the property; and III. ... ordering one of the plaintiffs who is in possession of the portion of the property to vacate the land, p. 1 Appellant's brief.

ISSUE: May a co-owner acquire exclusive ownership over the property held in common? -- NO HELD: Given that Rustico acquired the property individually, tends to it and is in possession together with the corresponding OCT, it does not discount the fact that the perfection of such was done through fraudulent means. His function of reacquisition only makes him a trustee in place of his other siblings. In addition, a torrens title does not make for a mode of extinguishment with regard to co-ownership. The court also refuses to recognize his claim of ownership by way of prescription given that he registered said land in 1955 and has been in his possession until 1974. Prescription, as a mode of terminating a relation of co-ownership, must have been preceded by repudiation (of the co-ownership). The act of repudiation, in turn is subject to certain conditions: (1) a co-owner repudiates the co-ownership; (2) such an act of repudiation is clearly made known to the other co-owners; (3) the evidence thereon is clear and conclusive, and (4) he has been in possession through open, continuous, exclusive, and notorious possession of the property for the period required by law. Rustico's claim falls short because he did not repudiate. In fact, he had been keeping substantive information for himself leaving all other co-owners oblivious with concern to his acts. So, the court finds it fit that although the span of time is indeed what prescription necessitates, it is still not conclusive nor meritorious to its effect. WHEREFORE, there being no reversible error committed by the respondent Court of Appeals, the petition is DENIED. The Decision sought to be reviewed is hereby AFFIRMED in toto. No pronouncement as to costs. MLAV Delima v. C.A. 201 SCRA 641 DOCTRINE: The issuance of the new title constituted an open and clear repudiation of the trust or co-ownership. FACTS: Lino Delima acquired a lot from the Friar Lands Estate in Cebu. He later died leaving as his only heirs three brothers and a sister namely: Eulalio Delima, Juanita Delima, Galileo Delima and Vicente Delima. After his death, a title was issued in the name of his legal heirs, as represented by Galileo Delima. Galileo Delima declared the lot in his name for taxation purposes and paid the taxes thereon from 1954 to 1965. Petitioners, who are the surviving heirs of Eulalio and Juanita Delima, filed with the Court of First Instance of Cebu an action for reconveyance and/or partition of property and for the annulment of the land title with damages against their uncles Galileo Delima and Vicente Delima. The trial court rendered a decision in favor of petitioners. The decision was appealed, and the CA reversed the lower court‘s decision. It held that all other

heirs (Eulalio, Juanita and Vicente Delima) had already relinquished and waived their rights to the property in favor of Galileo, considering that he alone paid the remaining balance of the purchase price and the realty taxes. ISSUE: WON Galileo Delima already acquired the property by prescription -- YES HELD: When a co-owner of the property in question executed a deed of partition and on the strength thereof obtained the cancellation of the title in the name of their predecessor and the issuance of a new one in his name, in effect denying or repudiating the ownership of the other co-owners over their shares, the statute of limitations started to run for the purposes of the action instituted by the latter seeking a declaration of the existence of the co-ownership and of their rights thereunder. Since an action for reconveyance of land based on implied or constructive trust prescribes after ten (10) years, it is from the date of the issuance of such title that the effective assertion of adverse title for purposes of the statute of limitations is counted. Evidence shows that TCT No. 2744 in the name of the legal heirs of Lino Delima, represented by Galileo Delima, was cancelled by virtue of an affidavit executed by Galileo Delima and that on February 4, 1954, Galileo Delima obtained the issuance of a new title in his name numbered TCT No. 3009 to the exclusion of his co-heirs. The issuance of this new title constituted an open and clear repudiation of the trust or co-ownership, and the lapse of ten (10) years of adverse possession by Galileo Delima from February 4, 1954 was sufficient to vest title in him by prescription. As the certificate of title was notice to the whole world of his exclusive title to the land, such rejection was binding on the other heirs and started as against them the period of prescription. Hence, when petitioners filed their action for reconveyance and/or to compel partition on February 29, 1968, such action was already barred by prescription. Whatever claims the other co-heirs could have validly asserted before can no longer be invoked by them at this time. DJTV Mariategui v. C.A. 205 SCRA 337 DOCTRINE: Prescription, as a mode of terminating a relation of co-ownership, must have been preceded by repudiation (of the co-ownership). The act of repudiation, in turn, is subject to certain conditions: (1) a co-owner repudiates the co-ownership; (2) such an act of repudiation is clearly made known to the other co-owners; (3) the evidence thereon is clear and conclusive; and (4) he has been in possession through open, continuous, exclusive, and notorious possession of the property for the period required by law. FACTS: Lupo Mariategui died without a will. During his lifetime, Lupo Mariategui contracted three (3) marriages. With his first wife he begot four (4) children. With his second wife, he begot a daughter. And with his third wife, he begot three children. At the time of his death, Lupo Mariategui left certain properties which he acquired when he was still unmarried. Lupo's descendants by his first and second marriages executed a deed of

extrajudicial partition whereby they adjudicated unto themselves lots in the Muntinglupa Estate. Thereafter, an Original Certiicate of Title (OCT) was issued in the name of the said heirs. Subsequently, the registered owners caused the subdivision of the said lot into for which separate transfer certificates of title were issued to the respective parties. Lupo's children by his third marriage filed with the lower court an amended complaint claiming that the lots were owned by their common father, Lupo Mariategui, and that, with the adjudication of the lots to their co-heirs, they (children of the third marriage) were deprived of their respective shares in the lots. Plaintiffs pray for partition of the estate of their deceased father and annulment of the deed of extrajudicial partition. The defendants filed a motion to dismiss on the grounds of lack of cause of action and prescription. The motion to dismiss was denied by the trial court. The plaintiffs elevated the case to the Court of Appeals (CA) but the CA upheld the trial court‘s decision. ISSUE: Whether or not prescription barred private respondents' right to demand the partition of the estate of Lupo Mariategui. -- NO HELD: Prescription does not run against private respondents with respect to the filing of the action for partition so long as the heirs for whose benefit prescription is invoked, have not expressly or impliedly repudiated the co-ownership. In other words, prescription of an action for partition does not lie except when the co-ownership is properly repudiated by the co-owner. Otherwise stated, a co-owner cannot acquire by prescription the share of the other co-owners absent a clear repudiation of co-ownership duly communicated to the other co-owners. Furthermore, an action to demand partition is imprescriptible and cannot be barred by laches. On the other hand, an action for partition may be seen to be at once an action for declaration of co-ownership and for segregation and conveyance of a determinate portion of the property involved. Petitioners contend that they have repudiated the co-ownership when they executed the extrajudicial partition excluding the private respondents and registered the properties in their own names. However, no valid repudiation was made by petitioners to the prejudice of private respondents. Assuming petitioners' registration of the subject lot was an act of repudiation of the co-ownership, prescription had not yet set in when private respondents filed the present action for partition. Petitioners' registration of the properties in their names did not operate as a valid repudiation of the co-ownership. In Adille vs. Court of Appeals (157 SCRA 455, 461-462 [1988]), the Court held: Prescription, as a mode of terminating a relation of co-ownership, must have been preceded by repudiation (of the co-ownership). The act of repudiation, in turn, is subject to certain conditions: (1) a co-owner repudiates the co-ownership; (2) such an act of repudiation is clearly made known to the other co-owners; (3) the evidence thereon is clear and conclusive; and (4) he has been in possession through open, continuous, exclusive, and notorious possession of the property for the period required by law. WHEREFORE, the petition is DENIED and the assailed decision of the Court of Appeals dated

December 24, 1980 is Affirmed. JRPA Estreller v. Ysmael G.R. No. 170264 DOCTRINE: Section 2 of Presidential Decree (P.D.) No. 2016, reinforced by P.D. No. 1517, which prohibits the eviction of qualified tenants/occupants, extends only to landless urban families who are rightful occupants of the land and its structures, and does not include those whose presence on the land is merely tolerated and without the benefit of contract, those who enter the land by force or deceit, or those whose possession is under litigation. Petitioners claim that they are lawful lessees of the property. However, they failed to prove any lease relationship or, at the very least, show with whom they entered the lease contract. Respondents, on the other hand, were able to prove their right to enjoy possession of the property. Thus, petitioners, whose occupation of the subject property by mere tolerance has been terminated by respondents, clearly do not qualify as "tenants" covered by these social legislations. FACTS: Petition for Review under Rule 45.Ysmael and Santos-Alvarez (respondents) filed with the RTC of Quezon City, a case for Recovery of Possession against Estreller et al.(petitioners), claiming ownership of the property subject of dispute, by virtue of a TCT issued by the Register of Deeds. Ysmael‘s allegations: On various dates in 1973, Estreller et al. entered the property through stealth and strategy and had since occupied the same; and despite demands refused to vacate the premises. Estreller et al.‘s counterclaims: Ysmael had no personality to file the suit since he only owned a small portion of the property, while Santos-Alvarez did not appear to be a registered owner thereof. That their occupation of the property was lawful, having leased the same from the Magdalena Estate, and later on from Alvarez. That the property has already been proclaimed by the Quezon City Government as an Area for Priority Development under P. D. Nos. 1517 and 2016, which prohibits the eviction of lawful tenants and demolition of their homes.RTC decision: in favour of Ysmael and Santos-Alvarez.CA decision: affirmed in toto the RTC. ISSUE: WON petitioners are rightful occupants of the property. -- NO HELD: Finally, petitioners' claim that they are entitled to the protection against eviction and demolition afforded by P.D. Nos. 2016, 1517, and Republic Act (R.A.) No. 7279, is not plausible. Section 6 of P.D. No. 1517 grants preferential rights to landless tenants/occupants to acquire land within urban land reform areas, while Section 2 of P.D. No. 2016 prohibits the eviction of qualified tenants/occupants. In Dimaculangan v. Casalla, the Court was emphatic in ruling that the protective mantle of P.D. No. 1517and P.D. No. 2016 extends only to landless urban families who meet these

qualifications: a) they are tenants as defined under Section 3(f) of P.D. No. 1517; b) they built a home on the land they are leasing or occupying; c) the land they are leasing or occupying is within an Area for Priority Development and Urban Land Reform Zone; and d) they have resided on the land continuously for the last 10 years or more. Section 3(f) of P.D. No. No. 1517 defines the term "tenant" covered by the said decree as the "rightful occupant of land and its structures, but does not include those whose presence on the land is merely tolerated and without the benefit of contract, those who enter the land by force or deceit, or those whose possession is under litigation." It has already been ruled that occupants of the land whose presence therein is devoid of any legal authority, or those whose contracts of lease were already terminated or had already expired, or whose possession is under litigation, are not considered "tenants" under the Section 3(f). Petitioners claim that they are lawful lessees of the property. However, they failed to prove any lease relationship or, at the very least, show with whom they entered the lease contract. Respondents, on the other hand, were able to prove their right to enjoy possession of the property. Thus, petitioners, whose occupation of the subject property by mere tolerance has been terminated by respondents, clearly do not qualify as "tenants" covered by these social legislations. ABB Pada-Kilario v. C.A. 322 SCRA 481

DOCTRINE: It is not required that partition among heirs be in writing and registered in order to be valid. The object of registration is to serve as constructive notice to third persons who might have interest on the said property. Thus, it follows that the intrinsic validity of partition not executed with the prescribed formalities is not undermined when no creditors are involved. FACTS: Silverio Pada filed an ejectment case against the spouses Ricardo and Verona Kilario. The Kilarios occupy a portion of the intestate estate of Jacinto Pada, grandfather of Silverio and have been living there since 1960 by sheer tolerance. Upon the death of Jacinto Pada, his heirs entered into extrajudicial partition of his estate in 1951. As a result thereof, Lot 5581 was allocated to Ananias and Marciano who became co-owners of the said lot. Ananias died and his daughter succeeded in his right as co-owner. Eventually, Juanita sold her right in the co-ownership to Engr. Paderes. Maria, Marciano‘s heir, on the other hand, sold her share to her cousin respondent Silverio Pada. The latter demanded spouses Kilario to vacate the lot for their use but the spouses Kilario refused. On June 1995, a complaint for ejectment was filed against the spouses Kilario. On July 1995 a deed of donation in their favor was executed by heirs of Amador Pada. ISSUE: Whether or not the partition was valid -- YES HELD:

The extrajudicial partition of the estate of Jacinto Pada among his heirs made in 1951 is VALID, even if executed in an unregistered private document. It is not required that partition among heirs be in writing and registered in order to be valid. The object of registration is to serve as constructive notice to third persons who might have interest on the said property. Thus, it follows that the intrinsic validity of partition not executed with the prescribed formalities is not undermined when no creditors are involved. Without creditors to take into consideration, it is competent for the heirs of an estate to enter into an agreement for distribution thereof in a manner and upon a plan different from those provided by the rules from which, in the first place, nothing can be inferred that a writing or other formality is essential for the partition to be valid. The partition of inherited property need not be embodied in a public document so as to be effective as regards the heirs that participated therein. The extrajudicial partition which the heirs of Jacinto Pada executed voluntarily and spontaneously in 1951 has produced a legal status. When they discussed and agreed on the division of the estate of Jacinto Pada, it is presumed that they did so in furtherance of their mutual interests. As such, their division is conclusive, unless and until it is shown that there were debts existing against the estate which had not been paid. No showing, however, has been made of any unpaid charges against the estate of Jacinto Pada. Thus, there is no reason why the heirs should not be bound by their voluntary acts. The belated act of Concordia, Esperanza and Angelito, who are the heirs of Amador Pada, of donating the subject property to petitioners after forty four (44) years of never having disputed the validity of the 1951 extrajudicial partition that allocated the subject property to Marciano and Ananias, produced no legal effect. The donation made by his heirs to petitioners of the subject property, thus, is void for they were not the owners thereof. In any case, it is too late for the heirs of Amador Pada to repudiate the legal effects of the 1951 extrajudicial partition as prescription and laches have equally set in. Having said this, the petitioners are then estopped from impugning the extrajudicial partition executed by the heirs of Jacinto Pada after explicitly admitting in their Answer that they had been occupying the subject property since 1960 without ever paying any rental as they only relied on the liberality and tolerance of the Pada family. Such admissions are binding to them insofar as the character of their possession of the subject property is concerned. FZC Maestrado v. C.A. 327 SCRA 678 DOCTRINE: A possessor or real estate property is presumed to have title thereto unless the adverse claimant establishes a better right. partition is the separation, division and assignment of a thing held in common among those to whom it may belong. If may be effected extrajudicially by the heirs themselves through a public instrument filed before the register of deeds. However, as between the parties, a public instrument is neither constitutive nor an inherent element of a contract of partition. Since registration serves as constructive notice to third persons, an oral partition by the heirs is valid if no creditors are affected. FACTS: These consolidated cases involve Lot No. 5872 and the rights of the contending parties thereto. The lot has an area of 57.601 sq.m. and is registered in the name of the deceased spouses Ramon and Rosario Chaves. The spouses died intestate in 1943 and 1944, respectively. They were urvived by six heirs. To settle the estate of said spouse, Angel Chaves, one of the heirs, initiated intestate proceedings and was appointed dministrator of said estates in the process. An inventory of the estates was made and thereafter, the heirs agreed on a project partition. The court approved the partition but a copy of said decision was missing. Nonetheless, the estate

was divided among the heirs. Subsequently, in 1956, the partition case effected and the respective shares of the heirs were delivered to them. Significantly, Lot No.5872 was not included in a number of documents. Parties offered different explanations as to the omission of said lot in the documents. Petitioners maintain the existence of an oral partition agreement entered into by all heirs after the death of their parents. To set things right, petitioners then prepared a quitclaim to confirm the alleged oral agreement. Respondents dispute voluntariness of their consent to the quitclaims. Six years after the execution of the quitclaims, respondents discovered that indeed subject lot was still a common property in the name of the deceased spouses. Eventually, an action for Quieting of Title was filed by petitioners on December 22, 1983. The trial court considered Lot No. 5872 as still a common property and therefore must be divided into six parts, there being six heirs. Petitioners appealed to the Court of Appeals which sustained the decision of the trial court. ISSUE: Whether or not the action for quieting of title had already prescribed. HELD: Lot No. 5872 is no longer common property of the heirs of the deceased spouses Ramon and Rosario Chaves. Petitioners‘ ownership over said lot was acquired by reason of the oral partition agreed upon by the deceased spouses‘ heirs sometime before 1956. That oral agreement was confirmed by the notarized quitclaims executed by the said heirs on August 16, 1977 and September 8, 1977. There was indeed an oral agreement of partition entered into by the heirs/parties. A possessor of real estate property is presumed to have title thereto unless the adverse claimant establishes a better right. In the instant case it is the petitioners, being the possessors of Lot No. 5872, who have established a superior right thereto by virtue of the oral partition which was also confirmed by the notarized quitclaims of the heirs. Partition is the separation,division and assignment of a thing held in common among those to whom it may belong. It may be effected extra-judicially by the heirs themselves through a public instrument filed before the register of deeds. However, as between the parties, a public instrument is neither constitutive nor an inherent element of a contract of partition. Since registration serves as constructive notice to third persons, an oral partition by the heirs is valid if no creditors are affected. Moreover, even the requirement of a written memorandum under the statute of frauds does not apply to partitions effected by the heirs where no creditors are involved considering that such transaction is not a conveyance of property resulting in change of ownership but merely a designation and segregation of that part which belongs to each heir. LNAC Pilapil v. Briones GR 150175 DOCTRINE: Since an implied trust is an obligation created by law, then respondents had 10 years within which to bring an action for reconveyance of their shares in Maximino‘s properties. The general rule is that an action for reconveyance of real property based on implied trust prescribes ten years from registration and/or issuance of the title to the property, not only because registration under the Torrens system is a constructive notice of title, but also because by registering the disputed properties exclusively in her name, Donata had already

unequivocally repudiated any other claim to the same. FACTS: Petitioners are the heirs of the late Donata Ortiz-Briones, consisting of her surviving sister, Rizalina Ortiz- Aguila; Rizalina‘s daughter, Erlinda Pilapil; and the other nephews and nieces of Donata, in representation of her two other sisters who had also passed away. Respondents, on the other hand, are the heirs of the late Maximino Briones, composed of his nephews and nieces, and grandnephews and grandnieces, in representation of the deceased siblings of Maximino. Maximino was married to Donata but their union did not produce any children. When Maximino died, Donata instituted intestate proceedings to settle her husband‘s estate. The CFI issued Letters of Administration appointing Donata as the administratrix of Maximino‘s estate. She submitted an Inventory of Maximino‘s properties, which included, among other things, parcels of land. The CFI subsequently issued an Order, awarding ownership of the real properties to Donata. Donata had the said CFI Order recorded in the Primary Entry Book of the Register of Deeds, and by virtue thereof, received new TCTs, covering the said properties, now in her name. Donata died. Erlinda, one of Donata‘s nieces, instituted a petition for the administration of the intestate estate of Donata. Erlinda and her husband, Gregorio, were appointed by the RTC as administrators of Donata‘s intestate estate. Silverio Briones, a nephew of Maximino, filed a Petition for Letters of Administration for the intestate estate of Maximino, which was initially granted by the RTC. The RTC also issued an Order, allowing Silverio to collect rentals from Maximino‘s properties. But then, Gregorio filed a Motion to Set Aside the Order, claiming that the said properties were already under his and his wife‘s administration as part of the intestate estate of Donata. Silverio‘s Letters of Administration for the intestate estate of Maximino was subsequently set aside by the RTC. The heirs of Maximino filed a Complaint against the heirs of Donata for the partition, annulment, and recovery of possession of real property. They alleged that Donata, as administratrix of the estate of Maximino, through fraud and misrepresentation, in breach of trust, and without the knowledge of the other heirs, succeeded in registering in her name the real properties belonging to the intestate estate of Maximino. RTC rendered its Decision in favor of the heirs of Maximino. Thus, the heirs of Donata filed the present Petition. ISSUE: Whether Donata succeeded in registering in her name the real properties belonging to the intestate estate of Maximino. -- YES HELD: In summary, the heirs of Maximino failed to prove by clear and convincing evidence that Donata managed, through fraud, to have the real properties, belonging to the intestate estate of Maximino, registered in her name. In the absence of fraud, no implied trust was established between Donata and the heirs of Maximino under Article 1456 of the New Civil Code. Donata was able to register the real properties in her name, not through fraud or mistake, but pursuant

to an Order issued by the CFI. The CFI Order, presumed to be fairly and regularly issued, declared Donata as the sole, absolute, and exclusive heir of Maximino; hence, making Donata the singular owner of the entire estate of Maximino, including the real properties, and not merely a co-owner with the other heirs of her deceased husband. Prescription of the action for reconveyance of the disputed properties based on implied trust is governed by Article 1144 of the New Civil Code. The following actions must be brought within ten years from the time the right of action accrues: (1) Upon a written contract; (2) Upon an obligation created by law; (3) Upon a judgment. Since an implied trust is an obligation created by, then respondents had 10 years within which to bring an action for reconveyance of their shares in Maximino‘s properties. The general rule is that an action for reconveyance of real property based on implied trust prescribes ten years from registration and/or issuance of the title to the property, not only because registration under the Torrens system is a constructive notice of title, but also because by registering the disputed properties exclusively in her name, Donata had already unequivocally repudiated any other claim to the same. By virtue of the CFI Order, dated 15 January 1960, Donata was able to register and secure certificates of title over the disputed properties in her name on 27 June 1960. The respondents filed their Complaint for partition, annulment, and recovery of possession of the disputed real properties only on 3 March 1987, almost 27 years after the registration of the said properties in the name of Donata. Therefore, respondents‘ action for recovery of possession of the disputed properties had clearly prescribed. Other than prescription of action, respondents‘ right to recover possession of the disputed properties, based on implied trust, is also barred by laches. The defense of laches, which is a question of inequity in permitting a claim to be enforced, applies independently of prescription, which is a question of time. Prescription is statutory; laches is equitable. Laches is defined as the failure to assert a right for an unreasonable and unexplained length of time, warranting a presumption that the party entitled to assert it has either abandoned or declined to assert it. This equitable defense is based upon grounds of public policy, which requires the discouragement of stale claims for the peace of society. It is uncontested that from the time of Maximino‘s death on 1 May 1952, Donata had possession of the real properties. She managed the real properties and even collected rental fees on some of them until her own death on 1 November 1977. After Donata‘s death, Erlinda took possession of the real properties, and continued to manage the same and collect the rental fees thereon. Donata and, subsequently, Erlinda, were so obviously exercising rights of ownership over the real properties, in exclusion of all others, which must have already put the heirs of Maximino on guard if they truly believed that they still had rights thereto. It is well established that the law serves those who are vigilant and diligent and not those who sleep when the law requires them to act. The law does not encourage laches, indifference, negligence or ignorance. On the contrary, for a party to deserve the considerations of the courts, he must show that he is not guilty of any of the aforesaid failings. TKDC Oliveros v. Lopez 168 SCRA 431

DOCTRINE: Under Article 494 and 1083 of the Civil Code, co-ownership of an estate should not exceed twenty years, while the agreement to keep a thing undivided should not exceed ten years. When parties stipulated a definite period to keep a thing undivided which exceeds the maximum allowed by law, said stipulation shall be void only as to the period beyond such maximum. Thus, co-ownership of an estate cannot exceed twenty years so property should be divided after twenty years. Each co-owner may demand at any time the partition of the thing owned in common insofar as his share is concerned. Article 494 specifically mandates that each co-owner may demand at any time the partition of the thing owned in common insofar as his share is concerned. FACTS: Upon his death, Lopez left the lot he owned to his widow and six children; the heirs did not initiate any move to legally partition the property. After many years, the widow and the eldest son, Candido, executed a deed of absolute sale of the undivided eastern portion of their interests in favor of spouses Oliveras & Minor; another deed of absolute sale of the undivided eastern part in favor of spouses Oliveras & Gaspar. The two Oliveras spouses had since possessed the properties. After many years, the counsel of the two Oliveras spouses wrote to the remaining heirs of Lopez reminding them of the Oliveras spouses‘ demand to partition the property so they could acquire their titles without court action. The heirs didn‘t answer so the Oliveras spouses filed a complaint for partition and damages. According to the Oliveras spouses, possession of the disputed properties was delivered to them with the knowledge and consent of the heirs; however, according to the heirs, no sale transpired as the vendors, the widow and Candido, could not sold specific portions of the property making Oliveras spouses‘ possession and occupation of specific portions of the properties illegal. Trial court ruled that the deeds of absolute sale are valid and ordered the segregation of the lot. ISSUES: 1. Whether or not the two deeds of absolute sale were null and void since the lot had not yet been partitioned. -- NO 2. Whether or not the action for partition has prescribed. -- NO HELD: 1. Under Article 494 and 1083 of the Civil Code, co-ownership of an estate should not exceed twenty years, while the agreement to keep a thing undivided should not exceed ten years. When parties stipulated a definite period to keep a thing undivided which exceeds the maximum allowed by law, said stipulation shall be void only as to the period beyond such maximum. Thus, co-ownership of an estate cannot exceed twenty years so property should be divided after twenty years. Each co-owner may demand at any time the partition of the thing owned in common insofar as his share is concerned. In this case, the heirs maintained the co-ownership beyond 20 years so when the widow and Candido sold definite portions of the lot, they validly exercised dominion over them because, by operation of law, the co-ownership had ceased. The filing of the complaint for partition by the Oliveras spouses who are legally considered as subrogated to the rights over the partitions of lot in their possession merely served as formality on the widow and Candido‘s act of terminating co-ownership.

2. Prescription may have barred the filing of complaint under Article 1144 (a) of the Civil Code; however, Article 494 specifically mandates that each co-owner may demand at any time the partition of the thing owned in common insofar as his share is concerned. In this case, although the complaint was filed after thirteen years from the execution of the deeds of sale, the Oliveras spouses‘ action for partition was timely and properly filed. AMD Sering v. Plazo 166 SCRA 84 DOCTRINE: 1. Anyone of the co-owners of an immovable may bring an action in ejectment. A co-owner may bring an ejectment action without joining the other co-owners, the suit being deemed instituted for the benefit of all. 2. The term, "action in ejectment," includes a suit of forcible entry (detentacion) or unlawful detainer (desahucio). FACTS: Alfredo Sering filed a forcible entry suit against respondent Spouses Restituto Plazo and Gertrudes Suan with the Municipal Court of del Carmen, Surigao del Norte. The case was adjudged against the Plazos, who then appealed to the CFI. Therein, the Plazos learned that the property subject of the suit was not owned solely by Sering but was owned in common by him and others. The Plazos moved for the impleading of the other co-owners as plaintiff, claiming that they were indispensable parties. The CFI agreed and ordered Sering to amend his complaint so as to include his co-owners as co-plaintiffs. Sering demurred, claiming that under the law anyone of the co-owners could bring suit for ejectment without joining the others. The Plazos contended that the invoked provision had no application to forcible entry actions, but only to suits of unlawful detainer. Because Sering failed to comply with the order for amendment of the complaint, the CFI dismissed his complaint, it also denied his MR. Hence, this petition. ISSUE: W/N Sering, as co-owner, can file an action for ejectment in behalf of and without joining the other co-owners. -- YES HELD: Article 487 of the Civil Code provides that anyone of the co-owners of an immovable may bring an action in ejectment. A co-owner may thus bring an ejectment action without joining the other co-owners, the suit being deemed instituted for the benefit of all. And the term, "action in ejectment," includes a suit of forcible entry (detentacion) or unlawful detainer (desahucio). As early as 1969, in the case Vencilao v. Camarento, the SC pertinently ruled as follows: ―Anent the question of whether an action of forcible entry and detainer should be brought in the name of all co-owners, We hold that under Article 487 of the new Civil Code, any of the coowners may bring the action... In forcible entry and detainer action(s) the matter to be determined is simply the question of prior physical possession. It having been alleged in the complaint that the plaintiff was in actual possession of the properties, certainly the plaintiff alone, who was in actual possession, could file the complaint.‖ CRF Adlawan v. Adlawan G.R. No. 161916

DOCTRINE: A co-owner by virtue of Art. 487 is allowed to bring an action without necessity of including all the co-owners as plaintiffs for it is presumed to be for the benefit of all BUT if the action of the plaintiff alone, the action should be dismissed. (batasnatin) FACTS: The instant ejectment suit stemmed from the parties‘ dispute over Lot 7226 and the house built thereon, covered by Transfer Certificate of Title No. 8842, registered in the name of the late Dominador Adlawan and located in Cebu. Petitioner averred that he is an acknowledged illegitimate son and the sole heir of Dominador. In ruling for the petitioner, the RTC held that the questioned January 31, 1962 deed of sale validly transferred title to Dominador and that petitioner is his acknowledged illegitimate son who inherited ownership of the questioned lot. The petitioner further claims that he allowed the respondents to occupy the property out of respect and generosity to respondents who are the siblings of his father who stayed on the questioned property since birth, provided they would vacate the same should his need for the property arise. When the petitioner verbally requested respondents to vacate the house and lot, but they refused and filed instead an action for quieting of title with the RTC. The respondents argued that even if petitioner is indeed Dominador‘s acknowledged illegitimate son, his right to succeed is doubtful because Dominador was survived by his wife, Graciana. This means that the petitioner is not the sole owner of Lot 7226. This is so because Dominador was survived not only by petitioner but also by his legal wife, Graciana, who died 10 years after the demise of Dominador on May 28, 1987. By intestate succession, Graciana and petitioner became co-owners of Lot 7226. ISSUE: Whether or not petitioner can validly maintain the instant case for ejectment against his coowners of lot 7226. -- NO HELD: No. The ejectment suit will not prosper as the petitioner filed it for his benefit alone and not for the benefits of the rest of co owners. ART. 487. Any one of the co-owners may bring an action in ejectment. This article covers all kinds of actions for the recovery of possession. Article 487 includes forcible entry and unlawful detainer (accion interdictal), recovery of possession (accion publiciana), and recovery of ownership (accion de reivindicacion). A co-owner may bring such an action without the necessity of joining all the other co-owners as co-plaintiffs because the suit is presumed to have been filed to benefit his co-owners. It should be stressed, however, that where the suit is for the benefit of the plaintiff alone who claims to be the sole owner and entitled to the possession of the litigated property, the action should be dismissed. MPF Cruz v. Leis 327 SCRA 570 DOCTRINE: Redemption by co-owner does not terminate the co-ownership nor give her title to the entire land subject of the co-ownership. FACTS:

● Leis and Isidro married each other in 1923. Isidro subsequently acquired from the Department of Agriculture and Natural Resources a parcel of land, which was titled in her name, with the description that she was a ―widow‖. Leis only passed away in 1973 without executing a will. ● Isidro then secured a loan from Cruz (PhP 15,000, with 5% interest) secured by a mortgage on the land from DANR, but failed to pay on due date. Isidro executed 2 contracts in favor of Cruz: a Deed of Absolute Sale and a Contract indicating a pacto de retro sale. Isidro still failed to repurchase the property within 1 year, so she consolidated the ownership of the land in favor of Cruz. ● 
 When Isidro died, Cruz demanded her heirs to vacate the premises. The heirs then filed a complaint with the RTC averring that the land was conjugal property having been purchased during their marriage. The RTC found in favor of the heirs. The case was appealed to the CA, but the CA merely affirmed the ruling because Cruz failed to get a judicial order to have the land consolidated in his name after failure of Isidro to comply with the requirements of the right to repurchase (Art. 1607). ISSUE: Whether or not the land in question is conjugal property, and therefore subject to the rules on co-ownership? HELD: Although the land was purchased during the marriage, upon Leis‘ death, the conjugal property regime ceased, and gave Isidro an equal portion of Leis‘ half of the property to be divided among his legitimes. Co-ownership of the land then began. However, upon failure of Isidro the heirs to exercise the right to repurchase, the ownership of the land transferred to Cruz. Despite the TCT being void for non-compliance with 1607, the ownership did not transfer back to the heirs, for compliance with 1607 is merely for purposes of registering the title in the Torrens System. AMDG Mariano v. CA 222 SCRA 736 DOCTRINE: Redemption of the whole property by a co-owner does not vest in him sole ownership over said property but will inure to the benefit of all co-owners. In other words, it will not put an end to the existing state of co-ownership. Redemption is not a mode of termination of a co-ownership FACTS: Decendent Francisco Gosienfiao, mortgage a residential lot to Rural Bank. Failing to pay the loan, the lot was foreclosed and was awarded to be bank for being the highest bidder. Francisco died intestate and was survived by his heirs, the third-party defendants and plaintiff-appellants. Within the lot‘s one year redemption period, one of the third-party defendants, Amparo, bought the land from the bank using her personal funds. It is noted that the rest of the third-party defendants executed a ―Deed of Assignment of the Right of Repurchase‖ to Amparo. Amparo sold the lot to then defendant spouses Leonardo and Avelina Mariano (now petitioners in this case) who built a house on the said lot. When one of the plaintiff-appellants, Grace, discovered such sale, she requested for a confrontation between her and the defendants in the presence of the Barangay Captain. Grace asked that the defendants to present her claim of the

said property which never happened. It is also noted that the spouses sold the questioned lot to their children, Lazaro and Dionicia. The plaintiff-appellants filed a complaint for ―recovery of possession and legal redemption with damages‖ against the spouses. The plaintiffs alleged that they are co-owners of the lot hence they have the right to it since they did not sell their shares. Also they alleged that they have the right of redemption with regard to the shares of other co-owners sold to the spuses. In their answer, the spouses raised that such lot was purchased by Amparo alone hence she became the sole owner of the land. As to the right of redemption, though such right indeed exists, the spouses alleged that it was already barred by Statute of Limitations or of by laches. The RTC ruled in favor of the spouses and held that the plaintiffs had no right of ownership or possession over the lot. The lower court further explained that when the lot was bought by the bank, their rights were reduced to a mere right of redemption. Having failed to redeem the land and Amparo solely repurchasing the land, the plaintiff‘s lost all their rights over the lot. However, the Court of Appeals reversed the lower court‘s decision and ruled the plaintiffs were the coowners of the lot. It explained that although Amparo used her own money when she bought the land, it did not make her the sole owner. Such redemption inured to the benefit of all the coowners. ISSUES: 1. WON a notice was given to the petitioners? -- NO 2. WON the petitioners lost their right to redeem although no notice was received by them? -- NO HELD: The right of legal redemption is granted within one month after the written notice was delivered to the party granted of such right. It has been previously held by this Court that such written notice is an indispensible requirement and the right of legal redemption will only run after such notice was presented. Also, a copy of a deed of sale was considered as a notice. Looking at the pieces of evidence, no written notice of the sale was ever presented to the petitioners even during the confrontation. The private respondents (then plaintiff-apellants) exercised their right of redemption at the first opportunity by tendering the repurchase price to the spouses which refused to accept it. A written notice must be given to remove all uncertainty as to the sale, its terms and its validity, and to quiet and doubt that the alienation is not definitive. Establishing that no notice was ever presented, the 30-day period has not yet started. Lastly, the Court of Appeals was correct in ruling that the redemption of Amparo alone did not terminate the co-ownership rather such action inured to the benefit of all co-owners. Redemption is not a recognized mode of extinguishing co-ownership. GCG Tan v. CA 172 SCRA 660 DOCTRINE: Consolidation of ownership by mortgagee after expiration of redemption period terminates co-ownership. A co-owner who redeems a property with her own funds after such consolidation becomes the sole owner thereof.

FACTS: Tan Tiong Tick, married to Tan Ong Hun was the registered owner of a parcel of land and its improvements in Binondo Manila. During his lifetime, he obtained a loan secured by a real estate mortgage over the land. The land was mortgaged to China Bank to secure payment for several obligations. Tan Tiong Tick and Tan Ong Hun died without paying their obligations. They had six children George Laurel, Teodora, Rosa, Rosita, Mauro Umali, and D. Annie Tan. China Bank foreclosed on the mortgage. Two weeks before the redemption period expired, China Bank and the Heirs of Tan Tiong Tick entered into a settlement. It provided that the heirs were given right to repurchase even after the redemption period but before August 3, 1973. The heirs failed to redeem before the legal redemption period so China Bank consolidated its ownership and was issued a new TCT. However, D. Annie Tan exercised the right to repurchase pursuant to the settlement using her personal funds. But the title to the land was registered in the name of all the heirs. D. Annie Tan filed an action to reconvey the property to her and damages. RTC ruled that the property was co-owned by the heirs. CA affirmed. ISSUE: Whether or not co-ownership among the heirs was dissolved by the foreclosure and consolidation of title by the bank after the redemption period has expired? -- YES RULING: Since the lot and its improvement were mortgaged by the deceased parents, there can be no question that a co-ownership existed among the heirs during the period given by law to redeem the foreclosed property. Redemption by one during this period would have inured to the benefit of all. The records show, however, that when the petitioner purchased the disputed property on August 30, 1974, any co-ownership among the brothers and sisters no longer existed. The period to redeem had expired more than one year earlier, on July 6, 1973. The respondent China Bank consolidated its ownership and a new title was issued in the bank's name. When the heirs allowed the one year redemption period to expire without redeeming their parents' former property and permitted the consolidation of ownership and the issuance of a new title, the co-ownership was extinguished. Since D. Annie Tan used her personal fund to repurchase the property, she is the lawful sole owner. The respondent China Banking Corporation is ordered to execute the deed of sale over the disputed property in favor of the petitioner alone. VCL IV Heirs of Maninding v. CA 276 SCRA 601 DOCTRINE: Prescription, as a rule, does not run in favor of a co-heir or co-owner as long as he expressly or impliedly recognizes the co-ownership. Co-owners cannot acquire by prescription the share of the other co-owners, absent a clear repudiation of the co-ownership. It must be

clearly shown that he has repudiated the claims of the others, and that they were apprised of his claim of adverse and exclusive ownership, before the prescriptive period would begin to run. FACTS: On 31 July 1979 Segunda Maningding died. Her heirs allegedly discovered the transfers made by Roque Bauzon in favor of his children only in 1986. Consequently, the heirs sought the partition of the properties as well as the accounting of the produce but were unsuccessful. With regard to the sugarland, Roque Bauzon denied having executed the Affidavit of SelfAdjudication presented by petitioners. He claimed that he acquired ownership over both the sugarland and the riceland by donation propter nuptias from his parents Ramon Bauzon and Sotera Zulueta on 21 April 1926 in consideration of his marriage to Petra Loresco. Since the death of Ramon Bauzon in 1948, Roque had been in open, continuous, notorious, adverse and actual possession of the subject properties. RTC ruled that the parcels of land formed part of the estate of Ramon Bauzon and his wife Sotera Zulueta which, upon their death, devolved by right of succession to their children Segunda Maningding, Maria Maningding, Juan Maningding and Roque Bauzon in equal proindiviso shares. The court a quo however awarded both parcels to Segunda Maningding and Roque Bauzon as co-owners in equal shares after finding that Juan Maningding and Maria Maningding had already executed an Affidavit of Quitclaim and Renunciation. It rejected the deed of donation for failure to prove its due execution and authenticity and ruled that the same was negated by the Affidavit of Quitclaim and Renunciation of Juan Maningding and Maria Maningding in favor of Roque Bauzon and nullified the deed of sale by Roque Bauzon in favor of Luis Bauzon as regards the riceland and to Eriberta Bauzon with respect to the sugarland. It concluded that Roque Bauzon could not have validly conveyed both parcels as one-half (1/2) of each parcel rightfully belonged to Segunda Maningding and her heirs. CA ruled that the properties validly pertained to Roque Bauzon by virtue of the donation propter nuptias. Consequently, the transfers made by Roque Bauzon must be given effect. However, upon motion for reconsideration, the same deed of donation was declared null and void by the appellate court for failure to comply with Art. 633 of the old Civil Code, the law then applicable, which required for the validity of the deed of donation to be in a public instrument. Nevertheless, the same court maintained that the properties belonged to Roque Bauzon by virtue of acquisitive prescription. ISSUE: Whether or not Bauzon acquired the ownership of the land by acquisitive prescription -- YES HELD: Bauzon acquired ownership over the subject properties by acquisitive prescription. Prescription, in general, is a mode of acquiring (or losing) ownership and other real rights through the lapse of time in the manner and under conditions laid down by law, namely, that the possession should be in the concept of an owner, public, peaceful, uninterrupted and adverse. Acquisitive prescription is either ordinary or extraordinary. Ordinary acquisitive prescription requires possession in good faith and with just title for ten (10) years. In extraordinary prescription ownership and other real rights over immovable property are acquired through uninterrupted adverse possession thereof for thirty (30) years, without need of title or of good faith. The disputed lots are unregistered lands. While tax declarations and receipts are not conclusive evidence of ownership, yet, when coupled with proof of actual possession, as in the instant case,

tax declarations and receipts are strong evidence of ownership. Even assuming that the donation proper nuptias is void for failure to comply with formal requisites, it could still constitute a legal basis for adverse possession. Sixty (60) years have already elapsed. Prescription, as a rule, does not run in favor of a co-heir or co-owner as long as he expressly or impliedly recognizes the co-ownership. Co-owners cannot acquire by prescription the share of the other co-owners, absent a clear repudiation of the co-ownership. It must be clearly shown that he has repudiated the claims of the others, and that they were apprised of his claim of adverse and exclusive ownership, before the prescriptive period would begin to run. The evidence relative to the possession, as a fact upon which the alleged prescription is based, must be clear, complete and conclusive in order to establish said prescription without any shadow of doubt. Therefore while prescription among co-owners cannot take place when the acts of ownership exercised are vague and uncertain, such prescription arises and produces all its effects when the acts of ownership do not evince any doubt as to the ouster of the rights of the other coowners. As disclosed by the records, Roque Bauzon and his heirs possessed the property from 1948 to 1986 to the exclusion of petitioners who were never given their shares of the fruits of the properties, for which reason they demanded an accounting of the produce and the conveyance to them of their shares. CRF Aguilar v. CA 227 SCRA 472 DOCTRINE: Any of the Co-owners may demand the sale of the house and lot at any time and the other cannot object to such demand; thereafter the proceeds of the sale shall be divided equally according to their respective interests. (batasnatin) FACTS: Petitioner Virgilio and respondent Senen are brothers and purchased a house and lot in Parañaque where their father could spend and enjoy his remaining years in a peaceful neighborhood. Initially, the brothers agreed that Virgilio's share in the co-ownership was twothirds while that of Senen was one-third. By virtue of a written memorandum dated 23 February 1970, Virgilio and Senen agreed that henceforth their interests in the house and lot should be equal, with Senen assuming the remaining mortgage obligation of the original owners with the Social Security System (SSS) in exchange for his possession and enjoyment of the house together with their father. It was further agreed that Senen would take care of their father and his needs since Virgilio and his family were staying in Cebu. After Maximiano Aguilar died in 1974, petitioner demanded from private respondent that the latter vacate the house and that the property be sold and proceeds thereof divided among them but the latter refused to do so thereby causing the petitioner to file a complaint. In his complaint, petitioner prayed that the proceeds of the sale, be divided on the basis of two-thirds (2/3) in his favor and one-third (1/3) to respondent. Petitioner also prayed for monthly rentals for the use of the house by respondent after their father died. The respondent alleged that he had no objection to the sale as long as the best selling price could be obtained; that if the sale would be effected, the proceeds thereof should be divided equally; and, that being a co-owner, he was entitled to the use and enjoyment of the property.

On 26 July 1979, rendering judgment by default against defendant, the trial court found him and plaintiff to be co-owners of the house and lot, in equal shares on the basis of their written agreement. However, it ruled that plaintiff has been deprived of his participation in the property by defendant's continued enjoyment of the house and lot, free of rent, despite demands for rentals and continued maneuvers of defendants, to delay partition. The trial court also upheld the right of plaintiff as co-owner to demand partition. Since plaintiff could not agree to the amount offered by defendant for the former's share, the trial court held that this property should be sold to a third person and the proceeds divided equally between the parties.The trial court likewise ordered defendant to vacate the property and pay plaintiff P1,200.00 as rentals 2 from January 1975 up to the date of decision plus interest from the time the action was filed. ISSUE: Whether or not the plaintiff as a co-owner demand a partition of the said property? -- YES HELD: The Court holds that on the basis of the pleadings of the parties and the evidence presented ex parte, petitioner and respondents are co-owners of subject house and lot in equal shares; either one of them may demand the sale of the house and lot at any time and the other cannot object to such demand; thereafter the proceeds of the sale shall be divided equally according to their respective interests. Article 494 of the Civil Code provides that no co-owner shall be obliged to remain in the coownership, and that each co-owner may demand at any time partition of the thing owned in common insofar as his share is concerned. Corollary to this rule, Art. 498 of the Code states that whenever the thing is essentially, indivisible and the co-owners cannot agree that it be, allotted to one of them who shall indemnify the others, it shall be sold and its proceeds accordingly distributed. This is resorted to (1) when the right to partition the property is invoked by any of the co-owners but because of the nature of the property it cannot be subdivided or its subdivision would prejudice the interests of the co-owners, and (b) the co-owners are not in agreement as to who among them shall be allotted or assigned the entire property upon proper reimbursement of the co-owners. JGY Coronel v. Constantino 397 SCRA 128 DOCTRINE: The sale of the subject property made by a co-owner is limited to the portion which may be allotted to her upon the termination of co-ownership over the subject property with her children. FACTS: The subject property consists of two parcels of land is originally owned by Aguinaldo. One-half was inherited by petitioners Emilia together with her sons Benjamin, Catalino and Ceferino. The other half was inherited by respondents Constantino and Buensuceso. Respondents filed a complaint for declaration of ownership, quieting of title against the Petitioners. Respondents allege that a certain Santos and Bernardo purchased the property belonging to Emilia and her sons by virtue of a deed of sale signed by Emilia. Santos and Bernardo in turn sold the same to Constantino and Buensuceso.TC ruled in favor of the respondents.

ISSUE: WON the contract of sale executed by a parent co-owner is enforceable with respect to the share of her children. -- NO HELD: It is clear that petitioner Benjamin did not sign the document and that the shares of Catalino and Ceferino in the subject property were not sold by them. The deed of sale is not a competent proof that petitioner Benjamin had sold his own share of the subject property. It cannot be disputed that Benjamin did not sign the document and therefore, it is unenforceable against him. Emilia executed the instrument in her own behalf and not in representation of her three children. Article 493 of the Civil Code states: ―Each co-owner shall have the full ownership of his part and of the fruits and benefits pertaining thereto, and he may therefore alienate, assign or mortgage it, and even substitute another person in its enjoyment, except when personal rights are involved. But the effect of the alienation or the mortgage, with respect to the co-owners, shall be limited to the portion which may be allotted to him in the division upon the termination of the co-ownership.‖ Consequently, the sale of the subject property made by Emilia in favor of Santos and Bernardo is limited to the portion which may be allotted to her upon the termination of her co-ownership over the subject property with her children.

POSSESSION (ART. 523-561) FXRL Yu v. Pacleb G.R. No. 130316 DOCTRINE: Should a question arise regarding the fact of possession, the present possessor shall be preferred; if there are two possessors, the one longer in possession; if the dates of the possession are the same, the one who presents a title; and if all these conditions are equal, the thing shall be placed in judicial deposit pending determination of its possession or ownership through proper proceedings. FACTS: ● Respondents sold a parcel of land divided into three partitions to Rebecca del Rosario ● Del Rosario sold the land to Ruperto Javier. ● The title to the property remained in the name of the respondents. None of the sales earlier mentioned were annotated on the title. ● Javier offered to sell a parcel of land to the spouses Yu (petitioners). ● Petitioners accepted the offer and formally took over the property after execution of a contract to sell. ● At the time of such turnover, a portion of the property was occupied by the son of the respondent and the wife thereof as tenants. ● The son allegedly surrendered possession of his portion of the property to the petitioners. He was made trustee by the petitioners. ● Petitioners also caused the annotation on the title of the property ● Petitioners allege that they exercised open, public, and peaceful possession of the property for three years. During this span of time, the respondent was in the US. ● Respondent allegedly entered the property using Force, Intimidation, Strategy, Threat, and Stealth (FISTS) and ousted the trustee of the property, replacing him with another one of his sons. ● Petitioners filed an action for forcible entry. ● MTC and RTC ruled in favor of the petitioners. ● CA reverses the decision. ISSUE: W/N the petitioners had prior physical possession of the subject property. -- NO HELD: The Civil Code states that possession is the holding of a thing or the enjoyment of a right. In the grammatical sense, to possess means to have, to actually and physically occupy a thing, with or without right. "Possession always includes the idea of occupation x x x. It is not necessary that the person in possession should himself be the occupant. The occupancy can be held by another in his name." Without occupancy, there is no possession. Two things are paramount in possession. First, there must be occupancy, apprehension or taking. Second, there must be intent to possess (animus possidendi). In the current case, the petitioners‘ contentions fail on three grounds. First, they never truly occupied the land as a lower court ruling categorically mentioned that they were never in

possession of the property and that the tax declarations for the two years prior the return of the respondent were made in the name of the latter. Further, the land was in the possession of the son of the respondent. Second, the petitioners acknowledged the fact that the son of the respondent and the wife thereof were tenants of the respondent. As a mere tenant, the son had no right at all to surrender the property in question. Most importantly, the title of the land in question remained in the name of respondent. "As the registered owner, petitioner had a right to the possession of the property, which is one of the attributes of ownership." The Civil Code states that possession as a fact cannot be recognized at the same time in two different personalities except in the cases of co-possession. Should a question arise regarding the fact of possession, the present possessor shall be preferred; if there are two possessors, the one longer in possession; if the dates of the possession are the same, the one who presents a title; and if all these conditions are equal, the thing shall be placed in judicial deposit pending determination of its possession or ownership through proper proceedings. RSDM De Jesus v. CA 217 SCRA 307 DOCTRINE: Under the present Civil Code, the prescriptive period required for acquisition of immovable property is ten years if the possession is in good faith, and thirty years if in bad faith FACTS: The property in dispute is a parcel of residential land situated in Dampol 2nd, Pulilan, Bulacan, bounded on the North by a Vereda: on the South, by the Provincial Road; on the East, by Catalino Tayag (Tayao); on the West, by Macario de Leon, containing an area of 2565 square meters and covered by Tax Declaration in the name of Victoriano Felipe Private respondent executed a sworn statement declaring herself the only heir of the deceased Victoriano Felipe and adjudicating to herself the ownership of the land in question. More than twelve years later, petitioners herein filed in the Court of First Instance, an action for recovery of ownership and possession and quieting of title to the abovementioned piece of land covered by Tax Declaration, alleging among others: "that their grandfather, Santiago de Jesus during his lifetime owned the residential lot; that Santiago de Jesus died before the outbreak of World War II, leaving three (3) sons, namely: Mariano, Exequiel, and Jose, all surnamed de Jesus; that Mariano de Jesus died on September 3, 1956 leaving eight (8) surviving children, namely: Edgardo, Remedios, Juanita, Juliano, Jose, Flordeliza, Reynaldo, and Ernesto, all surnamed de Jesus and all of them plaintiffs; that Exequiel de Jesus died on April 3, 1948, survived by two (2) children — Priscilo and Corazon, both surnamed de Jesus, also plaintiffs in this case; while Jose de Jesus died before the outbreak of World War II without any issue. CFI found for the plaintiffs. The Court of Appeals set aside the judgment of the trial court in a decision. ISSUE: Whether or not the petitioner has the right to the ownership and possession of the residential lot. -- YES HELD: Yes. The petitioner has the rigth to the ownership and possesion of the residential lot.

Private respondent's pretensions to acquisitive prescription may not succeed even under Act No. 190, the Code of Civil Procedure. Under Section 41 thereof, good faith and just title are not required for purposes of acquisitive prescription; adverse possession in either character ripens into ownership after the lapse of ten years. The just title required for acquisitive prescription to set in is not "titulo verdadero y valido" — such title which by itself is sufficient to transfer ownership without the necessity of letting the prescriptive period elapse, but only "titulo colorado" — or such title where, although there was a mode of transferring ownership, still something is wrong because the grantor is not the owner, and incidentally, it may perhaps be mentioned that prescription running even after the effectivity of the New Civil Code on August 30, 1950, continued to be governed by Section 41 of the Old Civil Code. Under the present Civil Code, the prescriptive period required for acquisition of immovable property is ten years if the possession is in good faith, and thirty years if in bad faith. Such open, continuous, exclusive and notorious occupation of the disputed property for thirty years must be conclusively established. Reckoned from the time she executed the affidavit of adjudication in 1961, eleven years after the New Civil Code had taken effect, private respondent's possession of the contested lot is far too short of the prescriptive period of thirty years considering that her possession is in bad faith. The filing of the petition for recovery of ownership and possession and quieting of title by petitioners on April 27, 1973 was well below the acquisitive prescriptive period for private respondent, which is thirty years under Article 1141 of the present Civil Code. In this case, the statutory period of prescription is deemed to have commenced when petitioners were made aware of a claim adverse to them, that is, when the affidavit of adjudication was duly registered with the Registry of Deeds which, at the earliest may be considered to be in 1974, when private respondent was able to secure a tax declaration in her name. MRAM SMPSM v. BCDA G.R. No. 142255 DOCTRINE: Prescription does not apply if the subject land is covered by a Torrens Title, as in the case at bar. Moreover, prescription does not run against the State. FACTS: As a background, in 1992, RA 7227 created the BCDA to "accelerate the sound and balanced conversion into alternative productive uses of the Clark and Subic military reservations and their extensions," and "to raise funds by the sale of portions of Metro Manila military camps." Pursuant to this Act, Pres. Ramos issued EO No. 40, series of 1992, specifying, among others, the portions of Metro Manila military camps to be utilized to generate capital for the BCDA. Among these Metro Manila military camps is Fort Bonifacio, located in the City of Makati and the Municipality of Taguig. Under EO No. 40, 214 hectares in Fort Bonifacio were earmarked for development and disposition to raise funds for BCDA projects and to use such funds to accelerate the sound and balanced conversion into alternative productive uses of the Clark and Subic military reservations and their extensions. The members of SMPMI, allegedly comprising over 20,000 families, are residents of Fort Bonifacio occupying a portion of it specifically Lot 4, Lot 3, and Lot 1 with an aggregate area of 97.58 hectares allegedly covered by SWO-00-001265 in the name of BCDA. Petitioner maintains that its members have been occupying peacefully and continuously these lots in Fort

Bonifacio. It alleges that Fort Bonifacio is covered by the TCT in the name of the United States of America, hence the Philippine Government.It further alleges that BCDA, pursuant to RA 7227, the Municipality of Taguig, through its Mayor, sent 30-day notices of eviction to its members. It asserts the illegality of the imminent eviction, for which the present action was filed, as the land which petitioner‘s members are occupying is still owned by the USA and not by the Philippine Government. It further asserts that Section 8 of RA 7227, which stipulates the area of Fort Bonifacio specifically covering 2,276 hectares, did not provide any technical description on what is indeed covered. Besides, it strongly argues that because of the lack of "tie line" locating the exact position claimed by BCDA, the latter cannot illegally stake its claim on the whole of Fort Bonifacio to the prejudice not only of its members but also of all persons or entities occupying said area. Petitioner also contends that what complicates the controversy is the approval of the BCDA plan by the Bureau of Land without due certification from the Land Registration Authority. ISSUE: Who between petitioner SMPMI and BCDA has the right of possession over the particular parcels of land which are subject of this petition. HELD: The Court ruled in favor of BCDA for the ff. reasons: 1. The Philippine Government, and now the BCDA, has title and ownership over Fort Bonifacio. 2. BCDA has convincingly shown that the in the name of the USA covering Fort Bonifacio was cancelled by TCT No. 61524 issued in 1958 in the name of the Republic of the Philippines. In 1995, the TCT was transferred in the name of BCDA. Thus, BCDA has valid titles over Fort Bonifacio which have become indefeasible and beyond question. On the other hand, SMPMI has not presented any title or deed to demonstrate ownership or any interest in the subject lots. 3. Third, it is clear from the records that BCDA has been granted a clear mandate by RA 7227, specifically by its Sections 7 and 8, and re-enforced by EO No. 40, series of 1992, to take over and administer Fort Bonifacio for its development and disposition to raise funds for BCDA projects, among others, the conversion of Clark and Subic military reservations and their extensions to alternative productive uses. 4. It is basic that ownership or dominion includes the right of possession. In traditional Roman law, jus possidendi or the right to possess is fundamentally not only an attribute of ownership but also a direct consequence of ownership. Thus, from BCDA‘s ownership of the subject lots originates the rights of possession, use, and disposition 5. Prescription does not apply if the subject land is covered by a Torrens Title, as in the case at bar. 6. Neither prescription nor laches runs against the State. Thus, even granting arguendo that the subject lands had been erroneously issued titles in favor of third parties, which is definitely not the case; neither prescription nor estoppel by laches applies against the State. In a catena of cases, we have consistently reiterated this hornbook doctrine. In Reyes v. Court of Appeals, it was held that: In so far as the timeliness of the action of the Government is concerned, it is basic that prescription does not run against the State. x x x The case law has also been: ‗When the government is the real party in interest, and is proceeding mainly to assert its own rights and recover its own property, there can be no defense on the ground of laches or limitation.‘ x x x

‗Public land fraudulently included in patents or certificates of title may be recovered or reverted to the State in accordance with Section 101 of the Public Land Act. Prescription does not lie against the State in such cases for the Statute of Limitations does not run against the State. The right of reversion or reconveyance to the State is not barred by prescription.‘ FMM Heirs of Soriano v. CA 363 SCRA 87 DOCTRINE: Possession and ownership are distinct legal concepts. There is ownership when a thing pertaining to one person is complete does not own it and cannot evict themly subjected to his will in a manner not prohibited by law and consistent with the rights of others. Ownership confers certain rights to the owner, among which are the right to enjoy the thing owned and the right to exclude other persons from possession thereof. On the other hand, possession is defined as the holding of a thing or the enjoyment of a right. Literally, to possess means to actually and physically occupy a thing with or without right. Possession may be had in one of two ways: possession in the concept of an owner and possession of a holder. FACTS: On 30 June 1967, the heirs of Adrian Soriano, who died intestate in 1947, leased the property to spouses David de Vera and Consuelo Villasista for a period of fifteen (15) years beginning 1 July 1967. The contract of lease provided that Roman Soriano (―Soriano‖), one of the children of the late Adriano, will be the caretaker of the property during the period of the lease. During the effectivity of the lease contract, the heirs of Adriano Soriano entered into an extrajudicial settlement of his estate. The property subject of this case was adjudicated to seven (7) of his rune (9) children pro-indiviso. On 11 January 1968, the property was divided into two (2) lots, Lot No. 60052 and Lot No. 8459. The former lot was assigned to Lourdes, Candido and the heirs of Dionisia while the latter lot was assigned to Francisco, Librada, Elcocadio and Soriano. The new owners of Lot No. 60052 sold the portions assigned to them to spouses Braulio and Aquilina Abalos. Likewise, the new owners of Lot 8459, except Soriano, sold their shares to the Spouses Abalos. On 14 March 1968, On March 14, 1968, the Spouses de Vera ousted Soriano as caretaker and appointed Isidro Versoza and Vidal Versoza as his substitutes. Thereafter, Soriano filed a case for reinstatement and reliquidation against the Spouses de Vera. On 30 September 1969, the Agrarian Court rendered a decision authorizing the ejectment of Roman. On appeal, the decision was reversed by the Court of Appeals (―CA‖). However, before it was executed, the parties entered into a post-decisional agreement wherein the spouses de Vera allowed Soriano to sub-lease the property until the termination of the original lease on 30 June 1982. This agreement was approved by the court in an order dated 22 December 1972. On 16 August 1976, the Spouses Abalos filed with the then Court of First Instance (―CFI‖) of Pangasinan at Lingayen an application for registration of title. The application claimed ownership of the entire lot No. 60052 and 3/4 pro-indiviso of Lot No. 8459. The Director of

Lands (―Republic‖) and Soriano filed separate oppositions to the application. The latter‘s opposition alleged that the two (2) lots subject of the application have not yet been subdivided and remained as one parcel; that he is the co-owner pro-indiviso of the combined area of the two (2) lots and not just to one-fourth (1/4) of Lot No. 8459 as alleged in the application; and that the applicant‘s source of ownership is voidable. The Republic subsequently conceded that the land applied for was private and disposable. The RTC, acting as a Land Registration Court, granted the application for Registration. On 13 April 1983, after the expiration of the original lease and the sub-lease in favor of Soriano, the Spouses Abalos filed a case for unlawful detainer against Soriano. This case, however, was dismissed on motion of the complainants, Spouses Abalos. For their part, Elcocadio, Librada, Soriano, Francisco, Lourdes, Candido and the heirs of Dionisia, filed a complaint to annul the deeds of sale they executed in favor of the Spouses Abalos or should the deeds be not annulled, to allow Soriano, Elcocadio and Librada to redeem those shares sold by Candido, Lourdes, Francisca and the heirs of Dionisia and to uphold Soriano‘s possession of the fishpond portion of the property as a tenant-caretaker. After the dismissal of the case for unlawful detainer, the Abalos spouses a motion for execution of the post-decisional order embodying the agreement of Soriano and the de Vera spouses allowing the former to sublease the property. In the mean time, Soriano died on 11 December 1985. Thus, the complaint in the civil case for annulment of document and/or redemption, ownership and damages, was amended to substitute Soriano‘s heirs. Several decisions by the Supreme Court (―SC‖) were rendered but the issue on possession remains challenged. ISSUE: Whether an owner is also entitled to possession. -- NO HELD: The Supreme Court held that possession and ownership are distinct legal concepts. There is ownership when a thing pertaining to one person is completely subjected to his will in a manner not prohibited by law and consistent with the rights of others. Ownership confers certain rights to the owner, among which are the right to enjoy the thing owned and the right to exclude other persons from possession thereof. On the other hand, possession is defined as the holding of a thing or the enjoyment of a right. Literally, to possess means to actually and physically occupy a thing with or without right. Possession may be had in one of two ways: possession in the concept of an owner and possession of a holder. A person may be declared owner but he may not be entitled to possession. The possession may be in the hands of another either as a lessee or a tenant. The SC also held that the exercise of their rights of ownership are subject to limitations that may be imposed by law. RGGM State Investment House v. CA 254 SCRA 368 DOCTRINE: A mortgagee who has foreclosed property is not considered in good faith when such mortgagee has or is expected to have knowledge of any defect in the title; a prior buyer in good faith, although merely under a contract to sell, is preferred over a mortgagee since if the

original owner delivered title, he would not anymore be able to mortgage the thing. FACTS: A contract to sell was executed between spouses Canuto and Oreta, and Solid Homes. The sale involved a parcel of land (511 sq. m.). Upon signing of the contract, Oreta made payment with the agreement that the balance shall be paid in installments. Meanwhile. Solid Homes executed several mortgages in favor of State Investment over its subdivided parcels of land, including the subject of land of the mentioned contract to sell. Such mortgage was foreclosed upon failure of Solid to comply with its obligations. Thereafter, Solid through a MOA, negotiated for the deferment of consolidation of ownership over the foreclosed properties. It further committed itself to redeem the properties. Spouses Canuto after a few years filed a complaint before the HLURB against Solid and State for failure on the part of Solid to execute the necessary absolute deed of sale as well as to deliver title to property subject of the contract to sell despite full payment. Solid alleged that its obligations under the contract have become so difficult for performance. Solid , in effect, asked to be partially released from its obligations by delivering another parcel of land in substitution to the subject of the sale. State, on the other hand averred that unless Solid pays the redemption price, it has a right to hold on to the foreclosed properties. However, HLURB ordered State to execute a deed of conveyance in favor of complainants and deliver the title to the land. Solid was then ordered to pay State the portion of the loan which corresponds to the value of the lot. this judgment was sustained by the Board of Commissioners, Office of the President, and Court of Appeals. ISSUES: 1. W/N spouses Oreta's unregistered rights are superior over State's registered mortgage over the property 2. W/N State has the right to rely on the face of the Torrens title HELD: 1. State's registered mortgage right over the property is inferior to that of respondents' unregistered right. The unrecorded sale is preferred for the reason that if the original owner (Solid) had parted with the ownership of the thing sold, he would no longer have the free disposal of it and would not be able to mortgage it. Registration of the mortgage is not important since it is understood to be without prejudice to the rights of third persons. 2. As a general rule, where there is nothing in the title to indicate any cloud or vice in the ownership thereof, the purchaser is not required to explore further. An exception to this is when the mortgagee or purchaser has knowledge of a defect or lack of title on the part of the vendor or that he was aware of sufficient facts to induce a reasonably prudent man to inquire further. In this case, petitioner knows full well that Solid is engaged in selling subdivision lots. Therefore, as founded on jurisprudence, it should have taken necessary precautions to ascertain any flaw. Moreover, the uniform practice of financing institutions is to investigate, examine, and assess real property offered as security. State is therefore not a mortgagee in good faith. MCSS DBP v. CA 316 SCRA 650

FACTS: DBP bought 91,188.30 square meters of land, consisting of 159 lots, in the proposed Diliman Estate Subdivision of the PHHC. However, the sale of the lots to DBP, Lots 2 and 4, which form part of said 159 lots, were still sold by PHHC to the spouses Nicandro, for which 2 deeds of sale were issued to them by PHHC. Upon learning of PHHC‘s previous transaction with DBP, the spouses filed a complaint against DBP and the PHHC to rescind the sale of Lots 2 and 4 by PHHC in favor of DBP. The CFI held that the sale of Lots 2 and 4, to DBP is null and void, for being in violation of Section 13 of the DBP Charter. ISSUE: WON spouses possess the legal personality to question the legality of the sale? -- YES HELD: The spouses stand to be prejudiced by reason of their payment in full of the purchase price for the same lots which had been sold to DBP by virtue of the transaction in question.The general rule is that the action for the annulment of contracts can only be maintained by those who are bound either principally or subsidiarily by virtue thereof. However, a person who is not obliged principally or subsidiarily in a contract may exercise an action for nullity of the contract if he is prejudiced in his rights with respect to one of the contracting parties, and can show the detriment which could positively result to him from the contract in which he had no intervention. NKVS San Miguel Corporation v. CA 185 SCRA 727 DOCTRINE: Tax declarations and receipts are not conclusive evidence of ownership or right of possession over a piece of land. They are merely indicia of a claim of ownership. FACTS: San Miguel Corporation purchased from Silverio Perez a parcel of land located in Sto. Tomas, Batangas. Claiming ownership in fee simple of the land, SMC filed before the then CFI, now RTC of Batangas an application for its registration under the Land Registration Act. The Solicitor General, opposed the application for registration contending that SMC's claim of ownership, that the parcel of land in question is part of the public domain, and that SMC, being a private corporation, is disqualified under Section 11, Article XIV of the Constitution from holding alienable lands of the public domain. The Solicitor General thereafter authorized the Provincial Fiscal of Batangas to appear in said case, subject to his supervision and control. Petitioner's claim that its predecessor-in-interest had open, exclusive and undisputed possession of Lot 684 for more than thirty years is anchored on certain documentary and testimonial evidence. Its documentary evidence consist of tax declaration No. 923 wherein it appears that in 1974, Silverio Perez declared as his own for taxation purposes, a certain riceland with an area of 1.5657 hectares located in Sta. Anastacia, Sto. Tomas, Batangas, and a certification of the Office of the Treasurer of Sto. Tomas to the effect that in 1977, Silverio Perez paid realty taxes for the land subject of tax declaration no. 923. ISSUE: W/N Tax declarations are evidence of ownership. -- NO HELD:

NO, it is not a conclusive evidence of ownership or right of possession of a piece of land. Tax declarations and receipts are not conclusive evidence of ownership or right of possession over a piece of land. They are merely indicia of a claim of ownership. Tax declarations only become strong evidence of ownership of land acquired by prescription, a mode of acquisition of ownership relied upon by petitioner in this case, when accompanied by proof of actual possession. Such proof of actual possession was sought to be provided by the testimony of vendor Silverio Perez that he had been in possession of the property since 1933 until he sold it to SMC in 1975; that the property was given to him by his parents when he got married; that no document evidenced that transfer; that it had been in the possession of his parents since 1925; that he had declared the property in his name for taxation purposes; that he had paid taxes therefor, and that he was in peaceful, continuous and exclusive possession of the property until its sale to SMC. Petitioner did not present other witnesses to corroborate Perez' testimony. Its other witness, Antonio M. de las Alas, Jr., a lawyer of the petitioner, simply testified that he handled the negotiations for the purchase of the property; that SMC was authorized to own and acquire property as shown by its articles of incorporation and by-laws; that since its acquisition in 1975, the property had been used as a hatchery farm of SMC; that SMC's possession in the concept of an owner had been continuous, adverse and against the whole world, and that the land was declared for taxation purposes still in the name of Silverio Perez . We hold that there is paucity of evidence of actual, notorious and exclusive possession of the property on the part of vendor Silverio Perez so as to attach to it the character of an express grant from the govemment. Indeed, as correctly held by the Court of Appeals, Silverio Perez's testimony, being uncorroborated, is simply self-serving and hence, undeserving of any weight. AMPS Equatorial Realty Development v. Mayfair Theatre 370 SCRA 56 DOCTRINE: Under Article 1380 to 1381(3) of the Civil Code, a contract otherwise valid may be rescinded by reason of injury to third persons, like creditors. Nonetheless, acquisition by a third person of the property subject of the contract is an obstacle to the action for its rescission where it is shown that such third person is in lawful possession of the subject of the contract and that he is in good faith. FACTS: Carmelo Inc. (Lessor) and Mayfair entered into two contracts of lease over certain portions of a two-storey building. Both contracts contained the following stipulation: That if the LESSOR should desire to sell the leased premises, the LESSEE shall be given 30-days exclusive option to purchase the same. In the event, however, that the leased premises is sold to someone other than the LESSEE, the LESSOR is bound and obligated, as it hereby binds and obligates itself, to stipulate in the Deed of Sale hereof that the purchaser shall recognize this lease and be bound by all the terms and conditions thereof.

Later on, Carmelo informed Mayfair that a certain Jose Araneta was offering to buy the whole property for US Dollars 1,200,000. Carmelo asked Mayfair if it was willing to buy the property for Six to Seven Million Pesos. Mayfair sent letter to Carmelo purporting to express interest in acquiring not only the leased premises but "the entire building and other improvements if the price is reasonable. However, Carmelo proceeded with the sale of the whole property which included the leased premises to Equatorial. Hence, Mayfair instituted the action a quo for specific performance and annulment of the sale of the leased premises to Equatorial. ISSUE: Is the contract of sale between Carmelo (lessor) and Equatorial still rescissible in spite of the possession of Equatorial? HELD: Still rescissible as the exception does not apply to Equatorial – not third party to the rescissible contract and possession was in bad faith. What Carmelo and Mayfair agreed to, by executing the two lease contracts, was that Mayfair will have the right of first refusal in the event Carmelo sells the leased premises. It is undisputed that Carmelo did recognize this right of Mayfair, for it informed the latter of its intention to sell the said property in 1974. There was an exchange of letters evidencing the offer and counter-offers made by both parties. Carmelo, however, did not pursue the exercise to its logical end. While it initially recognized Mayfair's right of first refusal, Carmelo violated such right when without affording its negotiations with Mayfair the full process to ripen to at least an interface of a definite offer and a possible corresponding acceptance within the "30-day exclusive option" time granted Mayfair, Carmelo abandoned negotiations, kept a low profile for some time, and then sold, without prior notice to Mayfair, the entire Claro M Recto property to Equatorial. Since Equatorial is a buyer in bad faith, this finding renders the sale to it of the property in question rescissible. We agree with respondent Appellate Court that the records bear out the fact that Equatorial was aware of the lease contracts because its lawyers had, prior to the sale, studied the said contracts. As such, Equatorial cannot tenably claim to be a purchaser in good faith, and, therefore, rescission lies. Contract of Sale was not voidable but rescissible. Under Article 1380 to 1381(3) of the Civil Code, a contract otherwise valid may nonetheless be subsequently rescinded by reason of injury to third persons, like creditors. According to Tolentino, rescission is a remedy granted by law to the contracting parties and even to third persons, to secure reparation for damages caused to them by a contract, even if this should be valid, by means of the restoration of things to their condition at the moment prior to the celebration of said contract. It is a relief allowed for the protection of one of the contracting parties and even third persons from all injury and damage the contract may cause, or to protect some incompatible and preferent right created by the contract. Rescission implies a contract which, even if initially valid, produces a lesion or pecuniary damage to someone that justifies its invalidation for reasons of equity. It is true that the acquisition by a third person of the property subject of the contract is an obstacle to the action for its rescission where it is shown that such third person is in lawful possession of the subject of the contract and that he did not act in bad faith. However, this rule

is not applicable in the case before us because the petitioner is not considered a third party in relation to the Contract of Sale nor may its possession of the subject property be regarded as acquired lawfully and in good faith KGS Maglucot v. Maglucot 329 SCRA 78 DOCTRINES: 1. Partition; The first phase of a partition and/or accounting suit is taken up with the determination of whether or not a co-ownership in fact exists, (i.e. not otherwise legally proscribed) and may be made by voluntary agreement of all the parties interested in the property. 2. Partition; The second phase commences when it appears that ―the parties are unable to agree upon the partition‖ directed by the court. In that event, partition shall be done for the parties by the court with the assistance of not more than three (3) commissioners. 3. Under the present rule, the proceeding of the commissioners without being confirmed by the court are not binding upon the parties. 4. Parties to a partition proceeding, who elected to take under partition, and who took possession of the portion allotted to them, are estopped to question title to portion allotted to another party. 5. In cases involving oral partition under which the parties went into possession, exercised acts of ownership, or otherwise partly performed the partition agreement, equity will confirm such partition and in a proper case decree title in accordance with the possession in severalty. FACTS: Lot 1639 was registered under the names of six persons. Thereafter, one of the co-owners and his predecessor-in-interest (Tomas) filed a petition to subdivide the lot, which was granted. As such, said lot was physically subdivided into A-F. Lot D (subject lot) was given to Roberto Maglucot. Portion of it was rented to Guillermo Maglucot; and subsequently, to Leopoldo and Severo Maglucot (respondents). The respondents built houses on their corresponding leased lots. After years, they stopped paying rentals as they are claiming ownership over the subject lot. Then, Tomas and other petitioners filed a case in the RTC for recovery of the possession for Lot D. They claimed that no partition took place, as it was not annotated in the title. The RTC ruled that there was partition; and ordered respondents to demolish the houses in Lot D and vacate the premises thereof. The CA reversed the decision and ruled that the sketch plan and tax declarations relied upon by petitioners are not conclusive evidence of partition. ISSUE: Was the oral and physical partition of Lot 1639 valid considering it was not annotated in the title? -- YES HELD: An action for partition is comprised of two phases: first, an order for partition which determines whether a co-ownership in fact exists, and whether partition is proper; and, second, a decision confirming the sketch or subdivision submitted by the parties or the commissioners appointed by

the court, as the case may be. The first phase of a partition and/or accounting suit is taken up with the determination of whether or not a co-ownership in fact exists, (i.e., not otherwise legally proscribed) and may be made by voluntary agreement of all the parties interested in the property. This phase may end with a declaration that plaintiff is not entitled to have a partition either because a co-ownership does not exist, or partition is legally prohibited. It may end, upon the other hand, with an adjudgment that a co-ownership does in truth exist, partition is proper in the premises and an accounting of rents and profits received by the defendant from the real estate in question is in order. In the latter case, the parties may, if they are able to agree, make partition among themselves by proper instruments of conveyance, and the court shall confirm the partition so agreed upon. In either case – i.e., either the action is dismissed or partition and/or accounting is decreed – the order is a final one, and may be appealed by any party aggrieved thereby. The second phase commences when it appears that "the parties are unable to agree upon the partition" directed by the court. In that event, partition shall be done for the parties by the court with the assistance of not more than three (3) commissioners. This second stage may well also deal with the rendition of the accounting itself and its approval by the court after the parties have been accorded opportunity to be heard thereon, and an award for the recovery by the party or parties thereto entitled of their just share in the rents and profits of the real estate in question. Such an order is, to be sure, final and appealable. However, this Court notes that the order of partition was issued when the ruling in Fuentebella vs. Carrascoso, which held that the order of partition is interlocutory, was controlling. In addition, the reports of the commissioners not having been confirmed by the trial court are not binding. In this case, both the order of partition and the unconfirmed sketch plan are, thus, interlocutory. Nevertheless, where parties do not object to the interlocutory decree, but show by their conduct that they have assented thereto, they cannot thereafter question the decree, especially, where, by reason of their conduct, considerable expense has been incurred in the execution of the commission. Respondents in this case have occupied their respective lots in accordance with the sketch/subdivision plan. They cannot after acquiescing to the order for more than forty (40) years be allowed to question the binding effect thereof. This case is to be distinguished from the order in the action for partition in Arcenas vs. Cinco. In that case, the order was clearly interlocutory since it required the parties " to submit the corresponding deed of partition to the Court for its approval." Here, the order appointed two commissioners and directed them merely to approve the sketch plan already existing and tentatively followed by the parties. Under the present rule, the proceedings of the commissioners without being confirmed by the court are not binding upon the parties. However, this rule does not apply in case where the parties themselves actualized the supposedly unconfirmed sketch/subdivision plan. The purpose of court approval is to give effect to the sketch/subdivision plan. In this case, the parties themselves or through their predecessors-in-interest implemented the sketch plan made pursuant to a court order for partition by actually occupying specific portions of Lot No. 1639 in 1952 and continue to do so until the present until this case was filed, clearly, the purpose of the court approval has been met. This statement is not to be taken to mean that confirmation of the commissioners may be dispensed with but only that the parties herein are estopped from raising this question by their own acts of ratification of the supposedly non-binding sketch/subdivision plan. The records of the case show that sometime in 1946 there was a prior oral agreement to tentatively partition Lot No. 1639. By virtue of this agreement, the original co-owners occupied specific portions of Lot No. 1639. It was only in 1952 when the petition to subdivide Lot No.

1639 was filed because two of the co-owners, namely Hermogenes Olis and heirs of Pascual Olis, refused to have said lot subdivided and have separate certificates of title. Significantly, after the 1952 proceedings, the parties in this case by themselves and/or through their predecessors-in-interest occupied specific portions of Lot No. 1639 in accordance with the sketch plan. Such possession remained so until this case arose, or about forty (40) years later. From its order in 1952, it can be gleaned that the CFI took notice of the tentative subdivision plan by oral partition of the parties therein. Further, it appears that said court was aware that the parties therein actually took possession of the portions in accordance with the sketch/subdivision plan. With this factual backdrop, said court ordered the partition and appointed two (2) commissioners to approve the tentative sketch/subdivision plan. It would not be unreasonable to presume that the parties therein, having occupied specific portions of Lot No. 1639 in accordance with the sketch/subdivision plan, were aware that it was that same sketch/subdivision plan which would be considered by the commissioners for approval. There is no showing that respondents by themselves or through their predecessors-in-interest raised any objections. On the contrary, the records show that the parties continued their possession of the specific portions of Lot No. 1639 pursuant to the sketch/subdivision plan. It has been previously held that a co-owner, who, though not a party to a partition accepts the partition allotted to him, and holds and conveys the same in severalty, will not be subsequently permitted to avoid partition. It follows that a party to a partition is also barred from avoiding partition when he has received and held a portion of the subdivided land especially in this case where respondents have enjoyed ownership rights over their share for a long time. Parties to a partition proceeding, who elected to take under partition, and who took possession of the portion allotted to them, are estopped to question title to portion allotted to another party. A person cannot claim both under and against the same instrument. In other words, they accepted the lands awarded them by its provisions, and they cannot accept the decree in part, and repudiate it in part. They must accept all or none. Parties who had received the property assigned to them are precluded from subsequently attacking its validity of any part of it.[38] Here, respondents, by themselves and/or through their predecessors-in-interest, already occupied of the lots in accordance with the sketch plan. This occupation continued until this action was filed. They cannot now be heard to question the possession and ownership of the other co-owners who took exclusive possession of Lot 1639-D also in accordance with the sketch plan. Partition may be inferred from circumstances sufficiently strong to support the presumption. Thus, after a long possession in severalty, a deed of partition may be presumed. It has been held that recitals in deeds, possession and occupation of land, improvements made thereon for a long series of years, and acquiescence for 60 years, furnish sufficient evidence that there was an actual partition of land either by deed or by proceedings in the probate court, which had been lost and were not recorded. And where a tract of land held in common has been subdivided into lots, and one of the lots has long been known and called by the name of one of the tenants in common, and there is no evidence of any subsequent claim of a tenancy in common, it may fairly be inferred that there has been a partition and that such lot was set off to him whose name it bears. It must be noted that there was a prior oral partition in 1946. Although the oral agreement was merely tentative, the facts subsequent thereto all point to the confirmation of said oral partition. By virtue of that agreement, the parties took possession of specific portions of the subject lot. The action for partition was instituted because some of the co-owners refused to have separate

titles issued in lieu of the original title. In 1952, an order for partition was issued by the cadastral court. There is no evidence that there has been any change in the possession of the parties. The only significant fact subsequent to the issuance of the order of partition in 1952 is that respondents rented portions of Lot No. 1639-D. It would be safe to conclude, therefore, that the oral partition as well as the order of partition in 1952 were the bases for the finding of actual partition among the parties. The legal consequences of the order of partition in 1952 having been discussed separately, we now deal with oral partition in 1946. Given that the oral partition was initially tentative, the actual possession of specific portions of Lot No. 1639 in accordance with the oral partition and the continuation of such possession for a very long period indicate the permanency and ratification of such oral partition. The validity of an oral partition is already wellsettled. In Espina vs. Abaya, we declared that an oral partition is valid. In Hernandez vs. Andal, reiterated in Tan vs. Lim, this Court has ruled, thus: on general principle, independent and in spite of the statute of frauds, courts of equity have enforce oral partition when it has been completely or partly performed. Regardless of whether a parol partition or agreement to partition is valid and enforceable at law, equity will proper cases where the parol partition has actually been consummated by the taking of possession in severalty and the exercise of ownership by the parties of the respective portions set off to each, recognize and enforce such parol partition and the rights of the parties thereunder. Thus, it has been held or stated in a number of cases involving an oral partition under which the parties went into possession, exercised acts of ownership, or otherwise partly performed the partition agreement, that equity will confirm such partition and in a proper case decree title in accordance with the possession in severalty. In numerous cases it has been held or stated that parol partition may be sustained on the ground of estoppel of the parties to assert the rights of a tenant in common as to parts of land divided by parol partition as to which possession in severalty was taken and acts of individual ownership were exercised. And a court of equity will recognize the agreement and decree it to be valid and effectual for the purpose of concluding the right of the parties as between each other to hold their respective parts in severalty. A parol partition may also be sustained on the ground that the parties thereto have acquiesced in and ratified the partition by taking possession in severalty, exercising acts of ownership with respect thereto, or otherwise recognizing the existence of the partition. A number of cases have specifically applied the doctrine of part performance, or have stated that a part performance is necessary, to take a parol partition out of the operation of the statute of frauds. It has been held that where there was a partition in fact between tenants in common, and a part performance, a court of equity would have regard to enforce such partition agreed to by the parties. JPOT Gabrito v. CA 167 SCRA 771 DOCTRINE: Pending final adjudication of ownership by the Bureau of Lands, the Court has jurisdiction to determine in the meantime the right of possession over the land. FACTS: The spouses Roberto Tan and Benita Ching-Tan filed a complaint in the Municipal Trial Court against defendants Maximo Gabrito, et al., alleging that they are the possessors and legal owners of the property situated at No. 107 Gordon Ave., New Kalalake, Olongapo City as evidenced by Tax Declaration No. 4-2046. The defendants are leasing portions of this parcel of land, each paying the corresponding monthly rentals due thereon. The Tans found it fit for them to make said lot a residential house for them instead. They furnished requests to Gabrito and

others stating their reasons and three months later they (Gabrito, et al.) were still being stubborn in keeping with the operations of their commercial spaces. On November 22, 1985, the Municipal Trial Court ruled and the dispositive portion of which reads: WHEREFORE, judgment is hereby rendered for all the defendants to vacate the parcel of land described in par. 3 of the complaint, removing therefrom the buildings and any other improvements respectively owned by them; and to pay plaintiffs the following as reasonable compensation for the use of the premises: Maximo Gabrito-aT P250.00 per month from April 1984 until he vacates the premises; Roger Libut-at P150.00 per month from May 1984 until he vacates the premises; Liza de Vera-at P150.00 per month from April 1984, until she vacates the premises; Carmelita Uy-at Pl 70.00 per month from April 1984, until she vacates the premises. for all defendants to pay, in equal shares, damages by way of attorney's fees in the amount of ONE THOUSAND PESOS ( P1,000.00 ) as well as costs SO ORDERED. (Rollo, p. 35). Gabrito, et al., found the findings contrary and elevated the matter to the Regional Trial Court as well as the Court of Appeals, leading up to the Supreme Court. ISSUES: 1. W/N an action for unlawful detainer is the proper action to oust petitioners from their occupation of the land in dispute? 2. Who has a better right to possess the land which definitely falls under the jurisdiction of the Municipal Trial Court and which of the summary procedures may properly be applied? HELD: In unlawful detainer, the defendant unlawfully withholds possession after the expiration or termination of his right thereto under any contract, express or implied. In such a case, prior physical possession is not required. Possession can also be acquired, not only by material occupation, but also by the fact that a thing is subject to the action of one's will or by the proper acts and legal formalities established for acquiring such right. Possession of land can be acquired upon the execution of the deed of sale thereof by its vendor. Actual or physical occupation is not always necessary. Although, there is still a dispute between the Bureau of Lands and the courts, unlawful detainer is still applicable and is indeed the correct action. Promulgated on March 28, 1988, the Court clearly stated that "pending final adjudication of ownership by the Bureau of Lands, the Court has jurisdiction to determine in the meantime the right of possession over the land." Corollary thereto, the power to order the sheriff to remove improvements and turn over the possession of the land to the party adjudged entitled thereto, belongs only to the courts of justice and not to the Bureau of Lands. In fact, the Bureau of Lands in its decision of June 7, 1987, admitted the jurisdiction of the courts to decide the case on the question of physical possession, although not on the question of ownership (Rollo, p. 179). Under the circumstances, a careful study of the records failed to show any cogent reason to disturb the findings of the Municipal Trial Court in Cities and of the Regional Trial Court, both of Olongapo City, and finally of the Court of Appeals. WHEREFORE, the decision of respondent Court of Appeals is AFFIRMED and the temporary restraining order is lifted. Costs against petitioners. SO ORDERED.

MLAV Vda. De Borromeo v. Pogoy 126 SCRA 217 DOCTRINE: Prescription for actions involving forcible entry and unlawful detainer is one year. The Barangay Lupon is required only where the parties thereto are "individuals" or a single human being as contrasted with a social group or institution. It applies only to cases involving natural persons, and not where any of the parties is a juridical person such as a corporation, partnership, corporation sole, testate or intestate, estate, etc. FACTS: The intestate estate of the late Vito Borromeo is the owner of a building in Cebu City. Said building has been leased and occupied by petitioner Petra Vda. de Borromeo at a monthly rental of P500.00. Private respondent Atty. Ricardo Reyes, administrator of the estate and a resident of Cebu City, served upon petitioner a letter demanding that she pay the overdue rentals, and thereafter to vacate the premises. When petitioner failed to comply, Atty. Reyes instituted an ejectment case against petitioner. Petitioner moves for the dismissal of the casce for want of jurisdiction as conciliation proceedings with the Lupon Barangay should have first been instituted since they both live in the same city. Respondent justified such omission through PD 1508, which allows the direct filing of an action in court where there was danger of prescribing under the Statute of Limitations. RTC dismissed the petition. Petitioner elevated the case to the Supreme Court via a petition for certiorari. ISSUES: 1. WON there was prescription -- NO 2. WON there was a need to go to the Lupon Barangay -- YES HELD Article 1147 of the Civil Code states that the period for filing actions for forcible entry and detainer is one year, and this period is counted from demand to vacate the premises. In the case at bar, the letter-demand was dated August 28, 1982, while the complaint for ejectment was filed in court on September 16, 1982. There is at least eleven (11) full months of the prescriptive period. Under the procedure outlined in PD 1508, the time needed for the conciliation proceeding before the Barangay Chairman and the Pangkat should take no more than 60 days. Thus, it cannot be truthfully asserted that his case would be barred by the Statute of Limitations if he had to course his action to the Barangay Lupon. Be that as it may, the instant petition should be dismissed. Referral of a dispute to the Barangay Lupon is required only where the parties thereto are "individuals" or a single human being as contrasted with a social group or institution. It applies only to cases involving natural persons, and not where any of the parties is a juridical person such as a corporation, partnership, corporation sole, testate or intestate, estate, etc.

Atty. Ricardo Reyes is a mere nominal party who is suing in behalf of the Intestate Estate of Vito Borromeo. The real party in interest is the intestate estate under administration and thus, it does not fall within the ―individual‖ description. DJTV Wilmon Auto Supply v. CA 208 SCRA 108 DOCTRINE: An ejectment suit cannot be suspended by an action filed in the RTC based on tenant‘s claim that his right of preemption (or prior purchase) was violated. FACTS: Wilmon Auto Supply Corporation, et al. (Wilmon) were lessees of a commercial building and bodegas standing on a registered land owned in common by Lucy Solinap, et al. The leases were embodied in uniformly worded deeds executed by the individual petitioners wherein one of the clauses provided for a reservation of rights—the lessor has the right to sell, mortgage, hypothecate or encumber the property so long as it requires the purchaser or mortgage creditors to respect the terms of the lease contract; provided further that lessee shall be duly informed about lessor‘s plan to sell the property. After the expiration of the period fixed in the lease agreements, the premises were sold to Star Group Resources and Development Inc. (Star Group). Thereafter, Star Group instituted an action in the Municipal Trial Court (MTC) for unlawful detainer against Wilmon. Wilmon refused to concede, and indeed impugned, Star Group's right to eject them. Wilmon then filed an action with the Regional Trial Court (RTC) and argued that in so selling the properties and seeking their ejectment therefrom, the lessors — and their buyer — had violated their leasehold rights because (i) they (the lessees) were not accorded the right of preemption, (b) the buyer was not required to honor the leases, and (c) the lessees were denied the option to renew their leases upon the expiration thereof. ISSUE: Whether or not an action of unlawful detainer filed in the MTC against a lessee — grounded on the expiration of the latter's lease — should be abated or suspended by an action filed in the RTC by the defendant lessee — on the claim that he is entitled to a right of preemption (or prior purchase) of the premises in question and wishes to have said right judicially enforced. -- NO HELD: An ejectment suit cannot be suspended by an action filed in the RTC based on tenant‘s claim that his right of preemption (or prior purchase) was violated. Actions in the RTC did not involve physical or de facto possession and, on not a few occasions, that the case in the RTC was merely a ploy to delay disposition of the ejectment proceeding, or that the issues presented in the former could quite as easily be set up as defenses in the ejectment action and there resolved. This is specially true in the cases at bar, where the petitioners-lessees' claims — that the lessors (and the buyer of the leased premises) had violated their leasehold rights because (a) they (the lessees) were not accorded the right of preemption, (b) the buyer was not required to respect their leases, and (c) the lessees were denied the option to renew their leases upon the expiration thereof — constituted their causes of action in the suits commenced by them in the Regional Court.

JGY Semira v. CA 230 SCRA 577 DOCTRINE: The issue of ownership can be decided provisionally for the sole purpose of resolving priority of possession and does not bar an action between the same parties involving title to the land. FACTS: Gutierrez owned a parcel of land which she sold to private respondent Buenaventura by means of a "Kasulatan ng Bilihan ng Lupa". Aside from the estimated area of 822.5 square meters appearing in the deed of sale, the following boundaries of the lot are also stated. Thereafter, private respondent entered the premises observing thereby the boundaries of the property and not the area given. Subsequently, he acquired two other parcels of land from the spouses Hornillas. Respondent sold the first lot to his nephew, Cipriano by means of another "Kasulatan ng Bilihan ng Lupa" where the lot was described with the same area and boundaries mentioned in the first "Kasulatan ng Bilihan ng Lupa" with the exception of the boundary on the east. Cipriano sold the lot to petitioner Semira. However, the area stated in the "Kasulatan ng Bilihan ng Lupa" 4 was 2,200 square meters and not 822.5 appearing in the previous document. As delimited by its boundaries, the lot is actually much bigger than 822.5 square meters. This was confirmed by the Taysan Cadastral Mapping Survey. A complaint for forcible entry was filed against petitioner by respondent. The latter claimed that the area of Lot 4221 was 822.5 square meters only and that the excess portion forcibly occupied by petitioner formed part of a lot which he acquired from the Hornillas. MTC dismissed the case since the issue of prior physical possession could not be resolved without first deciding on the ownership, dismissal was proper since forcible entry cases involve the sole issue of prior physical possession. ISSUE: WON the issue of ownership can be decided for the sole purpose of resolving priority of possesion. -- YES HELD: In the case at bench the issue of possession cannot be decided independently of the question of ownership. Where land is sold for a lump sum and not so much per unit of measure or number, the boundaries of the land stated in the contract determine the effects and scope of the sale, not the area thereof. Hence, the vendors are obligated to deliver all the land included within the boundaries, regardless of whether the real area should be greater or smaller than that recited in the deed. It should be emphasized, however, that the case is merely an action for forcible entry and that the issue of ownership was decided for the sole purpose of resolving priority of possession. Hence, any pronouncement made affecting ownership of the disputed portion is to be regarded merely as provisional, hence, does not bar nor prejudice an action between the same parties involving title to the land. JRPA Javier v. Veridiano

237 SCRA 565 DOCTRINE: A final judgment on forcible entry or unlawful detainer is NOT a bar against to an action for determination of ownership. A judgment rendered in a case for recovery of possession is conclusive only as to possession, not ownership. It is not a bar against an action for determination of ownership FACTS: Petitioner bought a land on a subdivision by filing a Miscellaneous Sales Application. Pending the approval of the sale, Ben Babol entered a portion adjacent to the land being bought by Petitioner. Petitioner claims that the occupied land by Babol is part of his land so he filed a forcible entry case. However, the trial court found Petitioner wrong and so sustained the possession of Babol. Later on, Babol would sell this portion to private Respondent. In the meantime, the application was approved and a TCT was delivered to Petitioner. This motivated the Petitioner to demand the land again, this time from Respondent, and this time on the basis of ownership. He filed a complaint for quieting of title and recovery of possession. Respondent countered that the first case on forcible entry constituted res judicata against the second complaint. ISSUE: W/N the first case on forcible entry was res judicata against the second case on quieting of title - NO HELD: For res judicata to arise, four requisites must concur: Final judgment, Court with competent jurisdiction, judgment based on merits, identity of parties and cause of action. The Court said that in the two cases, there were identity of parties because Respondent, having acquired the contested land by sale and tradition, is a successor in interest. A judgment in a forcible entry or detainer case disposes of no other issue than possession and declares only who has the right of possession, but by no means constitutes a bar to an action for determination of who has right or title of ownership. ABB Lim Kich Tong v. CA 195 SCRA 398 DOCTRINE: Any person deprived of possession of any land or building may file an action for forcible entry and unlawful detainer against the person unlawfully depriving or withholding possession from him. This relief is not only available to landlord, lessor but to lessee and tenant as well within one year from such unlawful deprivation or withholding of possession. FACTS: Respondent Reginaldo Lim and his family resided in room of a building owned by Petitioner Lim Kieh Tong until the former transferred to their new residence. However, the respondent retained possession of the room to keep his important belongings. At one point, when respondent wanted to go to his room, he found that his key was no longer compatible with the door‘s lock, that is, the lock was changed. He asked from petitioner the new key but having failed, he filed a writ of preliminary mandatory injunction plus damages with the MTC. Petitioner countered that

since the action is one for specific performance, the action should have been filed with the RTC. ISSUE: Whether or not the action is one for specific performance or forcible entry and detainer. HELD: The suit is actually one for forcible entry and detainer. Respondent remained in possession of the property but petitioner prevented him from enjoying his right by depriving him of the right of egress and ingress through the door of the building and the room. Any person deprived of possession of any land or building may file an action for forcible entry and detainer against the person unlawfully depriving or withholding possession from him. This relief is not only available to landlord, lessor but to lessee and tenant as well within one year from such unlawful deprivation or withholding of possession. FZC Penas v. CA 233 SCRA 744 DOCTRINE: The notice giving the lessee the alternative either to pay the increased rental or otherwise vacate the land is not the demand contemplated by the Rules of Court in unlawful detainer cases. When after such notice, the lessee elects to stay, he thereby merely assumes the new rental and cannot be ejected until he defaults in said obligation and necessary demand is first made. FACTS: Private respondent was the lessee of a property owned by petitioners based on a written lease contract (on a monthly basis). On January 18, 1990, petitioners (lessors) sent a letter terminating the lease contract with respondent and demanding that he vacate the premises. In the same letter, respondent was given an option to sign a new lease contract but with an increased rate (from P610 to P2,000). Respondent refused to vacate and refused to sign a new contract. He tried to pay the rentals at the old rate but petitioners refused to accept the same. Respondent deposited the amount with PNB in trust for petitioners but the petitioners never withdrew the said money. In August 1992 or after 2 years, petitioners again sent a letter to respondent demanding him to vacate the premises and pay back rentals for 2 years at the increased monthly rate. For failure to comply with the demands, petitioners filed this unlawful detainer case against respondent in September 1992. The MTC, RTC and the CA all ruled in favor of respondent-lessee and dismissed the case on the ground that more than one year has elapsed from the time petitioner sent his letter terminating the contract and demanding a new lease contract. ISSUE: Whether or not the unlawful detainer case was proper. -- YES HELD: Yes, the unlawful detainer case was proper. The Supreme Court ruled in favor of petitioners and ordered respondent to vacate the leased premises and pay back rentals at the increased rate. The settled rule is that when a lessee is given an option to vacate the premises or sign a new lease contract with increased rental, his choice to continuously stay means a new contract is made and he should pay the increased rental. In this case, respondent deposited an amount based on the old rental. Thus he defaulted in the payment. The demand to vacate sent in

August 1992 was proper since there was a breach of the contract on the part of the lessee. The case was filed in September 1992, and so it was properly filed within one year from the last letter of demand. LNAC Francel Realty Corporation v. CA G.R. No. 117051 DOCTRINE: The action here is not a simple action to collect on a promissory note; it is a complaint to collect amortization payments arising from or in connection with a sale of a subdivision lot under PD. Nos. 957 and 1344, and accordingly falls within the exclusive original jurisdiction of the HLURB to regulate the real estate trade and industry, and to hear and decide cases of unsound real estate business practices. FACTS: Petitioner Francel Realty Corporation filed a complaint for unlawful detainer against private respondent Francisco T. Sycip. Petitioner alleged that it had executed a Contract to Sell to private respondent Lot 16, Building No. 14 of the Francel Townhomes, at 22 Real Street, Maliksi, Bacoor, Cavite, for P451,000.00. The Contract to Sell provides that in case of default in the payment of two or more installments, the whole obligation will become due and demandable and the seller will then be entitled to rescind the contract and take possession of the property; the buyer will vacate the premises without the necessity of any court action and the downpayment will be treated as earnest money or as rental for the use of the premises. Petitioner alleged that private respondent failed to pay the monthly amortization since October 30, 1990 despite demands to update his payments and to vacate the premises, the latest of which was the demand made in the letter dated September 26, 1992, and that because of private respondent‘s unjust refusal to vacate, petitioner was constrained to engage the services of counsel. Petitioner prayed that private respondent be ordered to vacate the premises and pay a monthly rental beginning October 30, 1990 until he shall have vacated the premises, and other expenses of litigation. Private respondent moved to dismiss the complaint but his motion was denied by the MTC. He filed his answer in which he alleged that he had stopped paying the monthly amortizations because the townhouse unit sold to him by petitioner was of defective construction. He alleged that he had in fact filed a complaint for ―unsound real estate business practice‖ in the Housing and Land Use Regulatory Board against petitioner. Private respondent prayed that petitioner be ordered to pay damages. The MTC ruled that the answer was filed out of time on the ground that it was filed more than ten days after the service of summons. However, it dismissed the complaint for lack of jurisdiction. The MTC held that the case was cognizable by the HLURB. But it also ordered petitioner to pay private respondent damages, attorney‘s fees, and costs. On appeal the RTC affirmed the decision of the MTC. Petitioner filed a petition for review in the CA, however it was dismissed. Thus, this petition for review on certiorari. ISSUE:

Whether MTC correctly held itself to be without jurisdiction over petitioner‘s complaint. -- YES HELD: This is, therefore, not a simple case for unlawful detainer arising from the failure of the lessee to pay the rents, comply with the conditions of a lease agreement or vacate the premises after the expiration of the lease. Since the determinative question is exclusively cognizable by the HLURB, the question of the right of petitioner must be determined by the agency. Petitioner‘s cause of action against private respondent should instead be filed as a counterclaim in HLURB Case in accordance with Rule 6, §6 of the Rules of Court which is of suppletory application to the 1987 HLURB Rules of Procedure per §3 of the same. In the case of Estate Developers and Investors Corporation v. Antonio Sarte and Erlinda Sarte, the developer filed a complaint to collect the balance of the price of a lot bought on installment basis, but its complaint was dismissed by the RTC for lack of jurisdiction. It appealed the order to this Court. In dismissing the appeal, we held: The action here is not a simple action to collect on a promissory note; it is a complaint to collect amortization payments arising from or in connection with a sale of a subdivision lot under PD. Nos. 957 and 1344, and accordingly falls within the exclusive original jurisdiction of the HLURB to regulate the real estate trade and industry, and to hear and decide cases of unsound real estate business practices. Although the case involving Antonio Sarte is still pending resolution before the HLURB Arbiter, and there is as yet no order from the HLURB authorizing suspension of payments on account of the failure of plaintiff developer to make good its warranties, there is no question to Our mind that the matter of collecting amortizations for the sale of the subdivision lot is necessarily tied up to the complaint against the plaintiff and it affects the rights and correlative duties of the buyer of a subdivision lot as regulated by NHA pursuant to P.D. 957 as amended. It must accordingly fall within the exclusive original jurisdiction of the said Board, and We find that the motion to dismiss was properly granted on the ground that the regular court has no jurisdiction to take cognizance of the complaint. We hold that the MTC correctly held itself to be without jurisdiction over petitioner‘s complaint. But it was error for the MTC to grant private respondent‘s counterclaim for damages for expenses incurred and inconveniences allegedly suffered by him as a result of the filing of the ejectment case. Pursuant to Rule 6, §8 a party may file a counterclaim only if the court has jurisdiction to entertain the claim. Otherwise the counterclaim cannot be filed. Even assuming that the MTC had jurisdiction, however the award of damages to private respondent must be disallowed for the following reasons: (1) The MTC decision itself stated that the answer with its counterclaim was filed out of time or more than 10 days from private respondent‘s receipt of summons. In effect, therefore, private respondent did not make any counterclaim. (2) A reading of the MTC decision showed no justification for the award of moral and exemplary damages and attorney‘s fees. TKDC Azarcon v. Eusebio 105 Phil. 569 DOCTRINE: A person who is in possession of a parcel of land and is being ordered to leave said land while products are pending harvest has the right to a part of the net harvest.

FACTS: This involves a dispute over the possession of a parcel of public land; Eusebio filed a lease application for a parcel of said laid but Azarcon occupied a portion of that land under a homestead application. The conflict between the lessee and the homesteaders was investigated by the Director of Lands and again by the Secretary of Agriculture and natural Resources. Before the dispute could be settled, Eusebio filed a complaint with the Court of First Instance alleging that he acquired that parcel of land through a lease with the Bureau of Lands and while he is in possession of said land, Azarcon occupied a portion of it. Azarcon answered that he is in actual possession of a portion of the land under a homestead application and that the lease application of Eusebio was subsequent to his homestead application; he occupied the land since 1941 with interruption during the war and again in 1950 up to the time of the filing of the action. The trial court ruled in favor of Eusebio; while the case was appealed, a writ of execution was issued for the restitution of the land to Eusebio. Despite receipt of the notice of the writ of execution, Aznar entered the land to gather palay which was then pending harvest. ISSUE: Whether or not Aznar acted in bad faith when he entered the land to gather palay despite receipt of the notice of the writ of execution. -- NO HELD: While the court ordered Aznar to move out from the premises, it did not prohibit them from gathering the crop existing on the land. Under Article 545 of the Civil Code, a person who is in possession and who is being ordered to leave a parcel of land while products thereon are pending harvest has the right to a part of the net harvest. In this case, the order of execution did not expressly prohibit Aznar from gathering the pending fruits, which were the result of their possession and cultivation of the land, so it cannot be said that Aznar violated the courts' order. Their act in harvesting the pending fruits was not only justified by law but was not expressly prohibited by the court's order, and was even ratified when the court ordered the suspension of the execution. AMD Calagan v. CFI of Davao 95 SCRA 498 DOCTRINES: 1. Purpose of the law on homesteads: to conserve ownership in the hands of the homesteader and his family 2. Under Article 547, the homesteader desiring to repurchase should be given the option to require the vendee a retro to remove the useful improvements on the land subject of the sale a retro, such option is not granted the vendor a retro under Article 1616. FACTS: In 1954, Mangulon Calagan and his wife, Takura, were granted a Homesstead Patent and issued an Original Certificate of Title over a lot located in Digos, Davao del Sur. In 1955, Takura died and was survived by her husband and their children.

In 1961, Calagan sold a portion of their homestead to private respondent, Petra Sandoval. Sandoval borrowed Calagan‘s title to the land so she could have the sale annotated. In 1963, Calagan offered to repurchase the land but Sandoval refused such offer. Sandoval was only willing to comply if she was reimbursed the value of the house that she had constructed on the land. Petitioners filed an action for reconveyance against Sandoval. The CFI ruled in favor of petitioners. Sandoval was ordered to reconvey the land, given that petitioners pay her the value of the house she has constructed in good faith. Petitioners appealed, maintaining that the Court erred in ordering them to reimburse the value of the house. ISSUE: W/N petitioners should be ordered to reimburse the value of the house built in good faith on the land they seek to repurchase. -- NO HELD: It being obvious that petitioners are not exercising the option to refund the amount of the expenses incurred in building the house, private respondent, as the vendee a retro, may remove her house since this can be done without damage to the principal thing, as stipulated in Article 547 of the Civil Code. Petitioners should not, as opined by the trial Court, be made to refund the value of that house to private respondent, else, the salutary policy behind the Public Land Law would be thwarted and rendered meaningless. There is no provision in the Public Land Act (CA No. 141) which provides for the terms and conditions under which repurchase may be effected by a homesteader except that it should be made within 5 years from the date of conveyance. That Act is silent as to the nature of expenses that should be reimbursed by a repurchasing homesteader or his heir. However, under Art. 1616 of the New Civil Code, the vendor a retro cannot avail himself of the right of repurchase without returning to the vendee the price of the sale, the expenses of the contract and other legitimate payments, and the necessary and useful expenses made on the thing sold. The provision of Article 1616 of the Civil Code on redemption, is, in general, the applicable law to a homesteader desirous to redeem his property. However, considering the purpose of the law on homesteads, which is to conserve ownership in the hands of the homesteader and his family, Article 1616 of the Civil Code should be construed in conjunction with Articles 546 and 547 of the Civil Code prescribing the rules on refund of necessary and useful expenses, inasmuch as a vendee a retro is, as a rule, considered in good faith. Applying Article 547, the homesteader desiring to repurchase should be given the option to require the vendee a retro to remove the useful improvements on the land subject of the sale a retro, which option is not granted the vendor a retro under Article 1616. Under the latter Article, the vendor a retro must pay for the useful improvements introduced by the vendee a retro, otherwise, the latter may retain possession of the property until reimbursement is made. To allow a vendee a retro of a homestead, however, the right of retention until payment of useful expenses is made by the redemptioner would be to render nugatory the right of repurchase granted by law to a homesteader because all a vendee a retro can do to prevent repurchase is to build something on the homestead beyond the capacity to pay of the homesteader who seeks to repurchase CRF Cruz v. Pahati

98 Phil. 788 DOCTRINE: The common law principle that the one who has made the happening of fraud possible through misplaced confidence must suffer the consequence cannot be applied since there is an express provision covering the case. Article 559, a statutory provision, prevails over a common law principle. (batasnatin) FACTS: Pahati admitted having bought the automobile from Bulahan but when it was impounded, he returned the automobile to Bulahan who in turned surrendered the check for cancellation. Bulahan on his part claims that he acquired the automobile from Jesusito Belizo for value and without having any knowledge of any defect in the title of the latter. The lower court found that the automobile in question was originally owned by the Northern Motors, Inc. which later sold it to Chinaman Lu Dag. This Chinaman sold it afterwards to Jesusito Belizo and the latter in turn sold it to plaintiff. Belizo was then a dealer in second hand cars. One year thereafter, Belizo offered the plaintiff to sell the automobile for him claiming to have a buyer for it. Plaintiff agreed. At that time, plaintiff's certificate of registration was missing and, upon the suggestion of Belizo, plaintiff wrote a letter addressed to the Motor Section of the Bureau of Public Works for the issuance of a new registration certificate alleging as reason the loss of the one previously issued to him and stating that he was intending to sell his car. This letter was delivered to Belizo on March 3, 1952. He also turned over Belizo the automobile on the latter's pretext that he was going to show it to a prospective buyer. Subsequent facts show that the letter was falsified by Belizo to enable him to sell the car to Bulahan for a valuable consideration. ISSUE: Who has, therefore, a better right of the two over the car? HELD: The plaintiff has a better right to the car in question than defendant Bulahan for it cannot be disputed that plaintiff had been illegally deprived thereof because of the ingenious scheme utilized by Belizo to enable him to dispose of it as if he were the owner thereof. Plaintiff therefore can still recover the possession of the car even if defendant Bulahan had acted in good faith in purchasing it from Belizo. ART. 559. The possession of movable property acquired in good faith is equivalent to a title. Nevertheless, one who has lost any movable or has been unlawfully deprived thereof, may recover it from the person in possession of the same. If the possessor of a movable lost or of which the owner has been unlawfully deprived, has acquired it in good faith at a public sale, the owner cannot obtain its return without reimbursing the price paid therefor. Article 1505 of the same Code provides that "where goods are sold by a person who is not the owner thereof, and who does not sell them under authority or with the consent of the owner, the buyer acquires no better title to the goods than the seller had, unless the owner of the goods is by his conduct precluded from denying the seller's authority to sell. MPF Aznar v. Yapdiangco

13 SCRA 486 DOCTRINE: Ownership is not transferred by contract merely but by tradition or delivery. Contracts only constitute titles or rights to the transfer or acquisition of ownership, while delivery or tradition is the mode of accomplishing the same. Art. 559. The possession of movable property acquired in good faith is equivalent to a title. Nevertheless, one who has lost any movable or has been unlawfully deprived thereof may recover it from the person in possession of the same. If the possessor of a movable lost or which the owner has been unlawfully deprived, has acquired it in good faith at a public sale, the owner cannot obtain its return without reimbursing the price paid therefor. FACTS: ● Teodoro Santos advertised the sale of his FORD FAIRLANE 500 in a newspaper. L. De Dios went to the house of Teodoro and talked to his son Ireneo Santos and said that his uncle Vicente Marella is interested in buying the said car. ● The next day, Ireneo went to the house of Marella and they agreed to the price of P14,700 on the understanding that it will be paid after the car has been registered in the latter‘s name. ○ 
 A deed of sale was executed and the registration was changed to the name of Marella. Ireneo went to Marella to get the payment and deliver the car who informed him that he is P2,000 short of the money and that they need to go to his sister to get it. Ireneo, together with De Dios and an unidentified man went to a house. ● Once inside, De Dios asked Ireneo to wait.. After waiting in vain, he went down and discovered that the car was gone. ● Marella was able to sell the car to plaintiff-appellant Jose Aznar and while attending to registration; Philippine Constabulary seized the car due to the report of the incident. ISSUE: Who has the better right on the property involved (car)?? HELD: Teodoro Santos has the better right. Marella did not have any title to the property under litigation because the same was never delivered to him. He may have the contract but he never acquired valid title. Although the keys to the car may have been given to the unidentified companion, it may be done only because that companion took them to the place where the sister of Marella was supposed to live. The car was evidently stolen and that the buyer did not acquire any valid title thereto. Marella never had title to the car as the car wasn't ever delivered to him. While there was a deed of sale in his favor, he was only able to obtain possession of the car since he stole it from Santos. The applicable law is Article 559. The rule is to the effect that if the owner has lost a thing, or if he has been unlawfully deprived of it, he has a right to recover it, not only from its finder, thief or robber, but also from third persons who may have acquired it in good faith from such finder, thief or robber. The said article establishes 2 exceptions to the general rule of irrevindicabilty—to wit, the owner has lost the thing or has been unlawfully deprived thereof. In these cases, the possessor cannot retain the thing as against the owner who may recover it without paying any indemnity, except when the possessor acquired it in a public sale.

Furthermore, the common law principle that where one of two innocent persons must suffer a fraud perpetrated by another, the law imposes the loss upon the party who, by his misplaced confidence, has enable the fraud to be committed, cannot be applied in this case, which is covered by an express provision of law. AMDG De Garcia v. CA 37 SCRA 160 DOCTRINE: Article 559 of the Civil Code: The possession of movable property acquired in good faith is equivalent to a title. Nevertheless, one who has lost any movable or has been unlawfully deprived thereof may recover it from the person in possession of the same. If the possessor of a movable loss of which has been unlawfully deprived, has acquired it in good faith at a public sale, the owner cannot obtain its return without reimbursing the price paid thereof. As stated in Cruz v Pahati, the rightful owner may not be prevented from recovering his property by reason of good faith alone. The only exception that is acknowledged by the law is when the property was acquired in good faith at a public sale. Still, in such case, recovery may only take place after reimbursement. FACTS: Guevara purchased a diamond ring from R. Rebullida, Inc. The ring was eventually stolen from her. More than a year after it was stolen, while she was conversing with De Garcia, she noticed the latter wearing a ring which she recognized to be her stolen ring. Guevara asked De Garcia where she bought it and the latter answered that she purchased it from Mrs. Miranda who bought it from Ms. Hinahon who bought it from a certain Aling Petring. After Guevara explained to De Garcia that the ring is hers, she borrowed it and proceeded to Mr. Rebudilla. After examining the jewelry, Mr. Rebudilla confirmed that it was indeed the ring the Guevara bought from him. Guevara returned the ring and later on wrote and requested De Garcia to deliver the ring to her. Despite the written request as well as the writ of seizure delivered by the sheriff and served against her, De Garcia did not deliver the ring. The lower court ruled in favor of De Garcia. However, the Court of Appeals reversed the lower court‘s decision and ordered that the ring be returned to Guevara or pay Guevara equivalent price of the jewelry. One of the reasons cited by the CA was the dubious source narrated by De Garcia as regards how she acquired the ring. ISSUE: Whether or not the Court of Appeals erred in ruling that the ring must belongs to Guevara and must be returned to her – NO HELD: Reviewing the facts and reasoning of the CA, the SC did not see any error on the part of the CA in ruling in favor of Guevara. Applying Article 559 of the Civil Code, it is evident that Guevara was unlawfully deprived of her diamond ring. It is only proper that the ring be returned by De Garcia who has possession of it. Furthermore, as stated in Cruz v. Pahati, the rightful owner may not be prevented from recovering his property by reason of good faith alone. The only exception that is acknowledged by the law is when the property was acquired in good faith at a public sale. Still, in such case, recovery may only take place after reimbursement. Applying this to the case, De Garcia herself admitted that she did not acquire the ring from a public sale but

rather from Mrs. Miranda. In addition, Anzar v. Yapdiangco, reiterated that the reason behind the recovery of the rightful owner from a person who acquired the property in good faith was because no consent was given by the former from the dispossession of his property. The Court also discussed the existing common law principle which is: where one of two innocent persons must suffer by a fraud perpetrated by another, the law imposes the loss upon the part who, by his misplaced confidence, has enabled the fraud to be committed. However, such common law cannot be applied if there exists an express provision of the law such as Article 559 of the Civil Code. It follows from the above discussion that De Garcia‘s contention as regards possession in good faith is equivalent from a just title cannot be accepted by this Court. Looking further at the true meaning of Article 559, the title mentioned refers to a presumptive title only and not just title. Consequently, when the true owner has been unlawfully deprived of his property it is only just that it be returned to him. GCG Dizon v. Suntay 47 SCRA 160 DOCTRINE: The possession of movable property acquired in good faith is equivalent to a title. Nevertheless, one who has lost any movable or has been unlawfully deprived thereof may recover it from the person in possession of the same. If the possessor of a movable lost of which the owner has been unlawfully deprived, has/acquired it in good faith at a public sale, the owner cannot obtain its return without reimbursing the price paid therefore. The only exception the law allows is when there is acquisition in good faith of the possessor at a public sale, in which case the owner cannot obtain its return without reimbursing the price. FACTS: Plaintiff is the owner of a three-carat diamond ring valued at P5,500 and entered into a transaction with Clarita R. Sison. The plaintiff‘s ring was delivered to Clarita R. Sison for sale on commission. Upon receiving the ring, Clarita R. Sison executed and delivered to the plaintiff the receipt. After the lapse of a considerable time without Clarita R. Sison having returned to the plaintiff the latter‘s ring, the plaintiff made demands on Clarita R. Sison for the return of her ring but the latter could not comply with the demands. Three days after the ring above-mentioned was received by Clarita R. Sison, the ring was pledged by Melia Sison, niece of the husand of Clarita R. Sison, evidently in connivance with the latter, with the defendant‘s pawnshop for P2,600. This was done without the knowledge of the plaintiff. The plaintiff insistently demanded from Clarita R. Sison the return of her ring, the latter finally delivered to the former the pawnshop ticket, which is the receipt of the pledge with the defendant‘s pawnshop of the plaintiff‘s ring. When the plaintiff found out that Clarita R. Sison pledged, she took steps to file a case of Estafa against the latter with the fiscal‘s office. Subsequently thereafter, the plaintiff, wrote a letter to the defendant asking for the delivery to the plaintiff of her ring pledged with defendant‘s pawnshop. Since the defendant refused to return the ring, the plaintiff filed the present action with the CFI for the recovery of said ring. The plaintiff asked for the provisional remedy writ of replevin by the delivery of the ring to her, upon her filing the requisite bond, pending the final determination of

the action. The lower court issued the writ of replevin prayed for by plaintiff and the latter was able to take possession of the ring during the pendency of the action upon her filing the requisite bond. The lower court rendered judgment declaring that Suntay had the right to the possession of the ring in question. Petitioner Dizon, as defendant, sought to have the judgment reversed by the CA, which ruled in favor of Suntay. ISSUE: Whether or not Suntay had the right to the possession of the ring. -- YES HELD: Yes. The Supreme Court held that Suntay had the right to the possession of the ring. The controlling provision is Article 559 of the Civil Code. It reads thus: The possession of movable property acquired in good faith is equivalent to a title. Nevertheless, one who has lost any movable or has been unlawfully deprived thereof may recover it from the person in possession of the same. If the possessor of a movable lost of which the owner has been unlawfully deprived, has/acquired it in good faith at a public sale, the owner cannot obtain its return without reimbursing the price paid therefore. The only exception the law allows is when there is acquisition in good faith of the possessor at a public sale, in which case the owner cannot obtain its return without reimbursing the price. VCL IV EDCA Publishing and Distributing Corp. v. Santos 184 SCRA 614 DOCTRINE: Possession of movable property acquired in good faith is equivalent to title. There is no need to produce a receipt. FACTS: EDCA Publishing sold 406 books to a certain Professor Jose Cruz who ordered these by telephone, which was agreed to be payable on delivery. The books were subsequently delivered to him with the corresponding invoice, and he paid with a personal check. Cruz then sold the 120 of the books to Leonor Santos who asked for verification, and was then showed the invoice for the books. EDCA having become suspicious over a second order placed by Cruz even before clearing of his first check, made inquiries with the De la Salle College where he had claimed to be a dean and was informed that there was no such person in its employ. Further verification revealed that Cruz had no more account or deposit with the Philippine Amanah Bank, against which he had drawn the payment check. EDCA then went to the police, which set a trap and arrested Cruz on October 7, 1981. Investigation disclosed his real name as Tomas de la Peña and his sale of 120 of the books he had ordered from EDCA to Santos. On the night of the same date, EDCA sought the assistance of the police in Precinct 5 at the UN

Avenue, which forced their way into the store of the private respondents and threatened Leonor Santos with prosecution for buying stolen property. They seized the 120 books without warrant, loading them in a van belonging to EDCA, and thereafter turned them over to the petitioner. Protesting this high-handed action, the Santos sued for recovery of the books after demand for their return was rejected by EDCA. A writ of preliminary attachment was issued and the petitioner, after initial refusal, finally surrendered the books to the private respondents. MTC ruled that the ownership of the books of Santos. RTC and CA sustained the MTC‘s ruling. ISSUE: Whether or not the owner was unlawfully deprived of the property -- NO HELD: Santos was a good faith buyer after taking steps to verify the identity of the seller. When she was showed the invoice, she reasonably believed that he was a legitimate seller. With regard to unlawful deprivation, EDCA was not unlawfully deprived of the property by mere failure of consideration. There was already a perfected contract of sale. Proof was even substantiated when EDCA gave the invoice as proof of payment upon delivery of the books. This did not amount to unlawful taking, because by the delivery of EDCA to Cruz, ownership of the books already transferred to him. FXRL Ledesma v. CA 213 SCRA 195 DOCTRINE: Article 559 cannot be applied when the property was lawfully divested from the original owner. FACTS: ● September 1977, a person representing himself to be Jojo Consuji purchased two vehicles from Citywide Motors, Inc. ● ―Jojo‖ used a manager‘s check to pay for the vehicles ● The said vehicles were delivered via a contract of sale. ● The check was dishonored. ● It was discovered that ―Jojo Consuji‖ was in fact an Armando Suarez, a known and infamous conman. ● One of the vehicles was recovered, abandoned in Quezon City. The other vehicle was found to be sold to the petitioner. ● Ledesma claims that he had acquired the vehicle in good faith and is thus entitled to the same. ● The company filed for replevin and reacquired the vehicle from Ledesma after payment of a bond. ● The RTC ruled in favor of Ledesma and awarded him with possession of the vehicle plus damages. ● The CA ruled in favor of the company. ● The company relies on the strength of Article 559 of the Civil Code which provides that if the owner has lost a thing, or if he has been unlawfully deprived of it, he has a right to recover it not only from the finder, thief or robber, but also from third persons who may have acquired it in good faith from such finder, thief or robber. The said article establishes two (2) exceptions to the general rule of irrevendicability (sic), to wit: when the owner (1) has lost the thing, or (2) has been unlawfully deprived thereof. In these cases, the possessor cannot

retain the thing as against the owner who may recover it without paying any indemnity, except when the possessor acquired it in a public sale.‘ ISSUE: W/N the private respondent (the company) was unlawfully deprived of the vehicle in the first place so as to make Article 559 apply. -- NO HELD: The company was not unlawfully deprived of the vehicle in question. Therefore, the petitioner is a buyer in good faith and Article 559 may not apply. There was a perfected unconditional contract of sale between private respondent and the original vendee. The former voluntarily caused the transfer of the certificate of registration of the vehicle in the name of the first vendee — even if the said vendee used a fictitious name — and likewise voluntarily delivered the cars and the certificate of registration to the vendee Title thereto was forthwith transferred to the vendee. The subsequent dishonor of the check merely amounted to a failure of consideration which does not render the contract of sale void, but merely allows the prejudiced party to sue for specific performance or rescission of the contract, and to prosecute the impostor for estafa under the Revised Penal Code. Under the law on obligations and contracts, the parties to a contract of sale may stipulate that ownership in the thing sold shall not pass to the buyer until full payment of the purchase price only if there is a stipulation to that effect. Otherwise, the rule is that such ownership shall pass from the vendor to the vendee upon the actual or constructive delivery of the thing sold even if the purchase price has not yet been paid. Non-payment only creates a right to demand payment or to rescind the contract, or to criminal prosecution in the case of bouncing checks. But absent the stipulation above noted, delivery of the thing sold will effectively transfer ownership to the buyer who can in turn transfer it to another. RSDM Chua Kai v. Kapunan 104 Phil. 110 DOCTRINE: 1. That the acquirer and possessor in good faith, of a chattel or movable property is entitled to be respected and protected in his possession, as if he were the true owner thereof, until a competent court rules otherwise; 2. That being considered, in the meantime, as the true owner, the possessor in good faith cannot be compelled to I surrender possession nor to be required to institute an action for the recovery of the chattel, whether or not an indemnity bond is issued in his favo FACTS: Roberto Soto purchased from Youngstown Hardware, owned by Ong Shu, 700 corrugated galvanized iron sheets and 249 pieces of round iron bar for P6,137.70, and in payment thereof he issued a check drawn against the Security Bank and Trust Company for P7,000.00, without informing Ong Shu that he had no sufficient funds in said bank to answer for the same. When the check was presented for payment, it was dishonored for insufficiency of funds. Soto sold 165 sheets in Pangasinan and 535 sheets in Calapan, Mindoro. Of those sold in Pangasinan, 100 were sold to petitioner Chua Hai. When the case was filed in the Court of First Instance of

Manila against Roberto Soto, for estafa, the offended party filed a petition asking that the 700 galvanized iron sheets, which were deposited with the Manila Police Department, be returned to him, as owner of the Youngstown Hardware. Petitioner herein opposed the motion with respect to the 100 sheets that he had bought from Soto. Notwithstanding the opposition, the court ordered the return of the galvanized iron sheets to Ong Shu. Petitioner then presented a motion to reconsider the order, alleging that by the return thereof to the offended party, the court had not only violated the contract of deposit, because it was in that concept that petitioner had delivered the 100 sheets to the Manila Police Department, and that said return to Ong Shu amounted to a deprivation of his property without due process of law. It is also claimed that Article 105 of the Revised Penal Code, under whose authority the return was ordered, can be invoked only after the termination of the criminal case and not while said criminal case is still pending trial. ISSUE: Whether or not petitioner Chua has better right of the possession HELD: To deprive the possessor in good faith, even temporarily and provisionally, of the chattels possessed, violates the rule of Art. 559 of the Civil Code. The latter declares that possession of chattels in good faith is equivalent to title; i.e., that for all intents and purposes, the possessor is the owner, until ordered by the proper court to restore the thing to the one who was illegally deprived thereof. Until such decree is rendered (and it can not be rendered in a criminal proceeding in which the possessor is not a party), the possessor, as presumptive owner, is entitled to hold and enjoy the thing; and "every possessor has a right to be respected in his possession; and should he be disturbed therein he shall be protected in or restored to said possession established by the means established by the laws and the Rules of Court."(Art. 539, New Civil Code). The decision of the court below, instead of conforming to Arts. 559 and 539 of the Civil Code, directs possessor to surrender the chattel to the claimant Ong Shu before the latter has proved that he was illegally deprived thereof, without taking into account that the mere filing of a criminal action for estafa is no proof that estafa was in fact committed. Instead of regarding the possessor as the owner of the chattel until illegal deprivation is shown, the court below regards the possessor of the chattel not as an owner, but as a usurper, and compels him to surrender possession even before the illegal deprivation is proved. We see no warrant for such a reversal of legal rules.

USUFRUCT (ART. 562-612) MRAM NHA v. CA G.R. No. 148830 DOCTRINE: A usufruct gives a right to enjoy the property of another with the obligation of preserving its form and substance, unless the title constituting it or the law otherwise provides. FACTS: By virtue of Proclamation No. 481 issued by then Pres. Marcos, a portion of land in Quezon City owned by NHA was reserved for the site of National Government Center (NGC). Subsequently, President Marcos issued Proclamation No. 1670, which removed a seven-hectare portion from the coverage of the NGC. Proclamation No. 1670 gave Manila Seedling Bank Foundation, Inc. (MSBF) usufructuary rights over this segregated portion. MSBF occupied the area granted by Proclamation No. 1670. However, over the years, MSBF‘s occupancy exceeded the seven-hectare area subject to its usufructuary rights; it occupied approximately 16 hectares by 1987. By then the land occupied by MSBF was bounded by Epifanio de los Santos Avenue (EDSA) to the west, Agham Road to the east, Quezon Avenue to the south and a creek to the north. On 18 August 1987, MSBF leased a portion of the area it occupied to Bulacan Garden Corporation (BGC) and other stallholders. BGC leased the portion facing EDSA, which occupies 4,590 sq. m. of the 16-hectare area. On 11 November 1987, President Corazon Aquino issued Memorandum Order No. 127 (MO 127). This revoked the reserved status of ―the 50 hectares, more or less, remaining out of the 120 hectares of the NHA property reserved as site of the NGC.‖ MO 127 also authorized the NHA to commercialize the area and to sell it to the public. Acting on the power granted under MO 127, the NHA gave BGC 10 days to vacate its occupied area. Any structure left behind after the expiration of the 10-day period will be demolished by NHA. ISSUE: Whether or not the premises leased by BGC from MSBF is within the 7-hectare area that Proclamation No. 1670 granted to MSBF by way of usufruct? HELD: The Court held that MSBF abused their usufruct rights. Clearly, in the present case, Proclamation No. 1670 is the title constituting the usufruct. Proclamation No. 1670 categorically states that the 7-hectare area shall be determined ―by future survey under the administration of the Foundation subject to private rights if there be any.‖ MSBF, then, has the latitude to determine the location of its 7-hectare usufruct portion within the 16-hectare area. Although MSBF has the discretion to determine its 7-hectare usufruct, MSBF abused its right when it exceeded the 7-hectare portion granted to it by Proclamation No. 1670. The Court said that ―a usufruct is not simply about rights and privileges. A usufructuary has the duty to protect

the owner‘s interests. One such duty is found in Article 601 of the Civil Code which states: ART. 601. The usufructuary shall be obliged to notify the owner of any act of a third person, of which he may have knowledge, that may be prejudicial to the rights of ownership, and he shall be liable should he not do so, for damages, as if they had been caused through his own fault. A usufruct gives a right to enjoy the property of another with the obligation of preserving its form and substance, unless the title constituting it or the law otherwise provides.‖ The Court further said that ―at this point, the determination of the seven-hectare portion cannot be made to rely on a choice between the NHA‘s and MSBF‘s survey. There is a need for a new survey, one conducted jointly by the NHA and MSBF, to remove all doubts on the exact location of the seven-hectare area and thus avoid future controversies. This new survey should consider existing structures of MSBF. It should as much as possible include all of the facilities of MSBF within the seven-hectare portion without sacrificing contiguity.‖ It must be noted however that based on Art. 605, MSBF has only 22 years to exercise its usufruct since the Civil Code provides that the right can be exercised only within 50 year FMM Gaboya v. Cui 38 SCRA 85 DOCTRINE: The reserved usufructuary right of a seller on a property doesn‘t include rentals from the buildings subsequently constructed on the sale, but it entitles the usufructuary to a reasonable rental for the portion of the land being occupied by the building. FACTS: Don Mariano Cui was a widower who owned three lots (Nos. 2312, 2313 and 2319) situated in Cebu City with a total area of 2,658 square meters. He sold the three commercial lots to three of his children (Rosario C. de Encarnacion, Mercedes C. de Ramas and Antonia Ma. Cui) pro indiviso. Rosario C. de Encarnacion was unable to pay her corresponding share of the purchase price due to lack of funds. Thus, the sale to her was cancelled and the one-third of the property corresponding to her was returned to Don Mariano Cui. Because of the sale of the lots was pro indiviso and due to the cancellation of one of the three lots, Don Mariano and is children, Mercedes and Antonio became co-owners of the whole property. In the Deed of Sale, Don Mariano retained for himself the usufruct of the property, stating that he ―shall enjoy the fruits and rents of the property as long as his natural life shall last.‖ Subsequently, a building was erected on a portion of the property and was occupied by a Chinese businessman which paid a rent of P600 a month. Sometime after the sale to his two daughters, the latter applied to the Rehabilitation Finance Corporation (RFC) for a loan of P130,000 with which to construct a building presumably on a portion of the property sold to the two daughters. To facilitate the granting of the loan, Don Mariano executed an authority to mortgage his share of the property in favor of his two daughters.

The loan was eventually granted and secured by a mortgage on the property. However, Don Mariano, despite being one of the mortgagors, did not join in the construction of the building. The building was eventually constructed and the two daughters received rents therefrom and paid their loan. On 19 March 1949, Rosario C. Encarnacion, one of the daughters of Don Mariano filed a petition to declared their father incompetent and to have a guardian appointed for his property. The petition was granted and Victorino Reynes was appointed guardian of his property. On 15 June 1949, guardian Reynes filed a motion in the guardianship proceeding seeking authority to collect the rentals from the property. The motion, however, was denied by the judge. Subsequently, another complaint was filed by Don Mariano‘s guardian which alleged that the usufructuary right reserved in favor of Don Mariano extends to and includes the rentals of the building constructed by his two daughters on the land sold to them by their father; that the defendants retained those rentals for themselves; that the usufructuary rights of the vendor were of the essence of the sale, and their violation entitled him to rescind (or resolve) the sale. It prayed either for rescission with accounting, or for delivery of the rentals of the building with interests, attorneys' fees and costs. The Court of First Instance of Cebu denied the resolution of the foregoing complaint hence this petition by the Judicial Administrator of the Estate of Mariano Cui, Jesus M. Gaboya. ISSUE: Whether the usufruct reserved by Don Mariano in the deed of sale, over the property that were at the time vacant and unoccupied, gave the usufructuary the right to receive the rentals of the commercial building constructed by the two daughters with funds borrowed from the Rehabilitation and Finance Corporation, the loan being secured by a mortgage over the lots sold. -- NO HELD: The Supreme Court (the ―SC‖) held that the usufructuary rights of the late Don Mariano reserved in the deed of sale was over the land alone and did not entitle him to the rents of the building later constructed thereon by his two daughters. The SC also held that said usufructuary was entitled only to the reasonable rental value of the land occupied by the building aforementioned. RGGM Vda. De Albar v. Carangdang 106 Phil. 855 DOCTRINE: Under the law, usufruct is extinguished only by the total loss of the thing subject of the encumbrance. Any war damage payment received by the naked owner should also be subject to usufruct for life if such payment has not been used in the construction of a new building. A usufructuary who is the only recipient of all the benefits of the property subject of the usufruct, and who has bound himself to pay the real estate taxes on the property in a formal agreement approved by the court, should pay such taxes. FACTS:

Doña Rosario Fabie y Grey was the owner of the lot in the City of Manila with a building and improvements, and by a will left by her upon her death which was duly probated she devised the naked ownership of the whole property to Rosario Grey Vda. de Albar, et al. but its usufruct to Josefa Fabie for life. During liberation, as a consequence of the fire that gutted the building in many portions of Manila, the building on the Ongpin lot was burned, leaving only the walls and other improvements that were not destroyed by the fire. One Au Pit, a Chinaman, offered to lease the property for a period of five years, at the same time agreeing to construct on the lot a new building provided the naked owners as well as the usufructuary sign the agreement of the lease. As the usufructuary maintains that she has the exclusive right to cede the property by lease and to receive the full rental value by virtue of her right to usufruct while on the other hand the naked owners maintain that the right of usufruct was extinguished when the building was destroyed, the right of the usufructuary being limited to the legal interest on the value of the lot and the materials, in order that the agreement of lease may be affected, the parties agreed on a temporary compromise whereby the naked owners would receive P100.00, or 20% of the monthly rental of P500.00 and the usufructuary the balance of 80% or P400.00 of said monthly rental. It was likewise stipulated in the agreement that the title to the building to be constructed would accrue to the land upon it completion as an integral part of the lot covered by the transfer certificate of title issued in the name of the naked owners but subject to the right of usufruct of Josefa Fabie. The parties expressly reserved the right to litigate their respective claims after the termination of the contract of lease to determine which of said claims was legally correct. By reason of the destruction of the building on the Ongpin property, the United States War Damage Commission approved the claim that was presented for the damage caused to the property, paid to and received by the naked owners. In the meantime, the usufructuary paid the real estate taxes due on the property at Ongpin for the years 1945 to 1952. ISSUE: W/N the usufruct included the building and the land? W/N the usufructuary (FABIE) or naked owner (VDA DE ALBAR) should undertake the reconstruction? W/N the usufructuary should pay the real estate taxes? HELD: The usufruct for life extended to the land and the building. From the above, it is clear that when the deceased constituted the life usufruct on the rentals "fincas situadas" in Ongpin and Sto. Cristo streets, she meant to impose the encumbrance both the building and the land on which it is erected for indeed the building cannot exist without the land. And as this Court well said, "The land, being an indispensable part of the rented premises cannot be considered as having no rental value whatsoever." Moreover, in the Spanish language, the term "fincas" has a broad scope; it includes not only building but land as well. (Diccionario Ingles-Español, por Martines Amador) Since only the building was destroyed and the usufruct is constituted not only on the building but on the land as well, then the usufruct is not deemed extinguished by the destruction of the building for under the law usufruct is extinguished only by the total loss of the thing subject of the encumbrance (Article 603, old Civil Code). FABIE, the usufructuary has the discretion to reconstruct the building. Of course, this is addressed to the wisdom and discretion of the usufructuary who, to all intents and purposes is deemed as the administrator of the property. This has been clarified in the case of Fabie vs.

Gutierrez David, 75 Phil., 536, which was litigated between the same parties and wherein the scope of the same provision of the will has been the subject of interpretation. The usufructuary should pay the taxes. We find, however, merit in the contention that the real estate taxes paid by respondent in her capacity as usufructuary for several years previous to the present litigation should be paid by her, as she did, instead of by petitioners not only because she bound herself to pay such taxes in a formal agreement approved by the court in Civil Case No. 1569 of the Court of First Instance of Manila (Fabie vs. Gutierrez David, supra). In the case, which involved the same parties and the same properties subject to usufruct, the parties submitted an amicable agreement which was approved by the court wherein the usufructuary, herein respondent, bound herself to pay all the real estate taxes, special assessment and insurance premiums, and make all the necessary repairs on each of the properties covered by the usufruct and in accordance with said agreement, respondent paid all the taxes for the years 1945 to 1954 MCSS Baluran v. Navarro 79 SCRA 309 FACTS: Spouses Paraiso executed an agreement entitle ―BARTER‖ whereby they agreed to ―barter and exchange‖ with spouses Baluran their residential lot with the latter's unirrigated riceland. The documents allowed the parties to enjoy the material possession of their respective properties, reap the fruits of the unirrigated riceland for spouses Paraiso and build a house in the residential lot for spouses Baluran. Also, it was conditioned upon the event that if any of the children of Natividad Obencio, daughter of spouses Paraiso, shall choose to reside in this municipality and build a house in the residential lot, spouses Balura shall be obliged to return the lot with damages. Lastly, neither party shall encumber, alienate or dispose of in any manner their respective properties as bartered without the consent of the other. Antonio Obendencio, son of Natividad, filed a complaint to recover the residential lot. Avelino Baluran alleged that the barter agreement transferred to him the ownership of the residential lot in exchange for the unirrigated riceland. ISSUE: WON there was a transfer of ownership between the parties. -- NO HELD: It is a settled rule that to determine the nature of a contract courts are not bound by the name or title given to it by the contracting parties. The stipulations in said document are clear enough to indicate that there was no intention at all on the part of the signatories to convey the ownership of their respective properties; all that was intended, and it was so provided in the agreement, was to transfer the material possession thereof. In the third condition, the parties retained their right to alienate their respective properties which right is an element of ownership. All that the parties acquired was the right of usufruct which in essence is the right to enjoy the Property of another. Under the document, spouses Paraiso would harvest the crop of the unirrigate riceland while Baluran could build a house on the residential lot. The mutual agreement was subject to a resolutory condition, that the children of Natividad, shall choose to reside in the municipality and build his house on the lot, which would terminate the right of possession and use.

Usufruct may be constituted for any period of time and under such conditions as they may deem convenient and beneficial subject to the provision of the Civil Code. The manner of terminating or extinguishing the right of ususfruct is primarily determined by the stipulations of the parties which in this case is the happening of the event agreed upon. The right of usufruct of the parties is extinguished and each is entitled to a return of his property. NKVS Moralidad v. Pernes G.R. No. 152809 FACTS: While petitioner is in the U.S.A., where she stayed for a long time as teacher, that sometime in 1986, she received news from Arlene, petitioner‘s niece, that Mandug was infested by NPA rebels and many women and children were victims of crossfire between government troops and the insurgents. She immediately sent money to Araceli, Arlene‘s older sister, with instructions to look for a lot in Davao City where Arlene and her family could transfer and settle down. This was why she bought the parcel of land in Davao. Petitioner acquired the lot property initially for the purpose of letting Arlene move from Mandug to Davao City proper but later she wanted the property to be also available to any of her kins wishing to live and settle in Davao City. Following her retirement in 1993, petitioner came back to the Philippines to stay with the respondents‘ on the house they build on the subject property. In the course of time, their relations turned sour because members of the Pernes family were impervious to her suggestions and attempts to change certain practices concerning matters of health and sanitation within their compound. Violent confrontations meanwhile transpired, with the petitioner narrating that, at one occasion in July 1998, she sustained cuts and wounds when Arlene pulled her hair, hit her on the face, neck and back, while her husband Diosdado held her, twisting her arms in the process. Petitioner filed with the MTCC of Davao City an unlawful detainer suit against the respondent spouses. Petitioner alleged that she is the registered owner of the land on which the respondents built their house; that through her counsel, she sent the respondent spouses a letter demanding them to vacate the premises and to pay rentals therefor. In their defense, the respondents alleged having entered the property in question, building their house thereon and maintaining the same as their residence with petitioner‘s full knowledge and express consent. To prove their point, they invited attention to her written declaration of July 21, 1986, supra, wherein she expressly signified her desire for the spouses to build their house on her property and stay thereat for as long as they like. ISSUES: 1. W/N the agreement for the use of the land constituted the defendants as usufructs. 2. W/N the usufruct is terminated. 3. W/N the defendants, as usufructs, have the right of reimbursement. HELD: 1st Issue: YES, because of the written letter by the petitioner to the defendants, it created the

usufruct between the parties. What was constituted between the parties herein is one of usufruct over a piece of land, with the petitioner being the owner of the property upon whom the naked title thereto remained and the respondents being two (2) among other unnamed usufructuaries who were simply referred to as petitioner‘s kin. Usufruct is defined under Article 562 of the Civil Code in the following wise: ART. 562. Usufruct gives a right to enjoy the property of another with the obligation of preserving its form and substance, unless the title constituting it or the law otherwise provides. Usufruct, in essence, is nothing else but simply allowing one to enjoy another‘s property. It is also defined as the right to enjoy the property of another temporarily, including both the jus utendi and the jus fruendi, with the owner retaining the jus disponendi or the power to alienate the same. It is undisputed that petitioner, in a document dated July 21, 1986, supra, made known her intention to give respondents and her other kins the right to use and to enjoy the fruits of her property. There can also be no quibbling about the respondents being given the right "to build their own house" on the property and to stay thereat "as long as they like." 2nd Issue: YES, the usufruct was terminated. The term or period of the usufruct originally specified provides only one of the bases for the right of a usufructuary to hold and retain possession of the thing given in usufruct. There are other modes or instances whereby the usufruct shall be considered terminated or extinguished. For sure, the Civil Code enumerates such other modes of extinguishment: ART. 603. Usufruct is extinguished: (1) By the death of the usufructuary, unless a contrary intention clearly appears; (2) By expiration of the period for which it was constituted, or by the fulfillment of any resolutory condition provided in the title creating the usufruct; (3) By merger of the usufruct and ownership in the same person; (4) By renunciation of the usufructuary; (5) By the total loss of the thing in usufruct; (6) By the termination of the right of the person constituting the usufruct; (7) By prescription. The document executed by the petitioner dated July 21, 1986 constitutes the title creating, and sets forth the conditions of, the usufruct. Paragraph #3 thereof states "[T]hat anyone of my kins may enjoy the privilege to stay therein and may avail the use thereof. Provided, however, that the same is not inimical to the purpose thereof". What may be inimical to the purpose constituting the usufruct may be gleaned from the preceding paragraph wherein petitioner made it abundantly clear "that anybody of my kins who wishes to stay on the aforementioned property should maintain an atmosphere of cooperation, live in harmony and must avoid bickering with one another." That the maintenance of a peaceful and harmonious relations between and among kin constitutes an indispensable condition for the continuance of the usufruct is clearly deduced from the succeeding Paragraph #4 where petitioner stated "[T]hat anyone of my kins who cannot conform with the wishes of the undersigned may exercise the freedom to look for his own." In fine, the occurrence of any of the following: the loss of the atmosphere of

cooperation, the bickering or the cessation of harmonious relationship between/among kin constitutes a resolutory condition which, by express wish of the petitioner, extinguishes the usufruct. From the pleadings submitted by the parties, it is indubitable that there were indeed facts and circumstances whereby the subject usufruct may be deemed terminated or extinguished by the occurrence of the resolutory conditions provided for in the title creating the usufruct. Thus, the Court rules that the continuing animosity between the petitioner and the Pernes family and the violence and humiliation she was made to endure, despite her advanced age and frail condition, are enough factual bases to consider the usufruct as having been terminated. 3rd Issue: NO, they have no right to reimbursement. To reiterate, the relationship between the petitioner and respondents respecting the property in question is one of owner and usufructuary. Accordingly, respondents‘ claim for reimbursement of the improvements they introduced on the property during the effectivity of the usufruct should be governed by applicable statutory provisions and principles on usufruct. By express provision of law, respondents, as usufructuary, do not have the right to reimbursement for the improvements they may have introduced on the property. We quote Articles 579 and 580 of the Civil Code: Art. 579. The usufructuary may make on the property held in usufruct such useful improvements or expenses for mere pleasure as he may deem proper, provided he does not alter its form or substance; but he shall have no right to be indemnified therefor. He may, however, remove such improvements, should it be possible to do so without damage to the property. Art. 580. The usufructuary may set off the improvements he may have made on the property against any damage to the same. Given the foregoing perspective, respondents will have to be ordered to vacate the premises without any right of reimbursement. If the rule on reimbursement or indemnity were otherwise, then the usufructuary might, as an author pointed out, improve the owner out of his property. The respondents may, however, remove or destroy the improvements they may have introduced thereon without damaging the petitioner‘s property.

EASEMENTS OR SERVITUDES (ART. 613-693) AMPS Relova v. Lavarez 9 Phil. 149 DOCTRINE: Servitude may be created by usage for a considerable period of time. FACTS: The plaintiff is the owner of a tract of rice land, which is cultivated with the aid of water brought from the River Bangcabangca, through an aqueduct which passes over the land of the defendants. On the land of the defendants there was a dam (presa) with a small gate or aperture in its face which was used to control the flow of the water in the aqueduct, by permitting a greater or less quantity to escape in a drainage ditch, also on the land of the defendants. One of the defendants completely destroyed the dam and let all the water escape by the drainage ditch, so that none flowed on the land of the plaintiff. At the time when the dam was destroyed the plaintiff had some five cavanes of land prepared to plant rice, but because of the escape of the water resulting from the destruction of the dam he was unable to raise his crop, which was a complete failure. The trial court granted an injunction upon the fact that the aqueduct and dam in question had been in use by the plaintiff, as of right, for more than thirty years, and that he had an easement in the land of the defendants for the maintenance of the said aqueduct and dam, to restrain the defendants from interfering with the plaintiff's right to the use of the water in the aqueduct, in the manner heretofore established by custom. ISSUE: The existence of servitude for the maintenance of the dam upon the land of the defendants in favor of the land of the plaintiff. HELD: The aqueduct and the dam (presa) have been in existence for more than thirty years, during which period the owner of the land in question has always exercised the right to the reasonable use of the water in the aqueduct for irrigation purposes. (Arts. 527 and 528, Civil Code.) Counsel for the appellants contend that under the definition of a servitude which appears in article 530 of the Civil Code the existence of the servitude cannot be established unless it appears that from such servitude a benefit (beneficio) was, or might be, derived by the plaintiff landowner; and that since it appears from the testimony of the witnesses that the aperture in the dam was used for the purpose of controlling the flow of water in the aqueduct and for preventing damage by overflow to the lowlands over which the aqueduct runs, and since it appears that the lands of the plaintiff are higher than the lands of the defendants, therefore the aqueduct could never have been intended for the supply of water to the lands of the plaintiff and neither the dam nor the aqueduct could be of any benefit to these lands. This contention cannot be maintained in the face of the positive testimony as to the existence of the aqueduct and its use for many years to supply water to the lands in question. It may be that

the defendants had a right to open the aperture in the face of the dam to prevent a destructive overflow of water on their lands, but this would not give them the right to stop the flow of water altogether; nor does it tend to establish the contention of the defendants that the plaintiff landowner is not entitled to the benefit of the reasonable use of the water flowing in the aqueduct, since it does not appear that such use necessarily involved destructive overflows from the aqueduct, provided the flow of water therein was properly regulated by the opening of the aperture in the dam. AFFIRMED. KGS Solid Manila v. Bio Hong Trading 195 SCRA 748 DOCTRINES: 1. Servitudes are merely accessories to the tenements of which they form part, and even if they are possessed of a separate juridical existence, they cannot be alienated from the tenement or mortgaged separately. 2. The vendee of real property in which a servitude or easement exists, did not acquire the right to close that servitude or put up obstructions thereon, to prevent the public from using it. FACTS: Solid Manila Corp. (petitioner) owns a parcel of land located in Ermita, Manila. The same lies in the vicinity of another parcel owned by Bio Hong Trading (respondent). The latter‘s title came from a prior owner, and in their deed of sale, the parties thereto reserved as an easement of way approx. 914sqm converted as a private alley for the benefit of the neighboring estates. As a consequence, an annotation was entered in the respondent‘s title. The petitioners and its neighbors made use of the private alley and maintained and contributed to its upkeep. Thereafter, respondent constructed steel gates that precluded unhampered use. Respondent filed a case to remove said gates and to allow full access to the easement, which was granted. In the Court of Appeals, it was held that since respondent has acquired title to the property, ―merger‖ brought about an extinguishment of the easement. ISSUE: Whether the easement still exists or had been extinguished by merger? HELD: The easement still exists on the property of Bio Hong Trading. It is true that the sale did include the alley. On this score, the Court rejects the petitioner's contention that the deed of sale "excluded" it, because as a mere right-of-way, it cannot be separated from the tenement and maintains an independent existence. Thus: Art. 617. Easements are inseparable from the estate to which they actively or passively belong. Servitudes are merely accessories to the tenements of which they form part. Although they are possessed of a separate juridical existence, as mere accessories, they cannot, however, be alienated from the tenement, or mortgaged separately. The fact, however, that the alley in question, as an easement, is inseparable from the main lot is no argument to defeat the petitioner's claims, because as an easement precisely, it operates as

a limitation on the title of the owner of the servient estate, specifically, his right to use (jus utendi). As the petitioner indeed hastens to point out, the deed itself stipulated that "a portion thereof [of the tenement] measuring 914sqm, more or less, had been converted into a private alley for the benefit of the neighboring estates. . ." and precisely, the former owner, in conveying the property, gave the private owner a discount on account of the easement. Hence, and so we reiterate, albeit the private respondent did acquire ownership over the property –– including the disputed alley –– as a result of the conveyance, it did not acquire the right to close that alley or otherwise put up obstructions thereon and thus prevent the public from using it, because as a servitude, the alley is supposed to be open to the public. The Court is furthermore of the opinion, contrary to that of the Court of Appeals, that no genuine merger took place as a consequence of the sale in favor of the private respondent corporation. According to the Civil Code, a merger exists when ownership of the dominant and servient estates is consolidated in the same person. Merger then, as can be seen, requires full ownership of both estates. One thing ought to be noted here, however. The servitude in question is a personal servitude, that is to say, one constituted not in favor of a particular tenement (a real servitude) but rather, for the benefit of the general public. Personal servitudes are referred to in the following article of the Civil Code: Art. 614. Servitudes may also be established for the benefit of a community, or of one or more persons to whom the encumbered estate does not belong. In a personal servitude, there is therefore no "owner of a dominant tenement" to speak of, and the easement pertains to persons without a dominant estate, in this case, the public at large. Merger, as we said, presupposes the existence of a prior servient-dominant owner relationship, and the termination of that relation leaves the easement of no use. Unless the owner conveys the property in favor of the public –– if that is possible –– no genuine merger can take place that would terminate a personal easement. JPOT CID v. Javier 108 Phil. 850 DOCTRINE: "With regard to easements in the negative, from dominant to a servient, it must be accompanied by a 'formal act.' The law is explicit in stating that it should be in an instrument acknowledged by a notary public." FACTS: The review for certiorari arises with the conflict of Javier effectively blocking the easement of light and view, although both structures are in accordance with their corresponding lots. The conflict can be assisted by the interpretation of a formal act which has different applications in the Spanish Civil Code as well as our Civil Code. In the former, it was discussed that any act would suffice. As long as there is a positive act of prohibition whether orally or implied. However, the distinction was made apparent in the latter's discussion. The court does not aver and makes plain and distinguishes between a formal act. In this case, the effectivity of said act would be in the negative. This act (i.e. prohibition) would be coming from the dominant directed towards the servient. Moreover, such an act could only be validly appreciated once it is in a legal document that is notarized. Only with such acknowledgement can an easement be binding and bar anyone who committs to undermine and deprive the right of a land owner to his pleasures, in this

instance privacy, sunlight and a view. ISSUE: W/N respondents Irene P. Javier, et al., owners of a building standing on their lot with windows overlooking the adjacent lot, had acquired by prescription an enforceable easement of light and view arising from a verbal prohibition to obstruct such view and light, alleged to have been made upon petitioner's predecessor-in-interest as owner of the adjoining lot, both of which lots being covered by Torrens titles. HELD: NO. The requirement for an easement to be applicable would be that it be annotated and affixed together with the registry of the Torrens title specifying the prohibition in question. Also, Article 538 provides that the requirement to be fulfilled for prescription to run is that a formal act such as prohibition coming from a dominant estate to a servient one be in an instrument duly acknowledged by a notary public. Otherwise, there is no prescription or easement to speak of. MLAV Cortes v. Yu-Tibo 2 Phil. 24 DOCTRINE: An easement of light and view is a negative easement. When easement is negative, there should be a formal act of opposition for prescription to run. FACTS: Cortes‘ wife owns a house (No. 65) in which certain windows open on the adjacent property (No. 63), a neighboring house on the same street. This setting has been in existent since 1843. The tenant of the adjacent property raised the roof of house No. 63 in such a manner that half of the windows of house No. 65 has been covered, thus depriving it of a large part of air and light. Plaintiff contends that by the constant and uninterrupted use of the windows for 59 years, he acquired from prescription an easement of light in favor of house No. 65, and as a servitude upon house No. 63. Consequently, he has acquired the right to restrain the making of any improvement in the latter house which may be prejudicial to the enjoyment of the easement. Further, he contends that the easement of light is positive; and that therefore the period of possession for the purposes of the acquisition of a prescriptive title is to begin from the date on which the enjoyment of the same commenced, or, in other words, from the time that said windows were opened with the knowledge of the owner of the house No. 63, and without opposition on this part. Defendant contends that the easement is negative, and therefore the time for the prescriptive acquisition must begin from the date on which the owner of the dominant estate may have prohibited, by a formal act, the owner of the servient estate from doing something which would be lawful but for the existence of the easement. Lower court ruled in favor of the defendant. Plaintiff appealed the case. ISSUE: WON the easement is positive/negative HELD: Easement is negative.

The easement of light which is the object of this litigation is of a negative character, and therefore pertains to the class which cannot be acquired by prescription as provided by article 538 of the Civil Code, except by counting the time of possession from the date on which the owner of the dominant estate has, in a formal manner, forbidden the owner of the servient estate to do an act which would be lawful were it not for the easement. In consequence, the plaintiff, not having executed any formal act of opposition to the right of the owner of house No. 63 to make improvements which might obstruct the light of house No. 65, at any time prior to the complaint, has not acquired, nor could he acquire by prescription, such easement of light, no matter how long a time have elapsed. Because the period which the law demands for such prescriptive acquisition could not have commenced to run, the act with which it must necessarily commence not having been performed. DJTV Abellana v. CA 208 SCRA 316 DOCTRINES: ● The use of a footpath or road may be apparent but it is not a continuous easement because its use is at intervals and depends upon the acts of man. ● A right of way is not acquirable by prescription. FACTS: The petitioners who live on a parcel of land abutting the northwestern side of the Nonoc Homes Subdivision, sued to establish an easement of right of way over a subdivision road which, according to the petitioners, used to be a mere footpath which they and their ancestors had been using since time immemorial, and that, hence, they had acquired, through prescription, an easement of right of way therein. The construction of a wall by the respondents around the subdivision deprived the petitioners of the use of the subdivision road which gives the subdivision residents access to the public highway. They asked that the high concrete walls enclosing the subdivision and cutting of their access to the subdivision road be removed and that the road be opened to them. The private respondents denied that there was a pre-existing footpath in the place before it was developed into a subdivision. They alleged furthermore that the Nonoc Subdivision roads are not the shortest way to a public road for there is a more direct route from the petitioners' land to the public highway. After trial, the trial court ordered to demolish the subject fences or enclosures at the dead ends of Road Lots 1 and 3 of the Nonoc Homes Subdivision at their expense and to leave them open for the use of the plaintiffs and the general public. However, on appeal by the defendants and intervenors (now private respondents), the appellate court reversed the appealed judgment. It found that requisites essential for the grant of an easement of right of way are not obtaining in this case hence no alternative presents itself except reversal of the judgment. The appellate court denied petitioners' motion for reconsideration of the aforesaid decision. Hence, this petition for review.

ISSUES: Whether or not the Court of Appeals erred: 1. 1.in not holding that the easement claimed by them is a legal easement established by law (Art. 619. Civil Code) and acquired by them by virtue of a title under Art. 620, Civil Code and P.D. No. 957 through the National Housing Authority which has exclusive jurisdiction to regulate subdivision and condominium projects; 2. 2.in not holding that the footpaths and passageways which were converted into subdivision road lots have acquired the status of public streets in view of Section 4 of Municipal Ordinance No. 1, Series of 1969 of Talisay, Cebu which provides that subdivision roads shall be used not only for the exclusive use of the homeowners but also for the general public, and Section 5 of Ordinance No. 5, Series of 1974, which provides that "those subdivision road lots whose use by the public are (sic) deemed necessary by the proper authorities shall be made available for public use"; and 3. 3.in not determining whether or not the closure of the dead ends of road lots 1 and 3 of the Nonoc Homes Subdivision by the private respondents was legal. HELD: NO Petitioners' assumption that an easement of right of way is continuous and apparent and may be acquired by prescription under Article 620 of the Civil Code, is erroneous. The use of a footpath or road may be apparent but it is not a continuous easement because its use is at intervals and depends upon the acts of man. It can be exercised only if a man passes or puts his feet over somebody else's land. Hence, a right of way is not acquirable by prescription. Neither may petitioners invoke Section 29 of P.D. 957 which provides: Sec. 29.Right of Way to Public Road. — The owner or developer of a subdivision without access to any existing public road or street must secure a right of way to a public road or street and such right of way must be developed and maintained according to the requirement of the government authorities concerned. The above provision applies to the owner or developer of a subdivision (which petitioners are not) without access to a public highway. The petitioners' allegation that the footpaths which were converted to subdivision roads have acquired the status of public streets, is not well taken. In the first place, whether or not footpaths previously existed in the area which is now known as the Nonoc Homes Subdivision, is a factual issue which this Court may not determine for it is not a trier of facts. The municipal ordinances which declared subdivision roads open to public use "when deemed necessary by the proper authorities" simply allow persons other than the residents of the Nonoc Homes Subdivision, to use the roads therein when they are inside the subdivision but those ordinances do not give outsiders a right to open the subdivision walls so they can enter the subdivision from the back. As the private respondents pointed out in their Comment: The closure of the dead ends of road lots 1 and 3 is a valid exercise of proprietary rights. It is for the protection of residents in the subdivision from night prowlers and thieves. And the public is not denied use of the subdivision roads, only that the users must get inside the subdivision

through the open ends of the road lots that link the same to the public road. It is common to most, if not all subdivisions in Cebu, Metro Manila and other places, that points of ingress to and egress from the subdivision are the points where the subdivision roads intersect with public roads. It is of judicial notice that most, if not all, subdivisions are enclosed and fenced with only one or few points that are used as ingress to and egress from the subdivisions. WHEREFORE, finding no merit in the petition for review, the same is DENIED with costs against the petitioners. JGY Ronquillo v. Roco 103 Phil. 84 DOCTRINE: An easement of right of way being discontinuous, cannot be acquired through prescription but only by virtue of a title. FACTS: Petitioners Ronquillo have been in the continuous and uninterrupted use of a road or passage way which traversed the land of the Respondents Roco and their predecessors in interest, in going to Igualdad Street and the market place of Naga City, from their residential land and back, for more than 20 years. Respondents Roco and his men started constructing a chapel in the middle of the said right of way construction actually obstructed the continuous exercise of the rights of the plaintiffs over said right of way. Respondents also forcibly planted wooden posts, fenced with barbed wire and closed the road passage way. ISSUE: WON an easement of right of way can be acquired thru prescription -- NO HELD: The dismissal was based on the ground that an easement of right of way though it may be apparent is, nevertheless, discontinuous or intermittent and, therefore, cannot be acquired through prescription, but only by virtue of a title. Easements may be Continuous or discontinuous, apparent or non-apparent, discontinuous being those used at more or less long intervals and which depend upon acts of man. Continuous and apparent easements are acquired either by title or prescription, continuous nonapparent easements and discontinuous ones whether apparent or not, may be acquired only by virtue of a title. Both Manresa and Sanchez Roman are of the opinion that the easement of right of way is a discontinuous one. JRPA Amor v. Florentino 74 Phil. 404 FACTS: Maria Florentino owned a house and a camarin (warehouse). By a will, she transferred the house to Jose Florentino and the warehouse to Maria Florentino. Maria sold the warehouse to Amor. Amor then demolished the old warehouse in order to build a new 2-storey structure. The problem is it will shut off the light and air that come in through the window of the adjacent house

owned by Jose. Hence the latter files for prohibition claiming there is a negative easement prohibiting Amor from constructing any structure at any height that would block the window. Amor counters that there is no easement. Moreover, since the death of testator was before the Civil Code took effect, the rules on easement do not apply. ISSUES: 1. Whether or not there is an easement prohibiting Amor from doing said construction. 2. Whether or not the Civil Code may be applied HELD: 1. Yes. Easement are established by law or by will of the owners or by title. Under Art. 624, there is title by the doctrine of apparent sign. When the estate is subsequently owned by two different persons and the ―service‖ (it cannot be an easement before the transfer) is not revoked in the title nor removed, an easement is established. The Cortez case cannot be invoked by Amor because it involved acquisition by prescription. Art. 624 is acquisition by title. 2. Amor failed to prove that the death of the testator occurred before the effectivity of the Old Civil Code. The facts show that it happened after the effectivity of the said code so the law on easement is already applicable. In any case, even if we assume Amor‘s supposition, the law on easement was already integrated into the Spanish Law and in fact, had been established by Jurisprudence. Therefore, Amor is prohibitied from constructing the warehouse above the level of the window. ABB Gargantos v. Tan Yanon 108 Phil. 888 Doctrine: Art. 624 provides that when two adjoining estates were formerly owned by one person who introduced improvements on both such that the wall of the house constructed on the first estate extends to the wall of the warehouse on the second estate; and at the time of the sale of the first estate, there existed on the aforementioned wall of the house, doors, windows which serve as passages for light and view, there being no provision in the deed of sale that the easement of light and view will not be established, the apparent sign of easement between the two estates is established as a title. FACTS: The late Francisco Sanz was the previous owner of a land which he subdivided into several lots. One lot was sold to Guillermo Tengtio, who sold to Vicente Uy Veza. Another lot with a house constituted thereon was sold to Tan Yanon. A third portion with a warehouse was sold to Gargantos. The problem arose when Gargantos asked from the Municipality for a permit to demolish the warehouse in order to construct a higher one. Tan Yanon opposed the application for it would block his window and impair his right of light and view. Both the provincial fiscal and district engineer of Romblon recommended granting of the building permit to Gargantos. Tan Yanon then filed against Gargantos an action to restrain him from constructing a building that would prevent plaintiff from receiving light and enjoying the view through the window of his house, unless such building is erected at a distance of not less than three meters from the boundary line between the lots of plaintiff and defendant, and to enjoin the members of

Municipal Council of Romblon from issuing the corresponding building permit to defendant. ISSUE: Whether the property of respondent Tan Yanon has an easement of light and view against the property of petitioner Gargantos. -- YES HELD: Art. 624 provides that when two adjoining estates were formerly owned by one person who introduced improvements on both such that the wall of the house constructed on the first estate extends to the wall of the warehouse on the second estate; and at the time of the sale of the first estate, there existed on the aforementioned wall of the house, doors, windows which serve as passages for light and view, there being no provision in the deed of sale that the easement of light and view will not be established, the apparent sign of easement between the two estates is established as a title. FZC Valderama v. North Negros Sugar Central 48 Phil. 492 DOCTRINE: It is against the nature of the easement to pretend that it was established in favor of the servient estates, because it is a well settled rule that things serve their owner by reason of ownership and not by reason of easement. FACTS: Several hacienda owners in Manapla, Occidental Negros, entered into a milling contract with Miguel Osorio wherein the latter would build a sugar central of a minimum capacity of 300 tons for the milling and grinding of all the sugar cane to be grown by the hacienda owners who in turn would furnish the central with all the cane they might produce in their estates for 30 years from the execution of the contract. Later on, Osorio‘s rights and interests were acquired by the North Negros Sugar Co., Inc. 2 years after, the current petitioners, Catalino Valderrama, Emilio Rodriguez, Santos Urra et. al, made other milling contracts identical to the first one with the North Negros Sugar, Co., Inc. The hacienda owners, however, could not furnish the central sufficient cane for milling as required by its capacity, so the North Negros made other milling contracts with the various hacienda owners of Cadiz, Occidental Negros. This gave rise to the plaintiffs filing their complaint, alleging that the easement of way, which each of them has established in his respective hacienda, was only for the transportation through each hacienda of the sugar cane of the owner thereof, while the defendant maintains that it had the right to transport to its central upon the railroad passing through the haciendas of the plaintiffs, not only the sugar cane harvested in said haciendas, but also that of the hacienda owners of Cadiz, Occidental Negros. The CFI entered 1 single judgment for all of them, ruling in Valderrama et. al‘s favor finding that North Negros had no right to pass through the lands of the hacienda owners for the transportation of sugar cane not grown from their lands. Thus the appeal to the SC. ISSUE: WON the easement of way established was restricted to transporting only sugar cane from the hacienda owners‘ lands. -- NO HELD: The contract entered into by each of the hacienda owners contained a clause that granted the

North Negros an easement of way 7 meters wide for the period of 50 years upon their properties for the construction of a railroad. The owners allege ambiguity since it could permit the transportation of sugar cane which they did not produce which is contrary to their intent but the SC held that it is clear that the easement was established for the benefit of all producers and of the corporation as it is the intent of the milling contract. Since the easement is a voluntary, apparent, continuous easement of way in favor of the corporation, it is contrary to the nature of the contract that it is only limited to canes produced by the servient estates since it is a well settled rule that things serve their owner by reason of ownership and not by reason of easement. The owners also cannot limit its use for there is nothing in the contract prohibiting the central from obtaining other sources. Transporting cane from Cadiz also does not make it more burdensome since what is prohibited in Art. 543 of the CC is that in extending the road or in repairing it, it should occupy a greater area or deposit excavations outside the granted 7 meters. This does not happen in this case when the North Negros transports sugar cane from Cadiz, crossing the servient estates, since it continues to occupy the same area and the encumbrance is still the same regardless of the number of times it passes through the estates. Also the period of the easement is longer than the period of the milling contracts, so even if the owners no longer desire to furnish the central canes for milling, the North Negros still has the right to the easement for the remaining period so the contention that it should be limited to the canes produced by the owners has no basis. LNAC Javellana v. IAC 172 SCRA 280 DOCTRINE: They closed the entrance of the canal and demolished portions of the main dike thus impairing the use of the servitude by the dominant estate. And by so doing, plaintiffs violated not only the law on easement but also Presidential Degree No. 296 which enjoins any person, natural or juridical, to demolish structures or improvements which tend to obstruct the flow of water through rivers, creeks, esteros and drainage channels. For this canal did not serve merely to supply salt water to the school fishpond but also serves as drainage charged or channel of rainwater from adjacent lands to the Iloilo River. FACTS: Marsal & Co., Inc., and Marcelino Florete, Sr. is the present owner of the land adjoining the Iloilo River up to the adjacent lot where the L. Borres Elem. School is located. There existed a main canal from the Iloilo River which passes through the Marsal property and through a canal that traverses the school property going towards Lot 2344. Marsal & Co. closed the dike entrance and later on demolished the portions of the main dike connecting the main canal to the canal running through the school grounds. This closure caused flooding in the premises of the school and its vicinity because the canal serves as outlet of rain or flood water that empties into the river. This prompted the school and barangay officials to complain to higher authorities about the closure of the canal. When Florete was about to bury a pipe in lieu of an open canal, he was prevented from doing so by District Supervisor Javellana. Florete instituted a complaint for recovery of damages for allegedly denying his access to the use of the canal to his property.

RTC ruled in favor of Javellana. Florete appealed to the IAC which reversed the decision. Thus, Javellana instituted herein recourse. ISSUE: Whether an easement has been constituted on the subject property. -- YES HELD: A positive easement of water-right-of-way was constituted on the property of Florete as the servient estate in favor of the L. Borres Elementary School and the nearby lands as dominant estates since it has been in continuous use for no less than 15 years by the school fishpond as well as by the nearby adjacent lands. As a positive easement, Florete had no right to terminate the use of the canal without violating Art. 629 of the CC which provides that ―The owner of the servient estate cannot impair, in any manner whatsoever, the use of the servitude. Nevertheless if by reason of the place originally assigned or of the manner established for the use of the easement, the same should become very inconvenient to the owner of the servient estate, or should prevent him from making any important works, repairs or improvements thereon, it may be charged at his expense, provided he offers another place or manner equally convenient and in such a way that no injury is caused thereby to the owner of the dominant estate or to those who may have a right to the use of the easement.‖ Defendants' closure of the dike's entrance connecting the main canal with the canal running through the school premises, therefore, caused the flooding of the premises of the L. Borres Elementary School and its vicinity. This is so because during rainy season, said canal also serves as outlet of rain or flood waters that empties to the Iloilo River. Witnesses Ignacio Gencianeo, Francisco Regacho, Severo Maranon and Barangay Captain Antonio Sison were unanimous in declaring so. TKDC Benedicto v. CA 25 SCRA 145 DOCTRINE: The easement is perpetual in character and was annotated on all the transfer certificates of title issued to Heras and to Benedicto. Absence of anything that would show mutual agreement to extinguish the easement, the easement persists. FACTS: Heras filed an action with the Court of First Instance to recover a portion of land enclosed and walled by Benedicto and to demand the reopening of an easement of way between his and Benedicto‘s real property. Hendrick sold portions of her property to several personalities including Recto and Heras. When portion of the property was sold to Herras, he closed and walled the part of land serving as easement of way. Trial court found that the easement of way was found entirely within Benedicto‘s property contrary to the stipulation in the deed of sale between Hedrick and Recto that it should be between their properties with each contributing an equal portion of his property. Thus, trial court directed the parties to equally contribute to the maintenance of the passageway between Herras

and Benedicto. ISSUE: Whether or not Benedicto may enclose his property. HELD: In this case, the easement is perpetual in character and was annotated on all the transfer certificates of title issued to Heras and to Benedicto. Absence of anything that would show mutual agreement to extinguish the easement, the easement persists. TKDC Benedicto v. CA 25 SCRA 145 DOCTRINE: Under Art. 624, an easement may continue by operation of law. Alienation of the dominant and servient estates to different persons is not a ground for the extinguishment of easements, absent a statement extinguishing it. FACTS: Private respondent Antonio Cardenas was the owner of 2 parcels of land situated in Cebu City. An apartment building was constructed on Lot A, while on Lot B stands a 4-door apartment, a 2storey house, a bodega, and a septic tank for the common use of the occupants of Lots A and B. A small portion of the apartment building on Lot A also stands on Lot B. Cardenas sold Lot A to petitioner Eduardo C. Tañedo. On the same day, Cardenas also mortgaged Lot B to Tañedo as a security for the payment of a loan. Cardenas agreed that he would sell Lot B only to Tañedo in case he should decide to sell it. However, Cardenas sold Lot B to the respondent spouses Romeo and Pacita Sim. Upon learning of the sale, Tañedo offered to redeem the property from Romeo Sim, but the latter refused. Instead, Sim blocked the sewage pipe connecting the building of Eduardo Tañedo built on Lot A, to the septic tank in Lot B. He also asked Tañedo to remove that portion of his building enroaching on Lot B. As a result, Tañedo filed an action for legal redemption and damages against Spouses Sim and Antonio Cardenas, invoking the provisions of Art. 1622 of the Civil Code. Respondent judge, Juanito A. Bernad, dismissed the complaint for legal redemption, as well as petitioner‘s motion for reconsideration. Hence, this petition for review on certiorari. ISSUE: W/N the alienation of Lots A and B is a ground for the extinguishment of the easement of drainage. -- NO HELD: The finding of the trial court that Tañedo's right to continue to use the septic tank on Lot B ceased upon the subdivision of the land and its subsequent sale to different owners who do not have the same interest, also appears to be contrary to law. Article 631 of the Civil Code enumerates the grounds for the extinguishment of an easement. From its provisions, the alienation of the dominant and servient estates to different persons is not one of the grounds for the extinguishment of an easement. On the contrary, use of the

easement is continued by operation of law. Article 624 of the Civil Code provides: Art. 624. The existence of an apparent sign of easement between two estates, established or maintained by the owner of both, shall be considered, should either of them be alienated, as a title in order that the easement may continue actively and passively, unless, at the time the ownership of the two estates is divided, the contrary should be provided in the title of conveyance of either of them, or the sign aforesaid should be removed before the execution of the deed. This provision shall also apply in case of the division of a thing owned in common by two or more persons. In the instant case, no statement abolishing or extinguishing the easement of drainage was mentioned in the deed of sale of Lot A to Tañedo. Nor did Cardenas stop the use of the drain pipe and septic tank by the occupants of Lot A before he sold said lot. Hence, the use of the septic tank is continued by operation of law. Accordingly, the spouses Romeo and Pacita Sim the new owners of the servient estate (Lot B), cannot impair, in any manner whatsoever, the use of the servitude. CRF Alcantara v. Rita 372 SCRA 364 DOCTRINE: Construction of a house on the lot of another to facilitate the utilization of usufruct may constitute as personal easement pursuant to Article 614. (batasnatin) FACTS: Petitioner filed a complaint against Cornelio B. Reta, Jr. for the exercise of the right of right of first refusal under Presidential Decree No. 1517, injunction with preliminary injunction, attorney‘s fees and nullity of amicable settlement. The plaintiffs claimed that they were tenants or lessees of the land located in Barangay Sasa, Davao City, covered by Transfer Certificate of Title No. T72594, owned by Reta; that the land has been converted by Reta into a commercial center; and that Reta is threatening to eject them from the land. They assert that they have the right of first refusal to purchase the land in accordance with Section 3(g) of Presidential Decree No. 1517 since they are legitimate tenants or lessees thereof. On the other hand, Reta claimed that the land is beyond the ambit of Presidential Decree No. 1517 since it has not been proclaimed as an Urban Land Reform Zone; that the applicable law is Batas Pambansa Blg. 25 for failure of the plaintiffs to pay the rentals for the use of the land; and that the amicable settlement between him and Ricardo Roble was translated to the latter and fully explained in his own dialect. ISSUE: Whether or not the petitioners have the right of first refusal under Presidential Decree No. 1517. -- NO HELD: None of the petitioners is qualified to exercise the right of first refusal under P. D. No. 1517. The area involved has not been proclaimed an Urban Land Reform Zone (ULRZ).To be able to qualify and avail oneself of the rights and privileges granted by the said decree, one must be: (1) a legitimate tenant of the land for ten (10) years or more; (2) must have built his home on the land by contract; and, (3) has resided continuously for the last ten (10) years. Obviously, those who do not fall within the said category cannot be considered ―legitimate tenants‖ and, therefore,

not entitled to the right of first refusal to purchase the property should the owner of the land decide to sell the same at a reasonable price within a reasonable time. Respondent Reta allowed petitioner only usufruct to the property of another with the obligation of preserving its form and substance, unless the title constituting it or the law otherwise provides. Petitioner Roble was allowed to construct his house on the land because it would facilitate his gathering of tuba. This would be in the nature of a personal easement under Article 614 of the Civil Code. A contract has been defined as ―a meeting of the minds between two persons whereby one binds himself, with respect to the other, to give something or to render some service.‖ Clearly, from the moment respondent Reta demanded that the petitioners vacate the premises, the verbal lease agreements, which were on a monthly basis since rentals were paid monthly, ceased to exist as there was termination of the lease. Indeed, none of the petitioners is qualified to exercise the right of first refusal under P. D. No. 1517. Another factor which militates against petitioners‘ claim is the fact that there is no intention on the part of respondent Reta to sell the property. Hence, even if the petitioners had the right of first refusal, the situation which would allow the exercise of that right, that is, the sale or intended sale of the land, has not happened. P. D. No. 1517 applies where the owner of the property intends to sell it to a third party. MPF Costabella Corp. v. CA 193 SCRA 333 DOCTRINE:An easement of right of way is discontinuous and as such cannot be acquired by prescription. Convenience of the dominant estate is not a gauge for the grant of compulsary right of way. While a right of way is legally demandable, the owner of the dominant estate is not at liberty to impose one based on arbitrary choice. Under Article 650 of the Code, it shall be established upon two criteria: (1) at the point least prejudicial to the servient estate; and (2) where the distance to a public highway may be the shortest. FACTS: ● Petitioners owned a lot wherein they started constructing their beach hotel. Before such construction, the private respondent, in going to and from their respective properties and the provincial road, passed through a passageway, which traversed the petitioner‘s property. ○ As a result of the construction, this passageway, including the alternative route, was obstructed. ○ Private respondent filed for injunction plus damages. ● In the same complaint the private respondents also alleged that the petitioner had constructed a dike on the beach fronting the latter‘s property without the necessary permit, obstructing the passage of the residents and local fishermen, and trapping debris of flotsam on the beach. ● The private respondent also claim that they have acquired the right of way through prescription. ○ They prayed for the re-opening of the ―ancient road right of way‖ (what they called the supposed easement in this case) and the destruction of the dike. ● Petitioner answered by saying that their predecessor in interest‘s act of allowing them to pass was gratuitous and in fact, they were just tolerating the use of the private respondents. CA ruled in favor of the private respondents.

ISSUES: 1. Whether or not easement of right and way can be acquired through prescription? -- NO 2. Whether or not the private respondents had acquired an easement of right of way in the form of a passageway, on the petitioner‘s property? -- NO HELD: 1. Easement of right of way is discontinuous thus it cannot be subject to acquisitive prescription. 2. One may validly claim an easement of right of way when he has proven the: (1) the dominant estate is surrounded by other immovables and has no adequate outlet to a public highway; (2) proper indemnity has been paid; (3) the isolation was not due to acts of the proprietor of the dominant estate; (4) the right of way claimed is at point least prejudicial to the servient estate. The private respondent failed to prove that there is no adequate outlet from their respective properties to a public highway; in fact the lower court confirmed that there is another outlet for the private respondents to the main road (yet they ruled in favor of the private respondents). Apparently, the CA lost sight of the fact that the convenience of the dominant estate was never a gauge for the grant of compulsory right of way. There must be a real necessity and not mere convenience for the dominant estate to acquire such easement. Also, the private respondents made no mention of their intention to indemnify the petitioners. The SC also clarified that ―least prejudicial‖ prevails over ―shortest distance‖ (so shortest distance isn‘t necessarily the best choice). AMDG Villanueva v. Velasco 346 SCRA 99 DOCTRINE: A legal easement is one mandated by law, constituted for public use or for private interest and becomes a continuing property right. As a compulsory easement, it is inseparable from the estate to which it belongs as provided for Article 617 of the Civil Code. FACTS: Villanueva is currently registered owner of land that he bought from Pacific Bank Corporation. The bank is the mortgagee of the land and bought it from Maximo and Justina Gabriel at a public auction. It is noted that before Villanueva purchased the property, a two-meter wide easement of right of way was granted by the spouses Gabriel in favor of the Espinolas, the predecessor-in-interest of private respondents Sebastian and Lorilla. In addition, Gabriel constructed a small house that encroached one-meter of the easement right. A Civil Case was filed by Sebastian and Lorilla against the spouses Gabriel praying for the easement right to be enforced. The trial court and Court of Appeals ruled in their favor and ordered the demolition of the house. Consequently, an Alias Writ of Demolition was issued. It is noted that at this time, Villanueva is now the owner of the property. Villanueva filed a Third Party Claim with Prayer to Quash Alias of Demolition which was later on denied. The Court of Appeals also dismissed his petition for certiorari. Villanueva is arguing that the civil case decision cannot be enforced against him since (1) the easement right of way was not annotated in his title and (2) he was not a party in the civil case.

ISSUE: 1. Whether or not a right of way can be enforced against Villanueva although it was not annotated in his title – YES 2. Whether or not the civil case decision can be enforced against Villanueva even though he was not a party to the civil case – YES HELD: The Court of Appeals correctly identified that the contract of easement present in this case is both voluntary and legal easement. A legal easement is one mandated by law, constituted for public use or for private interest, and becomes a continuing property right. As a compulsory easement, it is inseparable from the estate to which it belongs, as provided for in Article 617 of the Civil Code. The essential requisites for an easement to be compulsory are: 1. The dominant estate is surrounded by other immovable and has no adequate outlet to a public highway 2. Proper indemnity has been paid 3. The isolation was not due to acts of the proprietor of the dominant estate 4. The right of way claimed is at a point at least prejudicial to the servient estate 5. To the extent consisted with the foregoing rule, where the distance from the dominant estate to a public highway may be the shortest. Having established that the easement right present in this case is legal in nature, it follows that the servient estate (Villanueva) is legally bound to adhere to his obligation of providing the dominant estate (Sebastian and Lorilla) its right of way. Applying this to the case, the one-meter wide easement is insufficient for the needs of the private respondents which is why Villanueva is obligated to demolish the house to be able to conform to what is stated in the contract of easement. As regards Villanueva‘s second argument, a decision in a case is conclusive and binding upon the parties as well as to its successor in interest by title. It is clear from the facts that the civil case was decided almost four years before Villanueva purchased the property. Being the successor in interest, the civil case decision binds Villanueva. GCG Cristobal v. CA 291 SCRA 122 DOCTRINE: To be entitled to a compulsory easement of right of way, the preconditions provided under Arts. 649 and 650 of the Civil Code must be established. These are: (1) that the dominant estate is surrounded by other immovables and has no adequate outlet to a public highway; (2) that proper indemnity has been paid; (3) that the isolation was not due to acts of the proprietor of the dominant estate; (4) that the right of way claimed is at a point least prejudicial to the servient estate and, in so far as consistent with this rule, where the distance from the dominant estate to a public highway may be the shortest. The burden of proving the existence of these prerequisites lies on the owner of the dominant estate. FACTS: Petitioners own a house and lot situated at No. 10 Visayas Avenue Extension, Quezon City. They have already been residing there since 1961.

Respondent Cesar Ledesma, Inc., on the other hand, is the owner of a subdivision at Barrio Culiat along Visayas Avenue. It included the disputed residential lots, Lot 1 and Lot 2. The said lots were originally part of a private road known as Road Lot 2 owned exclusively by Cesar Ledesma, Inc. When Visayas Avenue became operational as a national road in 1979, Cesar Ledesma Inc., filed a petition before the RTC of Quezon City to be allowed to convert Road Lot 2 into residential lots. The petition was granted. Road Lot 2 was converted into residential lots designated as Lot 1 and Lot 2. Subsequently, Cesar Ledesma, Inc. sold both lots to Macario Pacione in whose favor Transfer Certificates of Title were correspondingly issued. In turn, Macario Pacione conveyed the lots to his son and daughter-in-law, respondent spouses Jesus and Lerma Pacione. When the Pacione spouses, who intended to build a house on Lot 1, visited the property in 1987, they found out that the lot was occupied by a squatter named Juanita Geronimo. A portion was being used as a passageway by petitioners to and from Visayas Avenue. Accordingly, the spouses complained about the intrusion into their property to the barangay office. At the barangay conciliation proceeding, petitioners offered to pay for the use of a portion of Lot 1 as passageway but the Pacione spouses rejected the offer. When the parties failed to arrive at an amicable settlement, the spouses started enclosing Lot 1 with a concrete fence. Petitioners protested the enclosure alleging that their property was bounded on all sides by residential houses belonging to different owners and had no adequate outlet and inlet to Visayas Avenue except through the property of the Paciones. As their protest went unheeded, petitioners instituted an action for easement of right of way with prayer for the issuance of a temporary restraining order (TRO). The trial court issued a TRO directing the Pacione spouses to cease and desist from fencing the disputed property. The trial court dismissed the complaint holding that one essential requisite of a legal easement of a right of way was not proved. Petitioners appealed to the Court of Appeals. The appellate court affirmed the findings of the trial court. Their motion for reconsideration having been denied, petitioners filed the present petition together with the issue of legality or illegality of the conversion of Road Lot 2 into two (2) residential lots by the Cesar Ledesma, Inc. ISSUE: Whether or not there was a compulsory easement of right of way. HELD: The Supreme Court denied the petition. To be entitled to a compulsory easement of right of way, the preconditions provided under Arts. 649 and 650 of the Civil Code must be established. These are: (1) that the dominant estate is surrounded by othe immovables and has no adequate outlet to a public highway; (2) that proper indemnity has been paid; (3) that the isolation was not due to acts of the proprietor of the dominant estate; (4) that the right of way claimed is at a point least prejudicial to the servient estate and, in so far as consistent with this rule, where the distance from the dominant estate to a public highway may be the shortest. The burden of proving the existence of these

prerequisites lies on the owner of the dominant estate. In the present case, the first element is clearly absent. As found by the trial court and the Court of Appeals, an outlet already exist, which is a path walk located at the left side of petitioners' property and which is connected to a private road about five hundred (500) meters long. The private road, in turn, leads to Ma. Elena Street which is about 2.5 meters wide and, finally, to Visayas Avenue. This outlet was determined by the court a quo to be sufficient for the needs of the dominants estate, hence petitioners have no cause to complain that they have no adequate outlet to Visayas Avenue. Further, no evidence was adduced by petitioners to prove that the easement they seek to impose on private respondents' property is to be established at a point least prejudicial to the servient estate. For emphasis, Lot 1 is only 164 square meters and an improvident imposition of the easement on the lot may unjustly deprive private respondents of the optimum use and enjoyment of their property, considering that its already small area will be reduced further by the easement. Worse, it may even render the property useless for the purpose for which private respondents purchased the same. It must also be stressed that, by its very nature, and when considered with reference to the obligations imposed on the servient estate, an easement involves an abnormal restriction on the property rights of the servient owner and is regarded as a charge or encumbrance on the servient estate. Thus, it is incumbent upon the owner of the dominant estate to establish by clear and convincing evidence the presence of all the preconditions before his claim for easement of right of way be granted. Petitioners miserably failed in this regard. VCL IV Quimen v. CA 257 SCRA 163 DOCTRINE: Where the easement may be established on any of several tenements surrounding the dominant estate, the one where the way is shortest and will cause the least damage should be chosen. However, as elsewhere stated, if these two (2) circumstances do not concur in a single tenement, the way which will cause the least damage should be used, even if it will not be the shortest. FACTS: Petitioner Anastacia Quimen together with her brothers Sotero, Sulpicio, Antonio and sister Rufina inherited a piece of property situated in Pandi, Bulacan. They agreed to subdivide the property equally among themselves, as they did, with the shares of Anastacia, Sotero, Sulpicio and Rufina abutting the municipal road. Located directly behind the lots of Anastacia and Sotero is the share of their brother Antonio designated as Lot No. 1448-B-C which the latter divided into two (2) equal parts, now Lots Nos. 1448-B-6-A and 1448-B-6-B. The latter Lot is behind the property of Sotero, father of private respondent Yolanda Oliveros. Yolanda purchased Lot No. 1448-B-6-A from her uncle Antonio through her aunt Anastacia who was then acting as his administratrix. According to Yolanda, when petitioner offered her the property for sale she was hesitant to buy as it had no access to a public road. But Anastacia prevailed upon her to buy the lot with the assurance that she would give her a right of way on her adjoining property.

Thereafter, Yolanda constructed a house on the lot she bought using as her passageway to the public highway a portion of Anastacia's property. But when Yolanda finally offered to pay for the use of the pathway Anastacia refused to accept the payment. In fact she was thereafter barred by Anastacia from passing through her property. Later, Yolanda purchased the other lot of Antonio Quimen, Lot No. 1448-B-6-B, located directly behind the property of her parents who provided her a pathway between their house from the lot of Yolanda behind the sari sari store of Sotero, and Anastacia's perimeter fence. The store is made of strong materials and occupies the entire frontage of the lot measuring four (4) meters wide and nine meters (9) long. Although the pathway leads to the municipal road it is not adequate for ingress and egress. The municipal road cannot be reached with facility because the store itself obstructs the path so that one has to pass through the back entrance and the facade of the store to reach the road. Finally, Yolanda filed an action with the proper court praying for a right of way through Anastacia's property. The report was that the proposed right of way was at the extreme right of Anastacia's property facing the public highway, starting from the back of Sotero's sari-sari store and extending inward by one (1) meter to her property and turning left for about five (5) meters to avoid the store. TC dismissed Yolanda‘s complaint, but the CA reversed the decision declaring that she was entitled to a right of way on petitioner‘s property and that the way proposed by Yolanda would cause the least damage and detriment to the servient estate. ISSUE: Whether or not passing through the property of Yolanda's parents is more accessible to the public road than to make a detour to her property and cut down the avocado tree standing thereon. -- YES HELD: The conditions sine quo non for a valid grant of an easement of right of way are: (a) the dominant estate is surrounded by other immovables without an adequate outlet to a public highway; (b) the dominant estate is willing to pay the proper indemnity; (c) the isolation was not due to the acts of the dominant estate; and, (d) the right of way being claimed is at a point least prejudicial to the servient estate. The criterion of least prejudice to the servient estate must prevail over the criterion of shortest distance although this is a matter of judicial appreciation. While shortest distance may ordinarily imply least prejudice, it is not always so as when there are permanent structures obstructing the shortest distance; while on the other hand, the longest distance may be free of obstructions and the easiest or most convenient to pass through. In other words, where the easement may be established on any of several tenements surrounding the dominant estate, the one where the way is shortest and will cause the least damage should be chosen. However, as elsewhere stated, if these two (2) circumstances do not concur in a single tenement, the way which will cause the least damage should be used, even if it will not be the shortest. As between a right of way that would demolish a store of strong materials to provide egress to a public highway, and another right of way which although longer will only require an avocado tree to be cut down, the second alternative should be preferred. FXRL

Floro v. Llenado 244 SCRA 713 DOCTRINE: One may not claim a legal easement merely out of convenience. FACTS: ● Floro is the owner of the Floro Park Subdivision in Bulacan ● The subdivision has access roads from MacArthur Highway through road lot 4 ● Llenado is the owner of the Llenado Homes Subdivision, formerly known as Emmanuel Homes Subdivision prior to his purchase of the project. ● This subdivision is bounded on the south and separated from Floro Park Subdivision by Planas Creek. To its west lies a parcel of land owned by Marcial Ipapo ● Llenado Homes does not have any existing access to MacArthur Highway. However, a proposed access road traversing the property of Ipapo had been provided for in the subdivision plan of Emmanuel Homes which was approved by the HLURB. ● Llenado sought and was granted the (oral and provisional, as they were still drafting a formal contract) permission of Floro to use lots 4 and 5 of Floro Park Subdivision as a passage to and from MacArthur Highway. ● Several months later, Floro barricaded road lot 5 to prevent Llenado from further using the property on account of the damage done to the property due to the passage of heavy equipment. ● Llenado filed an easement claim with the RTC but was denied. ● The CA ruled in favor of Llenado and ordered Floro to remove the barricade. ISSUE: W/N Llenado can demand a compulsory easement of right of way over the existing roads of an adjacent subdivision instead of developing his subdivision's proposed access road as provided in his duly approved subdivision plan. -- NO HELD: A compulsory easement of right of way may be granted only upon the acquisitions of the conditions required by Articles 649 and 650 of the Civil Code. To wit, these conditions are: 1. That the dominant estate is surrounded by other immovables and has no adequate outlet to a public highway; 2. That proper indemnity has been paid 3. That the isolation was not due to acts of the proprietor of the dominant estate 4. That the right of way claimed is at a point least prejudicial to the servient estate and, in so far as consistent with this rule, where the distance from the dominant estate to a public highway may be the shortest. In this case, the elements are incomplete. The original subdivision development plan presented by Llenado indicates an existing and prior agreement which creates a right of way through the abandoned Ipapo ricefield. Ipapo had long agreed to these terms but Llenado apparently thought it too much work and cost to develop such road. It was easier for him to create an easement via the Floro property. Jurisprudence has dictated that one may not claim a legal easement merely out of convenience. It was convenience motivated Llenando to abandon the Ipapo access road development and pursue an access road through the Floro estate. He was stacking the cards in his favor to the unnecessary detriment of his neighbor.

RSDM Fransisco v. IAC 177 SCRA 527 DOCTRINE: The law makes it amply clear that an owner cannot, as respondent has done, by his own act isolate his property from a public highway and then claim an easement of way through an adjacent estate. FACTS: Ramos' Lot 860-A used to be a part of Lot 860 of the Malinta Estate. Lot 860 was owned by Cornelia and Frisca Dila, and had a frontage along Parada Road measuring 51.90 meters. Adjoining Lot 860 was Lot 226, owned by Eusebio Francisco, as aforestated; it also had a frontage along Parada Road of 62.10 meters. On December 3,1947, the co-owners of Lot 860 (Cornelia and Frisca Dila) executed a deed by which an undivided one-third portion of the land was donated to a niece, Epifania Dila, and another undivided one-third (1/3) portion to the children of a deceased sister, Anacleta Dila, and the remaining portion, also an undivided third, was declared to pertain exclusively to and would be retained by Cornelia Dila. Some months later, in March, 1972, after having set up a piggery on his newly acquired property, Ramos had his lawyer write to Eusebio Francisco — owner, as above mentioned, of the adjoining lot, Lot 266- to ask for a right of way through the latter's land. Negotiations thereafter had however failed to bring about a satisfactory arrangement. Francisco's proposal for an exchange of land at the rate of one (1) square meter from him to three (3) square meters from Ramos, as was supposedly the custom in the locality, was unacceptable to Ramos. Later that year, 1972, Ramos succeeded, through the intercession of Councilor Tongco of Valenzuela, in obtaining a three-meter wide passageway through Lot 860-B of Epifania Dila . Yet in August, 1973, he inexplicably put up a ten-foot high concrete wall on his lot, this was in August, 1973, and thereby closed the very right of way granted to him across Lot 860-B. It seems that what he wished was to have a right of passage precisely through Francisco's land, considering this to be more convenient to him, and he did not bother to keep quiet about his determination to bring suit, if necessary, to get what he wanted. Francisco learned of Ramos' intention and reacted by replacing the barbed-wire fence on his lot along Parada Road with a stone wall, also in August, 1973. Shortly thereafter, Francisco was served with summons and a copy of the complaint in Civil Case of the Court of First Instance of Bulacan, instituted by Ramos, as well as a writ of preliminary mandatory injunction directing him to remove his stone fence and keep his lot open for Ramos' use. The Court handed down its verdict, adversely to Francisco. Francisco appealed to the Court of Appeals. In its own decision, the latter affirmed the Trial Court's judgment. ISSUE: Whether or not Private Respondent was entitled to an easement of right of way through the land belonging to the Petitioner HELD: It is the fact already adverted to and which has never been disputed that respondent Ramos,

having already been granted access to the public road (Parada Road) through the other adjoining Lot 860-B owned by Epifania Dila — and this, at the time he was negotiating with petitioner for the similar easement over the latter's Lot 266 that he now claims — inexplicably gave up that right of access by walling off his property from the passageway thus established. The evidence, also uncontradicted, is that said passageway was 2.76 meters wide, or wide enough to accommodate a truck. The surveyor who at the instance of petitioner made a survey of the premises on September 13, 1973, shortly after Ramos had filed his complaint, verified the existence of said passageway from the presence of tire marks found on the scene and indicated on the sketch plan he prepared the path that it took from said respondent's Lot 860-A through Lot 860-B to Parada Road. That there was such a passageway was also confirmed by another witness, Parada Barrio Captain Fausto Francisco, one of those who had earlier tried to bring petitioner and respondent to an agreement about the proposed right of way through the property of the former. This witness declared, as already stated, that after the negotiations had been stalled by the failure of the parties to agree on the terms of a proposed land exchange that would have given Ramos access to Parada Road, said respondent had been able to obtain right of passage to the same public road over a 3-meter wide portion of Lot 860-B owned by Epifania Dila through the intercession of Councilor Tongco of Valenzuela. The evidence is, therefore, persuasively to the effect that the private respondent had been granted an adequate access to the public highway (Parada Road) through the adjacent estate of Epifania Dila even as he was trying to negotiate a satisfactory agreement with petitioner Francisco for another passageway through the latter's property. If at the time he filed suit against the petitioner, such access (through the property of Epifania Dila) could no longer be used, it was because he himself had closed it off by erecting a stone wall on his lot at the point where the passageway began for no reason to which the record can attest except to demonstrate the isolation of his property alleged in his complaint. But the law makes it amply clear that an owner cannot, as respondent has done, by his own act isolate his property from a public highway and then claim an easement of way through an adjacent estate. The third of the cited requisites: that the claimant of a right of way has not himself procured the isolation of his property had not been met indeed the respondent had actually brought about the contrary condition and thereby vitiated his claim to such an easement. It will not do to assert that use of the passageway through Lot 860-B was dffficult or inconvenient, the evidence being to the contrary and that it was wide enough to be traversable by even a truck, and also because it has been held that mere inconvenience attending the use of an existing right of way does not justify a claim for a similar easement in an alternative location. . MRAM Sta. Maria v. CA 285 SCRA 163 DOCTRINE: The requirements for an estate to be entitled to a compulsory servitude of right of way under the Civil Code are the ff.: 1. The dominant estate is surrounded by other immovables and has no adequate outlet to a public highway; 2. There is payment of proper indemnity; 3. The isolation is not due to the acts of the proprietor of the dominant estate; and 4. The right of way claimed is at the point least prejudicial to the servient estate; and insofar as consistent with this rule, where the distance from the dominant estate to a public highway may be the shortest .

FACTS: Respondent spouses Arsenio and Roslynn Fajardo are the registered owners of a piece of land (Lot 124), containing an area of 1,043 sq. m in Obando Bulacan. They acquired said lot under a Deed of Absolute Sale executed by the vendors Pedro M. Sanchez, et al. Lot 124 is surrounded by Lot 1, a fishpond, on the northeast; by Lot 126, owned by Florentino Cruz on the southeast; by Lot 6-A and a portion of Lot 6-B owned respectively by Sps Sta. Mr and Florcerfida Sta. Maria (petitioners); on the southwest,; and by Lot 122 owned by the Jacinto family on the northwest. The respondents filed a complaint against the defendants for the establishment of an easement of right of way, alleging that 1) Lot 124 is surrounded by properties belonging to other persons, 2) since they have no adequate outlet to the provincial road, an easement of a right of way passing through either of the alternative petitioners‘ properties which are directly abutting the provincial road would be plaintiffs‘ only convenient, direct and shortest access to and from the provincial road, 3) respondents‘ predecessors-in-interest have been passing through the properties of petitioners in going to and from their lot, 4) petitioners‘ mother even promised respondents‘ predecessors-in-interest to grant the latter an easement of right of way as she acknowledged the absence of an access from their property to the road, and 5) despite respondents‘ request for a right of way and referral of the dispute to the barangay officials, the petitioners refused to grant them an easement. ISSUE: Whether or not a compulsory right of way can be granted to the respondents who have two other existing passageways other than that of petitioners‘ and an alternative vacant lot fronting the provincial road also adjacent to plaintiff‘s property, which can be used in going to and from respondents‘ property? HELD: The Court held in the affirmative. The following are the requirements for an estate to be entitled to a compulsory servitude of right of way under the Civil Code, to wit: 1. The dominant estate is surrounded by other immovables and has no adequate outlet to a public highway (Art. 649, par. 1); 2. There is payment of proper indemnity (Art. 649, par. 1); 3. The isolation is not due to the acts of the proprietor of the dominant estate (Art. 649, last par.); and 4. The right of way claimed is at the point least prejudicial to the servient estate; and insofar as consistent with this rule, where the distance from the dominant estate to a public highway may be the shortest (Art. 650). Anent the first requisite, there is no dispute that the respondents‘ property is surrounded by other immovables owned by different individuals and that the property is without adequate outlet to a public highway. The second requisite is also present since, as established through testimony, respondents are willing to pay the corresponding damages provided for by law if granted the right of way. As for the third requisite, the isolation of their lot is not due to plaintiffs‘ acts. The property they purchased was already surrounded by other immovables leaving them no adequate ingress or egress to a public highway.

Lastly, as to the fourth requisite of ―least prejudice‖ and ―shortest distance,‖ the Court agreed with the lower court that this twin elements have been complied with in establishing the easement of right of way on defendants-appellants‘ properties. Among the 3 possible servient estates, it is clear that petitioners‘ property would afford the shortest distance from defendants‘ property to the provincial road. Moreover, it is the least prejudicial since as found by the lower court, ―it appears that there would be no significant structures to be injured in the defendants‘ property and the right-of-way to be constructed thereon would be the shortest of all the alternative routes pointed to by the defendants‖ FMM National Irrigation Authority v. CA 340 SCRA 661 DOCTRINE: When a land, originally public, is awarded to a private individual, a legal easement may be constituted and thus no just compensation is required. FACTS: On 28 June 1963, a free patent over three hectares of land situated in Barrio Baybayog, municipality of Alcala, province of Cagayan, was issued in the name of respondent‘s predecessor-in-interest, Vicente Manglapus and registered under a title in his name, subject to provisions including conditions on public easements and servitudes recognized and prescribed by law. Subsequently, Dick Manglapus (―Manglapus‖) acquired the lot from Vicente Manglapus by absolute sale. On 18 July 1974, the land was registered in Respondent‘s name under Transfer Certificate of Title No. T-26658 of the Register of Deeds for the Province of Cagayan. Sometime in 1982, the National Irrigation Authority (―NIA‖) entered into a contract with Villamar Development Construction. Under the contract, NIA was to construct canals in Amulung, Cagayan and Alcala, Cagayan. NIA then entered a portion of Manglapus' land and made diggings and fillings thereon.] On 14 March 1991, Manglapus filed with the Regional Trial Court (the ―RTC‖), Tuguegarao, Cagayan a complaint for damages against NIA. Manglapus alleged that NIA's diggings and fillings destroyed the agricultural use of his land and that no reasonable compensation was paid for its taking. Despite service of notice of the pretrial conference, NIA did not appear at the pretrial conference. On 3 December 1991, the trial court declared NIA in default and received Manglapus' evidence ex parte and on 23 December 1991, the RTC rendered a decision in favor of Manglapus. On 27 January 1992, NIA filed a motion to lift the order of default dated 3 December 1991, and to set aside the decision of 23 December 1991. On 3 June 1992, the RTC issued a resolution denying the motion for lack of merit. Thus on 17 July 1992, NIA filed a notice of appeal to the Court of Appeals (the ―CA‖). On 27 July 1992, the RTC gave due course to the appeal and ordered the transmission of the original records to the CA. Manglapus subsequently filed a motion for execution of judgment with the RTC which the NIA through the Solicitor General opposed. On 17 August 1992, the trial court declared that since the notice of appeal of NIA was given due course, the motion for

execution was "moot and academic." On 8 March 1994, the CA dismissed the appeal hence this petition. The NIA contended that the certificate of title covering the subject parcel of land contained a reservation granting the government a right of way over the land covered therein. ISSUE: Whether the NIA should pay Manglapus just compensation for the taking of a portion of his property for use as easement of a right of way. -- NO HELD: The Supreme Court held that Article 619 of the Civil Code provides that, "Easements are established either by law or by the will of the owners. The former are called legal and the latter voluntary easements." In the present case, we find and declare that a legal easement of a rightof-way exists in favor of the government. The land was originally public land, and awarded to respondent Manglapus by free patent. The ruling would be otherwise if the land were originally private property, in which case, just compensation must be paid for the taking of a part thereof for public use as an easement of a right of way. RGGM Abellana v. CA 208 SCRA 316 DOCTRINE: The use of a footpath or road may be apparent but it is not a continuous easement because its use is at intervals and depends upon the acts of man; A right of way is not acquirable by prescription. FACTS: Petitioners are owners of a parcel of land on the NW side of Nonoc Subdivision, Cebu. They sued to establish an easement of a right of way over a subdivision road, which they claim they‘ve acquired through prescription since their ancestors have been using these since time immemorial. They pray that the concrete wall surrounding the village be taken down to allow easy access to the public highway. RTC found for the petitioners. CA reversed, averring that road lots in subdivisions are private property and may only be used as a public highway once acquired by the government through donation, purchase or expropriation. ISSUE: Whether or not the easement of a right of way may be acquired by prescription?

HELD: No. Art. 620 of the Civil Code provides that only continuous and apparent easements may be acquired by prescription. The easement of a right of way cannot be considered continuous because its use is at intervals and is dependent on the acts of man. MCSS Encarnacion v. CA 195 SCRA 74 FACTS: The servient estate of respondent heirs are what stands between the dominant estate and the national road. When the servient estate was not yet enclosed with a concrete fence, persons going to the national highway just crossed the servient estate at no particular point. When a fence was constructed, a roadpath measuring 25 meters long and a meter wide was constituted to provide access to the highway, with one-half meter taken from the servient estate and another one-half from another lot. Petitioner's plant nursery business through sheer hard work flourished and he bought an ownerjeep which he could use for transporting his plants. However, the jeep could not pass through the roadpath. He offered the servient estate owners that they sell him one and one-half meters of their property to be added to the existing pathway. His request was turned down. During the trial, the attention of the lower court was called to the existence of another exit, a dried river bed, to the highway, only eighty meters away from the dominant estate. ISSUE: WON petitioner is entitled to a widening of an already existing easement of right-of-way. -- YES HELD: Just because the second egress is nearer to the highway by a difference of only 65 meters, are not amount the conditions specified by Article 649 of the Civil Code. While there is a dried river bed closer to the dominant tenement, that access is grossly inadequate. Generally, the right of way may be demanded:(1) when there is absolutely no access to a public highway, and (2) when, even if there is one, it is difficult or dangerous to use or is grossly insufficient. The river bed route is traversed by a semi-concrete bridge and there is no ingress nor egress from the highway. For the jeep to reach the level of the highway, it must literally jump four to five meters up. During the rainy season, the river bed is impassable due to the floods. Thus, it can only be used at certain times of the year. The river bed which make passage difficult, if nor impossible, it is if there were no outlet at all. There is a real and compelling need for such servitude in his favor. Article 651 provides that ―the width of the easement of right of way shall be that which is sufficient for the needs of the dominant estate, and may accordingly be changed from time to time.‖ It is the needs of the dominant property which ultimately determine the width of the passage, and these may vary from time to time. To force petitioner to leave his jeepney in the highway, exposed to the elements and to the risk of theft simply because it could not pass through the improvised pathway, is sheer pigheadedness on the part of the servient estate and can only be counter-productive for all the people concerned.

Petitioner should not be denied a passageway wide enough to accommodate his jeepney since that is a reasonable and necessary aspect of the plant nursery business. NKVS Vda. De Baltazar v. CA 245 SCRA 333 DOCTRINE: The owner of an estate may claim a compulsory right of way only after he has established the existence of four (4) requisites, namely, (1) the estate is surrounded by other immovables and is without adequate outlet to a public highway; (2) after payment of the proper indemnity; (3) the isolation was not due to the proprietor's own acts; and (4) the right of way claimed is at a point least prejudicial to the servient estate, and in so far as consistent with this rule, where the distance from the dominant estate to a public highway may be the shortest. FACTS: Daniel Panganiban is the owner of a parcel of residential land consisting of 117 square meters denominated as Lot no. 1027 located at Sta. Ines, Bulacan. Immediately to the front of said land is Lot 1026 of Loreto Vda. de Baltazar and her son Nestor Baltazar. Immediately behind is the Sta. Ana River. On either side are Lots 1025 and 1028 owned by Ricardo Calimon and Jose Legaspi, respectively. Braulio Street, a provincial road, runs along the frontage of Lots 1025, 1026 and 1028. Daniel Panganiban filed a complaint against the Baltazars who are owners of Lot 1026 for the establishment of a permanent and perpetual easement of right of way for him to have access to the provincial road. Petitioners opposed arguing that there exists two other rights of way adjacent to private respondent's property. Aside from the passageway which the plaintiff seeks to be established as a permanent easement, the property of the plaintiff is accessible to and from the provincial road via two (2) other passageways. RTC dismissed the case. The CA, however, found that the two passageways mentioned were mere temporary pathways, that the existence of the two passageways was not simultaneous and was granted by respondent's neighbors, Calimon and Legaspi only upon respondent's request when petitioner Baltazar closed the claimed passageway is supported by the evidence on record. In light of the above findings of the Court of Appeals, the underlying issue begging resolution is whether or not respondent Panganiban is entitled to claim an easement of right of way over the Baltazars' property. ISSUE: W/N Panganiban is entitled to the easement prayed for. -- YES HELD: By express provision of Articles 649 and 650 of the New Civil Code, the owner of an estate may claim a compulsory right of way only after he has established the existence of four (4) requisites, namely, (1) the estate is surrounded by other immovables and is without adequate outlet to a public highway; (2) after payment of the proper indemnity; (3) the isolation was not due to the proprietor's own acts; and (4) the right of way claimed is at a point least prejudicial to the servient estate, and in so far as consistent with this rule, where the distance from the dominant

estate to a public highway may be the shortest. It is not disputed that the first requisite has been established by the court a quo in its Order dated May 22, 1990. Respondent Panganiban's property is indeed surrounded by immovables on three sides and a river on the fourth. As for the second requisite, Francisco v. Intermediate Appellate Court states: There would indeed be some point in looking askance at a reading of the law which would impute to it a strict requirement to pay "proper indemnity" in advance of a suit the purpose of which, in addition to creating an easement, is precisely to fix the amount of the indemnity to be paid therefor. With respect to the third requisite, respondent Panganiban was likewise able to establish that the isolation of his property was not due to his own act for he merely bought Lot 1027. The respondents have been using as a right of way, has been "existing, recognized, acknowledged, tolerated and used by the appellant as a right of way for thirty (30) years during the lifetime of petitioner's grandfather, Fidel and his father, Onisimo Baltazar." It was also established that the right of way was "closed and obstructed by the petitioners when they closed the gate and placed plants across the gate of Lot 1026-B when petitioners constructed their present residence." As regards the fourth requirement, both parties agreed that the passage claimed by respondent as his right of way, compared to the other passageways, is the shortest distance from respondent's lot to Braulio Street. Petitioners could not have been inconvenienced by the passageway for, as borne out by the records, the same is separate and distinct from the gate used by them to enter their lot and residence. Such being the case, we conclude that respondent is entitled to claim a compulsory easement of right of way over petitioners' Lot 1026-B. AMPS David-Chan v. CA 268 SCRA 677 DOCTRINE: There are rigorous standards to be complied with by owners of the dominant estate before they may be granted with easement of right of way. These standards must be strictly complied with because easement is a burden on the property of another. Equity is not applied as statutory laws on the matter are existent. FACTS: Petitioner filed with the trial court an amended petition with prayer for preliminary prohibitory injunction, seeking to stop private respondent from fencing its property and depriving her of access to the highway. Petitioner alleged that her property was delineated on its northern and western sides by various business establishments. Adjoining her property along its southern boundary was the land of the Pineda family, while along the east-northeastern boundary, and lying between her property and the MacArthur Highway, was another lot with an area of approximately 161 square meters owned by private respondent. In short, petitioner's lot was almost completely surrounded by other immovables and cut off from the highway. Her only access to the highway was a very small opening measuring two feet four inches wide through the aforementioned property of

private respondent. Petitioner believed she was entitled to a wider compulsory easement of right of way through the said property of private respondent. The prospective subservient estate was a portion of a bigger lot, which was formerly owned by the Singian Brothers Corporation and was sold to private respondent without the knowledge and consent of petitioner, who was thereby allegedly prevented from exercising her right of pre-emption or right of redemption. The petition likewise prayed that judgment be rendered ordering private respondent to sell to petitioner the subject lot. Private respondent denied the allegations of petitioner. The parents and relatives of petitioner were never tenants or lessees of the former owner, Singian Brothers; rather, they were found to be illegally occupying the property as ruled by the MTC-San Fernando, Pampanga. As their affirmative and special defenses, Defendant Singian Brothers averred that the complaint of petitioner stated no cause of action because, being apparent and discontinuous, the right of way cannot be acquired by prescription. Petitioner was not a tenant of the Singian Brothers; therefore she was not entitled to a right of pre-emption or right of redemption. Finally, petitioner had another access to the National Highway which, however, she closed during the pendency of the case at the trial court when she extended the construction of her fence. ISSUES: 1. Is petitioner legally entitled to a right of way through private respondent's property? 2. Should traditional Filipino values as pakikisama be considered? HELD: 1. Not legally entitled to a right of way for failure to satisfy the requirements. Citing Articles 649 and 650 of the Civil Code, petitioner submits that "the owner of an estate may claim a compulsory right of way only after he (or she) has established the existence of four requisites, namely: (1) the estate is surrounded by other immovables and is without adequate out-let to a public highway; (2) proper indemnity is paid; (3) the isolation is not due to the proprietor's own acts; and (4) the right of way claimed is at a point least prejudicial to the servient estate and, insofar as consistent with this rule, where the distance from the dominant estate to a public highway may be the shortest. The Supreme Court upheld the factual findings of the lower courts thus: 1) Petitioner is not "without adequate outlet to a public highway". It was plaintiff who built a concrete fence on the southern boundary of her property to separate it from the property of the Pineda family. Worse, during the pendency of the case, she closed the 28-inch clearance which she could use as a means to reach the National Highway without passing through the property of defendant. 2) The appellate court likewise found that petitioner failed to satisfy the third requirement because she caused her own isolation by closing her access through the Pineda property. 3) Petitioner failed to prove she made a valid tender of the proper indemnity. 2. Equity should only be applied in the absence of statutory law or judicial rules of procedure.

Petitioner is not legally entitled to a right of way on the property of private respondent. Thus, such equitable arguments cannot prevail over the legal findings. There are rigorous standards to be complied with by owners of the dominant estate before they may be granted with easement of right of way. These standards must be strictly complied with because easement is a burden on the property of another. Before such inconvenience may be imposed by the Court, applicants must prove that they deserve judicial intervention on the basis of law, and certainly not when their isolation is caused by their own acts. In the latter case, they decide their detachment and must bear the consequences of such choice. KGS Almendras v. CA G.R. No. 110067 DOCTRINE: When the easement may be established on any several tenements surrounding the dominant estate, the one where the way is shortest and will cause the least damage should be chosen. FACTS: Almendras owns a land that is bounded on the north and on the east by lots owned by Eng and Yap, on the south by the lot owned by Bongo, and on the west by the properties of the Opones. On the western boundary of Almendras land abuts an existing private road, which passes through the several lots and leads to another private road (a permanent easement) located on the property of Tudtud, which in turn connects to the provincial road. Thereafter, Eng and Yap began building a concrete wall on his property on the northern and eastern sides of petitioner‘s lot. For this reason, Almendras wrote to them offering to buy a portion of their lot, so that he could have access to the provincial road. But her request was denied on the ground that there was an existing private road on the western side of Almendras‘ property providing adequate outlet to the provincial road. Eng and Yap claimed that granting petitioner‘s request would greatly reduce the value of his property, as the proposed right of way cuts across the middle of the property. Shortly thereafter, Bongo also fenced his property, thus closing off the southern boundary of petitioner‘s lot. As such, Almendras brought this action for the establishment of a right of way through Eng and Yap‘s land, which was granted by the trial court. Then, Opone subsequently closed off the western side of petitioner‘s property by erecting a fence on his lot, with the result that petitioner‘s property became inaccessible. ISSUE: Is Almendras entitled to a right of way through Eng and Yap‘s property? -- YES HELD: To begin with, the owner of a landlocked property has the right to demand a right of way through the neighboring estates. The easement must be established at the point which is least prejudicial to the servient estate and, whenever possible, the shortest to the highway. If these two conditions exist on different properties, the land where establishment of the easement will cause the least prejudice should be chosen. Thus, it has been held that ―where the easement may be established on any of several tenements surrounding the dominant estate, the one where the way is shortest and will cause the least damage should be chosen. However, . . . if these two (2) circumstances do not concur in a single tenement, the way which will cause the least damage should be used, even if it will not be the shortest.

In the case at bar, the trial court ruled that the easement should be constituted through the land of private respondents on the eastern side because it would be the shortest way to the provincial road, being only 17.45 meters long, compared to 149.22 meters if the easement was constituted on the Opone and Tudtud roads on the western and southern sides of petitioner‘s land. On the other hand, as already pointed out, the Court of Appeals, in pointing to the longer way, considered the fact that this was already existing and does not preclude its use by other parties than the individual owners of Lot 1-A to Lot 1-G and the owners of the land on which the connecting Tudtud road is found. The way may be longer and not the most direct way to the provincial road, but if the establishment of the easement in favor of petitioner on this roads will cause the least prejudice, then the easement should be constituted there. This seems to be reasoning of the Court of Appeals. However, this can only be determined if the several lot owners (i.e., the Opones and their buyers and those of Bienvenido Tudtud) are before the court, for the determination of the point least prejudicial to the owners of servient estates (if there are two or more possible sites for an easement) requires a comparative evaluation of the physical conditions of the estates. It is not possible to determine whether the estates which would be least prejudiced by the easement would be those of the owners of the Opone and Tudtud properties because they have not been heard. Although evidence concerning the condition of their estates has been presented by private respondents, it is impossible to determine with certainty which estate would be least prejudiced by the establishment of an easement for petitioner until these parties have been heard. Any decision holding them liable to bear the easement would not be binding on them since they are not parties to this action. The case was REMANDED to the RTC. JPOT Dionisio v. Ortiz 204 SCRA 745 DOCTRINE: Easement-Right of way is not applicable when the cause of its effectivity is the proprietary's own actuations. FACTS: Owners of lots are contiguous and adjacent each other leading to Howmart road and EDSA. Both parties consent to have a shared road by way of agreement involving Quezon City Industrial Estates (QCIE). This, in order to traverse Howmart road which is private and make known cooperation with fellow QCIE members having a shared interest. However, legal action ensued from the Regional Trial Court and Court of Appeals until January of 1990. The said internal agreement with QCIE only subsisted until December. Furthermore, said gate came into fruition because of respondents' willingness to subdivide their lot. Eventually, this would lead to unnecessary noise pollution and disturbances brought upon by the transportation business of the respondent. The gate in question leads into Dionisios' lot so they found it best to assemble a barricade. ISSUE: W/N the private respondents have an easement of right of way over Howmart Road -- NO

HELD: The court finds it untenable to appreciate the logic of an expired contract of agreement. Moreover, it finds it harder to take into account the claim of right of way when the land owner necessitated to avail said easement because of their own proprietary action. It is inadequate to make use of such a right when it is through one's own fault why there is a need for another gate for them to be able to access Howmart road. WHEREFORE, the petition is hereby GRANTED. The questioned decision of the Court of Appeals and the Order of the Regional Trial Court in Civil Case No. Q-89-3949 are SET ASIDE. The writ of preliminary injunction is hereby LIFTED. SO ORDERED. MLAV Cortes v. Yu-Tibo 2 Phil. 24 DOCTRINE: An easement of light and view is a negative easement. When easement is negative, there should be a formal act of opposition for prescription to run. FACTS: Cortes‘ wife owns a house (No. 65) in which certain windows open on the adjacent property (No. 63), a neighboring house on the same street. This setting has been in existent since 1843. The tenant of the adjacent property raised the roof of house No. 63 in such a manner that half of the windows of house No. 65 has been covered, thus depriving it of a large part of air and light. Plaintiff contends that by the constant and uninterrupted use of the windows for 59 years, he acquired from prescription an easement of light in favor of house No. 65, and as a servitude upon house No. 63. Consequently, he has acquired the right to restrain the making of any improvement in the latter house which may be prejudicial to the enjoyment of the easement. Further, he contends that the easement of light is positive; and that therefore the period of possession for the purposes of the acquisition of a prescriptive title is to begin from the date on which the enjoyment of the same commenced, or, in other words, from the time that said windows were opened with the knowledge of the owner of the house No. 63, and without opposition on this part. Defendant contends that the easement is negative, and therefore the time for the prescriptive acquisition must begin from the date on which the owner of the dominant estate may have prohibited, by a formal act, the owner of the servient estate from doing something which would be lawful but for the existence of the easement. Lower court ruled in favor of the defendant. Plaintiff appealed the case. ISSUE: WON the easement is positive/negative HELD: Easement is negative. The easement of light which is the object of this litigation is of a negative character, and therefore pertains to the class which cannot be acquired by prescription as provided by article 538 of the Civil Code, except by counting the time of possession from the date on which the

owner of the dominant estate has, in a formal manner, forbidden the owner of the servient estate to do an act which would be lawful were it not for the easement. In consequence, the plaintiff, not having executed any formal act of opposition to the right of the owner of house No. 63 to make improvements which might obstruct the light of house No. 65, at any time prior to the complaint, has not acquired, nor could he acquire by prescription, such easement of light, no matter how long a time have elapsed. Because the period which the law demands for such prescriptive acquisition could not have commenced to run, the act with which it must necessarily commence not having been performed. DJTV Purugganan v. Paredes 69 SCRA 69 DOCTRINE: In drainage or easement of receiving water falling from roofs, the distances prescribed in the Decree of Registration should not correspond to the width and length of the roof of the house but to the distance of the rainwater falling inside the land of the servient estate. FACTS: Plaintiff-appellee Emilio Purugganan is the owner of a piece of a residential lot subdivided as Lot 1 and Lot 2 (servient estate), adjacent to and bounded on the North by the lot of defendantappellant Felisa Paredes (dominant estate). The lots of the plaintiff-appellee are subject to an easement of drainage in favor of the defendants-appellants in the Decree of Registration of the Court of First Instance of Abra. Defendants-appellants constructed a house on their lot adjacent to Lots 1 and 2 of plaintiffappellee in such a manner that the southern side of their house is exactly on the brick wall, the southern side of which is the demarcation line between the plaintiff-appellee and the defendants-appellants, demolishing said brick wall and built thereon the southern wall of their house with 3 windows. The house constructed by the defendants-appellants is 2-½ meters longer than the length of roofing allowed in the Decree of Registration, and has an outer roofing (eaves) of 1.20 meters, protruding over the property of the plaintiff-appellee which is .20 meters wider than that allowed in the same Decree of Registration, and the rain water from the GI roofing falls about 3 meters inside Lots 1 and 2 of the plaintiff-appellee. Plaintiff filed a case prohibiting defendant from proceeding with the construction of the roof. Trial court, in a summary proceeding decided in favor of the plaintiff. CA affirmed trial court‘s decision. Hence this appeal. ISSUE: Whether or not the easement of drainage refers to the length of the roofing? -- NO HELD: The Supreme Court held that defendants-appellants have made a mistake in applying the distances prescribed in the Decree of Registration to the roofing of their house. They failed to comprehend the meaning of the phrase "servidumbre de vertiente de los tejados" constitutes on the land of plaintiff. Translated, it means the easement of receiving water falling from the roof which is an encumbrance imposed on the land of the plaintiff-appellee. Consequently, the distances prescribed in the Decree of Registration should not correspond to the width and length of the roof of the defendants-appellants' house but to the distance of the rain water falling inside the land of the plaintiff-appellee because the encumbrance is not the roof itself but the

rain water falling inside the property of the plaintiff-appellee. The summary judgment appealed from is affirmed with costs against defendants-appellants. JGY Valisno v. Adriano 161 SCRA 398 DOCTRINE: Water rights, such as the right to use a drainage ditch for irrigation purposes, which are appurtenant to a parcel of land, pass with the conveyance of the land, although not specifically mentioned in the conveyance FACTS: Plaintiff Valisno file against the Defendant Adriano an action for damages. The complaint alleged that the plaintiff is the absolute owner and actual possessor of a parcel of land in Nueva Ecija. Plaintiff bought the land from the defendant‘s sister, Honorata. Both parcels of land had been inherited by defendant and her sister from their father. At the time of the sale of the land to Valisno, the land was irrigated by water from the Pampanga River through a canal about 70 meters long, traversing the appellee's land. Adriano levelled a portion of the irrigation canal so that Valisno was deprived of the irrigation water and prevented from cultivating his 57-hectare land. Plaintiff Adriano filed in the Bureau of Public Works and Communications a complaint for deprivation of water rights. A decision was rendered in favor of the plaintiff. Defendant asked for a reinvestigation of the case and was granted. Meanwhile, plaintiff Valisno rebuilt the irrigation canal at his own expense because of urgency. He also filed a complaint for damages in the CFI against respondent. Defendant Adriano claims that he merely allowed his sister to use his water rights when she still owned the adjacent land. According to the appellant, the water right was the primary consideration for his purchase of Honorata's property, for without it the property would be unproductive. ISSUE: WON the water rights pass with the conveyance of the land. -- YES HELD: Water rights, such as the right to use a drainage ditch for irrigation purposes, which are appurtenant to a parcel of land, pass with the conveyance of the land, although not specifically mentioned in the conveyance. The purchaser's easement of necessity in a water ditch running across the grantor's land cannot be defeated even if the water is supplied by a third person. The fact that an easement by grant may also have qualified as an easement of necessity does detract from its permanency as property right, which survives the determination of the necessity. As an easement of waters in favor of the plaintiff has been established, he is entitled to enjoy it free from obstruction, disturbance or wrongful interference such as the appellee's act of levelling the irrigation canal to deprive him of the use of water from the Pampanga River. JRPA Trias v. Araneta 15 SCRA 241

DOCTRINE: Sellers of land may validly impose reasonable easements and restrictions as conditions for contracts of sales; the same may not be overturned by courts merely on the ground that it impacts dominical rights. FACTS: JM Tuason and Co. owned a piece of land that was part of a subdivision. Thru broker Araneta Inc (of Araneta Coliseum fame), this civic-minded company sold the land to Mr Lopez with the condition that said lot should never be used to erect a factory. This imposition was annotated to the TCT. A series of transfers and conveyances later, the lot ended up in the hands of the gorgeous Ms. Rafael Trias. She was dismayed with the annotation that stated ―5. That no factories be permitted in this section.‖ Ms. Trias felt that the annotation impaired her dominical rights and therefore illegal and existed as mere surplusage since existing zoning regulations already prevented the erection of factories in the vicinity. Worse, the annotation possibly hindered her plans to obtain a loan. She accordingly raised the issue to the court and received relief. Later on, Gregorio Araneta moved for reconsideration stating that the imposition resulted from a valid sales transaction between her predecessors in interest. He alleged that the court held no authority to overrule such valid easement and impaired the right to contract. ISSUE: Whether or not the imposition was valid. HELD: The imposition was valid. The prohibition is an easement validly imposed under art 594 which provides that ―every owner of a piece of land may establish easements he deems suitable xxx and not in contravention to the law, public policy and public order‖ The court ruled that the easement existed to safeguard the peace and quiet of neighboring residents. The intention is noble and the objectives benign. In the absence of a clash with public policy, the easement may not be eroded. The contention of surplusage is also immaterial. Zoning regulations may be repealed anytime, allowing the erection of factories. With the annotation, at the very least, the original intent to bar factories remains binding. ABB La Vista v. CA 278 SCRA 498 DOCTRINE: Like any other contractual stipulation, a voluntary easement cannot be extinguished except by voluntary recession of the contract establishing the servitude or renunciation by the owner of the dominant lots. FACTS: Mangyan road is a 15-meter wide road abutting Katipunan Avenue on the west, traverses the edges of La Vista Subdivision on the north and of the Ateneo de Manila University and Maryknoll College on the south. The said road was originally owned by the Tuasons who sold a portion of their land to Philippine Building Corporation. Included in such sale was half or 7.5

meters width of the Mangyan road. The said corporation assigned its rights, with the consent of the Tuasons, to Ateneo through a Deed of Assignment with Assumption of Mortgage. Ateneo later on sold to Maryknoll the western portion of the land. Tuason developed their land which is now known as La Vista. On January, 1976, Ateneo and La Vista acknowledged the voluntary easement or a mutual right of way wherein the parties would allow the other to use their half portion of the Mangyan road (La Vista to use Ateneo‘s 7.5 meters of the Mangyan road and also the other way around.) Ateneo auctioned off the property wherein Solid Homes Inc., the developer of Loyola Grand Villas, was the highest bidder. Ateneo transferred not only the property, but also the right to negotiate the easement on the road. However, La Vista did not want to recognize the easement thus they block the road using 6 cylindrical concrete and some guards over the entrance of the road blocking the entrance of the residents of Loyola Grand Villas. Solid Homes Inc. filed for injunction and La vista in turn filed a third party complaint against Ateneo. Some of the arguments of the petitioner were that Loyola residents had adequate outlet to a public highway using other roads and also that Ateneo has not yet finalized the negotiation of the easement. ISSUE: Whether or not there is an easement of right of way -- YES HELD: There was a voluntary easement of right of way which was acknowledged on January 1976 by the Tuasons and Ateneo. Being a voluntary easement, the four requisites for a compulsory easement need not be satisfied. Also, like any other contractual stipulation, the same cannot be extinguished except by voluntary recession of the contract establishing the servitude or renunciation by the owner of the dominant lots. In the case at bar, all the predecessors-ininterest of both parties recognized the existence of such easement and there was no agreement yet to revoke the same. The free ingress and egress along Mangyan Road created by the voluntary agreement is thus demandable.

NUISANCE (ART.694 TO 707) FZC Sitchon v. Aquino 98 Phil. 720 DOCTRINE: Houses constructed, with governmental authority, on public streets, and waterways obstruct at all times the free use by the public of said streets and waterways, and, accordingly, constitute nuisance per se, aside from public nuisances. As such, the summary removal thereof, without judicial process or proceedings may be authorized by the statute or municipal ordinance, despite the due process clause. FACTS: This decision stems from six (6) different suits. All of the petitioners implead Aquino (the City Engineer of Manila) as respondent so that he may be enjoined from causing the demolition of their respective houses situated in different areas along public streets in Manila inasmuch as these constitute public nuisances. All of the petitioners occupied the subject parcels of land initially entirely without consent. However, all of them subsequently paid concession fees or damages for the use of the land with the agreement that such payment and consent shall be without prejudice to an order to vacate. The time came when the City Engineer demanded that petitioners vacate the occupied streets. Unheeded, he threatened to demolish the houses. Petitioners contend that by virtue of arts. 700 and 702, the power to remove public nuisances is vested in the District Health Officer, not in the City Engineer. ISSUES: 1. Is there a public nuisance? 2. Does the City Engineer have authority to cause the abatement of the nuisance? HELD: There is a public nuisance. This case falls on article 694(4), classifying as a nuisance the obstruction of free passage of any public highway or street. It is public because it affects a community or neighbourhood. The constructions in fact constitute nuisances per se, obstructing at all times the streets. As such, the summary removal of these may be authorized by statute or ordinance. Aquino, as City Engineer, is vested with authority to effect the abatement of the nuisances through demolition. By virtue of the Revised Charter of Manila, such duty, among others, was placed upon him. Arts. 700 and 702 must yield to this provision not only because it is later law but also because of the principle that special provisions prevail over general ones. Moreover, an ordinance authorized the action sought to be taken by respondent. LNAC Velasco v. Manila Electric 40 SCRA 342 DOCTRINE: A noise may constitute an actionable nuisance but it must be a noise which affects injuriously the health or comfort of ordinary people in the vicinity to an unreasonable extent.

Injury to a particular person in a peculiar position or of specially sensitive characteristics will not render the noise an actionable nuisance. In the conditions of present living, noise seems inseparable from the conduct of many necessary occupations. Its presence is a nuisance in the popular sense in which that word is used, but in the absence of statute noise becomes actionable only when it passes the limits of reasonable adjustment to the conditions of the locality and of the needs of the maker to the needs of the listener. FACTS: Appellant Velasco bought from the People's Homesite and Housing Corporation three (3) adjoining lots. These lots are within an area zoned out as a "first residence" district by the City Council of Quezon City. Subsequently, the appellant sold two (2) lots to the Meralco, but retained the third lot, which was farthest from the street-corner, whereon he built his house. Appellee company started the construction of the sub-station in question and finished it the following November, without prior building permit or authority from the Public Service Commission. The facility reduces high voltage electricity to a current suitable for distribution to the company's consumers, numbering not less than 8,500 residential homes, over 300 commercial establishments and about 30 industries. The substation has a rated capacity of "2 transformers at 5000 Kva each or a total of 10,000 Kva without fan cooling; or 6250 Kva each or a total of 12,500 Kva with fan cooling". It was constructed at a distance of 10 to 20 meters from the appellant's house. The company built a stone and cement wall at the sides along the streets but along the side adjoining the appellant's property it put up a sawale wall but later changed it to an interlink wire fence. It is undisputed that a sound unceasingly emanates from the substation. Velasco contends that the sound constitutes an actionable nuisance under Article 694 of the Civil Code of the Philippines, reading as follows: A nuisance is any act, omission, establishment, business condition of property or anything else which: (1) Injures or endangers the health or safety of others; or (2) Annoys or offends the senses; XXX Subjection to the sound since 1954 had disturbed the concentration and sleep of said appellant, and impaired his health and lowered the value of his property. Wherefore, he sought a judicial decree for the abatement of the nuisance and asked that he be declared entitled to recover damages. Court dismissed the claim of the plaintiff. Plaintiff then appealed to this Court. ISSUE: Whether this sound constitutes an actionable nuisance. -- YES HELD: The general rule is that everyone is bound to bear the habitual or customary inconveniences that result from the proximity of others, and so long as this level is not surpassed, he may not

complain against them. But if the prejudice exceeds the inconveniences that such proximity habitually brings, the neighbor who causes such disturbance is held responsible for the resulting damage, being guilty of causing nuisance. While no previous adjudications on the specific issue have been made in the Philippines, our law of nuisances is of American origin, and a review of authorities clearly indicates the rule to be that the causing or maintenance of disturbing noise or sound may constitute an actionable nuisance. By jurisprudence, there can be no doubt but commercial and industrial activities which are lawful in themselves may become nuisances if they are so offensive to the senses that they render the enjoyment of life and property uncomfortable. It is no defense that skill and care have been exercised and the most improved methods and appliances employed to prevent such result. Of course, the creation of trifling annoyance and inconvenience does not constitute an actionable nuisance, and the locality and surroundings are of importance. The fact that the cause of the complaint must be substantial has often led to expressions in the opinions that to be a nuisance the noise must be deafening or loud or excessive and unreasonable. Usually it was shown to be of that character. The determining factor when noise alone is the cause of complaint is not its intensity or volume. It is that the noise is of such character as to produce actual physical discomfort and annoyance to a person of ordinary sensibilities, rendering adjacent property less comfortable and valuable. If the noise does that it can well be said to be substantial and unreasonable in degree; and reasonableness is a question of fact dependent upon all the circumstances and conditions. There can be no fixed standard as to what kind of noise constitutes a nuisance. It is true some witnesses in this case say they have been annoyed by the humming of these transformers, but that fact is not conclusive as to the nonexistence of the cause of complaint, the test being the effect which is had upon an ordinary person who is neither sensitive nor immune to the annoyance concerning which the complaint is made. In the absence of evidence that the complainant and his family are supersensitive to distracting noises, it is to be assumed that they are persons of ordinary and normal sensibilities. As can be anticipated, character and loudness of sound being of subjective appreciation in ordinary witnesses, not much help can be obtained from the testimonial evidence. That of plaintiff Velasco is too plainly biased and emotional to be of much value. The estimate of other witnesses on the point of inquiry are vague and imprecise, and fail to give a definite idea of the intensity of the sound complained of. TKDC Iloilo Cold Storage v. Municipal Council 24 Phil. 471 DOCTRINE: City Council cannot, by a mere resolution or motion, declare any particular thing a nuisance which has not theretofore been pronounced to be such by law, or so adjudged by judicial determination. FACTS: Plaintiff is the owner of an ice and cold storage plant. Nearby residents made complaints regarding the smoke that the plant emits saying that it was very injurious to their health and comfort. The defendant made investigations and later on passed a resolution which demands that the smokestacks of the said factory be elevated or else the factory operations will be closed or suspended. Plaintiff opposed by filing for injunction.

Iloilo Ice constructed an ice and cold storage plant in Iloilo City, after being granted authority by theMunicipal Council.After the plant was completed, residents who lived near the plant complained to the Council that smokecoming from the plant was injurious to their health and comfort.The Council formed a committee to investigate the complaints. The committee found that the complaintswere well founded, so the Council passed a resolution ordering Iloilo Ice to construct smokestacks; failureto comply with the same would force the municipal president to execute an order closing or suspendingthe operations of Iloilo Ice.Iloilo Ice commenced an action to enjoin the Council from carrying into effect the resolution, and apreliminary injunction was issued. They maintained that they were not obliged to comply with thedirective to elevate the smokestacks one hundred ft and that the Council threatened to require compliancewith the resolution without the intervention of the court.Council answered, denying the allegations in the complaint, asking that they be absolved from thecomplaint and that Iloilo Ice be declared to have no right to the remedy asked for. Iloilo Ice demurred to Council‘s answer. Demurrer was sustained so the Council appealed before the SC. ISSUE: Whether or not the resolution alone issued by the municipal council is sufficient to label and abate the supposed nuisance in this case? -- NO HELD: There are two kinds of nuisances: nuisances per se and per accidens. The former are recognized as nuisances under any and all circumstances. The latter are such only because of the special circumstances and conditions surrounding them. The former may be abated even by private individuals however the latter is different; it needs a determination of the facts which is a judicial function. The question of nuisance can conclusively be decided, for all legal uses, by the established courts of law or equity alone, and that the resolution of officers, or of boards organized by force of municipal charters, cannot, to any degree, control such decision. City Council cannot, by a mere resolution or motion, declare any particular thing a nuisance which has not theretofore been pronounced to be such by law, or so adjudged by judicial determination. In the present case it is certain that the ice factory of the plaintiff is not a nuisance per se. It is a legitimate industry, beneficial to the people and conducive to their health and comfort. The resolution is obviously not enough to abate the property of the plaintiff. AMD Hidalgo Enterprises v. Balandan 91 Phil. 488 DOCTRINE: Doctrine of Attractive Nuisance – One who maintains on his premises dangerous instrumentalities or appliances of a character likely to attract children in play, and who fails to exercise ordinary care to prevent children from playing therewith or resorting thereto, is liable to a child of tender years who is injured thereby, even if the child is technically a trespasser in the premises. The attractive nuisance doctrine, generally, is not applicable to bodies of water, artificial as well as natural, in the absence of some unusual condition or artificial feature other than the mere water and its location. FACTS: Petitioner Hidalgo Enterprises, Inc. was the owner of an ice-plant in San Pablo, Laguna.

Installed therein were two 9-feet-deep tanks full of water, with edges barely a foot high from the surface of the ground. While the compound was fenced, the tanks themselves were without any kind of fence or top cover. The plant had a wide gate entrance, usually left open for motor vehicles, customers, and anyone else to pass and enter the premises. There was no guard assigned on the gate. At about noon of April 16, 1948, plaintiff's son, Mario Balandan, an 8-year old boy, while playing with and in company of other boys of his age, entered the plant through the gate to take a bath in one of said tanks. While bathing, Mario sank to the bottom of the tank, only to be fished out later, already a cadaver, having died of "asphyxia secondary to drowning." Both the CFI of Laguna and the CA ruled in favor of Spouses Balandan. They both took the view that the petitioner maintained an attractive nuisance (the tanks), and neglected to adopt the necessary precautions to avoid accidents to persons entering its premises. It applied the doctrine of attractive nuisance, of American origin, recognized in this jurisdiction in Taylor vs. Manila Electric. The principle reason for the doctrine is that the condition or appliance in question, although its danger is apparent to those of age, is so enticing or alluring to children of tender years as to induce them to approach, get on or use it, and this attractiveness is an implied invitation to such children. ISSUE: W/N a water tank is an instrumentality or appliance considered as an attractive nuisance. -- NO HELD: The great majority of American decisions say no. There are numerous cases in which the attractive nuisance doctrine has not been held to be applicable to ponds or reservoirs, pools of water, streams, canals, dams, ditches, culverts, drains, cesspools or sewer pools. The reason why a reservoir of water is not considered an attractive nuisance was lucidly explained by the Indiana Appellate Court as follows: ―Nature has created streams, lakes and pools which attract children. Lurking in their waters is always the danger of drowning. Against this danger children are early instructed so that they are sufficiently presumed to know the danger; and if the owner of private property creates an artificial pool on his own property, merely duplicating the work of nature without adding any new danger… (he) is not liable because of having created an "attractive nuisance." Anderson vs. Reith-Riley Const. Co. Therefore, as petitioner's tanks are not classified as attractive nuisance, the question whether the petitioner had taken reasonable precautions becomes immaterial. And the other issue submitted by petitioner — that the parents of the boy were guilty of contributory negligence precluding recovery, because they left for Manila on that unlucky day leaving their son under the care of no responsible individual — needs no further discussion.

DONATION (ART. 725-772) CRF Republic v. Guzman 326 SCRA 90

DOCTRINE: There are three (3) essential elements of a donation: (a) the reduction of the patrimony of the donor; (b) the increase in the patrimony of the donee; and, (c) the intent to do an act of liberality or animus donandi. When applied to a donation of an immovable property, the law further requires that the donation be made in a public document and that there should be an acceptance thereof made in the same deed of donation or in a separate public document. In cases where the acceptance is made in a separate instrument, it is mandated that the donor should be notified thereof in an authentic form, to be noted in both instruments. FACTS: David Rey Guzman, a natural-born American citizen, is the son of the spouses Simerican citizen. In 1968 Simeon died leaving to his sole heirs Helen and David an estate consisting of several parcels of land located in Bulacan. On 29 December 1970 Helen and David executed a Deed of Extrajudicial Settlement of the Estate of Simeon Guzman dividing and adjudicating to themselves all the property belonging to the estate of Simeon. The taxes due thereon were paid and the parcels of land were accordingly registered in their names in undivided equal shares. On 10 December 1981 Helen executed a Quitclaim Deed assigning, transferring and conveying to her son David her undivided one-half (1/2) interest on all the parcels of land subject matter of the Deed of Extrajudicial Settlement of the Estate of Simeon Guzman. On 18 October 1989 David executed a Special Power of Attorney where he acknowledged thaeon Guzman, a naturalized American citizen, and Helen Meyers Guzman, an Amt he became the owner of the parcels of land subject of the Deed of Quitclaim executed by Helen on 9 August 1989 and empowering Atty. Lolita G. Abela to sell or otherwise dispose of the lots. On 1 February 1990 Atty. Lolita G. Abela, upon instruction of Helen, paid donor‘s taxes to facilitate the registry of the parcels of land in the name of David. The ownership of David was assailed and on the basis thereof, the Government filed before the Regional Trial Court of Malolos Bulacan a Petition for Escheat praying that one-half (1/2) of David's interest in each of the subject parcels of land be forfeited in its favor. On 9 August 1994 David Rey Guzman responded with a prayer that the petition be dismissed. Thus as a rule, only a Filipino citizen can acquire private lands in the Philippines. The only instances when a foreigner can acquire private lands in the Philippines are by hereditary succession and if he was formerly a natural-born Filipino citizen who lost his Philippine citizenship. Petitioner therefore contends that the acquisition of the parcels of land by David does not fall under any of these exceptions. It asserts that David being an American citizen could not validly acquire one-half (1/2) interest in each of the subject parcels of land by way of the two (2) deeds of quitclaim as they are in reality donations inter vivos. ISSUE:

Whether or not the quitclaim executed by Helen tantamount to a donation in favor of her son David? -- NO HELD: Not all the elements of a donation of an immovable property are present in the instant case. The language of the deed of quitclaim is clear that Helen merely contemplated a waiver of her rights, title and interest over the lands in favor of David, and not a donation. That a donation was far from Helen's mind is further supported by her deposition which indicated that she was aware that a donation of the parcels of land was not possible since Philippine law does not allow such an arrangement. There are three (3) essential elements of a donation: (a) the reduction of the patrimony of the donor; (b) the increase in the patrimony of the donee; and, (c) the intent to do an act of liberality or animus donandi. When applied to a donation of an immovable property, the law further requires that the donation be made in a public document and that there should be an acceptance thereof made in the same deed of donation or in a separate public document. In cases where the acceptance is made in a separate instrument, it is mandated that the donor should be notified thereof in an authentic form, to be noted in both instruments. Art. 633 of the Civil Code from whence Art. 749 came Manresa said: "If the acceptance does not appear in the same document, it must be made in another. Solemn words are not necessary; it is sufficient if it shows the intention to accept x x x x it is necessary that formal notice thereof be given to the donor, and the fact that due notice has been given must be noted in both instruments. Then and only then is the donation perfected. " Therefore, the provisions of the law not having been complied with, there was no effective conveyance of the parcels of land by way of donation inter vivos. There is no valid repudiation of inheritance as Helen had already accepted her share of the inheritance when she, together with David, executed a Deed of Extrajudicial Settlement of the Estate of Simeon Guzman on 29 December 1970 dividing and adjudicating between the two (2) of them all the property in Simeon‘s estate. By virtue of such extrajudicial settlement the parcels of land were registered in her and her son‘s name in undivided equal share and for eleven (11) years they possessed the lands in the concept of owner. MPF Heirs of Velasquez v. CA 325 SCRA 552 DOCTRINE: An action for partition will not lie if the claimant has no rightful interest over the subject property. A donation as a mode of acquiring ownership results in an effective transfer of title over the property from the donor to the donee and the donation is perfected from the moment the donor knows of the acceptance by the donee. And once a donation is accepted. the donee becomes the absolute owner of the property donated. FACTS: ● Spouses Leoncia de Guzman and Cornelio Aquino died intestate sometime in 1945 and 1947, respectively and were childless. ○ Leoncia de Guzman was survived by her sisters Anatalia de Guzman (mother of the plaintiffs) and Tranquilina deGuzman (grandmother of the defendants). During the existence of their marriage, spouses Aquino were able to acquire real properties. ● The plaintiffs alleged that Leoncia de Guzman, before her death, had a talk with 
 the plaintiffs‘ mother, Anatalia de Guzman, with plaintiff Santiago Meneses and 
 Tranquilina de Guzman and his son Cesario Velasquez in attendance; that in the


 conference Leoncia told Anatalia de Guzman, Tranquilina de Guzman and Cesario
 Velasquez that the documents of donation and partition which she and her husband 
 earlier executed were not signed by them as it was not their intention to give away all 
 the properties to Cesario Velasquez because Anatalia de Guzman who is one of her
 sisters had several children to support; Cesario Velasquez together with his mother 
 allegedly promised to divide the properties equally and to give the plaintiffs one-half
 (1/2) thereof; that they are entitled to ½ of each of all the properties in question being 
 the children of Anatalia de Guzman, full blood sister of Leoncia de Guzman. ● Plaintiffs also claim that after the death of Leoncia, defendants forcibly took 
 possession of all the properties and despite plaintiffs‘ repeated demands for partition,
 defendants refused. Plaintiffs pray for the nullity of any documents covering the
 properties in question since they do not bear the genuine signatures of the Aquino
 spouses, to order the partition of the properties between plaintiffs and defendants in
 equal shares and to order the defendants to render an accounting of the produce of the
 land in question from the time defendants forcibly took possession until partition shall 
 have been effected. ISSUE: Whether or not the action for partition should be sustained. -- NO HELD: No. In actions for partition, the court cannot properly issue an order to divide the
 property unless it first makes a determination as to the existence of co-ownership. The 
 court must initially settle the issue of ownership, the first stage in an action for partition. 
 Needless to state, an action for partition will not lie if the claimant has no rightful interest over the subject property. In fact, Section 1 of Rule 69 requires the party filing the action to state in his complaint the ―nature and the extent of his title‖ to the real estate. Until and unless the issue of ownership is definitely resolved, it would be premature to effect a partition of the properties. In this case, the properties sought to be partitioned by private respondents have 
 already been delivered to petitioners and therefore no longer part of the hereditary 
 estate which could be partitioned. After finding that no co-ownership exists between private respondents and petitioners, the court found no reason to discuss the other arguments raised by the petitioners in support of their petition. AMDG Aluad v. Aluad G.R. No. 176943 DOCTRINE: The donation being mortis causa, the formalities of a will must be observed. Absent that, the donation is void. The following are the characteristics of a donation mortis causa: 1. It conveys no title or ownership to the transferee before the death of the transferor; or what amounts to the same thing, that the transferor should retain the ownership (full or naked) and control of the property while alive; 2. That before the death of the transferor, the transfer should be revocable by the transferor at will, ad nutum; but revocability may be provided for indirectly by means of a reserved power in the donor to dispose of the properties conveyed; and

3. That the transfer should be void if the transferor should survive the transferee.

FACTS: Matilde and Crispin Aluad raised Maria and Zenaido Aluad (respondent). Crispin was the owner of Lot Nos. 674, 675, 676, 677, 680 and 682 and when he died Matilde adjudicated the said lands. Even before TCTs were issued to Matilde, she executed a Deed of Donation of Real Property Inter Vivos over the six lots in favor of Maria. The deed of donation will become effective upon the death of Matilde and until then Matilde may use, encumber or dispose any of the six lands. The deed of donation also mentioner that if Maria dies first, the deed of donation will be automatically be rescinded. After the TCTs were issued, Matilde sold Lot No. 676 to Zenaido and later on executed a last will and testament wherein Lot Nos., 675, 677, 682 and 680 were given to Maria while Lot No. 674 to Zenaido. It is noted that both Matilde and Maria are now deceased. The heirs of Maria (petitioners) filed an action to recover Lot No. 674 and 676 from Zenaido. In his answer, one of the contentions of Zeniado was that the deed of donation does not exist and that if it did, it was already revoked by Matilde. The trial court ruled on favor of the heirs however, the decision was reversed by the Court of Appeals in so far as Lot No. 676 is concerned. The Court of Appeals ruled that Deed of Donation that was executed was not Inter Vivos but one of Mortis Causa. Being a Donation Mortis Causa, its formalities under Article 805 were not complied since the donation was only witnessed by two persons and it had no attestation clause. Lot No. 674 cannot be awarded to Zenaido because the last will and testament was not probated. It is noted that Lot No. 674 was not awarded to the heirs. ISSUES: 1. Whether or not the Deed of Donation is one of Inter Vivos or Mortis Causa? -- MORTIS CAUSA 2. Whether or not Matilde (DONOR) could still sell to Zenaido Lot. 676 even after a deed of donation has been constructed? -- YES HELD: The SC agrees to the appellate court that the donation present in this case is a donation mortis causa. The deed of donation expressly stated that the donation will only take effect after the death of the Donor; it is clear that Matilde never intended to transfer the ownership of the lots to Maria during the lifetime of the former. To further support this, the deed also made an express mention that Matilde reserved the right to dispose the properties. Also, even after the deed of donation was constituted, Matilde still paid for the taxes, applied for free patents and the lots remained under her name. Having established that it is a donation mortis causa, it is important that the formalities under Article 805 must be followed. In this case, it was established that the donation was only witnessed by two persons and they only signed the attestation clause which is a separate requirement from signing the left pages of the will. The court emphasizes the distinction between these two signature requirements: x x x Article 805 particularly segregates the requirement that the instrumental witnesses sign each page of the will from the requisite that the will be ―attested and subscribed by [the instrumental witnesses]. The respective intents behind these two classes of signature[s] are distinct from each other. The signatures on the left-hand corner of every

page signify, among others, that the witnesses are aware that the page they are signing forms part of the will. On the other hand, the signatures to the attestation clause establish that the witnesses are referring to the statements contained in the attestation clause itself. Indeed, the attestation clause is separate and apart from the disposition of the will. An unsigned attestation clause results in an unattested will. Even if the instrumental witnesses signed the left-hand margin of the page containing the unsigned attestation clause, such signatures cannot demonstrate these witnesses‘ undertakings in the clause, since the signatures that do appear on the page were directed towards a wholly different avowal. Since the formalities were not observed, the donation mortis causa is void and transmitted no right to Maria. Even if they were observed, the deed was not probated hence, it still cannot be transmitted to her. Therefore, Matilde validly sold and transmitted Lot Nos. 674 and 676 to Zenaido with the condition that the will must be probated. GCG Danguilan v. IAC 168 SCRA 22 DOCTRINE: In order to determine whether or not said donation is valid and effective it should be sufficient to demonstrate that, as a contract, it embraces the conditions the law requires and is valid and effective, although not recorded in a public instrument. Donation of real property should be in a public instrument. FACTS: The respondent filed a complaint against the petitioner in the then Court of First Instance of Cagayan for recovery of a farm lot and a residential lot. She claimed that she had purchased from Domingo Melad the land now being unlawfully withheld by the defendant. In his answer, the petitioner denied the allegation and averred that he was the owner of the said lots of which he had been in open, continuous and adverse possession, having acquired them from Domingo Melad. The case was dismissed for failure to prosecute but was refilled. Danguilan presented three other witnesses to corroborate his statements and to prove that he had been living in the land since his marriage to Isidra and had remained in possession thereof after Domingo Melad's death. Two of said witnesses declared that neither the plaintiff nor her mother lived in the land with Domingo Melad. The trial court ruled based mainly on the issue of possession. Weighing the evidence presented by the parties, the judge held that the defendant was more believable and that the plaintiff's evidence was "unpersuasive and unconvincing." It was held that the plaintiff's own declaration that she moved out of the property and left it in the possession of the defendant was contradictory to her claim of ownership. She was also inconsistent when she testified first that the defendant was her tenant and later in rebuttal that he was her administrator. The decision concluded that where there was doubt as to the ownership of the property, the presumption was in favor of the one actually occupying the same, which in this case was the defendant. The appellate court merely affirmed the factual findings of the trial court except for an irrelevant modification, and it was only toward the end that it went to and resolved what it considered the lone decisive issues.

ISSUE: Whether or not there was a valid donation. -- NO

HELD: The donation being of real property, it is void for not complying with the requirements given by law. Donation of real property should be in a public instrument. In this case, it wasn‘t. The Supreme Court ruled, considering the language of the two instruments, that Domingo Melad did intend to donate the properties to the petitioner, as the private respondent contends. We do not think, however, that the donee was moved by pure liberality. While truly donations, the conveyances were onerous donations as the properties were given to the petitioner in exchange for his obligation to take care of the donee for the rest of his life and provide for his burial. The case at bar comes squarely under the doctrine laid down in Manalo v. De Mesa where the Court held: There can be no doubt that the donation in question was made for a valuable consideration, since the donors made it conditional upon the donees' bearing the expenses that might be occasioned by the death and burial of the donor Placida Manalo, a condition and obligation which the donee Gregorio de Mesa carried out in his own behalf and for his wife Leoncia Manalo; therefore, in order to determine whether or not said donation is valid and effective it should be sufficient to demonstrate that, as a contract, it embraces the conditions the law requires and is valid and effective, although not recorded in a public instrument. The deed of sale was allegedly executed when the respondent was only three years old and the consideration was supposedly paid by her mother, Maria Yedan from her earnings as a wage worker in a factory. This was itself a suspicious circumstance, one may well wonder why the transfer was not made to the mother herself, who was after all the one paying for the lands. The sale was made out in favor of Apolonia Melad although she had been using the surname Yedan her mother's surname, before that instrument was signed and in fact even after she got married. The averment was also made that the contract was simulated and prepared after Domingo Melad's death. It was also alleged that even after the supposed execution of the said contract, the respondent considered Domingo Melad the owner of the properties and that she had never occupied the same. VCL IV Aldaba v. CA 27 SCRA 263 DOCTRINE: A letter showing an intention to donate is not sufficient to prove donation; and most certainly not the form required by law in donations. FACTS: Two lots owned by Belen Aldaba are being disputed in this case. Petitoners Dr. Vicente Aldaba and Jane Aldaba, father and daughter, lived with Belen Aldaba for 10 years and took care of her until her death. Belen had presumptive heirs her surviving husband Estanislao Bautista, and her brother Cesar Aldaba (represented as the respondents in this case.) After the death of Belen,

the respondents asked the petitioners to leave the premises and upon their refusal, the former instituted an ejectment case. The petitioners argue that Belen really intended to donate the property to them as evidence by the note written by Belen to them which reads; June 18, 1953 Jane, Huag kayong umalis diyan. Talagang iyan ay para sa inyo. Alam nila na iyan ay sa inyo. Belen A. Bautista. They also argue that the property was for compensation of their services which amounted to P53,000. The respondents contend that the letter no way proves a donation. TC ruled that Estanislao Bautitsta is the absolute owner of the property in question. CA affirmed TC‘s ruling. ISSUE: Whether or not there was a disposition of property by Belen in favor of the petitioners -- NO HELD: For the following reasons: (1) The note was insufficient conveyance, and hence could not be considered as evidence of a donation with onerous caus. The note can be considered, at most, as indicative of the intention to donate. (2) no notarial document was executed by Belen to the petitioners during those 10 years. (3) P53,000 worth of services made by the petitioners no way proves the alleged donation. If at all, the petitioners believed that the gratuitous use of the property was not sufficient to compensate them for their services, they could have presented their claims in the intestate proceedings, which they themselves could have initiated, if none was instituted. The SC emphasized that there was no express agreement between the parties and that respondents Jane did not even expect to be compensated. FXRL Jutic v. CA 153 SCRA 269 DOCTRINE: An affidavit is an inadequate means of transferring title by donation. FACTS: ● The petitioners are the heirs Melquiades Seville, sibling of Arsenio Seville. ● Prior to his death, Arsenio executed an affidavit stating that he was the owner of two parcels of land and that upon his death, he desires to donate these parcels of land in favor of his sibling, Melquiades. ● With no spouse nor children, it was the siblings of Arsenio who served to be the heirs of his estate including the two lots allegedly donated in the affidavit. ● It is contended by the heirs of Melquiades that Arsenio designated the former to be the sole inheritor of the two parcels of land in question on the basis of the affidavit executed in favor of their father. ● The private respondents are his other siblings seeking to assert their rights over the said properties.

ISSUE: W/N there was a valid donation from Arsenio to Melquiades Seville. HELD: No. The affidavit is inadequate to render the donation valid. A close reading reveals that Exhibit 4 is not a donation inter vivos or mortis causa but a mere declaration of an intention and a desire. It is not a concrete and formal act of giving or donating. Further, the petitioners did not even assert any actions of ownership over the property in question. This is illustrated by the fact that the property was mortgaged by Arsenio with the knowledge of the Melquiades shows that ownership has not yet transferred. Additionally, when Arsenio died, payments to the loan for which the property was mortgaged stopped and were not continued by the petitioners. Consequently, the property was even foreclosed but was later on redeemed by Zoilo, one of the brothers of Arsenio who is also among the respondents. Petitioners have a rightful claim over the property based on the fact that they are heirs of Arsenio and not due to the alleged affidavit executed in favor of their father, Melquiades. It is also notable that the signed affidavit is a forgery because Arsenio Sevile was illiterate during his lifetime. The man was unable to write his name and only affixed his thumbmark in the aforementioned real estate mortgage. RSDM Howard v. Padilla 96 Phil. 983 DOCTRINE: Donations must be in accordance with the law.. FACTS: The property that was donasted is a conjugal in nature. The CA ruled that the donation was inter vivos, not mortis causa. It is valid and irrevocable. It is valid, however, only up to the extent of the share of the donor in the property. ISSUE: Is the donation inter vivos or mortis causa? HELD: The donation is mortis causa which takes effect upon the death of the donor. Therefore, the donation not having conformed with the formalities of the law, the same is void. MRAM Puig v. Penaflorida 16 SCRA 136 DOCTRINE: The reservation by the donor of the right to dispose of the property during her lifetime in the deed does not indicate that title had passed to the donee in her lifetime but that the donor merely reserves power to destroy the donation at any time. FACTS: On April 10, 1953, Carmen Ubalde Vda. de Parcon died in the City of Iloilo, without forced heirs,

leaving certain properties in the City and province of Iloilo. She left a will and was survived by nephews and nieces, children of her predeceased brother, Catalino Ubalde, and sister, Luisa Ubalde, married to Ariston Magbanua. It also appears that besides her will, the deceased had executed two notarial deeds of donation. One, entitled Donacion Mortis Causa, was executed in favor of her niece, Estela Magbanua, married to Mariano Peñaflorida, purporting to convey to the donee the properties covered by TCT Nos. 2338 and 18951 of the Registry of Deeds of Iloilo. Two, the deceased executed another deed of donation, also entitled "Escritura de Donacion Mortis Causa" in favor of the same donee, Estela Magbanua Peñaflorida, conveying to her three parcels of land covered by TCT Nos. 925, 927 and 11042 of the Register of Deeds of Iloilo . There was a condition in the instrument saying that ―if at the date of her death the donor had not transferred, sold, or conveyed one-half of lot 58 of the Pototan Cadastre to other persons or entities, the donee would be bound to pay to Caridad Ubalde, married to Tomas Pedrola, the amount of P600.00, and such payment was to be made on the date the donee took possession of Lot No. 58.‖ ISSUES: Are the two donations inter vivos or mortis causa? (It being admitted that in the latter event the donations are void for not being executed with testamentary formalities.) HELD: The Court held that the first donation is a valid donation inter vivos while the second one is a donation mortis causa. An essential characteristic of dispositions mortis causa is that the conveyance or alienation should be (expressly or by necessary implication) revocable ad nutum, i.e., at the discretion of the grantor or so-called "donor," simply because the latter has changed his mind. In consequence, the specification in a deed of the causes whereby the act may be revoked by the donor indicates that the donation is inter vivos, rather than a disposition mortis causa The Court further said that the designation of the donation as mortis causa, or a provision in the deed to the effect that the donation is "to take effect at the death of the donor" are not controlling criteria ; such statements are to be construed together with the rest of the instrument, in order to give effect to the real intent of the transferor. In case of doubt, the conveyance should be deemed donation inter vivos rather than mortis causa, in order to avoid uncertainty as to the ownership of the property subject of the deed. Tested by the foregoing principles, the donation of November 24, 1948, while somewhat ambiguous, should be held inter vivos in character. Admittedly, it is designated as "mortis causa," and specifies that it will take effect upon the death of the donor; but, as previously stated, these expressions are not controlling, and, in the instance, before us, are contradicted by other provisions indicating a contrary intent. Thus, ●

The conveyance of the properties described in the deed appears made in consideration of the undertaking of the donee, Estela Magbanua, to bear "all expenses for medical treatment, hospital expenses and/or burial of the Donor," without limiting the time when such expenses are to be incurred. In fact, the use of the words "y/o entierro" (and/or burial) strongly suggests that the illness and hospitalization expenditures to be borne by







the donee may or may not be connected with the donor's last illness. Emphasizing the onerous character of the transaction is the requirement that if the donee should predecease the donor, Governor Peñaflorida shall assume the obligations of the donee, "especialmente" (but not exclusively) "los gastos de ultima enfermedad y entierro de la donante" (par. 4), and this undertaking was assumed even if the properties donated would not go to Peñaflorida but to the donee's children and descendants (par. 3). It was evidently because of this liability, unconnected with the conveyance, that Peñaflorida had to sign the document together with the donee. It is easy to see that unless the conveyance were to be effective before the death of donor,, the obligations assumed by the donee and Governor Peñaflorida (her husband) would be without consideration (causa). Such conditions (consent, subject matter, causa or consideration and observance of the formalities or solemnities required by law) are all present in the deed of November 24, 1948. Again, while there is a clause that the donor reserved her right "to mortgage or even sell the donated property, when and if she should need funds to meet her own needs," this last sentence of the stipulation appears incompatible with the grantor's freedom to revoke a true conveyance mortis causa, a faculty that is essentially absolute and discretionary, whether its purpose should be to supply her needs or to make a profit, or have no other reason than a change of volition on the part of the grantor-testator. If the late Carmen Ubalde Vda. de Parcon wished or intended to retain the right to change the destination of her property at her sole will and discretion, there was no reason for her to specify the causes for which she could sell or encumber the property covered by her bounty. It is no objection to our view that the donation of November 24, 1948 should provide that it is not to be recorded until after the donor's death (par. 5), since the absence of registration would affect only subsequent purchasers, without denying the validity and obligatory effects of the conveyance as between the parties thereto.

As for the second deed of donation, the text thereof is clear that no proprietary right was intended to pass to the alleged "donee" prior to the "donor's" death, and that the same was a true conveyance mortis causa, which by law is invalid because it was not executed with the testamentary formalities required by the statutes in force at the time. Here, unlike in the previous donation the designation is donation mortis causa is confirmed by the fact that no signs contradict or limit the unqualified and unrestricted right of the donor to alienate the conveyed properties in favor of other persons of her choice at any time that she should wish to do so; it is so expressed in the deed, and it indirectly recognizes the donor's power to nullify the conveyance to the alleged "donee" whatever the "donor" wished to do so, for any reason or for no particular reason at all. As we have seen, this faculty is characteristic of conveyances post mortem or mortis causa: for the right of the transfer or to alienate the "donated" property to someone else necessarily imports that the conveyance to the "donee" will not become final and definite in favor of the latter until the death of the "donor" should exclude every possibility that the property maybe alienated to some other person. FMM Puig v. Penaflorida 294 SCRA 183 DOCTRINE: The real nature of a deed is to be ascertained by both its language and the intention of the parties as demonstrated by the circumstances attendant upon its execution. FACTS:

On 11 December 1979, a deed entitled ―Deed of Donation Inter Vivos‖ was executed by the late Aurora Virto Vda. De Montinola (―Montinola‖) in favor of her grandchildren, Catalino, Judy Cristina and Jesus Antonio, all surnamed Valderrama (the ―Valderramas‖) over a property located in Panay, Capiz covered by Transfer Certificate of Title (TCT) No. T-16105. The deed contained signature of the Valderramas in acknowledgment of their acceptance of the donation. Montinola‘s secretary, thereafter, presented the deed for recording in the Property Registry and the Register of Deeds cancelled TCT No. T-16105, the title under Montinola‘s name, and replaced it with TCT No. T-16622 in the name of the Valderramas. However, Montinola retained TCT No. T-16622 as well as the property until she transferred the same ten years later, on 10 July 1990, to Spouses Ernesto and Evelyn Sicad (the ―Sicads‖). On 12 March 1987, Montinola drew up a deed of revocation of the donation and caused it to be annotated as an adverse claim to TCT No. T-16622. On 24 August 1990, she then filed a petition with the Regional Trial Court (RTC) of Roxas City for the cancellation of TCT No. T-16622 and the reinstatement of TCT No. T-16105. Her petition was founded on the theory that the donation she executed was one of mortis causa which had to comply with the formalities of a will and since it had not, the donation was void and cannot be the basis for the cancellation of TCT No. T-16105 and the issuance of TCT No. T16622. In an opposition dated 29 August 1990, the Valderramas argued that the donation was one inter vivos which, having complied with the requirements set out in the Civil Code, was perfectly valid and efficacious. On 27 March 1991, the RTC rendered judgment holding that the donation was one inte rvivos and dismissed Montinola‘s petition for lack of merit. Montinola elevated the case to the Court of Appeals (―CA‖). She however died on 10 March 1993 while the case was pending. Thereafter, on 31 March 1993, a ―Manifestation and Motion‖ was filed by the Sicads in which they alleged that pursuant to a ―Deed of Definite Sale‖ dated 25 May 1992, they had become owners of the property covered by TCT No. T-16622. On 30 June 1995, the CA affirmed the RTC‘s decision. Hence, this petition to the Supreme Court (―SC‖). The Sicads contended that lower courts erred in their judgment in ruling the donation as one inter vivos because the circumstances surrounding the execution of the deed of donation, and the subsequent actions of Montinola incontrovertibly signify the latter‘s intent to transfer the property only after her death (mortis causa) and that Montinola did not intend to give effect to the donation. ISSUE: Whether the donation is mortis causa or inter vivos HELD: The donation is mortis causa. The SC held that the real nature of a deed is to be ascertained by both its language and the

intention of the parties as demonstrated by the circumstances attendant upon its execution. The SC found the following circumstances signifies that Montinola never intended the donation to take effect within her lifetime: (a) she expressed her wish that the donation take effect 10 years after her death; (b) she intercalated a new provision which states that ―however, the donees shall not sell or encumber the properties herein donated within 10 years after the death of the donor.‖; (c) she continued, as explicitly authorized in the deed itself, to possess he property, enjoy its fruits and otherwise exercise the rights of dominion, paying the property taxes as they fell due. All these she did until she transferred the property to the Sicad Spouses on July 10, 1990. The SC set aside the decisions of the lower courts. RGGM David v. Sison 76 Phil. 118 DOCTRINE: When the donor maintains the essential rights of ownership over the property during his lifetime, the donation is mortis causa. FACTS: The Administrator of deceased Ms. David was ordered by the court to pay the lawyer of deceased the amount of P18,000 for legal services rendered. The lawyer, on the other hand, wanted about P81,000. which was equivalent to 5% of the inventoried estate. Administrator thought that too high and said he‘d be happy to pay P3,000. Both parties based their claims on the central issue of the donation by Ms. David. If the donation was mortis causa, then the donation should be inventoried with the estate and therefore the lawyer can get his lofty demand. Should the donation have been intervivos, then the donation wouldn‘t be inventoried and therefore the administrator can pay the lower sum contended. The lower court analyzed a few paragraphs of the deed (in tagalog) and concluded that the deed partook of the nature mortis causa. "Na ang naturang "donor," Margarita David y Puato, alang-alang sa malaki niyang pagtiñgin, pagliñgap at pagmamahal sa mga nabanguit na "donees" Narcisa de la Fuente at Priscila de la Fuente, sa pamamagitan nang kasulatang ito, malayang ibinigay at ipinagkakaloob sa mga naturang Narcisa de la Fuente at Priscila de la Fuente, at sa kanilang mga tagapagmana, "albacea" at "Administradores", sa habang panahon, ang kanyang mga titulo, interes at participacion sa mag sumusunod na ari-arian na pawang malines sa lahat nang mga pananagutan: (Rec. on Appeal, pp. 209, 210.) Datapwa't ang lahat nang mga tubo at pakinabangan nang nagbibigay o "donor" na si Margarita David y Puato hanggang siya ay hindi binabawian nang buhay nang maykapal; at ang mga pinagbibigyan na si Narcisa de la Fuente at Priscila de la Fuente ay hindi maaaring maipagbili, maisangal, a maipagpalit o sa ano pa man paraan, kung walang kaalaman at pahintulot nang naturang Margarita David y Puato. ISSUE: Whether or not the Deed of Donation is Mortis Causa -- YES

HELD: The deed of donation makes it clear that all rents, proceeds, fruits, of the donated properties shall remain for the exclusive benefit and disposal of the donor, Margartia David, during her lifetime and that, without the knowledge consent of the donor, the donated properties could not be disposed of in any way, whether by sale, mortgage, barter, or in any other way possible, thus making the donees just as paper owners of the properties. The court then concluded that the donation in question is a donation mortis causa, because the combined effect of the circumstances surrounding the execution of the deed of donation and of the above-quoted clauses thereof could not have taken effect before the death of Margarita David. According to the terms of the deed, the most essential elements of ownership — the right to dispose of the donated properties and the right to enjoy the products, profits, possession — remained with Margarita David during her lifetime, and would accrue to the donees only after Margarita David's death. While the donation in question is a donation mortis causa, the court declined to rule that the donated properties should be included in the inventory of the estate and should follow the same proceedings as if they were not donated at all. The court then awarded the lawyer P10,000 instead of the P81,000 he craved. MCSS Maglasang v. Heirs of Cabatingan 383 SCRA 6 FACTS: Conchita Cabatingan executed in favor of her borhter, Nicolas, a ―Deed of Conditional Donation Inter Vivos for House and Lot‖, covering two parcels of land. The document stated that, ―in consideration of the love and affection of the DONOR‖, Conchita transfers, conveys, by way of donation the properties. However, the same was bound by the condition that, ―in the even that the DONEE should die before DONOR, the present donation shall be deemed automatically rescinded‖. Respondent heirs filed an action to declare the document void for failing to comply with the formalities of wills and testaments, considering donations mortis causa. ISSUE: WON the donations made are inter vivos or mortis causa. -- NO HELD: In a donation mortis causa, ―the right of disposition is not transferred to the donee while the donor is still alive. In determining whether a donation is one of mortis causa, the following characteristics must be take into account: (1) It conveys no title or ownership before death or the transferor should retain the ownership (full or naked) and control of the property while alive; (2) Before death, the transfer should be revocable by the transferor at will, even if the same may be done by indirect means of a reserved power; and (3) That the transfer should be void if the transferor should survive the transferee. If the donation is made in contemplation of the donor's death, then it is at the time that the donation takes effect, and it is a donation mortis causa which should be embodied in a last will and testament.

The said deed must be executed in accordance with the requisites on solemnities of wills and testaments under Articles 805 and 806 of the Civil Code. NKVS Bonsato v. CA 95 Phil. 481 FACTS: Respondents complaint charged that on December, 1949, Domingo Bonsato, then already a widower, had been induced and deceived into signing two notarial deeds of donations in favor of his brother Juan Bonsato and of his nephew Felipe Bonsato, respectively, transferring to them several parcels of land situated in the municipalities of Mabini and Burgos, Province of Pangasinan, both donations having been duly accepted in the same act and documents. Plaintiffs likewise charged that the donations were mortis causa and void for lack of the requisite formalities. The defendants, Juan Bonsato and Felipe Bonsato, answered averring that the donations made in their favor were voluntarily executed in consideration of past services rendered by them to the late Domingo Bonsato; that the same were executed freely without the use of force and violence, misrepresentation or intimidation; and prayed for the dismissal of the case and for damages in the sum of P2,000. After trial, the Court of First Instance rendered its decision on November 13, 1949, finding that the deeds of donation were executed by the donor while the latter was of sound mind, without pressure or intimidation; that the deeds were of donation inter vivos without any condition making their validity or efficacy dependent upon the death of the donor; but as the properties donated were presumptively conjugal, having been acquired during the coverture of Domingo Bonsato and his wife Andrea Nacario, the donations were only valid as to an undivided one-half share in the three parcels of land described therein. ISSUE: W/N the late Domingo Bonsato made donations inter vivos or dispositions post mortem in favor of the petitioners herein. -- DONATION INTER VIVOS. HELD: If the donation is a dispositions post mortem in favor of the defendants, then the documents should reveal any or all of the following characteristics: (1) Convey no title or ownership to the transferee before the death of the transferor; or, what amounts to the same thing, that the transferor should retain the ownership (full or naked) and control of the property while alive (Vidal vs. Posadas, 58 Phil., 108; Guzman vs. Ibea, 67 Phil., 633); (2) That before his death, the transfer should be revocable by the transferor at will, ad nutum; but revocability may be provided for indirectly by means of a reserved power in the donor to dispose of the properties conveyed (Bautista vs. Sabiniano, G. R. L-4326, November 18, 1952); (3) That the transfer should be void if the transferor should survive the transferee. None of these characteristics is discernible in the deeds of donation, executed by the late Domingo Bonsato. The donor only reserved for himself, during his lifetime, the owner's share of the fruits or produce a reservation that would be unnecessary if the ownership of the donated property remained with the donor. Most significant is the absence of stipulation that the donor

could revoke the donations; on the contrary, the deeds expressly declare them to be "irrevocable", a quality absolutely incompatible with the idea of conveyances mortis causa where revocability is of the essence of the act, to the extent that a testator can not lawfully waive or restrict his right of revocation. It is true that the last paragraph in each donation contains the phrase "that after the death of the donor the aforesaid donation shall become effective". However, said expression must be construed together with the rest of the paragraph, and thus taken, its meaning clearly appears to be that after the donor's death, the donation will take effect so as to make the donees the absolute owners of the donated property, free from all liens and encumbrances; for it must be remembered that the donor reserved for himself a share of the fruits of the land donated. Such reservation constituted a charge or encumbrance that would disappear upon the donor's death, when full title would become vested in the donees. AMPS Alejandro v. Geraldez 78 SCRA 245 DOCTRINE: A transfer mortis causa should be embodied in a last will and testament (Art. 728). It should not be called donation mortis causa. It is in reality a legacy. If not embodied in a valid will, the donation is void. FACTS: Andrea Diaz sued her brother, Angel Diaz, in the Court of First Instance for the partition of Lot 2502 donated to them by their parents. The Alejandros (compulsory heirs – husband and children of sister Olimpia Diaz) intervened in the said case, claiming one-third of Lot No. 2502 and that the donation of this lot to Andrea and Angel was a void mortis causa disposition. The trial court held that the said deed of donation was a donation mortis causa because the ownership of the properties donated did not pass to the donees during the donors' lifetime but was transmitted to the donees only "upon the death of the donors". However, it sustained the division of Lot No. 2502 into two equal parts between Angel Diaz and Andrea Diaz on the theory that the said deed of donation was effective "as an extra-judicial partition among the parents and their children. Consequently, the Alejandro intervenors were not given any share in Lot No. 2502. On appeal before the SC, the Alejandro intervenors contend that the said donation is mortis causa; that they are entitled to a one-third share in Lot No, 2502, and that the trial court erred in characterizing the deed as a valid partition. ISSUE: Whether the Alejandro intervenors should be awarded one-third of Lot No. 2502 square meters thereof, as intestate heirs of the Diaz spouses. HELD: Not entitled. Questioned donation valid. To resolve that issue, it is necessary to determine whether the deed of donation is inter vivos or mortis causa. The Code prescribes different formalities for the two kinds of donations. An inter vivos donation

of real property must be evidenced by a public document and should be accepted by the donee in the same deed of donation or in a separate instrument. In the latter case, the donor should be notified of the acceptance in an authentic form and that step should be noted in both instruments. (Art. 749, Civil Code. As to inter vivos donation of personal property, see art. 748). On the other hand, a transfer mortis causa should be embodied in a last will and testament (Art. 728, supra). It should not be called donation mortis causa. It is in reality a legacy (5 Manresa, Codigo Civil, 6th Ed., p. 107). If not embodied in a valid will, the donation is void (Narag vs. Cecilio, 109 Phil. 299; Aznar vs. Sucilla 102 Phil. 902; Tuazon vs. Posadas, 54 Phil. 289; Serrano vs. Solomon, 105 Phil. 998, 1002). From articles 728 to 732, it is evident that it is the time of effectivity (aside from the form) which distinguishes a donation inter vivos from a donation mortis causa . And the effectivity is determined by the time when the full or naked ownership (dominum plenum or dominium directum) of the donated properties is transmitted to the donees. (See Lopez vs. Olbes, 15 Phil. 540; Gonzales and Fuster Fabra vs. Gonzales Mondragon, 35 Phil. 105). The execution of a public instrument is a mode of delivery or tradition (Ortiz vs. Court of Appeals, 97 Phil. 46). The donation in the instant case is inter vivos because it took effect during the lifetime of the donors. It was already effective during the donors' lifetime, or immediately after the execution of the deed, as shown by the granting, habendum and warranty clause of the deed. In that clause it is stated that, in consideration of the affection and esteem of the donors for the donees and the valuable services rendered by the donees to the donors, the latter, by means of the deed of donation, wholeheartedly transfer and unconditionally give to the donees the lots mentioned and described in the early part of the deed, free from any kind of liens and debts. Thus, the habendum and warranty clause is the donors' declaration that they donate Lot No. 2502, the property in litigation, in equal shares to their children Angel Diaz and Andrea Diaz. The acceptance clause is another indication that the donation is inter vivos. Donations mortis causa, being in the form of a will, are never accepted by the donees during the donors' lifetime. Acceptance is a requirement for donations inter vivos. In the acceptance clause herein, the donees declare that they accept the donation to their entire satisfaction and, by means of the deed, they acknowledge and give importance to the generosity and solicitude shown by the donors and sincerely thank them. In the reddendum or reservation clause of the deed of donation, it is stipulated that the donees would shoulder the expenses for the illness and the funeral of the donors and that the donees cannot sell to a third person the donated properties during the donors' lifetime but if the sale is necessary to defray the expenses and support of the donors, then the sale is valid. The limited right to dispose of the donated lots, which the deed gives to the donees, implies that ownership had passed to them by means of' the donation and that, therefore, the donation was already effective during the donors' lifetime. That is a characteristic of a donation inter vivos. However, paragraph 3 of the reddendum in or reservation clause provides that "also, while we, the spouses Gabino Diaz and Severa Mendoza, are alive, our administration, right, and ownership of the lots mentioned earlier as our properties shall continue but, upon our death, the right and ownership of the donees to each of the properties allocated to each of them shall be fully effective." Evidently, the draftsman of the deed did not realize the discordant and ambivalent provisions thereof. The habendum clause indicates the transfer of the ownership over the donated properties to the donees upon the execution of the deed. But the reddendum

clause seems to imply that the ownership was retained by the donors and would be transferred to the donees only after their death. We have reflected on the meaning of the said contradictory clauses. All the provisions of the deed, like those of a statute and testament, should be construed together in order to ascertain the intention of the parties. Our conclusion is that the reddendum or reservation clause refers to the beneficial ownership (dominium utile) and not to the naked title and that what the donors reserved to themselves, by means of that clause, was the management of the donated lots and the fruits thereof. But, notwithstanding that reservation, the donation, as shown in the habendum clause, was already effective during their lifetime and was not made in contemplation of their death because the deed transferred to the donees the naked ownership of the donated properties. That conclusion is further supported by the fact that in the deed of donation, out of the eight lots owned by the donors, only five were donated. Three lots, Lots Nos. 4168, 2522 and 2521 were superflously reserved for the spouses or donors in addition to one- third of Lot No. 2377. If the deed of donation in question was intended to be a mortis causa disposition, then all the eight lots would have been donated or devised to the three children and daughter-in-law of the donors. The trial court's amended decision is reversed insofar as it pronounces that the deed of donation is void. That donation is declared valid as a donation inter vivos. The disputed lot should be partitioned in accordance with that deed between Andrea Diaz and Angel Diaz. KGS Reyes v. Masqueda 187 SCRA 661 DOCTRINES: 1. The title given to a deed of donation is not the determinative factor which makes the donation ―inter vivos‖ or ―mortis causa‖. 2. Characteristics of a donation inter vivos and mortis causa distinguished in Bonsato et al. v. Court of Appeals et al. 3. Whether a donation is inter vivos or mortis causa depends upon the nature of the disposition made. FACTS: Dr. Emilio Pascual died intestate and without issue. As such, his sister, Ursula, and the children of his late sisters became Dr. Pascual‘s heirs. The heirs filed Special Proceedings for the administration of the estate. Thereafter, Ursula filed a motion to exclude some properties from the inventory alleging that Dr. Pascual during his lifetime or in 1966 executed a ―Donation Mortis Causa‖ in her favor, which was granted by the court. Among the properties included in said donation was a lot in Tondo, Manila. Records show that in 1969 said property was donated inter vivos to Ofelia Parungao, who is then a minor. The latter‘s mother accepted it. Accordingly, said donation was duly registered. Subsequently, Ursula sold said property to the Reyes (involving 4 persons). The CFI ruled that the donation to Ursula is actually a donation inter vivos and that properties covered be excluded from the inventory of the estate of Dr. Pascual. The donation to Ofelia was declared null and void. ISSUE:

Is the donation made by Dr. Pascual to Ursula a donation inter vivos although the title given to the deed is ―donation mortis causa‖? -- YES HELD: It is, now a settled rule that the title given to a deed of donation is not the determinative factor which makes the donation "inter vivos" or "mortis causa" As early as the case of Laureta v. Manta, et al., this Court ruled that the dispositions in a deed of donation-whether "inter vivos" or "mortis causa" do not depend on the title or term used in the deed of donation but on the provisions stated in such deed. This Court explained in Concepcion v. Concepcion — ...But, it is a rule consistently followed by the courts that it is the body of the document of donation and the statements contained therein, and not the title that should be considered in ascertaining the intention of the donor. Here, the donation is entitled and called donacion onerosa mortis causa. From the body, however, we find that the donation was of a nature remunerative rather than onerous. It was for past services rendered, services which may not be considered as a debt to be paid by the donee but services rendered to her freely and in goodwill. The donation instead of being onerous or for a valuable consideration, as in payment of a legal obligation, was more of remuneratory or compensatory nature, besides being partly motivated by affection. We should not give too much importance or significance to or be guided by the use of the phrase 'mortis causa in a donation and thereby to conclude that the donation is not one of inter vivos. In the case of De Guzman et al. v. Ibea et al., this Court through Mr. Chief Justice Avancena said that if a donation by its terms is inter vivos, this character is not altered by the fact that the donor styles it mortis causa. In the case of Laureta v. Mata, et al., the court held that the donation involved was inter vivos. There, the donor Severa Magno y Laureta gave the properties involved as — ... a reward for the services which he is rendering me, and as a token of my affection toward him and of the fact that he stands high in my estimation, I hereby donate 'mortis causa to said youth all the properties described as follows: xxx xxx xxx I also declare that it is the condition of this donation that the donee cannot take possession of the properties donated before the death of the donor, and in the event of her death the said donee shall be under obligation to cause a mass to be held annually as a suffrage in behalf of my sold, and also to defray the expenses of my burial and funerals.‘ It will be observed that the present case and that of Laureta above cited are similar in that in both cases the donation was being made as a reward for services rendered and being rendered, and as a token of affection for the donee; the phrase 'mortis causa was used; the donee to take possession of the property donated only after the death of the donor; the donee was under obligation to defray the expenses incident to the celebration of the anniversary of the donor's death, including church fees. The donation in both cases were duly accepted. In said case of Laureta this Court held that the donation was in praesenti and not a gift in futuro. In the later case of Bonsato et al. v. Court of appeals, et al. this Court, distinguished the characteristics of a donation inter vivos and "mortis causa" in this wise: Did the late Domingo Bonsato, make donations inter vivos or dispositions post mortem in favor of the petitioners herein? If the latter, then the documents should reveal any or all of the following characteristics: (1) Convey no title or ownership to the transferee before the death of the transferor; or, what amounts to the same thing, that the transferor should retain the ownership (fun or naked) and control of the property while alive; (2) That before his death, the transfer should be revocable by the transferor at will, ad nutum; but revocability may be provided for indirectly by means of a reserved power in the donor to dispose of the properties conveyed; (3) That the transfer should

be void if the transferor should survive the transferee. These principles were repeated in the case of Castro v. Court of Appeals, to wit: Whether a donation is inter vivos or mortis causa depends upon the nature of the disposition made. 'Did the donor intend to transfer the ownership of the property donated upon the execution of the donation? If this is so, as reflected from the provisions contained in the donation, then it is inter vivos; otherwise, it is merely mortis causa, or made to take effect after death.' Applying the above principles to the instant petitions, there is no doubt that the so-called DONATION MORTIS CAUSA is really a donation inter vivos. The donation was executed by Dr. Pascual in favor of his sister Ursula Pascual out of love and affection as well as a recognition of the personal services rendered by the donee to the donor. The transfer of ownership over the properties donated to the donee was immediate and independent of the death of the donor. The provision as regards the reservation of properties for the donor's subsistence in relation to the other provisions of the deed of donation confirms the intention of the donor to give naked ownership of the properties to the donee immediately after the execution of the deed of donation. JPOT Gestopa v. CA 342 SCRA 105 DOCTRINE: Acceptance makes a donation inter vivos. There cannot be acceptance mortis causa in the lifetime of a donor because this would be in a form of a will. FACTS: The Danlags own six properties of unregistered land in Cebu. They donated four parcels of land mortis causa with reservations to Mercedes Pilapil who is an illegitimate child of the husband. After four years, the donors then again donated to Pilapil only this time it was inter vivos including the remaining two parcels. Afterwhich, the spouses sold two parcels to the Gestopas. It was first held in favor of the Gestopas and eventually reversed by the CA. It should be noted also that the husband of Mercedes was forced to purchase two parcels even though it was contrary to their interest. The Pilapils assumed responsibility over these lands and have tax declarations which were appreciated but put aside for reasons of them being easily obtained from the municipal offices, which after all, does not prove ownership. Some of the terms included in the Danlags issuance of donation was for them to enjoy the fruits of said parcels during their lifetime. Unfortunately, there were incidents when they were denied from getting coconuts. ISSUE: W/N the donation was inter vivos or mortis causa is the determination of whether the donor intended to transfer the ownership over the properties upon the execution of the deed. RULING: Said donation was absolutely made inter vivos. Spouses knew of the difference between the two modes and made use of them in remarkably distinguishing actions. To stress the court continues: "Note first that the granting clause shows that Diego (father) donated the properties out of love and affection for the donee. This is a mark of a donation inter vivos. Second, the reservation of lifetime usufruct indicates that the donor intended to transfer the naked ownership over the properties. As correctly posed by the Court of Appeals, what was the need for such

reservation if the donor and his spouse remained the owners of the properties? Third, the donor reserved sufficient properties for his maintenance in accordance with his standing in society, indicating that the donor intended to part with the six parcels of land. Lastly, the donee accepted the donation. In the case of Alejandro vs. Geraldez, 78 SCRA 245 (1977), we said that an acceptance clause is a mark that the donation is inter vivos. Acceptance is a requirement for donations inter vivos. Donations mortis causa, being in the form of a will, are not required to be accepted by the donees during the donors' lifetime." The only recourse best available for the spouses were to revoke said donation on grounds of officiousness or ingratitude, both of which were not invoked. WHEREFORE, the instant petition for review is DENIED. The assailed decision of the Court of Appeals dated August 31, 1993, is AFFIRMED. Costs against petitioners.

MLAV Quijada v. CA 299 SCRA 645 DOCTRINE: Donation is perfected once the acceptance by the donee is made known to the donor. Accordingly, ownership is immediately transferred and that ownership will only revert to the donor if the resolutory condition is not fulfilled. FACTS: Petitioners are the children of the late Trinidad Corvera Vda. de Quijada. Trinidad was one of the heirs of a two-hectare parcel of land. Trinidad Quijada together with her sisters and brother executed a conditional deed of donation of the two-hectare parcel of land in favor of the Municipality of Talacogon, the condition being that the parcel of land shall be used solely and exclusively as part of the campus of the proposed provincial high school in Talacogon. Trinidad remained in possession of the parcel of land despite the donation. She sold one (1) hectare of the subject parcel of land to respondent Regalado Mondejar. Subsequently, she verbally sold the remaining to respondent Mondejar without the benefit of a written deed of sale and evidenced solely by receipts of payment. In 1980, the heirs of Trinidad, who at that time was already dead, filed a complaint for forcible entry against respondent Mondejar. It was dismissed for failure to prosecute. Because the proposed provincial high school having failed to materialize, the municipality of Talacogon enacted a resolution reverting the land donated back to the donors. In the meantime, respondent Mondejar sold portions of the land to respondents Fernando Bautista, Rodolfo Goloran, Efren Guden and Ernesto Goloran. In their complaint, petitioners alleged that their deceased mother never sold the property. At the time of the alleged sale, the land belongs to the Municipality of Talacogon by virtue of the conditional deed of donation. Therefore, it renders the sale null and void. Respondents, in their answer, claimed that the land was sold to Mondejar. And that petitioners‘ action is already barred by prescription.

The lower court ruled in favor of the petitioners. It held that Trinidad Quijada had no legal title or right to sell the land to respondent Mondejar since the ownership at the time of sale belongs to the municipality. On appeal, the CA reversed and set aside the lower court‘s judgment ruling that the sale made by Trinidad Quijada to respondent Mondejar was valid as the former retained an inchoate interest on the lots by virtue of the automatic reversion clause in the deed of donation. ISSUE: WON the sale to Mondejar is void -- NO HELD The donation was subject to the condition that the donated property shall be "used solely and exclusively as a part of the campus of the proposed Provincial High School in Talacogon." It further provides that should the proposed high school be discontinued or in the future be closed, the donated property shall automatically revert to the donor. Such condition was validly imposed in the donation. The donation is perfected once the acceptance by the donee is made known to the donor. Accordingly, ownership is immediately transferred to the latter and that ownership will only revert to the donor if the resolutory condition is not fulfilled. At the time of the sales made, the alleged seller (Trinidad) could not have sold the lots since ownership was already transferred. So long as the resolutory condition subsists and is capable of fulfillment, the donation remains effective and the donee continues to be the owner. Such period, however, became irrelevant when the donee-Municipality manifested through a resolution that it cannot comply with the condition of building a school and the same was made known to the donor. Only then that ownership of the donated property reverted to the donor as provided in the automatic reversion clause of the deed of donation. The donor may have an inchoate interest in the donated property during the time that ownership of the land has not reverted to her. Such inchoate interest may be the subject of contracts including a contract of sale. In this case, however, what the donor sold was the land itself which she no longer owns. As to laches, petitioners' action is not yet barred by it. When petitioners initiated the suit, it cannot be said that they had slept on their rights for a long time. Be that at it may, there exists a valid sale. Sale, being a consensual contract, is perfected by mere consent, which is manifested the moment there is a meeting of the minds as to the offer and acceptance thereof on three (3) elements: subject matter, price and terms of payment of the price. Ownership by the seller on the thing sold at the time of the perfection of the contract of sale is not an element for its perfection. What the law requires is that the seller has the right to transfer ownership at the time the thing sold is delivered. The consummation, however, occurs upon the constructive or actual delivery of the subject matter to the buyer when the seller or her successors-in-interest subsequently acquires ownership thereof. Such circumstance happened in this case when petitioners became the owners of the subject property upon the reversion of the ownership of the land to them. Consequently, ownership is transferred to respondent Mondejar and those who claim their right from him. DJTV

Lagazo v. CA 287 SCRA 18

DOCTRINES: ● A simple or pure donation is one whose cause is pure liberality (no strings attached), while an onerous donation is one which is subject to burdens, charges or future services equal to or more in value than the thing donated. Under Article 733 of the Civil Code, donations with an onerous cause shall be governed by the rules on contracts; hence, the formalities required for a valid simple donation are not applicable. ● Acceptance of the donation by the donee is indispensable, its absence makes the donation null and void. FACTS: Petitioner filed an action seeking to recover from private respondent Cabanlit a parcel of land which the former claims to have acquired from his grandmother by donation. Private respondent, on the other hand, put up the defense that when the alleged donation was executed, he had already acquired the property by a Deed of Assignment from a transferee of plaintiff-appellee's grandmother. After trial, the lower court decided in favor of plaintiff-appellee and against defendant-appellant, rationalizing that the evidence presented by the former is more credible than that of the latter. Respondent Court of Appeals reversed trial court‘s decision and anchored its ruling upon the absence of any showing that petitioner accepted his grandmother's donation of the subject land. Citing jurisprudence that the donee's failure to accept a donation whether in the same deed of donation or in a separate instrument renders the donation null and void, Respondent Court denied petitioner's claim of ownership over the disputed land. The appellate court also struck down petitioner's contention that the formalities for a donation of real property should not apply to his case since it was an onerous one — he paid for the amortizations due on the land before and after the execution of the deed of donation — reasoning that the deed showed no burden, charge or condition imposed upon the donee; thus, the payments made by petitioner were his voluntary acts. Petitioner contends that the burdens, charges or conditions imposed upon a donation need not be stated on the deed of donation itself. Thus, although the deed did not categorically impose any charge, burden or condition to be satisfied by him, the donation was onerous since he in fact and in reality paid for the installments in arrears and for the remaining balance of the lot in question. Being an onerous donation, his acceptance thereof may be express or implied, as provided under Art. 1320 of the Civil Code, and need not comply with the formalities required by Art. 749 of the same code. His payment of the arrearages and balance and his assertion of his right of possession against private respondent clearly indicate his acceptance of the donation. ISSUES: 1. Whether or not the acceptance of a donation made in a separate instrument but not formally communicated to the donor may be considered complete, valid and subsisting. - NO 2. Whether or not the deed of donation which did not expressly impose any burden (the expressed consideration being purely one of liberality and generosity) but the recipient actually paid charges imposed on the property like land taxes and installment arrearages may be deemed onerous and thus governed by the law on ordinary contracts. -- NO

HELD. A simple or pure donation is one whose cause is pure liberality (no strings attached), while an onerous donation is one which is subject to burdens, charges or future services equal to or more in value than the thing donated. Under Article 733 of the Civil Code, donations with an onerous cause shall be governed by the rules on contracts; hence, the formalities required for a valid simple donation are not applicable. The Supreme Court ruled that the donation was simple, not onerous. Even conceding that petitioner's full payment of the purchase price of the lot might have been a burden to him, such payment was not however imposed by the donor as a condition for the donation. It is clear that the donor did not have any intention to burden or charge petitioner as the donee. Supreme Court agrees with the respondent Court that the payments made by petitioner were merely his voluntary acts. As a pure or simple donation, the donation is perfected from the moment the donor knows of the acceptance by the donee (Art. 734 of the Civil Code). Acceptance of the donation by the donee is, therefore, indispensable; its absence makes the donation null and void. Furthermore, if the acceptance is made in a separate instrument, the donor shall be notified thereof in an authentic form, and this step shall be noted in both instruments. WHEREFORE, the petition is DENIED and the assailed Decision is AFFIRMED. JGY Cagaoan v. Cagaoan 43 Phil. 554 DOCTRINE: Applying the rules on double sale, if the one who first registered the property is in bad faith, the one who first took possession of the property shall have the better claim. FACTS: Plaintiff Eugenio and the defendant Felix are brothers, the sons of Gregorio Cagaoan. Gregorio executed a deed of gift of four parcels of land in Pangasinan, in favor of Felix. He executed a similar deed in favor of Eugenio for a parcel of land which, apparently, is the same as that described as parcel No. 4 in the deed of gift executed in favor of Felix. Both of the deeds of gift are free from formal defects and were duly accepted by the donees. Eugenio went into possession of the parcel donated to him but he failed to get the donation recorded with the register of deeds. The deed given to Felix was duly recorded but he has never had possession of parcel No. 4. Gregorio died. Plaintiff Eugenio filed an action to be declared as the owner of the parcel donated to him. On the other hand, Felix asks that he be awarded the possession of the land. The trial court rendered judgment for the defendant ISSUE: WON the rules on double sale should be applied -- YES HELD: The case seems to use to be analogous to one where the same real property has been sold by the same vendor to two difference vendees. In such cases, under article 1473 of the Civil Code, the property goes to the vendee who first records his title in the registry of property. If the sale is

not recorded by either vendee, the property goes to the one who first takes possession of its in good, faith, and in the absence of both record and possession, to the one who present oldest title, provided there is good faith. It clearly appears that Felix Cagaoan had full notice of the plaintiff's claim to the land before he had his deed of gift recorded with the register of deeds. The plaintiff Eugenio Cagaoan having first taken possession in good faith must therefore be considered to have the better right to the land in question. JRPA JLT Agro v. Balasag 453 SCRA 211 DOCTRINE: Well-entrenched is the rule that all things, even future ones, which are not outside the commerce of man may be the object of a contract. The exception is that no contract may be entered into with respect to future inheritance, and the exception to the exception is partition inter vivos referred to in Article 1080. FACTS: Don Julian Teves contracted two marriages, first with Antonia Baena and had two kids namely Josefa and Emilio. After her death, he married Milagros Teves and they had four children namely: Maria Teves, Jose Teves, Milagros Teves and Pedro Teves. Thereafter, the parties to the case entered into a Compromise Agreement. When Antonia died an action for partition was instituted where the parties entered into a Compromise Agreement which embodied the partition of all the properties of Don Julian. On the basis of the compromise agreement, the CFI declared a tract of land known as Hacienda Medalla Milagrosa as property owned in common by Don Julian and his two children of the first marriage. The property was to remain undivided during the lifetime of Don Julian. Josefa and Emilio likewise were given other properties at Bais, including the electric plant, the ―movie property,‖ the commercial areas, and the house where Don Julian was living. The remainder of the properties was retained by Don Julian. On 16 November 1972, Don Julian, Emilio and Josefa executed a Deed of Assignment of Assets with Assumption of Liabilities in favor of J.L.T. Agro, Inc. (petitioner). Later, Don Julian, Josefa and Emilio also executed an instrument entitled Supplemental to the Deed of Assignment of Assets with the Assumption of Liabilities (Supplemental Deed) dated 31 July 1973. This instrument transferred ownership over Lot No. 63, among other properties, in favor of petitioner. The appellate court ruled that the supplemental deed, conveying ownership to JLT agro is not valid because the Compromise Agreement reserved the properties to Don Julian‘s two sets of heirs their future legitimes. The two sets of heirs acquired full ownership and possession of the properties respectively adjudicated to them and Don Julian himself could no longer dispose of the same. The appellate court in holding that the Supplemental Deed is not valid, added that it contained a prohibited preterition of Don Julian‘s heirs from the second marriage. ISSUE: 1. Was there preterition in the case? 2. 3. Whether or not the future legitime can be determined, adjudicated and reserved prior to

the death of Don Julian HELD: 1. None. Manresa defines preterition as the omission of the heir in the will. In the case at bar, Don Julian did not execute a will since what he resorted to was a partition inter vivos of his properties, as evidenced by the court approved Compromise Agreement. Thus, it is premature if not irrelevant to speak of preterition prior to the death of Don Julian in the absence of a will depriving a legal heir of his legitime. Besides, there are other properties which the heirs from the second marriage could inherit from Don Julian upon his death. 2. As a general rule, No. Well-entrenched is the rule that all things, even future ones, which are not outside the commerce of man may be the object of a contract. The exception is that no contract may be entered into with respect to future inheritance, and the exception to the exception is partition inter vivos referred to in Article 1080. The partition inter vivos of the properties of Don Julian is undoubtedly valid pursuant to Article 1347. However, considering that it would become legally operative only upon the death of Don Julian, the right of his heirs from the second marriage to the properties adjudicated to him under the compromise agreement was but a mere expectancy. It was a bare hope of succession to the property of their father. Being the prospect of a future acquisition, the interest by its nature was inchoate. Evidently, at the time of the execution of the supplemental deed in favor of petitioner, Don Julian remained the owner of the property since ownership over the subject lot would only pass to his heirs from the second marriage at the time of his death. ABB Roman Catholic Archbishop of Manila v. CA 198 SCRA 300 DOCTRINE: There is no need for prescription to be applied where a stipulation for automatic reversion is expressly provided for in the terms of the deed of donation. Hence, there is no need for a judicial declaration for the rescission of a contract because the law of the contract governs. FACTS: Private respondents spouses Eusebio de Castro and Martina Rieta executed a deed of donation in favor of the Roman Catholic Archbishop of Manila covering a parcel of land wherein a resolutory condition was imposed that donee shall not dispose or sell the property within a period of one hundred (100) years from the execution of the deed of donation, otherwise would render ipso facto null and void and such deed and property would revert back to donors. However, prior to the exhaustion of the period of one hundred (100) years, the Bishop of Imus executed a deed of absolute sale to spouses Florencio and Soledad Ignao for P114,000.00. Rieta then filed a complaint for the nullification of the deed of donation, reconveyance of the property with damages, and for the rescission of the contract. Ignao, in his answer said that the action for the rescission of the contract and reconveyance of the property has already prescribed. ISSUE: Whether or not the cause of action in the case at bar has already prescribed. -- NO

HELD: As a general rule, article 764 of the New Civil Code provides that "(t)his action shall prescribe after 4 years from the non-compliance with the condition, may be transmitted to the heirs of the donor, and may be exercised against the donee's heirs. But in the case at bar, there is no need for prescription to be applied where a stipulation for automatic reversion is expressly provided for in the terms of the deed of donation. Hence, there is no need for a judicial declaration for the rescission of a contract because the law of the contract governs. Judicial action is proper only when there is absence of a special provision granting the power of cancellation. However, the resolutory condition is held to be an undue restriction on the rights of ownership and is contrary to public policy. A donation is an effective transfer of title over the property from the donor to the donee. Once a donation is accepted, the donee becomes the absolute owner of the property donated. Although the donor may impose certain conditions in the deed of donation, the same must not be contrary to law, morals, good customs, public order and public policy. The condition imposed must not be perpetual or for an unreasonable period of time. FZC Genato v. Lorenzo 23 SCRA 618 DOCTRINE: The delivery by the donor and the acceptance by done must be simultaneous and the acceptance by a person other than the true done must be authorized by a proper power of attorney set forth in a public document FACTS: The property under dispute in this case is the 530 shares of stocks of Genato Commercal Corporation, which has P100 par value, of the deceased Simona B. De Genato (Director and secretary-treasurer of the said company). The petitioners herein, 2 heirs of Simona, are claiming that they own 530 shares of stocks of Genato Commercal Corporation because of the donation made by Simona to them. Respondents (other remaining heirs), however, are trying to recover from the petitioners, their co-heirs, the said stocks so they can include it in the intestate estate which should later be distributed among all the surviving children of the decedent. Four or five days after having Florentino Genato elected and designated as Assitant SecretaryTreasurer of the Corporation, 265 shares were issued in favour of Florentino Genato and another 265 were issued in favour of Francisco G. Genato. These were not presented as evidence in the course of the trial; they were merely mentioned by Florentino Genato in the course of his testimony as a witness. ISSUE: Whether or not there was a valid donation? -- NO HELD: There was no valid donation for lack of proper acceptance. Incontestably, one of the two donees was not present at the delivery, and there is no showing that Francisco Genato had authorized his brother, Florentino to accept for both of them. The delivery by the donor and the acceptance by done must be simultaneous and the acceptance by a person other than the true done must be authorized by a proper power of attorney set forth in a public document. None has been claimed to exist in this case.

LNAC Cruz v. Court of Appeals 140 SCRA 245 DOCTRINE: In the case of the subsequent adoption of a minor by one who had previously donated some or all of his properties to another, the donor may sue for the annulment or reduction of the donation within four years from the date of adoption, if the donation impairs the legitime of the adopted, taking into account the whole estate of the donor at the time of the adoption of the child. Of course, the burden of proof is on the plaintiff-donor, who must allege and establish the requirements prescribed by law, on the basis of which annulment or reduction of the donation can be adjudged. FACTS: Eduvigis J. Cruz, a childless widow, donated a 235.5 sq.m residential lot in San Isidro, Taytay Rizal together with the two-door apartment erected thereon to her grandnieces private respondents herein, in a deed of donation entitled "Kasulatan Sa Kaloobpala". The property was accordingly transferred to the names of private respondents. Cruz judicially adopted Cresencia Ocreto, a minor, after which she extrajudicially tried to revoke the donation, but the donees resisted, alleging that: (a) the property in question was co-owned by Eduvigis Cruz and her brother, the late Maximo Cruz, grandfather of the donees, hence the latter own 1/2 of the property by inheritance; and (b) Eduvigis Cruz owns another property, an agricultural land of more than two hectares situated in Barrio Dolores, Taytay, Rizal, hence the donation did not impair the presumptive legitime of the adopted child. Petitioner filed a complaint against the donees for revocation of donation in the CFI. Trial court rendered a decision revoking the donation. On appeal, the CA reversed the trial court and dismissed the complaint. Thus, prompted herein petition for review. ISSUE: Whether the CA correctly dismissed the complaint to annul the subject donation. -- YES HELD: In the case of the subsequent adoption of a minor by one who had previously donated some or all of his properties to another, the donor may sue for the annulment or reduction of the donation within four years from the date of adoption, if the donation impairs the legitime of the adopted, taking into account the whole estate of the donor at the time of the adoption of the child. (Civil Code, Articles 760, 761 and 763). Of course, the burden of proof is on the plaintiff-donor, who must allege and establish the requirements prescribed by law, on the basis of which annulment or reduction of the donation can be adjudged. Unfortunately, in the case at bar, the complaint for annulment does not allege that the subject donation impairs the legitime of the adopted child. Indeed it contains no indication at all of the total assets of the donor. Nor is there proof of impairment of legitime. On the contrary, there is unrebutted evidence that the donor has another piece of land (27,342 sq. m.) situated in Dolores, Taytay, Rizal worth P273,420.00 in 1977, although then subject to litigation.

The legal situation of petitioner-donor, as plaintiff, is made worse by the factual finding of the CA that the grandfather of the donees was the owner pro indiviso of one-half of the donated land, the effect of which is to reduce the value of the donation which can then more easily be taken from the portion of the estate within the free disposal of petitioner. TKDC De Luna v. Abrigo 181 SCRA 150 FACTS: De Luna donated a portion of a 75 sq. m. lot to the Luzonian University Foundation. The donation was embodied in a Deed of Donation Intervivos and was subject to certain terms and conditions. In case of violation or non-compliance, the property would automatically revert to the donor. When the Foundation failed to comply with the conditions, de Luna ―revived‖ the said donation by executing a Revival of Donation Intervivos with the following terms and conditions: 1) The Donee shall construct on the land and at its expense a Chapel, Nursery, and Kindergarten School to be named after St. Veronica 2) Construction shall start immediately and must be at least 70% completed three years from the date of the Deed unless the Donor grants extensions 3) Automatic reversion in case of violation The Foundation accepted and the donation was registered and annotated in the TCT. By a Deed of Segregation, the foundation was issued a TCT for area the lot donated while the remaining area was retained by the De Luna. The children and only heirs of the late De Luna (died after the donation) filed a complaint with the RTC for the cancellation of the donation on the ground that the terms were violated. The Foundation defended itself by saying that it had partially and substantially complied with the conditions and that the donor granted it an indefinite extension of time to complete construction. The RTC dismissed the petition on the ground of prescription (for being filed after 4 years). The heirs did not file an MR and went straight to the SC. ISSUE: Whether the action prescribes in 4 years (based on art. 764 NCC-judicial decree of revocation of the donation) or in 10 years (based on art. 1144 –enforcement of a written contract) HELD: 10 years The donation subject of this case is one with an onerous cause. Under the old Civil Code, it is a settled rule that donations with an onerous cause are governed not by the law on donations but by the rules on contract. On the matter of prescription of actions for the revocation of onerous donation, it was held that the general rules on prescription apply. The same rules apply under the New Civil Code as provided in Article 733 thereof which provides: Donations with an onerous cause shall be governed by the rules on contracts, and remuneratory donations by the provisions of the present Title as regards that portion which

exceeds the value of the burden imposed. It is true that under Article 764 of the New Civil Code, actions for the revocation of a donation must be brought within four (4) years from the non-compliance of the conditions of the donation. However, said article does not apply to onerous donations in view of the specific provision of Article 733 providing that onerous donations are governed by the rules on contracts. The rules on prescription and not the rules on donation applies in the case at bar. AMD Central Phil. University v. CA 246 SCRA 511 DOCTRINE: Onerous Donation: one executed for a valuable consideration which is considered the equivalent of the donation itself, e.g., when a donation imposes a burden equivalent to the value of the donation. When a person donates land to another on the condition that the latter would build upon the land a school, the condition imposed was not a condition precedent or a suspensive condition but a resolutory one. FACTS: In 1939, Don Ramon Lopez, Sr., then a member of the Board of Trustees of CPU, executed a deed of donation in favor of CPU over a parcel of land, with the following annotations— 1. The land described shall be utilized by the CPU exclusively for the establishment and use of a medical college with all its buildings as part of the curriculum; 2. The said college shall not sell, transfer or convey to any third party nor in any way encumber said land; 3. The said land shall be called "RAMON LOPEZ CAMPUS", and the said college shall be under obligation to erect a cornerstone bearing that name. Any net income from the land or any of its parks shall be put in a fund to be known as the "RAMON LOPEZ CAMPUS FUND" to be used for improvements of said campus and erection of a building thereon. 50 years later, private respondents, the heirs of Don Ramon Lopez, Sr., filed an action for annulment of donation, reconveyance and damages against CPU alleging that the latter had not complied with the conditions of the donation. They also argued that CPU negotiated with the National Housing Authority (NHA) to exchange the donated property with another land. In its answer CPU alleged that the right of private respondents to file the action had prescribed and it denied any violation of the conditions in the deed of donation. In 1991, the RTC ruled against CPU and ordered it to reconvey the property in favor of the heirs of the donor. Petitioner appealed to the CA, which reversed the RTC decision. It ruled that the annotations at the petitioner's certificate of title were resolutory conditions, breach of which should terminate the rights of the donee thus making the donation revocable. The CA also ruled that while the first condition mandated the use of the donated property for the establishment of a medical school, the donor did not fix a period within which the condition must be fulfilled, hence, until a period was fixed for the fulfillment of the condition, petitioner could not be considered as having failed to comply with its part of the bargain. Hence, this petition for review on certiorari. ISSUE:

W/N the donation was onerous and the conditions therein resolutory, although such conditions were given no fixed period. -- YES HELD: Based on the conditions in the deed of donation, the donation was onerous. When Don Ramon Lopez donated the parcel of land but imposed an obligation upon CPU to establish a medical college thereon, the donation must be for an onerous consideration. Under Art. 1181 of the Civil Code, in conditional obligations, the acquisition of rights, as well as the extinguishment or loss of those already acquired, shall depend upon the happening of the event which constitutes the condition. Thus, when a person donates land to another on the condition that the latter would build upon the land a school, the condition imposed was not a condition precedent or a suspensive condition but a resolutory one. It is not correct to say that the school had to be constructed before the donation became effective, that is, before the donee could become the owner of the land, otherwise, it would be invading the property rights of the donor. The donation had to be valid before the fulfillment of the condition. If there was no fulfillment or compliance with the condition, such as what obtains in the instant case, the donation may now be revoked and all rights which the donee may have acquired under it shall be deemed lost and extinguished. The claim of petitioner that prescription bars the instant action of private respondents is unavailing. When the obligation does not fix a period but from its nature and circumstances it can be inferred that a period was intended, the general rule provided in Art. 1197 of the Civil Code applies, which provides that the courts may fix the duration because the fulfillment of the obligation itself cannot be demanded until after the court has fixed the period for compliance and such period has arrived. This general rule, however, cannot be applied considering the different set of circumstances existing in the instant case. More than a reasonable period of 50 years has allowed petitioner to avail of the opportunity to comply with the condition even if it be burdensome, to make the donation in its favor forever valid. But, unfortunately, it failed to do so. Hence, there is no more need to fix the duration of a term of the obligation when such procedure would be a mere technicality and formality and would serve no purpose than to delay or lead to an unnecessary and expensive multiplication of suits. Records are clear and facts are undisputed that since the execution of the deed of donation up to the time of filing of the instant action, petitioner has failed to comply with its obligation as donee. Petitioner has slept on its obligation for an unreasonable length of time. Hence, it is only just and equitable now to declare the subject donation already ineffective and, for all purposes, revoked so that petitioner as donee should now return the donated property to the heirs of the donor by means of reconveyance. DISSENTING OPINION (Davide): I agree with the view in the majority opinion that the donation in question is onerous considering the conditions imposed by the donor on the donee which created reciprocal obligations upon both parties. Beyond that, I beg to disagree. First of all, may I point out an inconsistency in the majority opinion's description of the donation in question. In one part, it says that the donation in question is onerous. Yet, later on it states that the donation is basically a gratuitous one.

Second, the discussion on conditional obligations is unnecessary. There is no conditional obligation to speak of in this case. It seems that the "conditions" imposed by the donor and as the word is used in the law of donations is confused with "conditions" as used in the law of obligations. In his annotation of Article 764 of the Civil Code on Donations, Arturo M. Tolentino states clearly the context within which the term "conditions" is used in the law of donations, to wit: ―The word "conditions" in this article does not refer to uncertain events on which the birth or extinguishment of a juridical relation depends, but is used in the vulgar sense of obligations or charges imposed by the donor on the donee. It is used, not in its technical or strict legal sense, but in its broadest sense.‖ Clearly then, when the law and the deed of donation speaks of "conditions" of a donation, what are referred to are actually the obligations, charges or burdens imposed by the donor upon the donee and which would characterize the donation as onerous. In the present case, the donation is, quite obviously, onerous, but it is more properly called a "modal donation." A modal donation is one in which the donor imposes a prestation upon the donee. The establishment of the medical college as the condition of the donation in the present case is one such prestation. The conditions imposed by the donor Don Ramon Lopez determines neither the existence nor the extinguishment of the obligations of the donor and the donee with respect to the donation. In fact, the conditions imposed by Don Ramon Lopez upon the donee are the very obligations of the donation — to build the medical college and use the property for the purposes specified in the deed of donation. It is very clear that those obligations are unconditional, the fulfillment, performance, existence or extinguishment of which is not dependent on any future or uncertain event or past and unknown event, as the Civil Code would define a conditional obligation. It is incorrect to say that the "conditions" of the donation in the present case are resolutory conditions because, applying Article 1181 of the Civil Code, that would mean that upon fulfillment of the conditions, the rights already acquired will be extinguished. Obviously, that could not have been the intention of the parties. What the majority opinion probably had in mind was that the conditions are resolutory because if they are not complied with, the rights of the donee as such will be extinguished and the donation will be revoked. To my mind, though, it is more accurate to state that the conditions here are not resolutory conditions but, for the reasons stated above, are the obligations imposed by the donor. Third, I cannot subscribe to the view that the provisions of Article 1197 cannot be applied here. The conditions/obligations imposed by the donor herein are subject to a period. I draw this conclusion based on our previous ruling in Barretto vs. City of Manila, in which we said that when the contract of donation has no fixed period in which the condition should be fulfilled, the provisions of what is now Article 1197 are applicable and it is the duty of the court to fix a suitable time for its fulfillment. Indeed, from the nature and circumstances of the conditions/obligations of the present donation, it can be inferred that a period was contemplated by the donor. Don Ramon Lopez could not have intended his property to remain idle for a long period of time when in fact, he specifically burdened the donee with the obligation to set up a medical college therein and thus put his property to good use. There is a need to fix the duration of the time within which the conditions imposed are to be fulfilled. It is also important to fix the duration or period for the performance of the conditions/obligations in the donation in resolving the petitioner's claim that prescription has already barred the present action. I disagree once more with the ruling of the majority that the action of the petitioners is not barred by the statute of limitations. The mere fact that there is no time fixed as

to when the conditions of the donation are to be fulfilled does not ipso facto mean that the statute of limitations will not apply anymore and the action to revoke the donation becomes imprescriptible. Admittedly, the donation now in question is an onerous donation and is governed by the law on contracts (Article 733). Accordingly, the decision of the Court of Appeals must be upheld, except its ruling that the conditions of the donation are resolutory. CRF Parks v. Province of Tarlac 49 Phil. 142 DOCTRINE: A condition which cannot be complied with except after giving effect to the donation is not a condition precedent. (batasnatin) FACTS: On October 18, 1910, Concepcion Cirer and James Hill donated a parcel of land perpetually to the municipality of Tarlac, Province of Tarlac, under certain conditions(erection of a public school within six months) specified in the public document in which they made this donation which was accepted through its municipal president and was transferred in the name of the done. Later on, the donor sold this parcel to the herein plaintiff George L. Parks. On August 24, 1923, the municipality of Tarlac transferred the parcel to the Province of Tarlac. The plaintiff, George L. Parks, alleging that the conditions of the donation had not been complied with prayed that he be declared the absolute owner entitled to the possession of this parcel, that the transfer of the same by the municipality of Tarlac to the Province of Tarlac be annulled, and the transfer certificate issued to the Province of Tarlac cancelled. ISSUE: Whether or not the donation was coupled with a condition precedent? W/N the action to revoke has prescribed? -- NO HELD: Appellant contends that a condition precedent having been imposed in the donation and the same not having been complied with, the donation never became effective. We find no merit in this contention. The appellant refers to the condition imposed that one of the parcels donated was to be used absolutely and exclusively for the erection of a central school and the other for a public park, the work to commence in both cases within the period of six months from the date of the ratification by the parties of the document evidencing the donation. It is true that this condition has not been complied with. The allegation, however, that it is a condition precedent is erroneous. The characteristic of a condition precedent is that the acquisition of the right is not effected while said condition is not complied with or is not deemed complied with. In the present case the condition that a public school be erected and a public park made of the donated land, work on the same to commence within six months from the date of the ratification of the donation by the parties, could not be complied with except after giving effect to the donation. The donee could not do any work on the donated land if the donation had not really been effected, because it would be an invasion of another's title, for the land would have continued to belong to the donor so long as the condition imposed was not complied with. MPF Austria-Magat v. CA 375 SCRA 556

DOCTRINE: Whether the donation is inter vivos or mortis causa depends on whether the donor intended to transfer ownership over the properties upon the execution of deed. When the deed of donation provides that the donor will not dispose or take away the property donated, he in effect is making a donation inter vivos. FACTS: ● Basilisa Comerciante is a mother of five (5) children, namely, Rosario Austria, Consolacion Austria, herein petitioner Apolinaria Austria-Magat, Leonardo(deceased), and one of herein respondents, Florentino Lumubos. In 1953, Basilisa bought a parcel of residential land together with the improvement thereon covered and described in Transfer Certificate of Title No. RT-4036 (T-3268). On December 17, 1975, Basilisa executed a document designated as Kasulatan sa Kaloobpala (Donation). The said document which was notarized by Atty. Carlos Viniegra, reads as follows: ○ Na ako, si BASELISA COMERCIANTE, may sapat na gulang, Filipina, balo, at naninirahan sa blg. 809 L. Javier Bagong Pook, San Antonio, Lungsod ng Kabite, Filipinas, sa pamamagitan ng kasulatang itoy. xxx Na alang-alang sa mabuting paglilingkod at pagtingin na iniukol sa akin ng apat kong mga tunay na anak na sila: Kusang loob na ibinibigay ko at ipinagkakaloob ng ganap at hindi na mababawi sa naulit ng apat na anak ko at sa kanilang mga tagamagmana (sic), ang aking isang lupang residential o tirahan sampu ng aking bahay nahan ng nakatirik doon na nasa Bagong Pook din, San Antonio, Lungsod ng Kabite, at nakikilala bilang Lote no. 7, xxx Na ang Kaloob palang ito ay magkakabisa lamang simula sa araw na akoy pumanaw sa mundo, at sa ilalim ng kondision na: Magbubuhat o babawasin sa halaga ng nasabing lupa at bahay ang anumang magugul o gastos sa aking libing at nicho at ang anumang matitira ay hahatiin ng APAT na parte, parepareho isang parte sa bawat anak kong nasasabi sa itaas nito upang maliwanang (sic) at walang makkalamang sinoman sa kanila; At kaming apat na anak na naakalagda o nakadiit sa kasulatang ito ay TINATANGGAP NAMIN ang kaloob-palang ito ng aming magulang na si Basilisa Comerciante, at tuloy pinasasalamatan namin siya ng taos sa (sic) puso dahil sa kagandahan look (sic) niyang ito sa amin. Xxx (sgd.) ● Basilisa and her said children likewise executed another notarized document denominated as Kasulatan which is attached to the deed of donation. On February 6, 1979, Basilisa executed a Deed of Absolute Sale of the subject house and lot in favor of herein petitioner Apolinaria Austria-Magat for Five Thousand Pesos (P5,000.00). As the result of the registration of that sale, TCT No. RT-4036 in the name of the donor was cancelled and in lieu thereof TCT No. T-10434 was issued in favor of petitioner Apolinaria Austria-Magat on February 8, 1979. ● On September 21, 1983, herein respondents (representing their deceased mother Consolacion Austria), Ricardo, Mamerto and Segunda, all surnamed Sumpelo (representing their deceased mother Rosario Austria) and Florentino Lumubos filed before the Regional Trial Court of Cavite an action against the petitioner for annulment of TCT No. T-10434 and other relevant documents, and for reconveyance and damages. ISSUE: Whether or not the donation was inter vivos. -- YES HELD: The decisive proof that the deed is a donation inter vivos is in the provision that: Ibinibigay ko at ipinagkakaloob ng ganap at hindi mababawi sa naulit na apat na anak ko at sa kanilang mga tagapagmana, ang aking lupang residential o tirahan sampu ng aking bahay nakatirik doon xxx. This is a clear expression of the irrevocability of the conveyance. The irrevocability of the

donation is a characteristic of a donation inter vivos. By the words hindi mababawi, the donor expressly renounced the right to freely dispose of the house and lot in question. The right to dispose of a property is a right essential to full ownership. Hence, ownership of the house and lot was already with the donees even during the donors lifetime. xxx In the attached document to the deed of donation, the donor and her children stipulated that: Gayon din ang nasabing titulo ay hindi mapapasangla o maipagbibili ang lupa habang may buhay ang nasabing Basilisa Comerciante.xxx The stipulation is a reiteration of the irrevocability of the dispossession on the part of the donor. On the other hand, the prohibition to encumber, alienate or sell the property during the lifetime of the donor is a recognition of the ownership over the house and lot in issue of the donees for only in the concept of an owner can one encumber or dispose a property. The express irrevocability of the same (hindi na mababawi) is the distinctive standard that identifies that document as a donation inter vivos. The other provisions therein which seemingly make the donation mortis causa do not go against the irrevocable character of the subject donation. Thus, the court arrived at no other conclusion in that the petitioners cited provisions are only necessary assurances that during the donors lifetime, the latter would still enjoy the right of possession over the property; but, his naked title of ownership has been passed on to the donees; and that upon the donors death, the donees would get all the rights of ownership over the same including the right to use and possess the same. Furthermore, it also appeared that the provision in the deed of donation regarding the prohibition to alienate the subject property is couched in general terms such that even the donor is deemed included in the said prohibition (Gayon din ang nasabing Titulo ay hindi mapapasangla o maipagbibili ang lupa habang maybuhay ang nasabing Basilisa Comerciante). Both the donor and the donees were prohibited from alienating and encumbering the property during the lifetime of the donor. If the donor intended to maintain full ownership over the said property until her death, she could have expressly stated therein a reservation of her right to dispose of the same. The prohibition on the donor to alienate the said property during her lifetime is proof that naked ownership over the property has been transferred to the donees. It also supports the irrevocable nature of the donation considering that the donor has already divested herself of the right to dispose of the donated property. On the other hand, the prohibition on the donees only meant that they may not mortgage or dispose the donated property while the donor enjoys and possesses the property during her lifetime. However, it is clear that the donees were already the owners of the subject property due to the irrevocable character of the donation. JPOT Eduarte v. CA 253 SCRA 391 DOCTRINE: All crimes which offend the donor are considered manifests of ingratitude and are cause for revocation of donation. FACTS: Pedro Calapine donates half a parcel of his land to his niece, Helen Doria. For this benevolent act, he is blessed tenfold by greed and disloyalty. He willingly and knowingly gave only half of said land. However, there materialized a donation from him supposedly as well giving the other half to make whole said parcel. Moreover, these lands "donated" were made for profit and ironically enough, spirituality. This is when the Eduartes Romulo and Sally fall prey also.

Furthermore, a certain Calauan Christian Reform Church (CCRC) enters the fray. The niece, Helen, had the audacity to falsify said donation and donate yet again to attain false pretense of forgiveness. This, by way of giving the land to a church except of course the residence. The Eduartes, unknowingly trust Doria and purchase the lot altogether. Pedro then moves to reprimand his niece by setting forth machinations to effectively revoke his donation. ISSUE: W/N the act of Helen Doria of falsification of documents is tantamount to ingratitude towards Pedro Calapine which would lead to the effective revocation of donation? -- YES HELD: This act is of pure treason. Any action that the donor takes offense to is equal to ingratitude. It is saddening that an individual can have such a shameful display of thanks relating to family. The mere act of falsifying documents to reserve the right to half the land to which you have already been granted half of is appaling to say the least. WHEREFORE, the instant petition for review is DENIED. The assailed decision of the Court of Appeals dated August 31, 1993, is AFFIRMED. Costs against petitioners. JPOT Noceda v. CA 313 SCRA 504 DOCTRINE: Usurpation with regard to donee towards donor is a definite act of ingratitude and neecessitates only to be proved to effect revocation. FACTS: Aurora Directo, Rodolfo Noceda and Maria Arbizo extra-judicially partitioned a land in Zambales. Aurora coincidentally donated on the same day of the partition to Rodolfo who happens to be her nephew. After several surveys conducted by a certain Geodetic Engineer Eugene Quejada of the Bureau of Lands and specific boundary adjustments, Aurora opted to fence her property accordingly. This, for reasons of making known and presuppose which of the vast land is indeed in her rightful possession. Even though there was an apparent exercise of giving and safety measures so as to avoid conflict, it appears to have been averred by Rodolfo. It was made true by his actions of passing through said fences and intentionally staying at nipa huts designated inside Aurora's land. What comes into contention is the manner Rodolfo makes his action. ISSUE: W/N usurpation of Rodolfo is enough to suffice revocation of donation? -RULING: In this case, Rodolfo is directly showing his disregard of Aurora's wishes intentionally going beyond the boundaries Directo so painstakingly made plain. Usurpation is the undermining of one's authority. Noceda not only negates her will but makes it obvious in the sense of flaunting it even with the presence of Maria Arbizo. We find that both the trial court and the respondent Court had carefully considered the questions of fact raised below and the respondent Court‗s conclusions are based on the evidence on record. No cogent reason exists for disturbing such findings. We also note that petitioner in this petition merely rehashed the same issues and arguments raised in the

respondent Court in whose decision we find no reversible error. Clearly, petitioner failed to present any substantial argument to justify a reversal of the assailed decision. WHEREFORE, the petition for review is hereby DENIED. Costs against appellant. SO ORDERED.

View more...

Comments

Copyright ©2017 KUPDF Inc.
SUPPORT KUPDF